You are on page 1of 412

-

2
3
4
5
Item: 1 of40
0. ld : 4385
~'?Mark <]
Previous
C>
Next
!I
Lab Values
~
Notes
~
Calculator
,
Reverse Color
~
Text Zoom

6 A 68-year-old man is brought to the emergency department after an episode of coffee-g round
7 emesis followed by lightheadedness He has had black, tarry stools fo r the past few days. The
8 patient has been taking over-the-counter ibuprofen for osteoarthritis. His other medical problems
9 include hypertension and removal of 2 hyperplastic colon polyps 4 years ago. The patient drinks
10 2 or 3 beers on weekends. He does not smoke. Temperature is 36.7 C (98 F), blood pressure is
11 112/72 mm Hg, pulse is 11 0/min, and respirations are 16/min. His lungs are clear to
12
auscultation. The abdomen is slightly tender to deep palpation in the epigastrium Laboratory
13
14 results after the initial resuscitation are as follows
15 Complete blood count
16
17 Hemoglobin 6.8 g/dL
18
19 Platelets 130,000/mm•
20
21 Leukocytes 9,500/mm'
22
23
24 Serum chemistry
25
26 Sodium 140 mEq/L
27
28 Potassium 3.5 mEq/L
29
Bica rbonate 27 mEq/L
30
31 Blood urea nitrogen 50 mg/dL
32
33 Creatinine 1.7 mg/dL
34
35
36
37 Coagulation studies
38 1.2 (08-1 1)
INR
39
40
-
2
3
4
5
Item: 1 of40
0. ld : 4385
~'?Mark

Leukocytes
<J
Previous
C>
Next

9,500/mm'
61
Lab Values
~
Notes
~
Calculator
,
Reverse Color
~
Text Zoom

6
7
8 Serum chemistry
9
10 Sodium 140 mEq/L
11 Potassium 3.5 mEq/L
12
13 Bicarbonate 27 mEq/ L
14
15 Blood urea nitrogen 50 mg/dl
16
17 Creatinine 1.7 mg/dl
18
19
20 Coagulation studies
21
22 INR 1.2 (08-1 1)
23
24
In addition to intravenous fl uid resuscitation, which of the following is the most appropriate next
25
26 step in management of this patient?
27 -
28 ® A Fresh frozen plasma infusion
29
30 ® B. Intravenous famotidine
31 ® C. Intravenous octreotide
32
33 ® D. Packed red blood cell transfusion
34 ® E. Platelet transfusion
35
® F. W hole blood transfusion
36
37
38 Submit
39 T

40
-
2
3
4
5
Item: 1 of40
0. ld : 4385

Explanation:
~'?Mark <]
Previous
C>
Next
a
Lab Values
~
Notes
~
Calculator
,
Reverse Color
~
Text Zoom

6
7
8
9 Red blood cell transfusion thresholds
10
11
12 Hemoglobin
Recommendation
13 (g/dl)
14
15
16 <7 • Generally indicated
17
18
19
• Cardiac surgery
20 7-8 • Oncology patients in treatment
21 • Heart failure
22
23
24 • Symptomatic anemia
25 • Ongoing bleeding
8-10
26 • Acute coronary syndrome
27 • Non cardiac surgery
28
29
30 >10 • Not generally indicated
31
@UWorld
32
33
34 This patient's presentation (hematemesis, melena, anemia, and a blood urea nitrogen to
35 creatinine ratio >20) is consistent with upper gastrointestinal bleeding (UGIB), likely due to
36 nonsteroidal anti-inflammatory drug use. Supportive measures for UGIB include supplemental
37 oxygen, bowel rest, and intravenous fluids through large-bore catheters. An intravenous proton
38 pump inhibitor should also be administered for acid suppression Packed red blood cell (PRBC)
39 transfusion can increase oxygen-carrying capacity in patients with significantly low hemoglobin
40
-
2
3
4
5
Item : 1 of40
0. ld : 4385
. I I I ... I I I I
~'?Mark
I I " I ...
<]
Previous
.. I I I
C>
Next
..

considered for patients with acute coronary syndrome Patients with active bleeding and
I I I I • I I ., I ..
!I
Lab Values
~
Notes
~
Calculator
,
Reverse Color
~
Text Zoom

hypovolemia may need PRBC transfusion at higher hemoglobin levels due to the initial
6
7 hemoglobin concentration not fully reflecting blood loss. In addition, the hemoglobin level may
8 drop significantly as blood volume is replaced by the infusion of crystalloid solutions and the
9 mobilization of interstitial fluid.
10
11 (Choice A) Fresh frozen plasma contains all clotting factors and plasma proteins from one unit of
12 blood. It is usually indicated for severe coagulopathy (eg, liver disease, disseminated
13 intravascular coagulation) with active bleeding Fresh frozen plasma is generally not needed to
14 correct a minimally abnormal INR (<1.6) , which is a common finding in gastrointestinal bleeding.
15
16 {Choice B) Proton pump inhibito rs reduce rebleeding and the need for transfusions, and help
17 stabilize clots in patients with UGIB. Histamine-2 blockers, such as famotidine, have not
18 demonstrated such benefits and consequently are not recommended as first-line therapy for
19
UGIB.
20
21 (Choice C) Somatostatin analogs such as octreotide are a mainstay in management of va riceal
22 bleeding This patient has relatively limited alcohol use and no manifestations of cirrhosis, making
23
24 variceal bleeding less likely
25
{Choice E) Platelet transfusions are typically given for a platelet count <10,000/mm> (increased
26
27 risk of spontaneous hemorrhage) or for a platelet count <50,000/mm' with active bleeding
28
(Choice F) W hole blood transfusion, which includes PRBCs in addition to plasma, may be used
29
30 in patients with severe hemorrhage (eg, major trauma) requiring massive blood transfusions to
31 ass.ist in volume expansion.
32
33 Educational objective:
34 Packed red blood cell transfusions are recommended in acute gastrointestinal bleeding for
35 patients with hemoglobin <7 g/dl. A higher threshold of hemoglobin <9 g/dl is considered for
36 unstable patients with acute co ronary syndrome or with active bleeding and hypovolemia
37
38 References:
39
40
-
3
4
5
6
1
Item: 2 of 40
0. ld : 2213
~'?Mark <J
Previous
C>
Next

A 35-year-old woman comes to the physician with a 5-month history of episodic retrosternal pain
61
Lab Values
~
Notes
~
Calculator
,
Reverse Color
~
Text Zoom

7 that radiates to the interscapular region The episodes typically last 15 minutes, although they
8 occasionally can last a few hours, and are precipitated by emotional stress and hot or cold food.
9 She regurgitates food intermittently The patient's past medical history is unremarkable, and she
10 takes no medications. There is no fam ily history of coronary artery disease. Her vital signs are
11 within normal limits. Physical examination shows no abnormalities. A lipid profile is within normal
12
limits. An electrocardiogram ( ECG) during a pain episode shows a normal sinus rhythm without
13
14 ST-segment changes Sublingual nitroglyce rin tablets alleviate the pain A stress test fails to
15 reproduce the symptoms or to induce ST!T-wave changes Chest x-ray, upper gastrointestinal
16 endoscopy, and echocardiography show no abnormalities. W hich of the following is the most
17 appropriate next step in management of this patient?
18
19
20 ID A 24-hour pH monitoring
21 6 B. Coronary angiogram
22
23 ID C. CT scan of the chest with contrast
24 6 D. Esophageal motility studies (manometric recordings)
25
26 ® E. Pulmonary perfusion/ventilation scintigraphy
27
28
29 Submit
30
31
32
33
34
35
36
37
38
39
40
-
3
4
5
6
1
Item: 2 of 40
0. ld : 2213

.,
~'?Mark
D. Esophageal motility studies (manometric recordings) [80%]
E. Pulmonary perfusion/ventilation scintigraphy [0%]
<J
Previous
C>
Next
61
Lab Values
~
Notes
~
Calculator
,
Reverse Color
~
Text Zoom

7
8 Explanation:
9
10 This patient's symptoms (ie, spontaneous pain, odynophagia for cold and hot food) are
11 suggestive of diffuse esophageal spasm Resol ution of her chest pain after taking nitroglycerin
12 is also consistent with the diagnosis. Nitrates (and calcium channel blockers) relax not only
13 myocytes in coronary vessels, but also those in the esophagus, thereby alleviating the pain.
14
Esophagography may or may not show other anomalies (eg, co rkscrew shape). Esophageal
15
16 manometry should show repetitive, non-peristaltic , high-amplitude contractions, either
17 spontaneously or after ergonovine stimulation.
18
19 {Choice A) Radiation of pain to the back and its precipitation by emotional stress make the
20 diagnosis of motility disorder more likely than gastroesophageal reflux disease (GERD) GERD
21 most commonly causes burning discomfort (heartburn) rather than radiating pain and is
22 associated with esophagitis on endoscopy. Furthermore, if GERD were suspected, the patient
23 should be started on an empiric trial of a proton pump inhibitor rather than 24-hour pH monitoring
24
25 {Choice B) Most cardiac etiologies have been fairly well excluded by this workup, although this
26 patient's symptoms may also raise suspicion for variant ( Prinzmetal) angina However , several
27 features of her presentation, including the duration of pain (up to several hours) , regurgitation,
28 and the absence of ST changes during one of the episodes, make these less likely
29
30 (Choice C) A CT scan of the chest with contrast would be helpful in diagnosing a pulmonary
31
embolism or an aortic dissection. However, this patient's symptoms, including provocation by
32
stress and food, regurgitation, and relief by nitroglyce rin , make both of these diagnoses less
33
34 likely
35
36 {Choice E) Repetitive pulmonary embolism is unlikely with normal vital signs and no shortness of
37 breath.
38
Educational objective:
39
40 .. ... . .. .. . .. . . .. ... . ... . .. .. . ... .. . .. ... ... ... ... ... ... .. ... ... . .. . .. .
~
-------------------------------------------------------------------------------------------------------------------------------
Feedback SuWend EnQ ock
-
3
4
5
6
1
Item : 2 of 40
0 . ld : 2213
' . '- •• <I • ,- -
~'?Mark
•• ol • • -
<]
Previous
-- •• ., . I

Esophagography may or may not show other anomalies (eg, co rkscrew shape). Esophageal
C>
Next

manometry should show repetitive, non-peristaltic , high-amplitude contractions, either


- - . ' <I -, ,.. • - • ..,
!I
Lab Values
~
Notes
~
Calculator
,
Reve rse Color
~
Tex t Zoom

spontaneously or after ergonovine stimulation.


7
8 (Choice A) Radiation of pain to the back and its precipitation by emotional stress make the
9
diagnosis of motility disorder more likely than gastroesophageal reflux disease (GERD) GERD
10
11
most commonly causes burning discomfort (heartburn) rather than radiating pain and is
12 associated with esophagitis on endoscopy Furthermore, if GERD were suspected, the patient
13 should be started on an empiric trial of a proton pump inhibitor rather than 24-hour pH monitoring
14
15 (Choice B) Most ca rdiac etiologies have been fairly well excluded by this workup, although this
16 patient's symptoms may also raise suspicion for variant ( Prinzmetal) angina However , several
17 features of her presentation, including the duration of pain (up to several hours), regurgitation,
18 and the absence of ST changes during one of the episodes, make these less likely
19
20 (Choice C) ACT scan of the chest with contrast would be helpful in diagnosing a pulmonary
21 embolism or an aortic dissection. However , this patient's symptoms, including provocation by
22 stress and food , regurgitation, and relief by nitroglycerin, make both of these diagnoses less
23 likely
24
25 (Choice E) Repetitive pulmonary embolism is unlikely with normal vital signs and no shortness of
26
breath.
27
28 Educational objective:
29
Episodes of dysphagia, regurgitation, and/or chest pain precipitated by emotional stress should
30
31 raise the suspicion for esophageal motility disorder such as diffuse esophageal spasm.
32 Manometry establishes the diagnosis of diffuse esophageal spasm.
33
34 References:
35
36 1. Management of spastic disorders of the esophagus.
37
38
39 Time Spent 2 seconds Copyright © UWorld Last updated: [09/ 18/2016)
40

~
-------------------------------------------------------------------------------------------------------------------------------
Feedback SuWend EnQ ock
~'?Mark <J C> 61 ~ ~ , ~

--
Item: 3 of 40
2 0. ld : 4504 Previous Next Lab Values Notes Calculator Reverse Color Text Zoom

4
5
6 A 37-year-old man comes to the physician fo r a routine health maintenance examination. He has
7 been doing well and has no significant symptoms He has a 30-pack-year smoking history. The
8 patient does not use alcohol or illicit drugs. His father was diagnosed with colon cancer at age 50
9 and treated with colectomy. His oldest brother was diagnosed with non-small cell lung cancer at
10 age 50. His mother has Graves' disease, and his maternal grandfather had co ronary artery
11 disease. W hich of the following is the most appropriate initial screening test for this patient?
12
13
14 (:) A Colonoscopy at age 40
15
e; B. Colonoscopy at age 50
16
17 e> C. Exercise electroca rdiography
18
19 e; D. Low-dose CT scan of the chest
20 e> E. Sputum cytology
21
22 e; F. Thy roid-stimulating hormone level
23
24
25 Submit
26
27
28
29
30
31
32
33
34
35
36
37
38
39
40
~'?Mark C> !I ~ ~ , ~

--
Item: 3 of 40 <]
2 0. ld : 4504 Previous Next Lab Values Notes Calculator Reverse Color Text Zoom

4 Explanation:
5
6 The American Cancer Society and the United States Preventive Services Task Force (USPSTF)
7 strongly encourage routine colon cancer screening in all patients age ~50. Screening can be
8 performed using high-sensitivity fecal occult blood testing (FOBT) annually, flexible
9 sigmoidoscopy every 5 yea rs combined with FOBT every 3 years, or colonoscopy every 10
10 yea rs All 3 strategies decrease colon cancer mortality. Colonoscopy is the most sensitive and
11
specific test, but it is also the most costly and expertise dependent
12
13
Colonoscopy at age 50 is appropriate for patients with average colon cancer risk (Choice
14
B). However , this patient's family history places him at increased risk, and he requires screening
15
16 at an earlier age Patients with a history of colon cance r in a fi rst-degree relative should be
17 screened at age 40 or 10 yea rs before the age of the relative's diagnosis This patient's father
18 was diagnosed with colon cancer at age 50, and he should begin screening at age 40.
19
20 (Choice C) An exercise electrocardiogram (ECG) stress test is a useful diagnostic study in
21 patients with a moderate risk of coronary artery disease based on symptoms and risk
22 factors. Although this patient has risk factors for coronary artery disease, he is currently
23 asymptomatic and would not benefit from a stress ECG at this time.
24
25 (Choices D and E) This patient with a heavy smoking history and a family history of lung cancer
26 is at increased risk for lung cance r. Sputum cytology does not effectively screen for lung cance r
27 or decrease mortality from the disease. Low-dose chest CT is recommended yearly fo r lung
28
cancer screening in patients who are age 55-80, have a ~30-pack-year smoking history, and
29
are currently smoking or quit within the past 15 years.
30
31
(Choice F) The USPSTF guidelines conclude that there is currently insufficient evidence on
32
33 screening for thyroid disease in asymptomatic patients with a family history of thy roid disease.
34
35
Educational objective:
36 Patients at average risk of developing colon cancer should begin screening at age 50 with high-
37 sensitivity fecal occult blood testing (FOBT) annually, flexible sigmoidoscopy every 5 yea rs
38 combined with FOBT every 3 years, or colonoscopy every 10 yea rs Patients with an affected
39 fi rst-degree relative should begin screening at age 40 or 10 yea rs before the age of the relative's
40
2
Item: 4 of40 ~'?Mark <J C> 61 ~ ~ , ~

-
0. ld : 2934 Previous Next Lab Values Notes Calculator Reverse Color Text Zoom
3

5
6 A 42-year-old man comes to the emergency department with sudden-onset severe abdominal
7 pain and 2 episodes of vomiting. He rates his pain as 9/ 10 in severity with radiation to the back.
8 He has never experienced these symptoms before and has no other medical problems. The
9 patient admits to binge drinking last night while watching a football game on television. His
10 temperature is 37.7 C (99 9 F), blood pressure is 112/70 mm Hg, pulse is 102/min, and
11 respirations are 16/min. There is epigastric tenderness on palpation of the abdomen but no
12
rigidity There is no evidence of hematemesis, and a guaiac smear is negative Initial laboratory
13
14 findings include a normal hemoglobin and leukocytosis A chest radiograph and
15 electrocardiogram are normal. W hich of the following is the best next step in management of this
16 patient?
17
18
e> A. Abdominal computed tomography scan
19
20 e; B. Abdominal ultrasound
21
22
e> C. Abdominal upright x-rays
23 e; D. Endoscopic retrograde cholangiopanc reatography
24
25 (:) E. Serum amylase and lipase
26
27
28 Subm it
29
30
31
32
33
34
35
36
37
38
39
40
2
Item: 4 of40 ~'?Mark <] C> a ~ ~ , ~

-
0. ld : 2934 Previous Next Lab Values Notes Calculator Reverse Color Text Zoom
3

5 Clinical features of acute pancreatitis


6
7
• Chronic alcohol use
8
9 • Gallstones
10 • Hyperlipidemia (types I, IV & V)
11
12 Etiology • Drugs (eg, didanosine, azathioprine, valproic acid)
13 • Infections (eg, cytomegalovirus, Legionella, Aspergillus)
14
15 • Trauma
16 • Iatrogenic (post-ERCP)
17
18
19 Diagnosis requ ires 2 of the following:
20 • Acute epigastric abdominal pain often radiating to the
21
22 back
23 • t Amylase/lipase >3 times normal limit
24
25
• Abdominal imaging showing focal or diffuse pancreatic
26 enlargement with heterogeneous enhancement with
27 Clinical intravenous contrast (CT) or diffusely enlarged
28 presentation & hypoechoic pancreas (ultrasound)
29
30 Other findings:
31
32 • Nausea, vomiting, leukocytosis
33 • Severe disease with possible abdominal tenderness,
34
fever, tachypnea, hypoxemia & hypotension
35
36 • ALT level >150 units/L- biliary pancreatitis
37
38
39 • Pleural effusion
40
2
Item: 4 of40 ~'?Mark <] C> a ~ ~ , ~

-
0. ld : 2934 Previous Next Lab Values Notes Calculator Reverse Color Text Zoom
3

5 • Pleural effusion
6
7 • Ileus
Complications
8 • Pancreatic pseudocyst/abscess/necrosis
9
10 • Acute respiratory distress syndrome
11
©UWo~d
12
13
This patient's recent binge drinking, severe epigastric pain radiating to his back and leukocytosis
14
suggest acute pancreatitis Diagnosis of acute pancreatitis requires 2 of the following c riteria:
15
16 acute onset of severe epigastric pain radiating to the back, increased amylase or lipase >3 times
17 the upper limit of normal, and characteristic abdominal imaging findings (eg, focal or diffuse
18 pancreatic enlargement).
19
20 Early in pancreatitis, the pancreas synthesizes digestive enzymes but cannot secrete them.
21 These enzymes leak out of the acinar cells into the systemic circulation. Amylase rises within
22 6-12 hours of symptom onset and may remain elevated for 3-5 days. Lipase rises within 4-8
23 hours of symptom onset but remains elevated longer than amylase (8-14 days). As a result,
24 lipase is more useful and sensitive than amylase for diagnosis (especially in alcoholics and
25 patients presenting later in the disease course)
26
27 Imaging is not required for diagnosis in patients with typical abdominal pain and significantly
28
elevated amylase and/or lipase. Contrast-enhanced computed tomography scan of the abdomen
29
may be performed in patients with unclear diagnosis or in those who fail to improve with
30
31 conservative management (to identify infection or necrosis) This patient has characteristic
32 abdominal pain and would first need laboratory testing. If the diagnosis is still unclear , he may
33 require imaging such as abdominal computed tomography scan (Choice A)
34
35 {Choice B) Abdominal ultrasound may reveal a classic diffusely enlarged and hypoechoic
36 pancreas in acute panc reatitis However , nearly 30% of patients have an ileus with bowel gas
37 that prevents full visualization of the panc reas and may not clearly identify inflammation or
38 necrosis. Once the diagnosis of panc reatitis is confirmed, ultrasound may be useful to identify
39 ce rtain causes (eg, gallstones).
40
2
Item: 4 of40 ~'?Mark <] C> !I ~ ~ , ~

-
0. ld : 2934 Previous Next Lab Values Notes Calculator Reverse Color Text Zoom
3
conservative management (to identify infection or necrosis). This patient has characteristic
5 abdominal pain and would first need laboratory testing If the diagnosis is still unclear, he may
6 require imaging such as abdominal computed tomography scan (Choice A).
7
8 {Choice B) Abdominal ultrasound may reveal a classic diffusely enlarged and hypoechoic
9 pancreas in acute pancreatitis However, nearly 30% of patients have an ileus with bowel gas
10 that prevents full visualization of the pancreas and may not clearly identify inflammation or
11 necrosis. Once the diagnosis of pancreatitis is confirmed, ultrasound may be useful to identify
12
certain causes (eg, gallstones)
13
14
(Choice C) Abdom inal x-rays may be normal in mild pancreatitis or show nonspecific ileus in
15
severe disease. In either case, they are not as useful for diagnosing acute pancreatitis. An
16
17 upright chest x-ray, however, might be useful in this patient as it could show air under the
18 diaphragm if there were suspicion for bowel perforation.
19
20 {Choice D) Endoscopic retrograde cholangiopancreatography (ERCP) is not indicated for the
21 routine diagnosis of acute pancreatitis However, early ERCP for suspected biliary pancreatitis
22 may decrease morbidity and mortality ERCP is also effective for evaluating patients with
23 recurrent pancreatitis or draining pancreatic pseudocysts.
24
25 Educational objective:
26 Uncomplicated patients with acute onset of severe epigastric pain radiating to the back and
27 increased amylase or lipase (>3 times normal) do not need confirmatory imaging fo r diagnosing
28 acute pancreatitis. Contrast-enhanced computed tomography scan of the abdomen may be
29 performed in patients with unclear diagnosis or those who fail to improve with conservative
30
management. Abdominal ultrasound may identify gallstones as the cause of pancreatitis
31
32
33 References:
34
1. American College of Gastroenterology guideline: management of acute
35
36 pancreatitis.
37
38
39 Time Spent 2 seconds Copyright© UWorld Last updated [06/30/2016)
40
2
Item: 5of40 ~'?Mark <J C> 61 ~ ~ , ~
0. ld : 2205 Previous Next Lab Values Notes Calculator Reverse Color Text Zoom
3

-
4

6
7
8
9
A 26-year-old man comes to the emergency department with 4 weeks of intermittent lower
abdominal pain and cramps accompanied by rectal urgency, bloody diarrhea, nausea, and
decreased appetite His symptoms have become more severe over the past 2 days. The
patient's medical history is unremarkable, and he has had no recent travel or antibiotic use. His
10 temperature is 38.5 C (101 .3 F). blood pressure is 90/50 mm Hg, pulse is 130/min, and
11 respiratory rate is 15/min. Abdominal examination reveals distension, diffuse tenderness to
12
palpation, and decreased bowel sounds. There is no rebound tenderness. Rectal examination
13
14 shows marked tenderness and mucus mixed with blood in the vault Laboratory results are as
15 follows:
16 Hemoglobin 10.2 g/dL
17
18 Leukocytes 31 ,600/mm'
19
20 Platelets 459,000/mm'
21 -
22 Erythrocyte sedimentation
54 mm/hr
23 rate
24
25 Intravenous fluids are started, with improvement of blood pressure to 103/58 mm Hg Which of
26
the following is the most appropriate next step in management of this patient?
27
28
29 6 A Abdominal x-ray
30
31 6 B. Barium enema
32 e; C. Colonoscopy with biopsies
33
6 D. Emergent surgery
34
35 6 E. Opioid analgesics
36 6 F. Stool sample for ova and parasites
37
38 e; G. Sulfasalazine
39
40
2
Item: 5of40
0. ld : 2205
~'?Mark <]
Previous
C>
Next
a
Lab Values
~
Notes
~
Calculator
,
Reverse Color
~
Text Zoom
3

-
4

6
7
8
9
Explanation:

Toxic megacolon
10
11
12 • lBO
Risk factors
13 • Clostridium difflcile infection
14
15
16 • Systemic toxicity
17 (eg, fever, tachycardia, hypotension)
18
19 Diagnosis • Bloody diarrhea
20 • Abdominal distension/peritonitis
21
• Marked colonic distension on imaging
22
23
24 • Bowel rest, NG suction, antibiotics
25
26 • +/- Corticosteroids if IBO-associated
Management
27 • Surgery if unresponsive to
28 medical management
29
30 IBD = inflammatory bowel disease; NG = nasogastric.
31
32 © UWorld
33
34 This patient with weeks of lower abdominal pain, bloody diarrhea, and fecal urgency likely has
35 undiagnosed inflammatory bowel disease ( lBO) (eg, ulcerative colitis). His acute worsening
36 with fever, abdominal distension, leukocytosis, hypotension, and tachycardia suggests toxic
37 megacolon Other manifestations of toxic megacolon include altered mental status, peritonitis,
38 and electrolyte abnormalities. If due to severe colitis with massive colonic distension, toxic
39 megacolon may be the first presentation of IBD and is potentially lethal.
40
2
Item : 5of40 ~'?Mark <] C> !I ~ ~ , ~
0 . ld : 2205 Previous Next Lab Values Notes Calculator Reverse Color Tex t Zoom
3

-
'"' I I '"' I I'"' '"'I o1.. · ''' • • ·" o1 I '«~ J• Ill' o1 o1 I'"' t • I •t
4 quickly, confirm the diagnosis, and exclude perforation Conservative management (successful
in >50% of cases) can initially be instituted and includes bowel rest, nasogastric suction, and
6
7 either corticosteroids with broad spectrum-antibiotics (due to lBO) or antibiotics targeted at
8 Clostridium difficile.
9
10 (Choices B and C) Although a barium enema may be helpful in cases of suspected ulcerative
11 colitis, it is contraindicated in suspected toxic megacolon due to the risk of colonic perforation.
12 Complete endoscopic evaluation would be appropriate to confirm I BO if this patient had presented
13 earlier. However, in the setting of toxic megacolon, such a procedure is not recommended due to
14 the risk of perforation Limited evaluation (eg, proctoscopy or sigmoidoscopy without bowel
15 preparation) can be considered to help identify the etiology once the diagnosis has been
16 confirmed via x-ray
17
18 (Choice D) Severe cases of toxic megacolon not responding to medical therapy may require
19 emergent surgery (eg, subtotal colectomy and end-ileostomy)
20
21 (Choice E) Opioids are inadvisable in this patient as their antimotility effects can promote colonic
22 perforation. Other antimotility agents (eg, loperamide) and anticholinergic agents should also be
23
24 discontinued.
25
(Choice F) In general, parasitic causes of diarrhea (eg, Entamoeba histolytica) do not lead to
26
27 sepsis and, in the absence of a travel history, are much less likely than other causes.
28
(Choice G) Toxic megacolon should be confirmed before treatment is initiated; intravenous
29
30 corticosteroids are preferred Sulfasalazine for lBO should be started only once acute symptoms
31 have resolved.
32
33 Educational objective:
34 Toxic megacolon may be the initial presentation of inflammatory bowel disease. Key to the
35 diagnosis are radiologic evidence of colonic distension with manifestations of severe systemic
36 toxicity (eg, fever, leukocytos is, hemodynamic instability) About 50% of patients improve with
37 conservative management and corticosteroids.
38
39
40
2
Item: 6 of 40 ~'?Mark <J C> 61 ~ ~ , ~
0. ld : 4431 Previous Next Lab Values Notes Calculator Reverse Color Text Zoom
3
4

-
5

7
8
9
10
A 53-year-old woman comes to the urgent care clinic with right-sided abdominal pain that started
2 days ago. She describes the pain as constant, burning, and severe enough to interfere with
sleep There is no associated nausea, vomiting, or diarrhea. The patient tried treating the
symptoms with a few doses of over-the-counte r antacids and ibuprofen, which brought no relief.
Her medical history is significant fo r breast cance r diagnosed a yea r ago, for which she
11 underwent a modified radical mastectomy followed by chemotherapy; the last course was
12
completed 3 weeks ago On physical examination, her temperature is 36.7 C (98 F), blood
13
14 pressure is 120/70 mm Hg, pulse is 80/min, and respirations are 16/min. Her lung fields are
15 clear to auscultation and her abdomen is soft and nondistended with no rebound, guarding, or
16 rigidity. The liver span is 8 em and the spleen is not palpable Bowel sounds are normal. Lightly
17 brushing the skin to the right of the umbilicus elicits intense pain W hich of the following
18 additional features is this patient most likely to develop?
19
20
21 0 A Asc ites
22 0 B. Black stool
23
24 0 C. Cough and hemoptysis
25 6 D. Fever and jaundice
26
27 0 E. Hollow viscus perforation
28 0 F. Intestinal obstruction
29
30 ® G. Skin lesions
31
32
Submit
33
34
35
36
37
38
39
40
2
Item: 6 of 40 ~'?Mark <] C> !I ~ ~ , ~
0. ld : 4431 Previous Next Lab Values Notes Calculator Reverse Color Text Zoom
3
4

-
5

7
8
9
10
Explanation:

This patient with burning, localized pain and regional hyperesthesia/allodynia, in the context of
recent cancer treatment, has common features of herpes zoster (shingles) Pain from shingles
may precede the onset of the classic vesicular rash by several days, during which the diagnosis
11 may not be obvious. The possibility of shingles should be considered in patients with regional
12 pain who have no conclusive evidence of disease in the local internal organs.
13
14 Shingles may occur at any age, but it is most common after age 50 and the risk increases with
15 age It is frequently triggered by severe physical stress (such as cancer treatment, as in this
16 patient) or immunosuppressed states, but many cases are spontaneous Shingles develops when
17 there is reactivation of the varicella zoster virus in a dorsal root ganglion, where it has remained
18 dormant since a past chickenpox infection. This results in pain and a vesicular rash in a
19 dermatomal distribution along the course of the nerve. In some cases, patients may develop
20 persistent hypersensitivity of afferent pain fibers leading to chronic pain known as post-herpetic
21
neuralgia. Treatment with antiviral medications (acyclovir, valacyclovir, or famc iclovir) in the first
22
23 few days of a shingles outbreak can shorten the duration of symptoms and decrease the risk of
24 post-herpetic neuralgia.
25
26 (Choice A) Ascites is seen most commonly in patients with advanced liver disease (cirrhosis) or
27 chronic kidney disease. This patient has no liver abnormalities noted in the history or
28 examination findings, and development of ascites would be unexpected
29
30 (Choice B) Black stools (melena) are a typical symptom of upper gastrointestinal hemorrhage
31 (above the ligament of Treitz) Peptic ulcer would be the most likely explanation Although
32 ibuprofen increases the risk of ulcer , this patient had symptoms prior to taking the medication.
33
34 (Choice C) Patients with recent malignancy would be at increased risk for a number of
35 pulmonary conditions, such as pulmonary embolism or pneumonia, which could cause referred
36 pain to the abdomen. However, this patient's abdominal hyperesthesia is not consistent with a
37 pulmonary disorder.
38
39 I I I I
40

~ Feedback SuWend EnQock


2
Item : 6 of 40 ~'?Mark <] C> !I ~ ~ , ~
0 . ld : 4431 Previous Next Lab Values Notes Calculator Reverse Color Text Zoom
3 p
I g > I I p I I I
4

-
5

7
8
9
10
{Choice B) Black stools (melena) are a typ ical symptom of upper gastrointestinal hemorrhage
(above the ligament of Treitz) Peptic ulcer would be the most likely explanation Although
ibuprofen increases the risk of ulcer , this patient had symptoms prior to taking the medication.

{Choice C) Patients with recent malignancy would be at increased risk fo r a number of


11
pulmonary conditions, such as pulmonary embolism or pneumonia, which could cause referred
12 pain to the abdomen. However , this patient's abdominal hyperesthesia is not consistent with a
13 pulmonary disorder.
14
15 (Choice D) Fever and jaundice in association with right-sided abdominal pain would suggest the
16 possibility of acute cholangitis. Cholangitis is most often associated with biliary obstruction,
17 usually due to gallstones. In the absence of nausea or right upper-quadrant tenderness, biliary
18 obstruction is less likely
19
20 (Choice E) Patients with impending bowel perforation, as in acute appendicitis, often develop
21 initial symptoms in the periumbilical area. Abdominal examination should yield more specific
22 clues to th is possibility , and hyperesthesia would be unlikely
23
24 (Choice F ) Small-bowel obstruction is a common cause of abdominal pain, but it most often
25 occurs in patients with adhesions from prior abdominal surgery. It is usually associated with
26
nausea and abnormal bowel sounds.
27
28 Educational objective:
29
Herpes zoster (shingles) is due to reactivation of varicella zoste r virus from a dorsal root
30
31
ganglion Patients experience pain and a ves icular rash in a dermatomal distribution. The pain
32 may precede the rash by several days, during which the diagnosis may not be apparent
33
34 Refer ences:
35
36 1. Herpes zoster: diagnostic, therapeutic, and preventive approaches.
37
38
39 Time Spent 2 seconds Copyright © UWorld Last updated: [10/ 15/2016)
40
2
Item: 7of40 ~'?Mark <J C> 61 ~ ~ , ~
0. ld : 3467 Previous Next Lab Values Notes Calculator Reverse Color Text Zoom
3
4

...
5
6

8
9
A 65-year-old man comes to the office with 2 weeks of dysuria and turb id, foul-smelling urine.
He has also noticed air bubbles while urinating. He has no fever, hematuria, hematochezia,
weight loss, or pelvic or flank pain The patient has a long history of straining on urination and a
weak urine stream and is currently being treated for benign prostatic hyperplasia Four weeks
10 ago, he was seen in the emergency department for persistent lower quadrant abdominal pain and
11 diagnosed with acute diverticulitis. He was discharged on an oral antibiotic . Current temperature
12
is 98.7 F (37 1 C), blood pressure is 132/84 mm Hg, and pulse is 72/min. The abdomen is soft
13
14 and mildly tender in the left lower quadrant There is no costovertebral angle tenderness. Rectal
15 examination shows a smooth, enlarged, nontender prostate Laboratory studies show normal
16 complete blood count and serum chemistries, including serum creatinine. Urinalysis is positive
17 fo r numerous white cells and bacteria. Urine culture grows Escherichia coli, Proteus mirabilis,
18 and Klebsiella pneumoniae. W hich of the following is the most likely cause of this patient's
19 current condition?
20
21
22 <0 A Acute bacterial prostatitis
23 <0 B. Bladder cancer
24
25 <0 C. Colovesical fistula
26 <0 D. Emphysematous pyelonephritis
27
28 <0 E. Prostatic abscess
29 <0 F. Staghorn calculi
30
31
32 Submit
33
34
35
36
37
38
39
40
2
Item: 7of40
0. ld : 3467
~'?Mark <]
Previous
C>
Next
a
Lab Values
~
Notes
~
Calculator
,
Reverse Color
~
Text Zoom
3
4

...
5
6

8
9
Explanation:

Clinical features of colovesical fistula


10
11
12 • Diverticular disease (sigmoid most common)
13 Etiology • Crohn disease
14 • Malignancy (colon, bladder, pelvic organs)
15
16
• Pneumaturia (air in urine)
17 Clinical
18 • Fecaluria (stool in urine)
presentation
19 • Recurrent urinary tract infections (mixed flora)
20
21 • Abdominal CT with oral or rectal (not intravenous) contrast
22 Diagnosis
• Colonoscopy to exclude colonic malignancy
23
24 ©UWorld
25
26 This patient's presentation is consistent with colovesical fistula (connection between the colon
27 and bladder), which can be a complication of acute diverticulitis. The mechanism is usually due
28 to direct extension of a ruptured diverticulum or erosion of a diverticular abscess into the
29 bladder. Patients typically develop fecaluria (stool in the urine) or pneumaturia (air in the urine)
30
that usually occurs at the end of urination as the gas collects at the top of the bladder. Patients
31
32 can also develop recurrent urinary tract infections (sometimes due to mixed flora with coliform
33 organisms) or other nonspecific symptoms that can sometimes delay the diagnosis Colovesical
34 fistula can also occur in patients with Crohn disease or malignancy (usually of the colon)
35
36 Abdominal CT scan with oral or rectal (not intravenous) contrast can confirm the diagnosis by
37 showing contrast material in the bladder with thickened colonic and vesicular walls.
38 Colonoscopy is usually recommended in patients diagnosed with colovesical fistula to exclude
39 colonic malignancy Treatment is typically surgical after resolution of the infection.
40
2
Item : 7of40 ~'?Mark <] C> !I ~ ~ , ~
0. ld : 3467 Previous Next Lab Values Notes Calculator Reverse Color Text Zoom
3
4 showing contrast material in the bladder with thickened colonic and vesicular walls.

...
5
6

8
9
Colonoscopy is usually recommended in patients diagnosed with colovesical fistula to exclude
colonic malignancy Treatment is typically surgical after resolution of the infection.

(Choices A and E) Acute bacterial prostatitis typically presents with acute onset of systemic
findings (eg, fever, chills, malaise), irritative urinary symptoms (eg, dysuria, frequency),
10 pelvic/perineal pain, and cloudy urine. Prostatic abscess, a focal purulent collection, is a
11 compl ication of acute bacterial prostatitis. However, both conditions usually cause an exquisitely
12 tender prostate on rectal examination and are not usually associated with pneumaturia.
13
14 (Choice B) Bladder cance r classically presents with irritative urinary symptoms and painless
15 hematuria. This patient's pneumaturia and absence of hematuria make this less likely
16
17 (Choice D) Emphysematous pyelonephritis (pyelonephritis due to a gas-producing infection,
18 typically in patients with diabetes) can present with abrupt or gradual onset of fever, chills, flank
19
or abdominal pain, and nausea/vomiting This patient's absence of fever and flank pain makes
20
this less likely
21
22
(Choice F) Staghorn calculi, which are typ ically due to struvite stones filling the entire intrarenal
23
24 collecting system, can lead to recurrent urinary tract infections. Patients can develop irritative
25 urinary symptoms, flank pain, or hematuria. However, staghorn calculi are not usually associated
26 with pneumaturia
27
28 Educational objective:
29 Colovesical fistula is most commonly due to diverticular disease and presents with pneumaturia,
30 fecaluria, or findings consistent with urinary tract infection. Abdominal CT scan with oral or rectal
31 (not intravenous) contrast can confirm the diagnosis by showing contrast material in the bladder
32 with thickened colonic and vesicular walls.
33
34
35
References:
36 1. A case of colovesical fistula induced by sigmoid diverticulitis .
37
38
39 ..... ..... ...... • ••• (!) 'jll ol • • <1 .... • • • •

40

~ Feedback SuWend EnQ ock


2
Item: 8of40 ~'?Mark <J C> 61 ~ ~ , ~
0. ld : 4434 Previous Next Lab Values Notes Calculator Reverse Color Text Zoom
3
4
5
6 A 45-year-old man comes to the emergency department with persistent abdominal pain The pain

-
7

9
10
11
12
began more than 12 hours ago and is predominantly epigastric . He has vom ited several times
since the pain started. The pain is worse with walking and lying supine and is improved with
sitting up or leaning forward. The patient's other medical problems include hyperlipidemia treated
with simvastatin and hypertension treated with lisinopril He smokes a pack of cigarettes daily
He drinks 4-6 cans of beer a day and several more on weekends. He does not use illicit drugs.
The patient's temperature is 37.8° C (100° F) , blood pressure is 100/70 mm Hg, pulse is 110/min,
13
14 and respirations are 20/min. Abdominal examination shows mild epigastric tenderness without
15 guarding or rebound. Chest x-ray shows a left-sided pleural effusion. W hich of the following is
16 most likely in this patient?
17
18
0 A Acute cholecystitis
19
20 0 B. Acute pancreatitis
21
0 C. Intra-abdominal abscess
22
23 6 D. Mesenteric ischemia
24 0 E. Myocardial infarction
25
26 0 F. Peptic ulcer perforation
27
® G. Pneumonia
28
29 ® H. Pyelonephritis
30
31
32 Submit
33
34
35
36
37
38
39
40
2
Item: 8of40
0. ld : 4434
~'?Mark <]
Previous
C>
Next
a
Lab Values
~
Notes
~
Calculator
,
Reverse Color
~
Text Zoom
3
4 Clinical features of acute pancreatitis
5
6

-
7

9
10
11
12 Etiology
• Chronic alcohol use
• Gallstones
• Hyperlipidemia (types I, IV & V)
• Drugs (eg, didanosine, azathioprine, valproic acid)
13 • Infections (eg, cytomegalovirus, Legionella, Aspergillus)
14
15 • Trauma
16 • Iatrogenic (post-ERCP)
17
18
19 Diagnosis requires 2 of the following:
20 • Acute epigastric abdominal pain often radiating to the
21 back
22
23 • 1 Amylase/lipase >3 times normal limit
24 • Abdominal imaging showing focal or diffuse pancreatic
25 enlargement with heterogeneous enhancement with
26 intravenous contrast (CT) or diffusely enlarged
27 Clinical
presentation & hypoechoic pancreas (ultrasound)
28
29
30 Other findings:
31
32
• Nausea, vomiting, leukocytosis
33 • Severe disease with possible abdominal tenderness,
34 fever, tachypnea, hypoxemia & hypotension
35
36
• ALT level >150 units/L- biliary pancreatitis
37
38 • Pleural effusion
39
40
2
Item : 8of40
0 . ld : 4434
~'?Mark <]
Previous
C>
Next
a
Lab Values
~
Notes
~
Calculator
,
Reverse Color
~
Text Zoom
3
4
• • g
5 • Nausea, vomiting, leukocytosis
6

-
• Severe disease with possible abdominal tenderness,
7 fever, tachypnea, hypoxemia & hypotension
9 • ALT level >1 50 units/L- biliary pancreatitis
10
11
12 • Pleural effusion
13 • Ileus
14 Complications
15 • Pancreatic pseudocyst/abscess/necrosis
16 • Acute respiratory distress synd rome
17
18 ©UWorld
19
20 This patient's presentation is most consistent with a diagnosis of acute pancreatitis Patients with
21 acute pancreatitis typically have sudden onset of persistent, severe epigastric abdominal pain,
22 nausea, and vom iting The pain frequently radiates to the back, and patients may get partial relief
23 by sitting up and leaning forward. Physical examination findings vary with the severity of acute
24 pancreatitis They range from minimal tenderness to palpation in the epigastric area in mild
25 cases to severe tenderness in the epigastrium or diffusely over the abdomen with guarding and
26
rebound tenderness.
27
28
Gallstones and chronic alcohol abuse account for about 75% of all cases of acute pancreatitis.
29
Nearly one-third of patients have chest x-ray abnormalities, including pleural effusions,
30
31 atelectasis, elevated hemidiaphragm, or pulmonary infiltrates. These complications are often due
32 to activated pancreatic enzymes (eg, phospholipase, trypsin) and cytokines (eg, tumor necrosis
33 factor) that are released from the inflamed pancreas into the circulation and cause inflammation
34 in other parts of the body. Other complications include ileus, acute respiratory distress
35 syndrome, and renal failure.
36
37 {Choice A) Acute cholecystitis typically presents as right upper-quadrant pain that classically
38 radiates to the right shoulder. It is often accompanied by nausea, vomiting, feve r, and
39 leukocytosis Examination usually shows increased discomfort during palpation around the area
40
2
Item : 8of40 ~'?Mark <] C> !I ~ ~ , ~
0 . ld : 4434 Previous Next Lab Values Notes Calculator Reverse Color Text Zoom
3
4 {Choice A) Acute cholecystitis typically presents as right upper-quadrant pain that classically
5 radiates to the right shoulder. It is often accompanied by nausea, vomiting, feve r, and
6 leukocytosis Examination usually shows increased discomfort during palpation around the area

-
7

9
10
11
12
of the gallbladder fossa during deep inspiration (Murphy's sign) However , this patient's pain
radiation to the back, left-sided pleural effusion, and alcoholism make pancreatitis a much more
likely diagnosis

(Choice C) Intra-abdominal abscess often presents in a subacute or insidious fashion with fever ,
nausea, vom iting, and/or abdominal pain This patient's acute symptom onset is less consistent
13
with an abdominal abscess.
14
15 (Choice D) Mesenteric ischemia usually presents with severe acute periumbilical abdominal pain
16
that is out of proportion to findings on physical examination. Risk factors include advanced age,
17
18 diffuse atherosclerosis, valvular abnormality, ca rdiac arrhythmias, or recent myocardial infarction
19 (MI) The history of alcohol abuse and chest x-ray findings of left pleural effusion make
20 pancreatitis the more likely diagnosis in this patient
21
22 {Choice E) An atypical Ml (especially inferior Ml) can present with nausea, vomiting, and
23 epigastric pain However, pain improving while sitting up or leaning forward is not consistent with
24 MI.
25
26 {Choice F) Patients with perforated peptic ulce r usually present with the sudden onset of
27 epigastric pain, nausea, vomiting, hematemesis, and peritoneal signs (eg, guarding, rigidity,
28 rebound tenderness) on physical examination. Chest x-ray shows free air under the diaphragm
29
30 (Choice G) Pneumonia can cause low-grade feve r and unilateral pleural effusion, usually
31 secondary to empyema or a parapneumonic effusion. However , this patient's x-ray did not show
32 any pulmonary infiltrates. In addition, the history of recurrent vomiting and persistent epigastric
33
pain/tenderness makes pneumonia less likely
34
35 (Choice H) Acute pyelonephritis classically presents with nausea, vomiting, fever, flank pain,
36
and costovertebral angle tenderness.
37
38
Educational objective:
39
! ... 1 .,1 ... ., ., ., ., .. 1 ., .. ., Ill II ., Jt '"' I 1 ., t ., I ! Ill ., t ... ., 11
40

~ Feedback SuWend EnQock


2
Item : 8of40 ~'?Mark <] C> !I ~ ~ , ~
0 . ld : 4434 Previous Next Lab Values Notes Calculator Reverse Color Text Zoom
3
p p I g py • I I I g ,
4
diffuse atherosclerosis, valvular abnormality, cardiac arrhythmias, or recent myocardial infarction
5
6 (MI) . The history of alcohol abuse and chest x-ray findings of left pleural effusion make

-
7 pancreatitis the more likely diagnosis in this patient

9 (Choice E) An atypical Ml (especially inferior Ml) can present with nausea, vomiting , and
10 epigastric pain However , pain improving while sitting up or leaning fo rward is not consistent with
11 MI.
12
13 {Choice F) Patients with perforated peptic ulce r usually present with the sudden onset of
14 epigastric pain, nausea, vomiting, hematemesis, and peritoneal signs (eg, guarding, rigidity,
15 rebound tenderness) on physical examination. Chest x-ray shows free air under the diaphragm
16
17 {Choice G) Pneumonia can cause low-g rade fever and unilateral pleural effusion, usually
18 secondary to empyema or a parapneumonic effusion. However , this patient's x-ray did not show
19 any pulmonary infiltrates. In addition, the history of recurrent vom iting and persistent epigastric
20 pain/tenderness makes pneumonia less likely
21
22 (Choice H) Acute pyelonephritis classically presents with nausea, vomiting, fever , flank pain,
23
and costovertebral angle tenderness.
24
25 Educational objective:
26
Acute pancreatitis classically causes nausea, vom iting, and epigastric pain Alcohol abuse and
27
28
gallstone disease are the most common causes of acute pancreatitis. Complications include
29 pleural effusion, acute respiratory distress syndrome, ileus, and renal failu re.
30
31 References:
32
33 1. Pleural effusions in acute pancreatitis.
34 2. New diagnostic criteria of acute pancreatitis.
35
36 3. Diagnosis and management of acute pancreatitis.
37
38
39 Time Spent 2 seconds Copyright © UWo rld Last updated: [08/ 06/ 2016)
40
2
Item : 9 of40 ~'?Mark <J C> 61 ~ ~ , ~
0. ld : 2907 Previous Next Lab Values Notes Calculator Reverse Color Text Zoom
3
4
5
6 A 54-year-old man comes to the office due to 6 months of fatigue, anorexia, and a 7-kg (154-lb)
7 unintentional weight loss. The patient has noticed that despite his weight loss, his pants are

-
8

10
11
12
13
getting tighter and his socks leave deep imprints on his legs. He has a history of hypertension fo r
which he takes amlodipine. He is unaware of any family history as he was adopted The patient
has smoked 5-6 c igarettes daily for the past 30 years and drinks 1 or 2 cans of beer on
weekends. He has a remote history of injection drug use. Blood pressure is 134/82 mm Hg,
pulse is 84/min, and BMI is 23 kg/m2 Jugular venous pressure is normal. The patient has
14 bilateral gynecomastia and several spider angiomas on his upper trunk. Cardiac examination
15 reveals normal heart sounds without any murmurs. Breath sounds are decreased at the bases.
16 The abdomen is soft, nontender, and distended with shifting dullness. There is bilateral lower
17 extremity pitting edema. Which of the following is the most likely underlying cause of this
18 patient's condition?
19
20
21 0 A Adverse medication effect
22 0 B. Chronic viral infection
23
24 0 C. Cor pulmonale
25 0 D. Hypothyroidism
26
27 0 E. Lung cancer
28 0 F. Nephrotic syndrome
29
30
31 Submit
32
33
34
35
36
37
38
39
40
2
Item : 9 of40 ~'?Mark <] C> !I ~ ~ , ~
0. ld : 2907 Previous Next Lab Values Notes Calculator Reverse Color Text Zoom
3
4 This patient with pedal edema, ascites (shifting dullness), bilateral gynecomastia, and spider
5 angiomata likely has cirrhosis and portal hype rtension Other cl inical features that suggest
6 cirrhosis include palmar erythema, caput medusa, and splenomegaly The most common causes
7

-
of cir rhosis in the United States include viral hepatitis (C more than 8 ), chronic alcoholism,
8
nonalcoholic fatty live r disease (NAFLD ), and hemochromatosis.
10
11
The initial evaluation of patients with suspected cirrhosis consists of obtaining a medical history,
12 including inquiring about medications, social habits (eg, alcohol use, drug use, high-risk sexual
13 activity), and family history (eg, to exclude hemochromatosis) Given this patient's history of
14 intravenous dr ug use, the most likely cause of his cirrhosis is an infection with viral hepatitis
15 (which, along with alcohol use, accounts fo r - 50% of cases of cirrhosis). He has no significant
16 alcohol history He is not overweight and is therefore unlikely to have NAFLD. His family history
17 is unknown, but he has no stigmata of hemochromatosis (eg, bronze diabetes, ca rdiomyopathy ,
18 arthropathy). Laboratory studies, including viral hepatitis serologies and iron studies, should be
19
obtained. If these are unrevealing, the patient may require further investigation fo r the less
20
21 common causes of cirrhosis.
22
(Choice A) Amlodipine can cause pedal edema, but not cirrhosis. Medications (eg,
23
24 methotrexate, isoniazid) can cause cirrhosis, but this is less common.
25
(Choice C) Cor pulmonale can cause fatigue and leg edema but not gynecomastia or spider
26
27 angiomata; it would be unlikely in the absence of other manifestations (eg, exertional dyspnea,
28 syncope, angina, loud second heart sound, prominent a wave)
29
30 (Choice D) Hypothyroidism can cause fatigue, pleural and pericardia! effusions, ascites, and
31 pedal edema; however, these findings are rare in the absence of severe hypothyro idism (eg, cold
32 intolerance, constipation, weight gain, bradycardia, goiter) Hypothyro idism would be unlikely to
33 cause gynecomastia or spider angiomata
34
35 (Choice E) Lung cancer could lead to fatigue, weight loss, anorexia, edema of the legs, and
36 ascites in patients with advanced disease (eg, metastatic compression of the lymphatics,
37 peritoneal ca rcinomatosis), but it would not likely cause gynecomastia or spider angiomata.
38
39 {Choice F) Nephrotic syndrome can cause anasarca but is not usually associated with spider
40
2
Item : 9 of40 ~'?Mark <] C> !I ~ ~ , ~
0. ld : 2907 Previous Next Lab Values Notes Calculator Reverse Color Text Zoom
3 .. .. . . . . . . .. . . .. . .. ... .
4 common causes of cirrhosis.
5
6 (Choice A) Amlodipine can cause pedal edema, but not cirrhosis. Medications (eg,
7 methotrexate, isoniazid) can cause cirrhosis, but this is less common.

-
8

10
11
12
13
(Choice C) Cor pulmonale can cause fatigue and leg edema but not gynecomastia or spider
angiomata; it would be unlikely in the absence of other manifestations (eg, exertional dyspnea,
syncope, angina, loud second heart sound, prominent a wave)

(Choice D) Hypothyro idism can cause fatigue, pleural and pericardia! effusions, ascites, and
14
pedal edema; however, these findings are rare in the absence of severe hypothy roidism (eg, cold
15
intolerance, constipation, weight gain, bradycardia, goiter) Hypothy roidism would be unlikely to
16
17 cause gynecomastia or spider angiomata
18
19
(Choice E) Lung cance r could lead to fatigue, weight loss, anorexia, edema of the legs, and
20 ascites in patients with advanced disease (eg, metastatic compression of the lymphatics,
21 peritoneal ca rcinomatosis), but it would not likely cause gynecomastia or spider angiomata.
22
23 (Choice F) Nephrotic syndrome can cause anasarca but is not usually associated with spider
24 angiomata or gynecomastia.
25
26 Educational objective:
27 Cirrhosis should be suspected in any patient with evidence of chronic liver disease (eg, ascites,
28 spider angiomata, gynecomastia, splenomegaly). Viral hepatitis, chronic alcohol abuse,
29 nonalcoholic fatty liver disease, and hemochromatosis are the most common causes of c irrhosis
30 in the United States.
31
32
References:
33
34 1. Cirrhosis and chronic liver failure: part I. Diagnosis and evaluation.
35
36 2. Treatment of patients with cirrhosis.
37
38
39 Time Spent 3 seconds Copyright © UWorld Last updated: [10/ 10/2016)
40
2
Item : 10of40 ~'?Mark <] C> !J ~ ~ , ~
0. ld : 4623 Previous Next Lab Values Notes Calculator Reverse Color Text Zoom
3
4
5
6 A 52-year-old man comes to the physician due to black stools. He also has occasional
7 abdominal discomfort and nausea but has no hematemesis. Food seems to help his abdominal
8 pain, so he eats frequently during the day and occasionally snacks at night. As a consequence,

--
9

11
12
13
14
the patient has gained 5 pounds over the last year. He says his diet is lacking in vegetables and
fruit. The patient drinks 1 to 2 cans of beer nightly but does not use tobacco or illicit drugs. His
father died of colon cancer, and his mother died from stroke. Physical examination shows mild
epigastric abdominal tenderness and a right-sided carotid bruit. Fecal occult blood testing is
positive W hich of the following is the most likely diagnosis?
15
16 0 A Colon cancer
17
18 0 B. Diverticulosis
19 0 C. Gastric carcinoma
20
21 6 D. Inflammatory bowel disease
22 0 E. Ischemic colitis
23
24 0 F. Mallory-Weiss tear
25 ® G. Mesenteric ischemia
26
27 e; H. Peptic ulcer disease
28
29
30 Submit
31
32
33
34
35
36
37
38
39
40
2
Item : 10 of40
0. ld : 462 3
~'?Mark <]
Previous
C>
Next
a
Lab Values
~
Notes
~
Calculator
,
Reverse Color
~
Text Zoom
3
4
5
6 Peptic ulcers
7
8

--
9

11
12
13
14
15
16
17
18
19
20
21
22 Duodenal ulcer
23
24
25
26
27 Stomach ulcer
28
29
30
31
32
33 @ UWo~d
34
35 This patient's epigastric abdominal discomfort and melena are likely due to peptic ulcer disease
36
(PUD). PUD refers to ulcerations in the stomach or duodenum that are most commonly caused
37
by Helicobacter pylori infection or nonsteroidal anti-inflammatory drugs. Patients may have
38
epigastric pain, nausea, and/or early satiety in association with food. The classic symptoms of
39
40 . ... . ... . . .. . . .. .. .. .. .. ... .. ... . .. . ... . . .. . . .. .. ... . . .. . ..
~ Feedback SuWend EnQ ock
2
3
4
5
6
7

....
8
9

11
12
This patient's epigastric abdominal discomfort and melena are likely due to peptic ulcer disease
(PUD). PUO refers to ulcerations in the stomach or duodenum that are most commonly caused
by Helicobacter pylori infection or nonsteroidal anti-inflammatory drugs. Patients may have
epigastric pain, nausea, and/or early satiety in association with food. The classic symptoms of
13 duodenal ulcer occur in the absence of a food buffer and can include epigastric pain 2-5 hours
14 after meals, on an empty stomach, or at night. Patients can have melena due to processed blood
15 from upper gastrointestinal (GI) bleeding (eg, proximal to the ligament of Treitz). In fact, PUO is
16
one of the most common causes of upper Gl bleeding. Diagnosis of PUD is made with upper Gl
17
18 endoscopy.
19
(Choices A and B) Colon cancer and diverticulosis are possible etiologies of lower Gl bleeding
20
that typically present with hematochezia as opposed to melena. Colon cancer may also result in
21
22 changes in bowel habits (eg, constipation, changes in stool caliber).
23
24 (Choice C) Gastric cancer can cause melena and abdominal pain but is usually accompanied
25 by weight loss and anorexia instead of weight gain
26
27 (Choice D) Ulcerative colitis typ ically causes hematochezia as opposed to melena. Melena can
28 occur with Crohn disease involving the upper Gl tract or small boweL Nonetheless, in flammatory
29 bowel disease is typically accompanied by diarrhea and systemic symptoms (eg, fatigue, fever,
30 weight loss), which are not evident in this patient.
31
32 (Choices E and G) Atherosclerotic vascular disease (eg, ca rotid stenosis) is a risk factor fo r
33 chronic ischemic colitis and mesenteric ischemia. However, ischemic colitis is typically
34 associated with hematochezia as opposed to melena. In addition, patients with mesenteric
35 ischemia report pain that is exacerbated by eating, which often leads to food aversion and weight
36
loss as opposed to weight gain.
37
38 (Choi ce F) Mallory-Weiss syndrome refers to hematemesis that occurs after repeated episodes
39
t .. I I I l ! I II I I .; I I t i .. I ., I II t "' I ~· t ... I I .. I I t .. • t ., I II .. ., I '" I : I
40
~ -------- - ----------
Feedback
-
SuW.nd EnQock
-
2
Item : 10of40 ~'?Mark <] C> !I ~ ~ , ~
0. ld : 4623 Previous Next Lab Values Notes Calculator Reverse Color Text Zoom
3 - -
4 that typically present with hematochezia as opposed to melena. Colon cancer may also result in
5 changes in bowel habits (eg, constipation, changes in stool caliber)
6
7 (Choice C) Gastric cancer can cause melena and abdominal pain but is usually accompanied
8 by weight loss and anorexia instead of weight gain

--
9

11
12
13
14
{Choice D) Ulcerative colitis typically causes hematochezia as opposed to melena. Melena can
occur with Grahn disease involving the upper Gl tract or small bowel. Nonetheless, inflammatory
bowel disease is typically accompanied by diarrhea and systemic symptoms (eg, fatigue, feve r,
weight loss), which are not evident in this patient
15 (Choices E and G) Atherosclerotic vascular disease (eg, carotid stenosis) is a risk factor for
16 chronic ischemic colitis and mesenteric ischemia. However, ischemic colitis is typically
17
associated with hematochezia as opposed to melena. In addition, patients with mesenteric
18
19
ischemia report pain that is exacerbated by eating, which often leads to food aversion and weight
20 loss as opposed to weight gain.
21
22 (Choice F) Mallory-Weiss syndrome refers to hematemesis that occurs after repeated episodes
23 of retching/vomiting and is often accompanied by epigastric or back pain. The absence of
24 vomiting and hematemesis in this patient make the diagnosis unlikely
25
26 Educational objective:
27 Peptic ulcer disease (PUD) may present with epigastric pain, nausea, and/or early satiety in
28 association with food . Symptoms of duodenal ulcer classically occur in the absence of a food
29 buffer (eg, 2-5 hours after meals, on an empty stomach, or at night) Melena is a manifestation
30 of upper gastrointestinal bleeding, with PUD being one of the most common causes.
31
32
References:
33
34 1. Diagnosis and management of upper gastrointestinal bleeding.
35
36 2. Peptic ulcer disease.
37
38
39 Time Spent 3 seconds Copyright © UWorld Last updated: [09/ 12/2016)
40
2
Item : 11 of40 ~'?Mark <] C> !I ~ ~ , ~
0. ld : 2978 Previous Next Lab Values Notes Calculator Reverse Color Text Zoom
3
4
5
6 A 45-year-old woman comes to the emergency department with one day of fever , chills, nausea,
7 and vomiting She has had progressively worsening right upper quadrant abdominal pain for the
8 past 2 days. The patient has a history of depression as a teenager and attempted suicide. She
9 currently takes no medications and does not drink alcohol or use recreational drugs. Her

--
10

12
13
14
15
temperature is 39.5 C (103.1 F), blood pressure is 90/48 mm Hg, pulse is 98/min, and
respirations are 30/min. Her BMI is 36 kg/m2 . The patient appears very ill and confused. Skin
and sclera are icteric . Cardiopulmonary examination is normaL Marked tenderness and
guarding are present at the right upper quadrant of the abdomen. She has no flank dullness.
Laboratory results are as follows
16
Liver studies
17
18 Total bilirubin 7.2 mg/dL
19
20 Direct bilirubin 5.4 mg/dL
21
22 Alkaline phosphatase 71 4 U/L
23
24 Aspartate aminotransferase
75 U/L
25 (SGOT)
26 Alanine aminotransferase
27 60 U/L
(SGPT)
28
29 81 U/L (1 0-140
30 Lipase
U/L)
31
32
33
34 Complete blood count
35
36 Hemoglobin 14.1 g/dL
37
Platelets 306,000/mm>
38
39 Leukocytes 19,200/mm•
40
2
Item : 11 of40 ~'?Mark <J C> 61 ~ ~ , ~
0. ld : 2978 Previous Next Lab Values Notes Calculator Reverse Color Text Zoom
3 • , I
p p
4
5 Aspa rtate aminotransferase
6 75 U/L
(SGOT)
7
8 Alanine aminotransferase
9 60 U/L
(SGPT)

--
10

12
13
14
15
Lipase
81 U/L (10-1 40
U/L)

16 Complete blood count


17
Hemoglobin 14.1 g/dL
18
19 Platelets 306,000/mm'
20
21 Leukocytes 19,200/mm'
22
23 Neutrophils 81%
24
Lymphocytes 16%
25
26
27 Further workup in this patient would most likely show which of the following?
28
29
30 <0 A Common bile duct dilation
31 <0 B. Elevated ascitic fluid neutrophil count
32
33 <0 C. Inflamed and edematous enlargement of the pancreas
34 <0 D. Markedly elevated serum acetaminophen levels
35
36
<0 E. Short, annular strictures alternating with normal bile duct
37
38 Submit
39
40
2
Item : 11 of40
0. ld : 2978
~'?Mark <]
Previous
C>
Next
a
Lab Values
~
Notes
~
Calculator
,
Reverse Color
~
Text Zoom
3
4
5 Explanation:
6
7
8
9 Acute cholangitis

--
10

12
13
14
15
Clinical
presentation



Fever, jaundice, right upper quadrant pain (Charcot triad)
Mental status changes, hypotension (Reynolds pentad)
Liver failure
• Acute kidney injury
16
17
18 • Biliary dilation on ultrasound or CT scan
19
20 Diagnosis • t Alkaline phosphatase, gamma-glutamyl transpeptidase,
21 direct bilirubin
22 • Leukocytosis, f C-reactive protein
23
24
25 • Biliary drainage: Endoscopic retrograde
26 cholangiopancreatography with sphincterotomy or
27 Treatment percutaneous transhepatic cholangiography
28
29 • Broad-spectrum antibiotics: Beta-lactam/beta-lactamase
30 inhibitor, third-generation cephalosporin + metronidazole
31
@UWo~d
32
33
34
This patient with fever, jaundice, and right upper quadrant abdominal pain (Charcot triad) most
35 likely has acute cholangitis (AC) Confusion and hypotension (Reynolds pentad) are also
36 sometimes seen in severe AC. If not treated promptly, AC can lead to septic shock. Laboratory
37 results usually show leukocytosis and neutrophilia in addition to elevations in alkaline
38 phosphatase, gamma-glutamyl transpeptidase, and direct bilirubin.
39
40 .. - - - - - -- - - - . -- - - ·- -- - .. ·- - - . - - -- - - - - - - - - - --

~ Feedback SuWend EnQ ock


2
Item : 11 of40 ~'?Mark <] C> !I ~ ~ , ~
0. ld : 2978 Previous Next Lab Values Notes Calculator Reverse Color Text Zoom
3
results usually show leukocytosis and neutrophilia in addition to elevations in alkaline
4
5 phosphatase, gamma-glutamyl transpeptidase, and direct bilirubin.
6
7 Biliary stasis predisposes to AC , and the most common causes are due to bile duct obstruction
8 from gallstones, malignancy, or stenosis. In the setting of stasis, the bile-blood barrier can be
9 disrupted, allowing bacteria and toxins from the hepatobiliary system to translocate into the blood

--
10

12
13
14
15
stream. Ultrasound or CT scan can be helpful in confi rming the diagnosis and most frequently
shows common bile duct dilation. Supportive ca re, broad-spectrum antibiotics, and biliary
drainage, preferably by endoscopic retrograde cholangiopancreatography with sphincte rotomy,
are the mainstays of treatment Other options for biliary decompression include percutaneous
transhepatic cholangiography and open surgical decompression
16 (Choice B) Spontaneous bacterial peritonitis commonly presents with abdominal distension and
17 pain; feve r can be present Altered mental status can be seen if hepatic encephalopathy is
18
precipitated. The most common predisposing factors are cirrhosis and ascites. This patient has
19
20 no history of cirrhosis and lacks evidence of ascites on examination.
21
22 (Choice C) Acute panc reatitis can also present with fever and abdominal pain; however, marked
23 hyperbilirubinemia is less common, and the absence of elevated lipase makes this much less
24 likely
25
26 (Choice D) Although prior suic ide attempt is the greatest risk factor for future attempts, this
27 patient's history does not include recent acetaminophen ingestion In addition, feve r is less
28 common initially, and transaminases are frequently >3000 U/L in acetaminophen overdose.
29
30 (Choice E) Primary sclerosing cholangitis is a chronic disease. Although it has a cholestatic
31 pattern on liver tests, patients are frequently asymptomatic or have chronic fatigue and pruritus
32 on presentation. Acute hypotensive illness is not typicaL
33
34 Educational objective:
35 Fever, jaundice, and right upper quadrant abdominal pain (Charcot triad) are consistent with
36 acute cholangitis. Confusion and hypotension are also sometimes seen. Ultrasound or CT scan
37 most commonly shows common bile duct dilation.
38
39
40
2
Item: 12 of40 ~'?Mark <] C> !I ~ ~ , ~
0. ld : 2204 Previous Next Lab Values Notes Calculator Reverse Color Text Zoom
3
4
5
6 A 69-year-old man has had 3 days of pain in the lower left abdomen. The pain was intermittent
7 but has been constant over the past day. He has had nausea and fever for the past 2 days and is
8 urinating more frequently than usuaL The patient has a history of chronic constipation and right
9 inguinal hernia repair 20 years ago. He currently takes no medications. His temperature is 38.6
10 C (101 3 F), blood pressure is 122/80 mm Hg, pulse is 98/min, and respirations are 16/min. His

-
11

13
14
15
16
abdomen is soft but tender to palpation in the lower left quadrant Deep palpation is difficult and
reveals no masses or organomegaly. Bowel sounds are present Rectal examination reveals
normal prostate without nodules or enlargement Costovertebral angle tenderness is absent
Laboratory results are as follows
Leukocytes 14,500/mm'
17
18 Hemoglobin 13.1 g/dL
19
20 Blood urea nitrogen 18 mg/dL
21
22 Creatinine 1.0 mg/dL
23
24
25 Urinalysis
26
27 Red blood cells 1/hpf
28
29 Leukocytes 2-3/hpf
30
31 Leukocyte esterase negative
32 Nitrites negative
33
34
35 W hich of the following is the most appropriate next test to diagnose this patient's condition?
36
37
38 6 A Abdominal CT scan with contrast
39 6 B. Barium contrast enema
40
2
Item : 12 of40 ~'?Mark <J C> 61 ~ ~ , ~
0. ld : 2204 Previous Next Lab Values Notes Calculator Reverse Color Text Zoom
3 - , - ., I "' - I I t .. I - e .. ' •I " - II I "' -I - - -- "' - "' "' I - , - .,
4 normal prostate without nodules or enlargement Costovertebral angle tenderness is absent
5 Laboratory results are as follows
6
7 Leukocytes 14,500/mm>
8
9 Hemoglobin 13.1 g/dL
10

-
11 Blood urea nitrogen 18 mg/dL

Creatinine 1 0 mg/dL
13
14
15
16 Urinalysis
17
18 Red blood cells 1/hpf
19
Leukocytes 2-3/hpf
20
21 Leukocyte esterase negative
22
23 Nitrites negative
24
25
26 Which of the following is the most appropriate next test to diagnose this patient's condition?
27
28 ® A Abdominal CT scan with contrast
29
30 ® B. Barium contrast enema
31 ® C. Sigmoidoscopy
32
33 ® D. Small bowel follow-through
34 ® E. Upright abdominal film
35
36
® F. Urine culture
37
38 Submit
39
40
2
Item : 12 of40
0. ld : 2204
~'?Mark <]
Previous
C>
Next
a
Lab Values
~
Notes
~
Calculator
,
Reverse Color
~
Text Zoom
3
4
5 Explanation:
6
7
8
9 Clinical features of acute diverticulitis
10

-
11

13
14
15
16
Clinical
presentation
• Abdominal pain (usually lower left quadra nt)
• Fever, nausea & vomiting
• Ileus (perito neal irritation)

17 Diagnosis • Abdominal CT (oral & intravenous contrast)


18
19
20 • Bowel rest
Management
21 • Antibiotics (eg, ciprofloxacin, metronidazole)
22
23
24 Complications • Abscess, obstruction, fistula , perforation
25
© UWorld
26
27
This patient's cl inical presentation is conce rning fo r acute diverticulitis (inflammation due to
28
29 microperforation of a diverticulum). Chronic constipation and a low-fiber, high-fat diet are risk
30 factors for dive rticular disease. Patients with diverticulitis present with abdominal pain (usually
31 lower left quadrant), feve r, nausea/vomiting, and leukocytosis Some (10%-15%) have urinary
32 urgency, frequency, or dysuria due to bladder irritation from an inflamed sigmoid colon.
33 Abdominal CT scan (oral and intravenous contrast) is the best diagnostic test fo r dive rticulitis,
34 diffe rentiating it from other diseases (eg, colon cancer, kidney infection) Findings suggestive of
35 diverticulitis include inc reased inflammation in pericolic fat, presence of diverticula, bowel wall
36 thickening, soft tissue masses (eg, phlegmons), and pericolic fluid collections suggesting abscess.
37
38 (Choice B) Barium contrast enema can detect strictures or inflammation in ulcerative colitis or
39 C h d. It . t . d. t d . th tt' I f d. rt· IT fl I rf t' h b I d
40

~ Feedback SuWend EnQock


2
Item : 12 of40 ~'?Mark <] C> !I ~ ~ , ~
3
4
0. ld : 2204
. . . "' .. . . "' . . . . . . . ... . ., .
Previous Next Lab Values Notes Calculator Reverse Color Text Zoom

5 (Choice B) Barium contrast enema can detect strictures or inflammation in ulcerative colitis or
6 Crohn disease. It is contraindicated in the setting of diverticulitis until perforation has been ruled
7 out
8
9 (Choice C) Sigmoidoscopy or colonoscopy is contraindicated in the setting of acute dive rticulitis
10

-
as it may cause perforation; colonoscopy can be done to exclude underlying malignancy
11 following resolution of the diverticulitis.
13 {Choice D) Small bowel follow-through is performed by giving oral contrast to the patient and
14
using fluoroscopy to follow the contrast into the small bowel. The test is usually indicated fo r
15
diagnosing small bowel pathology (eg, stricture, obstruction, masses) and is not as useful for
16
17 diagnosing diverticulitis in the colon.
18
19 (Choice E) Upright abdominal film can reveal nonspecific findings sometimes associated with
20 acute dive rticulitis (eg, dilated small/large bowel with air-fl uid levels due to ileus or obstruction, soft
21 tissue densities due to abscess) It can also occasionally reveal air under the diaphragm or in
22 the retroperitoneum (perforation). However, abdominal CT is more sensitive (~94%) as well as
23 specific ( -99%) for diagnosing diverticulitis and diffe rentiating it from other causes of abdominal
24 pain.
25
26 (Choice F) The patient has a normal rectal examination without costovertebral angle tenderness
27 or marked pyuria (increased leukocytes in the urine), making genitourinary causes of abdominal
28 pain less likely His urinary symptoms are likely due to bladder irritation from diverticulitis.
29
30 Educational objective:
31 Abdominal CT scan is the best diagnostic test for diagnosing acute diverticulitis and differentiating
32
it from other causes of abdominal pain
33
34
35 References:
36
1. Differentiating sigmoid diverticulitis from carcinoma on CT scans: mesenteric
37
inflammation suggests diverticulitis.
38
39 2. Diagnostic imaging of acute abdominal pain in adults.
40
2
Item : 13 of40 ~'?Mark <] C> !I ~ ~ , ~
0. ld : 46 25 Previous Next Lab Values Notes Calculator Reverse Color Text Zoom
3
4
5
6 A 56-year-old man is brought to the emergency department due to worsening confusion. He has
7 been forgetful and irritable for the past several days, and today, the patient's wife found him in
8 bed, lethargic and disoriented. He has had no fever, headache, abdominal pain, or vomiting
9 The patient drinks a half to a full pint of vodka daily and was hospitalized previously for acute
10 alcoholic pancreatitis. Temperature is 36.9 C (984 F), blood pressure is 116/68 mm Hg, and
11

-
pulse is 84/min. The patient is somnolent but awakens to physical stimuli. He follows simple
12
instructions but is not oriented to time or place. His speech is slurred. Lung fields are clea r to
14 auscultation and cardiac examination reveals normal heart sounds with no murmur. The abdomen
15 is moderately distended with shifting dullness. The patient moves all extremities equally and has a
16 flapping tremor of his outstretched, dorsiflexed hands. Deep tendon reflexes are 3+ diffusely.
17 Laboratory results are as follows
18
Complete blood count
19
20 Hematocrit 32%
21
22 Platelets 110,000/mm'
23
24 Leukocytes 4,100 /mm'
25
26
27 Serum chemistry
28
29 Sodium 132 mEq/L
30
31 Potassium 3.8 mEq/L
32
Blood urea nitrogen 14 mg/dL
33
34 0.8 mg/dL
Creatinine
35
36 Glucose 140 mg/dL
37
38
39
40
2
Item : 13 of40 ~'?Mark <J C> 61 ~ ~ , ~
0. ld : 46 25 Previous Next Lab Values Notes Calculator Reverse Color Text Zoom
3
4 Sodium 132 mEq/L
5
6 Potassium 3.8 mEq/L
7
8 Blood urea nitrogen 14 mg/dl
9
Creatinine 0.8 mg/dl
10
11

-
Glucose 140 mg/dl
12

14
15 Live r function studies
16
17 Albumin 2.8 g/dl
18
Alkaline phosphatase 170 U/L
19
20 Aspa rtate aminotransferase 97 U/L
21
22 Alanine aminotransferase 46 U/L
23
24 W hich of the following treatments will most likely improve this patient's mental status?
25
26
27 0 A Initiate broad-spectrum antibiotics
28 0 B. Initiate chlordiazepoxide
29
30 0 C . Initiate dantrolene
31 0 D. Initiate hype rtonic saline
32
33 0 E. Initiate intravenous thiamine
34 0 F. Initiate lactulose
35
0 G. Prescribe a high-p rotein diet
36
37
38 Submit
39
40
2
Item : 13 of40
0. ld : 46 25
~'?Mark <]
Previous
C>
Next
a
Lab Values
~
Notes
~
Calculator
,
Reverse Color
~
Text Zoom
3
4
5 Explanation:
6
7
8
9 Hepatic encephalopathy
10
11

-
12

14
15
16
17
Precipitating
factors
• Drugs (eg, sedatives, narcotics)
• Hypovolemia (eg, diarrhea)
• Electrolyte changes (eg, hypokalemia)
• 1 nitrogen load (eg, Gl bleeding)
• Infection (eg, pneumonia, UTI, SSP)
18 • Portosystemic shunting (eg, TIPS)
19
20
21
• Sleep pattern changes
22 Clinical • Altered mental status
23 presentation • Ataxia
24 • Asterixis
25
26
27 • Correct precipitating causes
28 (eg, fluids, antibiotics)
29 Treatment
30
• l blood ammonia concentration
(eg, lactulose, rifaximin)
31
32
Gl = gastrointestinal; SBP = spontaneous bacterial peritonitis;
33 TIPS = transjugular intrahepatic portosystemic sh un~ UTI = urinary tract infection.
34
@UWorld
35
36 This patient with likely cirrhosis (heavy alcohol use, ascites, thrombocytopenia) now has
37
progressive confusion, slurred speech, and asterixis (flapping tremor with outstretched hands),
38
which is suggestive of hepatic encephalopathy (HE) HE refers to impaired central nervous
39
40 -·· ..... -·
. .. .. . - . . . . ...... ... . . ' ...
~ Feedback SuWend EnQ ock
2
Item : 13 of40 ~'?Mark <] C> !I ~ ~ , ~
3
4
0. ld : 46 25
. .. . . .... ., ... Previous
"' ... ...
Next
"'."'
Lab Values Notes Calculator Reverse Color Text Zoom

progressive confusion, slurred speech, and asterixis (flapping tremor with outstretched hands),
5 which is suggestive of hepatic encephalopathy (HE) HE refers to impaired central nervous
6 system function in patients with c irrhosis and is due in part to ammonia neurotoxicity from
7
impaired liver function. Treatment involves identifying any under lying precipitant (eg, infection,
8
9
sedative medications, gastrointestinal bleed) and lowering serum ammonia.
10
Nonabsor bable disaccharides (eg, lactulose, lactitol) are preferred for lowering serum
11

-
12 ammonia. Colonic bacteria metabolize lactulose to short-chain fatty acids (eg, lactic acid, acetic
acid). This acidifies the colon to stimulate conversion of the absorbable ammonia to the
14 nonabsorbable ammonium (an ammonia trap) and causes bowel movements (which facilitates
15 fecal nitrogen excretion) The medication is titrated to produce 2 or 3 semiformed stools daily.
16
17 {Choice A) Broad-spectrum antibiotics would be appropriate in a patient with a suspected or
18 confi rmed infection, which would generally present with fever, elevated leukocyte count, and other
19 signs of infection (eg, tachyca rdia)
20
21 (Choice B) Chlordiazepoxide is a long-acting benzodiazepine used to treat alcohol withdrawal,
22 which would usually cause autonomic instability (eg, tachyca rdia, hypertension) along with mental
23 status changes
24
25 (Choice C) Dantrolene is used in neuroleptic malignant syndrome and malignant hype rthermia
26 Patients with these conditions usually have severe muscle rigidity, fever, and tachycardia
27
28 (Choice D) Hypertonic saline is usually used in patients with hyponatremia and severe
29 symptoms (eg, coma, seizure). A serum sodium level of 132 mg/dl is mild hyponatremia, which
30
is not uncommon in patients with cirrhosis (hype rvolemic hyponatremia) and would be an unlikely
31
32 cause of this patient's symptoms
33
34 (Choice E) Thiamine is used to treat and prevent Wernicke encephalopathy , which may be seen
35 in alcoholics due to poor dietary intake of thiamine. It presents as altered mental status, ataxia,
36 and nystagmus, but asterixis is not generally present
37
38 {Choice G) A high-protein diet is generally not recommended in patients with cirrhosis as it can
39 precipitate HE by causing increased serum ammonia levels.
40
2
Item : 13 of40 ~'?Mark <] C> !I ~ ~ , ~
0. ld : 46 25 Previous Next Lab Values Notes Calculator Reverse Color Text Zoom
3 I I I I
4
signs of infection (eg, tachyca rdia)
5
6
(Choice B) Chlordiazepoxide is a long-acting benzodiazepine used to treat alcohol withdrawal,
7
which would usually cause autonomic instability (eg, tachycardia, hypertension) along with mental
8
9 status changes
10
11 (Choice C) Dantrolene is used in neuroleptic malignant syndrome and malignant hyperthermia

-
12 Patients with these conditions usually have severe muscle rigidity, fever, and tachycardia

14 (Choice D) Hypertonic saline is usually used in patients with hyponatremia and severe
15 symptoms (eg, coma, seizure). A serum sodium level of 132 mg/dl is mild hyponatremia, which
16 is not uncommon in patients with cirrhosis (hypervolemic hyponatremia) and would be an unlikely
17 cause of this patient's symptoms
18
19 (Choice E) Thiamine is used to treat and prevent Wernicke encephalopathy , which may be seen
20 in alcoholics due to poor dietary intake of thiamine. It presents as altered mental status, ataxia,
21 and nystagmus, but asterixis is not generally present
22
23 {Choice G) A high-protein diet is generally not recommended in patients with cirrhosis as it can
24 precipitate HE by causing increased serum ammonia levels.
25
26 Educational objective:
27 Treatment of hepatic encephalopathy involves identifying the underly ing precipitant and lowering
28 serum ammonia. Nonabsorbable disaccharides (eg, lactulose, lactitol) are preferred fo r lowering
29
serum ammonia.
30
31
32 References:
33
1. Hepatic encephalopathy in chronic liver disease: 2014 Pr actice Guideline by the
34
35
American Association for the Study of Liver Diseases and the European
36 Association for the Study of the Liver.
37
38
39 Time Spent 5 seconds Copyright © UWorld Last updated: [10/ 10/2016)
40
2
Item : 14 of 40 ~'?Mark <J C> 61 ~ ~ , ~
0. ld : 4362 Previous Next Lab Values Notes Calculator Reverse Color Text Zoom
3
4
5
6 A 43-year-old man comes to the office due to a 6-month history of intermittent upper abdominal
7 pain associated with nausea. The patient describes episodes of dull epigastric pain that are
8 usually worse 15-30 minutes after meals and last for a few hours. Pain is not relieved with
9 antacids but improves when leaning forward . He also has had occasional diarrhea. The patient
10 has lost 6.8 kg (15 lb) over the last 12 months. Five yea rs ago, he was hospitalized for 3 days
11 with acute abdominal pain He has no other medical problems and takes no medications. The
12

-
patient smokes a pack of cigarettes a day and consumes alcohol almost daily His fam ily history
13
is significant for diabetes mellitus in his mother and prostate cancer in his father. W hich of the
15 following will most likely indicate a diagnosis for this patient's condition?
16
17
18
o A Anti-tissue transglutaminase antibodies
19 o B. Cancer-associated antigen 19-9 levels
20
21 o C. CT scan of the abdomen
22 0 D. Mesenteric angiogram
23
24 o E. Serum lipase levels
25 o F. Upper gastrointestinal endoscopy
26
27
28 Submit
29
30
31
32
33
34
35
36
37
38
39
40
2
Item : 14 of 40 ~'?Mark <] C> a ~ ~ , ~
3
4
0. ld : 4362
. ... . .' Previous Next Lab Values Notes Calculator Reverse Color Text Zoom

F. Upper gastrointestinal endoscopy [23%]


5
6
7 Explanation:
8
9
10
11
12 Overview of chronic pancreatitis

-
13

15
16
17
18
Etiology
• Alcohol use
• Cystic fibrosis (common in children)
• Ductal obstruction (eg, malignancy, stones)
19 • Autoimmune
20
21
22 • Chron ic epigastric pain with intermittent
23 Clinical pain-free intervals
24 presentation • Malabsorption-steatorrhea, weight loss
25
26 • Diabetes mellitu s
27
28 • Amylase/lipase can be normal & nondiagnostic
29 Laboratory
30 results/imaging • CT scan or MRCP can show calcifications,
31 dilated ducts & enlarged pancreas
32
33
34 • Pain management
35 • Alcohol & smoking cessation
36 Treatment
37 • Frequent, small meals
38 • Pancreatic enzyme supplements
39
40
2
Item : 14 of 40 ~'?Mark <] C> !I ~ ~ , ~
0. ld : 4362 Previous Next Lab Values Notes Calculator Reverse Color Text Zoom
3
4
• •
5
alcohol consumption (including prolonged use of socially acceptable amounts). CP typ ically
6 presents with chronic epigastric abdominal pain that can radiate to the back and is partially
7 relieved by sitting upright or leaning forward . Patients can have intermittent pain-free intervals
8 lasting from months to a yea r Early CP can present with acute attacks that become continuous
9 as the condition progressively worsens.
10
11 Patients with CP develop progressive pancreatic inflammation that causes nonreversible exocr ine
12 and endocrine functional damage. Diarrhea, steator rhea, and weight loss can develop due to

-
13

15
16
17
18
fat malabsorption from reduced levels of exocrine pancreatic enzymes (eg, amylase, protease,
lipase) CP eventually causes pancreatic endocrine fa ilure with glucose intolerance or overt
diabetes. Pancreatic calcifications seen on abdominal plain films or CT scan are helpful fo r
establishing the diagnosis In addition, CT scan helps exclude other etiologies (eg, pancreatic
cancer, pseudocyst)
19
20 {Choice A) Anti-tissue transglutam inase antibodies are typ ically positive in celiac disease, which
21 can cause iron deficiency anemia, steatorrhea, and weight loss. However , celiac disease
22 typ ically causes minor gastrointestinal (GI) symptoms; significant abdominal pain relieved by
23 position change is extremely unlikely .
24
25 (Choice B) Serum cance r-associated antigen (CA) 19-9 levels are elevated in panc reatic
26 cancer, which usually presents with abdominal pain, weight loss, and jaundice However , CA
27 19-9 levels should not be used to diagnose suspected panc reatic cance r (due to limited sensitivity
28 and specificity) Abdominal CT scan is usually the fi rst step
29
30 (Choice D) Mesente ric angiogram can diagnose chronic mesenteric ischemia, which can
31 present with dull abdominal pain (usually after eating) and unintentional weight loss due to
32 avoidance of food . Most cases are due to atherosclerosis, for which this patient has no
33
significant risk factors. Diarrhea and improvement in pain with position changes are uncommon.
34
35 (Choice E) In contrast to widespread inflammation in acute pancreatitis, CP causes patchy
36
inflammation and fibros is (burned-out pancreas) and can present with normal or only slightly
37
38 elevated serum amylase and lipase. Abdominal imaging showing pancreatic calcifications is the
39 best way to confirm CP.
40
2
Item : 14 of 40 ~'?Mark <] C> !I ~ ~ , ~
0. ld : 4362 Previous Next Lab Values Notes Calculator Reverse Color Text Zoom
3 • . - , - ,. e I ,. ,. -,. ••• - .,. - • • ,. • •• ,- - ,. • -,. - "

4 can cause iron deficiency anemia, steatorrhea, and weight loss. However, celiac disease
5 typically causes minor gastrointestinal (GI) symptoms; significant abdominal pain relieved by
6
position change is extremely unlikely .
7
8 (Choice B) Serum cancer-associated antigen (CA) 19-9 levels are elevated in pancreatic
9
cancer, which usually presents with abdominal pain, weight loss, and jaundice However, CA
10
11 19-9 levels should not be used to diagnose suspected pancreatic cancer (due to limited sensitivity
12 and specificity) Abdominal CT scan is usually the first step

-
13

15
16
17
18
(Choice D) Mesenteric angiogram can diagnose chronic mesenteric ischemia, which can
present with dull abdominal pain (usually after eating) and unintentional weight loss due to
avoidance of food. Most cases are due to atherosclerosis, for which this patient has no
significant risk factors. Diarrhea and improvement in pain with position changes are uncommon.

19 (Choice E) In contrast to widespread inflammation in acute pancreatitis, CP causes patchy


20 inflammation and fibrosis (burned-out pancreas) and can present with normal or only slightly
21 elevated serum amylase and lipase. Abdominal imaging showing pancreatic calcifications is the
22 best way to confi rm CP.
23
24 (Choice F) Upper Gl endoscopy is indicated for suspected peptic ulcer disease, gastritis,
25 dysphagia, or hematemesis. The diagnosis of CP can often be made without
26 invasive/endoscopic studies.
27
28 Educational objective:
29
Chronic pancreatitis is a progressive inflammatory disorder of the pancreas characterized by
30
31 recurrent bouts of upper abdominal pain, diarrhea/steatorrhea. and weight loss. Diagnosis is
32 established by the presence of panc reatic calcifications on CT scan or plain film.
33
34 References:
35
36 1. Chronic pancreatitis: a diagnostic dilemma.
37
38
39 Time Spent 2 seconds Copyright © UWorld Last updated: [09/20/2016)
40
2
Item : 15of40 ~'?Mark <] C> !I ~ ~ , ~
0. ld : 4321 Previous Next Lab Values Notes Calculator Reverse Color Text Zoom
3
4
5
6 A 46-year-old man comes to the office due to a sensation of right upper-quadrant fullness. He
7 has been obese since early adolescence. Past medical history is notable for type 2 diabetes
8 mellitus and hypertension His current medications include metformin, candesartan, and
9 hydrochlorothiazide. The patient does not use tobacco, alcohol, or illicit drugs. His father has
10 type 2 diabetes mellitus. Temperature is 36.7 C (98 F), blood pressure is 138/90 mm Hg, pulse is
11
72/min, and respirations are 16/min. BMI is 36 kg/m2 . On examination, the patient has
12
acanthosis nigricans over the neck creases and hepatomegaly Laboratory results are as follows:
13

--
14

16
17
18
19
Albumin

Total bilirubin
4.0
mg/dl

1.0
mg/dl

20 0.8
Direct bilirubin
21 mg/dl
22
23 Alkaline phosphatase 100 U/ L
24
25 Aspa rtate aminotransferase
122 U/L
26 (SGOT)
27
Alanine aminotransferase
28 131 U/L
29 (SGPT)
30
168
31 Blood glucose (fasting)
mg/dl
32
33
34 Viral hepatitis serologies, antimitochondrial antibodies, antinuclear antibodies, serum
35 ceruloplasmin, and transferrin saturation are normal. W hich of the following is the most likely
36 pathophysiologic mechanism responsible for this patient's increased liver enzymes?
37
38
39
40
2
Item : 15of40 ~'?Mark <J C> 61 ~ ~ , ~
0. ld : 4321 Previous Next Lab Values Notes Calculator Reverse Color Text Zoom
3 ., ., I I ., tJ• '"' '"' ' •., '"' ., I ... ,., I '"'I., J o~ll ., I 'I '"' I ., '"'., I tH
4
5 4.0
6 Albumin
mg/dl
7
8 1.0
Total bilirubin
9 mg/dl
10
11 0.8
Direct bilirubin
12 mg/dl
13

--
14 Alkaline phosphatase 100 U/L
Aspartate aminotransferase
16 122 U/L
(SGOT)
17
18
Alanine aminotransferase
19 131 U/L
(SGPT)
20
21 168
22 Blood glucose (fasting)
mg/dl
23
24
25 Viral hepatitis serologies, antimitochondrial antibodies, antinuclear antibodies, serum
26 ce ruloplasmin, and transferrin saturation are normal. W hich of the following is the most likely
27 pathophysiologic mechanism responsible for this patient's increased live r enzymes?
28
29
30 <0 A Decreased lipolys is
31 <0 B. Glucagon overproduction
32
33 <0 C. Increased glucoco rticoid production
34 <0 D. Inc reased hepatic glycogen synthesis
35
36
<0 E. Insulin resistance
37
38 Submit
39
40
2
Item : 15 of40 ~'?Mark <] C> a ~ ~ , ~
3
4
0. ld : 4321
. "' . .. . '
Previous Next Lab Values Notes Calculator Reverse Color Text Zoom

B. Glucagon overproduction [4%)


5
6 C. Increased glucocorticoid production [3%)
7 D. Inc reased hepatic glycogen synthesis [18%)
8
9 .; E. Insulin resistaAce [52%)
10
11
12 Explanation:
13

--
14

16
17
18
19
Nonalcoholic fatty liver disease

• Hepatic steatosis on imaging or biopsy


Definition • Exclusion of significant alcohol use
20
21 • Exclusion of other causes of fatty liver
22
23 • Mostly asymptomatic
24 • Metabolic syndrome
Clinical
25 • +/- Steatohepatitis (AST/ALT ratio <1)
features
26
• Hyperechoic texture on ultrasound
27
28
29 • Diet & exercise
Treatment
30 • Consider bariatric surgery if BMI ;::35
31
32 AST = aspartate aminotransferase; AlT = alanine aminotransferase.
33
@UWortd
34
35 This patient's presentation (hepatocellular injury in the absence of viral hepatitis or autoimmune
36
disease) is most consistent with nonalcoholic fatty liver disease (NAFLD) Affected patients
37
38 are typically middle-aged, obese, and have features of the metabolic syndrome (eg, central
39 obesity, diabetes mellitus, hyperlipidemia, hypertension) NAFLD can range from bland steatosis
40
2
Item : 15of40 ~'?Mark <] C> !I ~ ~ , ~
0. ld : 4321 Previous Next Lab Values Notes Calculator Reverse Color Text Zoom
3 I- ~- I I - " • • •• • • ,. • · ·• · · ·-. A-- .. .. -
4 are typically middle-aged , obese, and have features of the metabolic syndrome (eg, central
5
obesity, diabetes mellitus, hyperlipidemia, hypertension) NAFLD can range from bland steatosis
6
to inflammation and necrosis (steatohepatitis) to fibrosis and cirrhosis. Histologically, NAFLD can
7
8 resemble alcohol-induced live r disease (macrovesicular fat deposition, peripheral displacement of
9 nuclei) but occurs in patients with minimal or no alcohol history
10
11 NAFLD can be due to increased transport of free fatty acids (FFA) from adipose tissue to the
12 liver, decreased oxidation of FFA in the liver, or decreased clearance of FFA from the liver (due
13 to decreased VLDL production) It is frequently related to peripheral insulin resistance leading

--
14

16
17
18
19
to increased peripheral lipolysis, triglyceride synthesis, and hepatic uptake of fatty acids.
Hepatic FFA increases oxidative stress and production of proinflammatory cytokines (eg, tumor
necrosis factor-alpha)

(Choice A) Insulin decreases lipolysis in adipose cells. Insulin resistance, as seen in most
patients with NAFLD, leads to inc reased lipolysis and FFA release. FFA are then transported to
20 the liver, where they participate in pathogenesis of NAFLD and further contribute to insulin
21
resistance by impairing insulin-dependent glucose uptake and increasing hepatic
22
23
gluconeogenesis.
24
25
(Choice B) Some patients with type 2 diabetes can have increased glucagon production.
26 Although glucagon increases gluconeogenesis and glycogenolysis, it does not play a role in
27 hepatic fat accumulation.
28
29 (Choice C) Cushing syndrome can contribute to insulin resistance and NAFLD, but it would be
30 unlikely in this patient with near lifelong obesity
31
32 (Choice D) Hepatic glycogen synthesis is usually decreased in type 2 diabetes and does not
33 play a role in hepatic steatosis.
34
35 Educational objective:
36 Nonalcoholic fatty liver disease resembles alcohol-induced liver disease but occurs in patients
37 with minimal or no alcohol history. It is associated with insulin resistance. Severity can range
38 from bland steatosis to steatohepatitis to fibrosis and cirrhosis.
39
40
2
Item : 16 of40 ~'?Mark <J C> 61 ~ ~ , ~
0. ld : 4405 Previous Next Lab Values Notes Calculator Reverse Color Text Zoom
3
4
5
6 A 40-year-old man comes to the office for evaluation of dysphagia. For the past 3 years, the
7 patient has had difficulty swallowing solid foods and liquids, with symptoms worsening recently .
8 He reports that it is easier to swallow standing upright He has occasional regurgitation of
9 undigested food and has lost 5 kg (11 lbs) over the past 6 months. The patient has no chest pain
10 or muscle weakness. His only other medical problem is generalized anxiety disorder fo r which he
11 takes sertraline. He does not use tobacco, alcohol, or recreational drugs Vital signs are
12
normal. His neck is supple without masses. Cardiopulmonary examination shows no
13
14 abnormalities. Muscle strength is 5/5 in all 4 extremities, and deep tendon reflexes are 2+

-
15 symmetrically Barium esophagogram is shown in the exhibit. W hich of the following is the
patient's most likely diagnosis?
17
18
6 A Achalasia
19
20 <0 B. Esophageal cancer
21
6 C. Esophageal web
22
23 ® D. Globus sensation
24
e; E. Polymyos itis
25
26 0 F. Zenke r dive rticulum
27
28
29 Submit
30
31
32
33
34
35
36
37
38
39
40
2
Item : 16 of40 ~'?Mark <] C> !I ~ ~ , ~
0. ld : 4405 Previous Next Lab Values Notes Calculator Reverse Color Text Zoom
3
4 E. Polymyositis [0%]
5
6 F. Zenke r dive rticulum [2%]
7
8
Explanation:
9
10
11
12
13
14

-
Chronic dysphagia to both solids and liquids, regurgitation, difficulty belching, and mild weight
15
loss are all common manifestations of achalasia. Other symptoms include chest pain and
17 heartburn; therefore, many patients are initially diagnosed with gastroesophageal reflux. On
18 average, patients have symptoms for approximately 5 yea rs before receiving a diagnosis of
19 achalasia.
20
21 Achalasia is due to impaired peristalsis of the distal esophagus and impaired relaxation of the
22 lower esophageal sphincter (LES) This prevents food or liquid from passing through the LES
23 until the hydrostatic pressure in the esophageal column is greater than the closing pressure of the
24 sphincter Being in the upright position inc reases the pressure in the esophagus and results in
25 more effective swallowing
26
27 Manometry is the most sensitive test and key to diagnosis. Barium esophagram, which may show
28 a smooth "bird-beak" narrowing near the LES, can be helpful in patients with nondiagnostic
29 manometry.
30
31 (Choice B) Esophageal cancer classically presents with dysphagia to solids, especially bread
32
and meat, although tumors can eventually cause dysphagia to liquids as well Tobacco and
33
34 alcohol use are major risk factors. In addition, the prolonged time course of this patient's
35 symptoms and his relatively young age are less consistent with malignancy
36
37 (Choice C) Esophageal webs are most commonly located in the upper esophagus and only
38 cause mild focal narrowing (dysphagia to solids but not liquids). They are often associated with
39 iron deficiency (Plummer-Vinson syndrome)
40
2
Item : 16 of40 ~'?Mark <] C> !I ~ ~ , ~
0. ld : 4405 Previous Next Lab Values Notes Calculator Reverse Color Text Zoom
3
4 (Choice B) Esophageal cancer classically presents with dysphagia to solids, especially bread
5 and meat, although tumors can eventually cause dysphagia to liquids as well Tobacco and
6 alcohol use are major risk factors. In addition, the prolonged time course of this patient's
7 symptoms and his relatively young age are less consistent with malignancy
8
9 (Choice C) Esophageal webs are most commonly located in the upper esophagus and only
10 cause mild focal narrowing (dysphagia to solids but not liquids). They are often associated with
11 iron deficiency (Plummer-Vinson syndrome)
12
13 (Choice D) Globus sensation is a diagnosis of exclusion and is characterized by the sensation
14

-
of a lump in the back of the throat It is a functional disorder and does not cause any
15
abnormalities on barium esophagram
17
(Choice E) Polymyositis can present with dysphagia but usually affects the striated muscle in the
18
19
upper third of the esophagus, and is associated with other symptoms of muscle weakness (eg,
20 difficulty climbing stairs).
21
22 (Choice F) Zenker diverticulum, caused by an outpouching at the cricopharyngeallevel of the
23 esophagus, most commonly occurs in patients age >60 and presents with dysphagia, halitosis,
24 and fullness of the throat
25
26 Educational objective:
27 Achalasia commonly presents with chronic dysphagia to both solids and liquids, regurgitation,
28 difficulty belching, and weight loss. Achalasia is caused by impaired peristalsis of the distal
29 esophagus and failure of the lower esophageal sphincter to relax when food boluses reach it
30 Manometry is key to diagnosis
31
32
References:
33
34 1. Achalasia.
35
36 2. Achalasia: A review of clinical diagnosis, epidemiology, treatment and outcomes.
37
38
39 Time Spent 3 seconds Copyright© UWorld Last updated: [05/02/2016)
40
2
Item : 17 of 40 ~'?Mark <J C> 61 ~ ~ , ~
0. ld : 2200 Previous Next Lab Values Notes Calculator Reverse Color Text Zoom
3
4
5
6 A 54-year-old man comes to the clinic complaining of bitter taste and substernal burning 30-40
7 minutes after meals. The burning is relieved with antacids and worsened by lying supine. He has
8 also noticed that food gets stuck in his chest if he does not chew properly He has had an
9 unintentional weight loss of 4.5 kg (10 lb) over the past 3 months. The patient has a 40-pack-year
10 smoking history. He has no fam ily history of cancer. He is afebrile with stable vital signs The
11 physical examination is unremarkable. Chest x-ray and electrocard iogram show no
12
abnormalities. The stool guaiac test is negative Which of the following is the most appropriate
13
14 next step in management of this patient?
15

-
16

18
19
20
21
6 A Esophageal manometry
e; B. Esophageal pH monitoring
6 C. Helicobacter pylori stool antigen testing
e; D. Trial of famotidine
22 6 E. Trial of lansoprazole
23
24 e; F. Upper gastrointestinal endoscopy
25
26
27 Submit
28
29
30
31
32
33
34
35
36
37
38
39
40
2
Item : 17 of 40 ~'?Mark <] C> !I ~ ~ , ~
0. ld : 2200 Previous Next Lab Values Notes Calculator Reverse Color Text Zoom
3
4
Patients with typical GERD require upper gastrointestinal (GI) endoscopy if they have alarm
5 symptoms (dysphagia, odynophagia, weight loss, anemia, Gl bleeding, or recurrent vomiting) or
6 are men age >50 with chronic (>5 years) symptoms and cancer risk factors (eg, tobacco use)
7 Patients with esophagitis on endoscopy can receive treatment depending on the diagnosis (eg,
8 autoimmune disease, Barrett's esophagus) Patients without esophagitis on endoscopy usually
9 require further evaluation (eg, esophageal manometry)
10
11 Patients with typical GERD symptoms who do not meet initial endoscopy criteria can receive an
12 initial trial of daily proton pump inhibitor (PPI). Patients with refractory symptoms should try
13 another PPI or increase the use of PPI to twice daily Patients with persistent symptoms likely
14
require further testing such as endoscopy or esophageal pH monitoring This patient's
15

-
presentation (age >50, unintentional weight loss, dysphagia, tobacco use) warrants initial
16
endoscopy (rather than PPI or H2 blockers) to assess for complications of GERD (Choices 0
18 and E)
19
20 (Choices A and B) Esophageal manometry and pH monitoring should be considered in patients
21 with persistent GERD symptoms or normal upper Gl endoscopy to assess fo r other conditions
22 (eg, motility disorders) that can occasionally mimic GERD.
23
24 (Choice C) Helicobacter pylori can cause gastric and duodenal ulcers. Testing is indicated in
25 patients with active or past history of peptic ulcer disease. Routine screening and empiric
26 treatment fo r H pylori infection may also be considered for patients who have dyspepsia but not
27 GERD.
28
29 Educational objective:
30 Patients with typical gastroesophageal reflux symptoms require an upper gastrointestinal (GI)
31 endoscopy if they have alarm symptoms (dysphagia, odynophagia, weight loss, anemia, Gl
32
bleeding, or recurrent vomiting) or are men age >50 with chronic (>5 years) symptoms and
33
34 cancer risk factors (eg, tobacco use). All other patients can receive an empiric trial of proton
35 pump inhibitor therapy and further evaluation if refractory to therapy
36
37 References:
38
39 1. Guidelines for the diagnosis and management of gastroesophageal refl ux
40
2
Item : 18 of40 ~'?Mark <J C> 61 ~ ~ , ~
0. ld : 4086 Previous Next Lab Values Notes Calculator Reverse Color Text Zoom
3
4
5
6 A 55-year-old man comes to the office for a routine preventive examination. He feels well but has
7 had mild constipation fo r several years He does not have any major medical problems and is not
8 on any prescription or over-the-counter medication. The patient smoked one-and-a-half packs of
9 cigarettes daily for 30 years but quit several years ago, and drinks an average of 1 alcoholic
10 beverage daily . The patient's diet is notable for daily intake of fast foods and heavy consumption
11 of dairy products Phys ical examination is unremarkable. A screening colonoscopy is
12
performed and abnormalities are noted in the sigmoid colon as shown in the image below.
13
14
15
16

--
17

19
20
21
22
The patient is concerned about the findings and asks what he could do to decrease the chance of
developing complications W hich of the following is the best recommendation for this patient?

<0 A Avoid nuts and seeds


23
24 <0 B. Consider surgical options
25 <0 C. Eliminate alcohol intake
26
27 e; D. Increase dietary fiber intake
28 <0 E. Reduce caffeine intake
29
30 <0 F. Start daily low-dose aspirin
31
32
33 Submit
34
35
36
37
38
39
40
2
Item : 18 of40 ~'?Mark <] C> !I ~ ~ , ~
0. ld : 4086 Previous Next Lab Values Notes Calculator Reverse Color Text Zoom
3
beverage daily. The patient's diet is notable fo r daily intake of fast foods and heavy consumption
4
5 of dairy products Physical examination is unremarkable. A screening colonoscopy is
6 performed and abnormalities are noted in the sigmoid colon as shown in the image below.
7
8
9
10
11
12
13
14
15
16

--
17

19
20
21
22
23
24
25
26
27
28
29
30 The patient is concerned about the findings and asks what he could do to decrease the chance of
31 developing complications W hich of the following is the best recommendation fo r this patient?
32
33
34 A Avo id nuts and seeds [6%)
35 B. Consider surgical options [1%)
36
37 C. Eliminate alcohol intake [1%)
38 ~ D. Increase dietary fiber intake [92%)
39
40
2
Item : 18 of40 ~'?Mark <J C> 61 ~ ~ , ~
0. ld : 4086 Previous Next Lab Values Notes Calculator Reverse Color Text Zoom
3
C. Eliminate alcohol intake [1%)
4
5 ~ D. Increase dietary fiber intake [92%)
6 E. Reduce caffeine intake [0%)
7
8 F. Start daily low-dose aspirin [0%)
9
10
11 Explanation:
12
13 Diverticula are outpouchings of the colonic wall that fo rm at points of weakness, most commonly
14 in the sigmoid colon. Diverticulosis is a common finding on endoscopy , and the incidence
15 inc reases with age. It is usually asymptomatic , but potential complications include diverticular
16 hemorrhage and diverticulitis (acute feve r, abdominal pain, and possible perforation with abscess

--
17

19
20
21
22
formation). Diverticulosis is strongly associated with chronic constipation; the increased
colonic pressures seen in constipation may promote formation of diverticula, and conversely , the
altered motility seen in diverticulosis may contribute to constipation

The impact of dietary fiber on the initial formation of diverticulosis is unclear. However, the
incidence of acute diverticular complications is lower in individuals with a high intake of fruit
23
and vegetable fiber. Physical activity is also inve rsely co rrelated with the risk of complications.
24
Factors associated with an increased risk of complications include heavy meat consumption,
25
26 aspirin or nonsteroidal anti-inflammatory drug use (Choice F) , obesity, and possibly smoking
27
28
{Choice A) In the past, patients with diverticulosis were advised to avoid nuts and seeds under
29 the assumption that they may obstruct the diverticular lumen and precipitate acute diverticulitis.
30 However , recent studies have found no link between intake of these foods and inc idence of
31 diverticulitis.
32
33 {Choice B) Surgery is indicated only in the event of complications, such as recurrent diverticular
34 bleeding, perforation, or peritonitis
35
36 (Choices C and E) Alcohol intake is associated with inc reased formation of dive rticula.
37 However , it is not associated with risk of diverticular complications, and this patient's light intake is
38 unlikely to be a significant factor. Caffeine intake also does not correlate with risk of
39 complications
40
2
Item : 18 of40 ~'?Mark <] C> !I ~ ~ , ~
3
4
. ., . . . . . . . . . .
0. ld : 4086
.~ .
Previous
. Next
. .. . "'.
Lab Values Notes Calculator Reverse Color Text Zoom

colonic pressures seen in constipation may promote fo rmation of diverticula, and conve rsely, the
5 altered motility seen in diverticulosis may contribute to constipation.
6
7 The impact of dietary fiber on the initial formation of diverticulosis is unclear. However , the
8 incidence of acute diverticular complications is lower in individuals with a high intake of fr uit
9
and vegetable fiber . Physical activity is also inve rsely correlated with the risk of complications.
10
Factors associated with an inc reased risk of compl ications include heavy meat consumption,
11
12 aspirin or nonsteroidal anti-inflammatory drug use (Choice F), obesity , and possibly smoking
13
14 (Choice A) In the past, patients with diverticulosis were advised to avoid nuts and seeds under
15 the assumption that they may obstruct the diverticular lumen and precipitate acute diverticulitis.
16 However, recent studies have found no link between intake of these foods and incidence of

--
17

19
20
21
22
diverticulitis.

(Choice B) Surgery is indicated only in the event of complications, such as recurrent diverticular
bleeding, perforation, or peritonitis.

(Choices C and E) Alcohol intake is associated with increased formation of diverticula.


23 However, it is not associated with risk of diverticular complications, and this patient's light intake is
24 unlikely to be a significant factor. Caffeine intake also does not co rrelate with risk of
25 compl ications
26
27 Educational objective:
28 Dive rticulosis is associated with chronic constipation. It is usually asymptomatic , but potential
29 complications incl ude hemorrhage and diverticulitis. The risk of complications is lower with a high
30
intake of fruit and vegetable fibe r, and higher with heavy meat consumption, aspirin or
31
32 nonsteroidal anti-inflammatory drug use, obesity , and possibly smoking
33
34 References:
35
36 1. Colonic diverticular disease: medical treatments for acute diverticulitis.
37
38
39 Time Spent 43 seconds Copyright © UWorld Last updated [08/ 18/2016)
40
2
Item : 19 of40 ~'?Mark <J C> 61 ~ ~ , ~
0. ld : 2203 Previous Next Lab Values Notes Calculator Reverse Color Text Zoom
3
4
5
6 A 46-year-old man comes to the emergency department due to several episodes of vomiting The
7 last episode of emesis contained blood. Five hours ago, he had a fatty meal and several
8 alcoholic drinks. The patient's medical history is notable for alcohol abuse. He also has chronic
9 dyspepsia, for which he was evaluated 3 days ago with upper gastrointestinal endoscopy and
10 abdominal ultrasound. The ultrasound revealed an enlarged, hyperechoic live r and gallstones in
11 the gallbladder , and the endoscopy found mild esophagitis and gastritis His temperature is 36.6
12
C (97 9 F) , blood pressure is 120/70 mm Hg, pulse is 95/min, and respirations are 15/min. The
13
14 abdomen is soft and nontender. The rectal examination shows no melena. Laboratory results are
15 as follows
16

...
Hemoglobin 12.8 g/dL
17
18 Leukocytes 5,400/mm>

20 Blood urea nitrogen 26 mg/dL


-
21
22 Creatinine 1.1 mg/dL
23
Aspartate aminotransferase (SGOT) 100 U/L
24
25 Alanine aminotransferase (SGPT) 45 U/L
26
27 Bilirubin 0.7 mg/dL
28
29
30 Nasogastric suction returns normal stomach contents mixed with bright red blood. W hich of the
31 following is the most likely explanation for this patient's bloody vomiting?
32
33 e> A Effort rupture of the esophagus
34
35 6 B. Endoscopy-related esophageal perforation
36 6 C. Hemobilia
37
38 6 D. Mucosal tear at the gastroesophageal junction
39 e> E. Rupture of esophageal varices
40
2
Item: 19 of40 ~'?Mark <J C> 61 ~ ~ , ~
0. ld : 2203 Previous
Next Lab Values Notes Calculator Reverse Color Text Zoom
3 -~---- ........ - -·----------- ~------

4 dyspepsia, for which he was evaluated 3 days ago with upper gastrointestinal endoscopy and
5 abdominal ultrasound. The ultrasound revealed an enlarged, hyperechoic liver and gallstones in
6 the gallbladder, and the endoscopy found mild esophagitis and gastritis His temperature is 36.6
7 C (97 9 F), blood pressure is 120/70 mm Hg, pulse is 95/min, and respirations are 15/min. The
8
abdomen is soft and nontender. The rectal examination shows no melena. Laboratory results are
9
10 as follows:
11 Hemoglobin 12.8 g/dL
12
13 Leukocytes 5,400/mm>
14
15 Blood urea nitrogen 26 mg/dL
16

...
17
18

20
21
Creatinine
Aspartate aminotransferase (SGOT)

Alanine aminotransferase (SGPT)


1.1 mg/dL

100 U/ L

45 U/ L
22 Bilirubin 0.7 mg/dL
23
24
Nasogastric suction returns normal stomach contents mixed with bright red blood. Which of the
25
26 following is the most likely explanation for this patient's bloody vomiting?
27
28 ® A Effort rupture of the esophagus
29
30 ® B. Endoscopy-related esophageal perforation
31 ® C. Hemobilia
32
33 ® D. Mucosal tear at the gastroesophageal junction
34 ® E. Rupture of esophageal varices
35
36
® F. Stress gastritis
37
38 Submit
39
40
2
Item : 19 of40
0. ld : 2203
~'?Mark <]
Previous
C>
Next
a
Lab Values
~
Notes
~
Calculator
,
Reverse Color
~
Text Zoom
3
4
5 Explanation:
6
7
8
9 Mallory-Weiss tear
10
11
12 • Sudden increase in abdominal pressure
13 (eg, forceful retching)
14
Etiology • Mucosal tear in esophagus or stomach
15
16 (submucosal arterial or venous plexus bleeding)

...
17
18

20
21
Clinical
presentation
• Risk factors: Hiatal hemia, alcoholism

• Vomiting, retching
• Hematemesis
22 • Epigastric pain
23
24
25 Diagnosis • Longitudinal laceration on endoscopy
26
27
28 • Most heal spontaneously
Treatment
29 • Endoscopic therapy for persistent bleeding
30
~UWorld
31
32
33 This patient with alcohol abuse, alcoholic hepatitis (2:1 ratio of aspartate aminotransferase to
34 alanine aminotransferase), esophagitis, and gastritis now has acute, bright red hematemesis
35 following multiple episodes of vomiting This presentation is highly suggestive of a Mallory-Weiss
36 tear (MWT) MWT is characterized by longitudinal tears in the mucosa near the
37 gastroesophageal junction, with bleeding due to injury to the submucosal arteries or veins. It
38 typically occurs due to a sudden increase in intraabdominal pressure (eg, retching , blunt
39 abdominal trauma)
40
2
Item : 19 of40 ~'?Mark <] C> !I ~ ~ , ~
0. ld : 2203 Previous Next Lab Values Notes Calculator Reverse Color Text Zoom
3
4 gastroesophageal junction, with bleeding due to injury to the submucosal arteries or veins. It
5 typically occurs due to a sudden increase in intraabdominal pressure (eg, retching , blunt
6 abdominal trauma).
7
8 MWT is a common cause of upper gastrointestinal (GI) hemorrhage and is often seen in
9 association with alcohol abuse and hiatal hernia. The diagnosis can be confi rmed on
10 endoscopy. Bleeding stops spontaneously in 90% of patients. Those with ongoing bleeding
11 can be treated endoscopically with electrocoagulation or local injection of epinephrine.
12
13 (Choices A and B) Esophageal rupture can be caused by severe retching (Boerhaave
14 syndrome), penetrating trauma, or as a complication of endoscopy. It is characterized by acute
15 chest pain, subcutaneous emphysema, and often a left-sided pleural effusion. W hen esophageal
16

...
rupture occurs due to endoscopy , it is usually apparent shortly after the procedure
17
18
(Choice C) Hemobilia (bleeding in the biliary tree) is an uncommon cause of upper Gl
hemorrhage. It is typically seen following abdominal trauma or surgery Endoscopy without
20
21 manipulation of the biliary system would not cause hemobilia.
22
23 (Choice E) Esophageal varices are submucosal veins that have dilated due to portal
24 hypertension Eventual rupture causes upper Gl bleeding However, varices would likely have
25 been visible on this patient's endoscopy 3 days earlier.
26
27 (Choice F) Stress ulcers develop in the setting of severe and prolonged physiological stress. Gl
28 bleeding from stress ulcers is usually seen in patients in an intensive care or burn unit setting
29
30 Educational objective:
31 Mallory-Weiss tear occurs due to a sudden increase in intraabdominal pressure (eg, retching),
32 leading to a mucosal tear and hematemesis. The diagnosis can be confi rmed on endoscopy
33 Bleeding stops spontaneously in most patients, but those with ongoing bleeding can be treated
34 endoscopically
35
36
37 References:
38 1. Endoscopic management of Mallory-Weiss tearing.
39
40
2
Item: 20of40 ~'?Mark <J C> 61 ~ ~ , ~
0. ld : 3936 Previous Next Lab Values Notes Calculator Reverse Color Text Zoom
3
4
5
6 An 80-year-old woman is brought to the physician by her son due to fatigue She lives alone and
7 suffers from bilateral knee osteoarthritis, which significantly limits her mobility. Her only other
8 medical problem is hypertension Her medications include lisinopril, chlorthalidone, and
9 prescription naproxen. She also takes low-dose aspirin because it is "good fo r the heart." Her
10 blood pressure is 146/92 mm Hg and pulse is 78/min. The patient has conjunctival pallor Both
11 knees are deformed due to bony overgrowth but there is no significant effusion or erythema.
12
Extension of both knees is limited. Her renal function, assessed 2 months ago, was normal.
13
14 W hich of the following is the most likely cause of pallor in this patient?
15
16 ® A. Anemia of chronic disease
17
18 ® B. Aplastic anemia

-
19

21
22
23
24
® C. Drug-induced hemolysis
® D. Hypothyroidism
® E. Iron deficiency anemia
® F. Vitamin B12 deficiency
25
26
27 Submit
28
29
30
31
32
33
34
35
36
37
38
39
40
2
Item : 20of40 ~'?Mark <] C> !I ~ ~ , ~
0. ld : 3 936 Previous Next Lab Values Notes Calculator Reverse Color Text Zoom
3 J, : I y ~'
4
5
6 Explanation:
7
8 Conjunctival pallor on physical examination is nonspecific but suggests a significant underlying
9 anemia. Iron deficiency anemia is the most likely diagnosis in this patient given her concurrent
10 nonsteroidal anti-inflammatory drug (NSAID) and aspirin use. Both NSAIDs and aspirin can
11
cause gastritis and/or gastric ulce rs leading to chronic gastrointestinal blood loss and depletion of
12
13 iron stores. Evaluation of this patient would likely incl ude blood counts, iron studies, and fecal
14 occult blood testing If iron deficiency is confirmed, especially if there is evidence of ongoing
15 blood loss, definitive diagnosis can be obtained with upper (and usually lower) gastrointestinal
16 endoscopy Even mild iron deficiency should be evaluated thoroughly as low-grade
17 gastrointestinal bleeding may herald a later catastrophic hemorrhage Management typically
18 includes withholding NSAIDs and aspirin and initiating antisecretory medication (proton pump

-
19

21
22
23
24
inhibitors)

Elderly patients may present additional challenges in evaluating acute anemia. Besides other
potential comorbidities that can cause chronic anemia at baseline (eg, renal insufficiency,
myelodysplastic syndromes, occult malignancy, nutritional deficiencies), a significant number of
elderly patients will have a baseline anemia for which no etiology is apparent, the so-called
25
26 "idiopathic anemia of ageing." They are also more likely to have additional comorbidities such as
27 congestive heart failure or chronic lung disease, which make them poorly tolerant of even mild
28 anemia.
29
30 (Choice A) Anemia of chronic disease (anemia of chronic inflammation) is caused by
31 suppression of red cell production by inflammatory cytokines. It commonly occurs in
32 inflammatory arthropathies (eg, rheumatoid arth ritis, lupus) but is not associated with
33 osteoarthritis.
34
35 {Choice B) Aplastic anemia is a disorder of bone marrow stem cells that is often induced by
36 exposure to various drugs, chemicals, ionizing radiation, or viruses. It can also be
37 idiopathic The incidence inc reases with age, but it is significantly less common than iron
38
deficiency anemia at any age
39
40
2
Item: 20of40 ~'?Mark <] C> !I ~ ~ , ~
0. ld : 3936 Previous Next Lab Values Notes Calculator Reverse Color Text Zoom
3
4
5 (Choice A) Anemia of chronic disease (anemia of chronic inflammation) is caused by
6 suppression of red cell production by inflammatory cytokines It commonly occurs in
7 inflammatory arth ropathies (eg, rheumatoid arthritis, lupus) but is not associated with
8 osteoarthritis.
9
10 (Choice B) Aplastic anemia is a disorder of bone marrow stem cells that is often induced by
11 exposure to various drugs, chemicals, ionizing radiation, or viruses. It can also be
12 idiopathic . The incidence increases with age, but it is significantly less common than iron
13 deficiency anemia at any age.
14
15 {Choice C) Although the risk of drug-induced hemolysis (eg, dapsone with G6PD deficiency)
16 may be higher in older patients taking multiple medications, it is an uncommon cause of
17
anemia. None of this patient's medications are strongly associated with hemolysis
18

-
19

21
22
23
24
(Choice D) Undiagnosed hypothyroidism is common in elderly patients and can cause a number
of nonspecific symptoms including fatigue, depression, and mental status abnormalities. It can
also cause anemia, but the anemia of hypothyroidism is generally mild and less likely to be
apparent on examination than iron deficiency anemia.

25 (Choice F) Although biochemical vitamin 812 deficiency, as defined by low serum 812 levels, is
26 relatively common, overt anemia due to 812 deficiency is significantly less common. The
27 clinician may wish to measure 812 levels, especially in a patient with macrocytic
28 anemia. However, iron deficiency anemia is more common in elderly patients taking a
29 combination of naproxen and aspirin
30
31 Educational objective:
32 Nonsteroidal anti-inflammatory drugs are a common cause of iron deficiency anemia, often
33 through chronic blood loss from the gastrointestinal tract I ron deficiency anemia should prompt
34
a thorough evaluation fo r the cause as early low-grade bleeding can herald later catastrophic
35
hemorrhage. Elderly patients often have a low-grade chronic anemia at baseline and may not
36
37 tolerate additional blood loss.
38
39 Time Spent 2 seconds Copyright © UWorld Last updated: [10/ 11/2016)
40
2
Item : 2 1 of40 ~'?Mark <J C> 61 ~ ~ , ~
0. ld : 2969 Previous Next Lab Values Notes Calculator Reverse Color Text Zoom
3
4
5
6 A 45-year-old male with history of cirrhosis related to hepatitis C infection presents with
7 abdominal distention and exertional shortness of breath. He had an episode of variceal bleeding
8 one yea r ago. He admits drinking alcohol occasionally He takes high doses of furosemide and
9 spironolactone. He also takes nadolol. He has no history of stroke or heart disease. His blood
10 pressure is 98/65 mmHg and heart rate is 55/min. Physical examination reveals decreased
11 breath sounds at the right lung base, significant abdominal distention with flank dullness, and 2+
12
peripheral edema. Periumbilical venous distention is appreciated He has no jaundice or hand
13
14 tremor . His short term memory is intact On chest x-ray , there is a moderate-to-severe right
15 pleural effusion and a small left pleural effusion. Right-sided thoracentesis is performed and
16 demonstrates a transudate. W hich of the following is the best management option fo r this patient?
17
18
e> A. Chest tube placement
19

-
20

22
23
24
25
e; B. Nitrates and hydralazine
e> C. Pleurodesis with doxycycline
e; D. Surgical porto-systemic shunt
(:) E. Transjugular intrahepatic portosystemic shunt (TIPS)
26
27
28 Subm it
29
30
31
32
33
34
35
36
37
38
39
40
2
3
4 E. Transjugular intrahepatic portosystemi c shunt (TIPS) [61 %)
5
6
7 Explanation:
8
9 Patients with cirrhosis and portal hypertension frequently have abdominal ascites and peripheral
10 edema secondary to low albumin levels and resulting abnormal extracellular fluid volume
11 regulation A small number of these patients may also develop pleural effusions which are not
12 secondary to an underlying ca rdiac or pulmonary abnormality; this is often referred to as hepatic
13 hydrothorax. Hepatic hydrothorax generally results in transudative pleural effusions and is much
14
more common on the right side. It is thought to occur secondary to small defects in the
15
16 diaphragm which permit abdominal ascites fluid to pass into the pleural space The best option
17 for treatment is liver transplantation, although depending on other factors this may not be an
18 option fo r all patients The primary treatment for most patients is a therapeutic thoracentesis
19 followed by a salt restricted diet and diuretics. However , this patient has already been on

-
20

22
23
24
25
diuretics and yet a moderate to large pleural effusion has still developed The next best treatment
for this patient would be a transjugular intrahepatic portosystemic shunt (TIPS)

{Choice A) Chest tube placement is not recommended as the pleural fluid will reaccumulate and
may result in large volumes of fl uid loss and electrolyte depletion

26 (Choice B) The combination of nitrates and hydralazine is sometimes used as a treatment for
27 systolic heart failure. However , hepatic hydrothorax by definition occurs in patients without
28 underlying cardiac dysfunction to account for their pleural effusion.
29
30 (Choice C) W hile pleurodesis is commonly used to treat recurrent malignant pleural effusions,
31
there is little evidence for its efficacy in the treatment of hepatic hydrothorax It could be
32
considered as an alternative option in patients where a TIPS procedure is contraindicated.
33
34
(Choice D) Surgical porto-systemic shunts are rarely performed as TIPS placement has a lower
35
36 morbidity
37
Educational objective:
38
Hepatic hydrothorax is a cause of transudative pleural effusions in patients with cirrhosis who
39
40
. .. .. .. . . ... .. ... . . ... ... ... . .. ... . .. .. .. . .. .. ... .. .... .. . ... . .
~ Feedback SuWend EnQock
2
Item : 2 1 of40 ~'?Mark <] C> !I ~ ~ , ~
0. ld : 2969 Previous Next Lab Values Notes Calculator Reverse Color Text Zoom
3
4 edema secondary to low albumin levels and resulting abnormal extracellular fluid volume
5 regulation A small number of these patients may also develop pleural effusions which are not
6 secondary to an underlying ca rdiac or pulmonary abnormality; this is often referred to as hepatic
7 hydrothorax. Hepatic hydrothorax generally results in transudative pleural effusions and is much
8 more common on the right side. It is thought to occur secondary to small defects in the
9 diaphragm which permit abdominal ascites fluid to pass into the pleural space. The best option
10 for treatment is liver transplantation, although depending on other factors this may not be an
11 option for all patients. The primary treatment for most patients is a therapeutic thoracentesis
12
followed by a salt restricted diet and diuretics. However, this patient has already been on
13
14 diuretics and yet a moderate to large pleural effusion has still developed The next best treatment
15 for this patient would be a transjugular intrahepatic portosystemic shunt (TIPS)
16
17 (Choice A) Chest tube placement is not recommended as the pleural fl uid will reaccumulate and
18 may result in large volumes of fluid loss and electrolyte depletion
19

-
20 {Choice B) The comb ination of nitrates and hydralazine is sometimes used as a treatment for
systolic heart fa ilure. However, hepatic hydrothorax by definition occurs in patients without
22 underlying cardiac dysfunction to account for their pleural effusion.
23
24 (Choice C) W hile pleurodesis is commonly used to treat recurrent malignant pleural effusions,
25 there is little evidence for its efficacy in the treatment of hepatic hydrothorax It could be
26 considered as an alternative option in patients where a TIPS procedure is contraindicated.
27
28 (Choice D) Surgical porto-systemic shunts are rarely performed as TIPS placement has a lower
29 morbidity
30
31 Educational objective:
32 Hepatic hydrothorax is a cause of transudative pleural effusions in patients with cirrhosis who
33 have no underlying card iac or pulmonary disease to account for development of such an
34
effusion. Hepatic hydrothorax usually results in a right-sided pleural effusion. Initial treatment is
35
usually with salt restriction and diuretics. TIPS placement is considered in patients with
36
37 refractory hepatic hydrothorax
38
39 Time Spent 3 seconds Copyright © UWorld Last updated: [08/ 15/2016)
40
2
Item: 22 of 40 ~'?Mark <J C> 61 ~ ~ , ~
0. ld : 4595 Previous Next Lab Values Notes Calculator Reverse Color Text Zoom
3
4
5
6 A 34-year-old woman comes to the physician with daily crampy abdominal pain for the last 2
7 years The pain sometimes occurs after meals but is not always preceded by eating It is often
8 accompanied by passage of small, loose stools and mucus, after which she feels significantly
9 better. The patient reports that these symptoms interfere with her daily activities, including work.
10 She has no fever, weight loss, or blood in her stools. She takes over-the-counter medications for
11 gas and constipation. Her mother died of colon cancer at age 65. W hich of the following is the
12
most likely finding on further evaluation of this patient?
13
14
15 r() A Crypt abscesses
16
17 6 B. Duodenal ulcer
18 (() C. Folic acid deficiency
19
20 6 D. Intestinal villous atrophy

-
21

23
24
25
26
(() E. Normal colonic mucosa
6 F. Perianal fistula

Submit
27
28
29
30
31
32
33
34
35
36
37
38
39
40
2
Item: 22 of 40
0. ld : 4595
~'?Mark <]
Previous
C>
Next
a
Lab Values
~
Notes
~
Calculator
,
Reverse Color
~
Text Zoom
3
4
5 Explanation:
6
7
8
9 Clinical features of irritable bowel syndrome
10
11
12 Recurrent abdominal pain/discomfo rt 2:3 days/mo nth for the
13 past 3 mo nths & 2:2 of t he follow ing:
Ro me
14
diag nostic • Symptom improvement with bowel movement
15
16 c riteria • Change in frequency of stool
17 • Change in form of stool
18
19
20 Signs/sympto ms suggesting etio logies other t han IBS

-
21

23
24
25
26
Warning
signs/
symptoms
• Rectal bleeding
• Nocturnal (awakens from or prevents sleep) or worsening
abdominal pain
• Weight loss

27 • Abnormal laboratory findings (eg, anemia & electrolyte disorders)


28
©UWo~d
29
30 This patient most likely has irritable bowel syndrome (ISS), a functional disorder of the
31
gastrointestinal tract (with no identifiable organic cause) It is the most common gastrointestinal
32
33 diagnosis in North America, with a prevalence of 10%-15%. IBS presents most commonly in
34 young women as chronic, crampy abdominal pain with alternating episodes of constipation and
35 diarrhea. Passage of stool often relieves the pain It can also present with nonspecific symptoms
36 such as gastroesophageal refl ux, dysphagia, early satiety, and chest pain IBS is further
37 subclassified as diarrhea-predominant, constipation-predominant, or mixed. IBS was previously
38 considered a diagnosis of exclusion. However, patients with IBS symptoms based on the ROME
39 Ill criteria, no alarm features, and no family history of inflammatory bowel disease or colorectal
40
2
Item : 22 of 40 ~'?Mark <] C> !I ~ ~ , ~
0. ld : 45 9 5 Previous Next Lab Values Notes Calculator Reverse Color Text Zoom
3 U g g y p I I g, g I I p g
4
5
loss, and abnormal laboratory studies (eg, anemia, elevated inflammatory markers) do not
6 suggest IBS. These require further investigation to rule out other etiologies.
7
8 (Choice A) Crypt abscesses are characteristic for ulcerative colitis, which presents with fever ,
9 colicky abdominal pain, chronic diarrhea with bloody stool, and we ight loss.
10
11 {Choice B) A duodenal ulcer typically presents with epigastric pain, gastrointestinal bleeding,
12 and anemia. It is not a common cause of diarrhea.
13
14 (Choice C) Folic acid deficiency can occur due to poor dietary intake, alcoholism, increased
15 demand (eg, pregnancy), or as a medication side effect It does not usually cause abdominal
16 pain and diarrhea.
17
18 (Choice D) Intestinal villous atrophy is the hallmark of celiac disease, an autoimmune disease
19 resulting in gluten intolerance. Classic presentation includes gastrointestinal distress (eg,
20 diarrhea, abdominal distension), malabsorption, and nutritional deficiencies. Atypical

-
21

23
24
25
26
presentation includes minor abdominal complaints, iron deficiency anemia, increased
transaminases, arthritis, or neurologic symptoms Symptoms are strongly co rrelated to dietary
intake of gluten-containing foods .

(Choice F) Perianal fistulas suggest possible Grahn's disease. These patients typically have
abdominal pain, weight loss, and extraintestinal manifestations (eg, arthritis, skin disorder s,
27
28
nephrolithiasis)
29
Educational objective:
30
31 Irritable bowel syndrome is a functional disorder of the gastrointestinal tract characterized by
32 chronic abdominal pain with diarrhea and/or constipation Patients with symptoms consistent with
33 the ROME Ill criteria and no alarm features do not require extensive workup fo r diagnosis.
34
35 References:
36
37 1. Irritable bowel syndrome.
38
39
40
2
Item: 23 of 40 ~'?Mark <] C> !I ~ ~ , ~
0. ld : 2923 Previous Next Lab Values Notes Calculator Reverse Color Text Zoom
3
4
5
6 A 22-year-old male college student of Middle Eastern origin presents to clinic complaining of
7 insidious onset yellowing of his skin that was first noted by his older sister yesterday . He has no
8 other complaints and has an unremarkable medical history He states that he has been fasting for
9 the past four days fo r religious reasons. Laboratory evaluation reveals the following :
10
11 Live r studies
12
Total bilirubin 3.4 mg/ dL
13
14 Indirect bilirubin 2.8 mg/ dL
15 Aspartate aminotransferase (SGOT) 32 U/L
16 Alanine aminotransferase (SGPT) 40 U/L
17
18 Complete blood count
19 Hemoglobin 13.5 g/L
20
Erythrocyte count 5.2 mln/mm3
21

--
22 MCHC 32%
MCV 89 fl
24 Reticulocytes 1%
25 Platelets 240,000/mm3
26 Leukocyte count 7,100/mm3
27
28 Hepatitis Panel
29
30 HBsAg Negative
31 Anti-HBsAg Positive
32 Anti-HBcAg Negative
33 Anti-HAY lgM Negative
34 Anti-HAV lgG Negative
35 Anti-HCV Negative
36
37
W hich of the following is the most likely diagnosis in this patient?
38
39
40
2
Item : 23 of 40 ~'?Mark <J C> 61 ~ ~ , ~
0. ld : 2923 Previous Next Lab Values Notes Calculator Reverse Color Text Zoom
3 - .. I I I I I
4 Erythrocyte count 5.2 mln/mm3
5
MCHC 32%
6
7 MCV 89 f1
8 Reticulocytes 1%
9 Platelets 240,000/mm3
10 Leukocyte count 7,100/mm3
11
12 Hepatitis Panel
13
14 HBsAg Negative
15 Anti-HBsAg Positive
16 Anti-HBcAg Negative
17 Anti-HAV lgM Negative
18 Anti-HAV lgG Negative
19 Anti-HCV Negative
20
21

--
Wh ich of the following is the most likely diagnosis in this patient?
22

24 <0 A Crigler-Nanar syndrome type 1


25
<0 B. Crigler-Nanar syndrome type 2
26
27 <0 C . Gilbert's syndrome
28 <0 D. Hereditary spherocytosis
29
30 <0 E. Rotor syndrome
31 <0 F. G6PD deficiency anemia
32
33 <0 G. Autoimmune hepatitis
34 <0 H. Acute hepatitis B infection
35
36
<0 I. Wilson's disease
37
38 Submit
39
40
2
Item: 23 of 40 ~'?Mark <] C> !I ~ ~ , ~
0. ld : 2923 Previous Next Lab Values Notes Calculator Reverse Color Text Zoom
3
4
5 Explanation:
6
7 The hepatic metabolism of bilirubin occurs in the following four stages: uptake from the
8 bloodstream; storage within the hepatocyte; conjugation with glucuronic acid; and biliary
9 excretion. In the normal individual, serum total bilirubin is 0.2-1 mg/dl, of which < 0.2 mg/dl is
10 the direct fraction. Typically, elevated conjugated bilirubin levels are suggestive of hepatobiliary
11 disease (eg, cirrhosis or hepatitis) because the bilirubin conjugates will reflux back into the
12
plasma when the secretion of conjugated bilirubin into the bile is slowed. In contrast, elevated
13
14 unconjugated bilirubin levels typ ically indicate an increased level of bilirubin fo rmation (such as
15 that seen in hemolys is) or a slowing in bilirubin conjugation (such as that seen in this patient, who
16 has Gilbert's syndrome)
17
18 Gilbert's syndrome is a fam ilial disorder of bilirubin glucuronidation in which the production of
19 UDP glucuronyl transferases (enzymes that mediate glucuronidation of various substances) is
20 reduced. The genetic defect is mapped to the promoter region of the UDP glucuronyl transferase
21 gene. Approximately 9% of individuals in Western countries are homozygous for this mutation,

--
22

24
25
26
27
with another 30% heterozygous and asymptomatic

Clinical manifestations of Gilbert's syndrome include icterus secondary to a mild, predominantly


unconjugated hyperbilirubinemia (normal levels in these patients are < 3 mg/dl) Those patients
who are symptomatic tend to have nonspecific complaints, including malaise, fatigue, or
abdominal discomfort. Certain events, such as hemolysis, fasting or consuming a fat-free diet,
28
29 physical exertion, febrile illness, stress, or fatigue are thought to be triggers for
30 hyperbilirubinemia in patients with Gilbert's syndrome.
31
32 The diagnosis is suggested in those patients with no apparent liver disease who have mild
33 unconjugated hyperbilirubinemia thought to be provoked by one of the classic triggers
34 Presumptive diagnosis can be made when the unconjugated hype rbilirubinemia persists with
35 repeat testing, but liver function tests, complete blood count, blood smear, and reticulocyte count
36 are normal.
37
38 Treatment is generally considered to be unnecessary in patients with Gilbert's syndrome
39 However , its mode of inheritance should be discussed with patients as a means of preventing
40
2
Item: 23 of 40 ~'?Mark <] C> !I ~ ~ , ~
0. ld : 2923 Previous Next Lab Values Notes Calculator Reverse Color Text Zoom
3
4 However , its mode of inheritance should be discussed with patients as a means of preventing
5 needless testing in similarly affected fam ily members.
6
7 {Choice A) Crigler-Nanar syndrome type 1 is an autosomal recessive disorder of bilirubin
8 metabolism characterized by severe jaundice and neurologic impairment due to kernicterus
9 (bilirubin encephalopathy) In these infants, the indirect bilirubin levels are typically 20-25 mg/dl
10 but can rise to as high as 50 mg/dl Live r enzymes and histology are normal. If intravenous
11 phenobarbital is administered, the serum bilirubin remains unchanged Phototherapy or
12 plasmapheresis are typically helpful in the short term, though liver transplant is the only curative
13 option
14
15 {Choice B) Crigler-Nanar syndrome type 2 is a milder autosomal recessive disorder of bilirubin
16 metabolism characterized by lower serum bilirubin levels (<20 mg/dl) and survival into adulthood
17
with no kernicterus or neurologic impairment Live r enzymes and histology are normal. If
18
19 intravenous phenobarbital is administered, the serum bilirubin is reduced. Treatment is often
20 unnecessary in milder cases, though periodic administration of phenobarbital or clofibrate can
21 reduce serum bilirubin levels if necessary

--
22

24
25
26
27
(Choices 0 and F) Chronic hemolysis can result in an unconjugated hyperbilirubinemia with
anemia. Triggers such as fasting or stress are not characteristic of hemolysis His hemoglobin
and reticulocyte count are with in normal limits. Since this patient is totally asymptomatic and it
appears his condition was brought on by fasting, chronic hemolysis is an unlikely diagnosis.

28 (Choice E) Roto r syndrome is a benign condition in which there is a defect of hepatic storage of
29 conjugated bilirubin, resulting in its leakage into the plasma Chronic and mild hyperbilirubinemia
30 of both unconjugated and conjugated fo rms develops, without any suggestion of hemolysis. Live r
31
function tests are normal, and treatment is unnecessary
32
33 Educational objective:
34
Gilbert's syndrome, Crigler-Nanar type 1, and Crigler-Nanar type 2 are three familial disorders
35
characterized by varying degrees of severity and unconjugated hyperbilirubinemia Know how to
36
37 diffe rentiate them.
38
39 Time Spent 5 seconds Copyright © UWorld Last updated: [06/24/2016)
40
2
Item : 24 of 40 ~'?Mark <] C> !I ~ ~ , ~
0. ld : 4278 Previous Next Lab Values Notes Calculator Reverse Color Text Zoom
3
4
5
6 A 50-year-old man is admitted to the hospital with worsening jaundice, anorexia, general malaise,
7 and right upper quadrant abdominal pain over the past 3 days. The patient's other medical
8 problems include major depression with an attempted suicide 30 years ago His temperature is
9 38.3 C (101 F), blood pressure is 130/86 mm Hg, pulse is 86/min, and respirations are 16/min.
10 Cardiopulmonary examination is unremarkable. Scleral icterus and tender hepatomegaly are
11 present. Laboratory results are as follows:
12
13 Complete blood count
14
15 Hemoglobin 12 g/dL
16
17 Mean co rpuscular
102 !Jm'
18 volume
19
Platelets 120,000/mm'
20
21 Leukocytes 13,000/mm'
22

--
23

25
26
27
28
Serum chemistry

Urea nitrogen 20 mg/dL

Creatinine 1.2 mg/dL


29
30
31
32 Liver function studies
33
34 Albumin 3.4 g/dL
35
Total bilirubin 5.3 mg/dL
36
37 D irect bilirubin 3.7 mg/dL
38
39 Alkaline phosphatase 105 U/L
40
2
Item : 24 of 40 ~'?Mark <J C> 61 ~ ~ , ~
0. ld : 4278 Previous Next Lab Values Notes Calculator Reverse Color Text Zoom
3 '- I • I I -

4
5 Albumin 3.4 g/dl
6
7 Total bilirubin 5.3 mg/dl
8
Direct bilirubin 3.7 mg/dl
9
10 Alkaline phosphatase 105 U/L
11
12 Aspartate
212 U/L
13 aminotransferase
14
15 Alanine
99 U/L
16 aminotransferase
17
18 Amylase 91 U/L
19
20
21 Coagulation studies
22

--
23

25
26
27
28
INR 1.2 (08-11 )

Which of the following would be the most helpful in establishing a diagnosis in this patient?

® A Family history
29
30 ® B. Medication history
31 ® C. Right upper quadrant ultrasound
32
33 ® D. Serum acetaminophen level
34 ® E. Substance abuse history
35
® F. Travel history to a developing country
36
37
38 Submit
39
40
2
Item : 24 of 40 ~'?Mark <] C> !I ~ ~ , ~
0. ld : 4278 Previous Next Lab Values Notes Calculator Reverse Color Text Zoom
3
4
5 Alcoholic hepatitis (AH) best explains this patient's presentation with fever , jaundice, anorexia,
6 tender hepatomegaly, mild (<300 U/L) elevation in aminotransferases (aspartate
7 aminotransferase [AST] and alanine aminotransferase [ALT]) with an AST:AL T ratio ~2:1 ,
8 macrocytic anemia (mean corpuscular volume [MCVJ >1 OO/!Jm3) , thrombocytopenia, and mild
9 elevation in the International Normalized Ratio. Alcohol use should be confi rmed and quantified
10 by first obtaining the patient's social history and discussing substance use. Patients with AH
11 commonly have a history of chronic , heavy alcohol use (>7 drinks/day) and sometimes develop
12
AH symptoms after an acute increase in consumption
13
14 Diagnosis is clinical and often does not require further studies in patients with consistent history
15
and laboratory results. Patients with risk factors fo r diseases such as acute viral hepatitis require
16
further investigation. Radiographic imaging may reveal fatty liver disease, cirrhosis, or
17
18 ascites. Treatment involves abstinence, supportive ca re (eg, hydration and nutrition support), and
19 acid suppression Liver biopsy can be helpful if there is diagnostic uncertainty
20
21 (Choice A) Family history can be helpful in identifying causes of inherited live r disease (eg,
22 W ilson disease, hemochromatosis); however, an AST:ALT ratio <::2 1 in the setting of acute

--
23

25
26
27
28
hepatitis (tender hepatomegaly , feve r) is highly suggestive of AH. In addition, this patient's age
makes inherited disease less likely

(Choice B) Methotrexate could cause elevated MCV, bone marrow suppression (anemia,
thrombocytopenia) , and liver function test abnormalities; however , an ASTALT ratio !::2 1 makes
AH more likely
29
30 (Choice C) Cholangitis can cause fever, abdominal pain, and jaundice {Charcot's triad) and
31 could be evaluated with a right upper quadrant ultrasound. However, cholangitis would typically
32 cause a more severe presentation (eg, sepsis) with a marked elevation in the alkaline
33
phosphatase level; in addition, an AST:ALT ratio <::2 1 with tender hepatomegaly and an elevated
34
35 MCV are atypical for cholangitis and more suggestive of heavy alcohol use with AH.
36
(Choices 0 and F) Acetaminophen intoxication and travel-associated diseases (eg, viral
37
38 hepatitis, malaria, leptospirosis) may present similarly; however , these conditions do not explain
39
40 ... the macrocytic anemia and ASTALT ratio <::2: 1. In addition, aminotransferase levels are typically
- - - - - ----- - -- - - - - - -- - -- -- -

~ Feedback SuWend EnQ ock


2
Item : 24 of 40 ~'?Mark <] C> !I ~ ~ , ~
0. ld : 4278 Previous Next Lab Values Notes Calculator Reverse Color Text Zoom
3
4 acid suppression Liver biopsy can be helpful if there is diagnostic unce rtainty
5
6 {Choice A) Family history can be helpful in identifying causes of inherited liver disease (eg,
7 W ilson disease, hemochromatosis); however , an AST ALT ratio ~2:1 in the setting of acute
8 hepatitis (tender hepatomegaly , fever) is highly suggestive of AH. In addition, this patient's age
9 makes inherited disease less likely.
10
11 (Choice B) Methotrexate could cause elevated MCV, bone marrow suppression (anemia,
12 thrombocytopenia), and liver function test abnormalities; however , an AST:ALT ratio ~21 makes
13 AH more likely
14
15 (Choice C) Cholangitis can cause fever , abdominal pain, and jaundice (Charcot's triad) and
16 could be evaluated with a right upper quadrant ultrasound. However , cholangitis would typically
17
cause a more severe presentation (eg, sepsis) with a marked elevation in the alkaline
18
19
phosphatase level; in addition, an AST ALT ratio ~2 1 with tender hepatomegaly and an elevated
20 MCV are atypical for cholangitis and more suggestive of heavy alcohol use with AH.
21
22 (Choices D and F) Acetaminophen intoxication and travel-associated diseases (eg, viral

--
23 hepatitis, malaria, leptospirosis) may present similarly; however, these conditions do not explain
the macrocytic anemia and AST:ALT ratio ~2 : 1. In addition, aminotransferase levels are typically
25 >1 ,000 U/ L in acetaminophen intoxication and viral hepatitis.
26
27 Educational objective:
28 Alcoholic hepatitis (AH) is most commonly seen in patients with a history of chronic heavy alcohol
29 use. AH is characterized by fever , jaundice, anorexia, and tender hepatomegaly with laboratory
30 results showing an aspartate aminotransferase to alanine aminotransferase ratio of ~2 : 1.
31
32
References:
33
34 1. Alcoholic hepatitis: Challenges in diagnosis and management.
35
36 2. Diagnosis and management of alcoholic liver disease.
37
38
39 Time Spent 6 seconds Copyright © UWorld Last updated: [06/ 14/2016)
40
2
Item: 25of40 ~'?Mark <] C> !J ~ ~ , ~
0. ld : 2596 Previous Next Lab Values Notes Calculator Reverse Color Text Zoom
3
4
5 A 44-year-old white male comes to the office and says, "I have had this persistent, upper
6 abdominal pain for the past 2 months. My wife has peptic ulce r, and I think I also have an ulcer
7 down there, so I take some of her medicines. At first, the medicines made the pain go away , but
8 soon after the pain proved to be too tough for the medicines. I hardly want to eat anything now.
9
Do you think all this is due to some reaction to those medicines?" Physical examination reveals
10
mild tenderness in the epigastric region Esophagogastroduodenoscopy reveals a mass in the
11
12 pylorus, which the biopsy identifies as an infiltrating, low-grade, gastric mucosa-associated
13 lymphoid tissue (MALT) lymphoma Further work-up concludes that there are no regional lymph
14 nodes involved. W hat is the best next step in the management of this patient?
15
16 C) A Radical gastrectomy with en bloc resection of spleen and regional nodes
17 C) B. Give a comb ination of omeprazole, clarithromyc in, and amoxicillin.
18
19 C) C. Give combination chemotherapy
20 C) D. Radiotherapy
21
22 C) E. Observation and strict surveillance
23

-
24

26
27
28
29
Subm it

30
31
32
33
34
35
36
37
38
39
40
2
Item: 25of40 ~'?Mark <J C> 61 ~ ~ , ~
0. ld : 2596 Previous Next Lab Values Notes Calculator Reverse Color Text Zoom
3
4 C. Give combination chemotherapy [18%]
5 D. Radiotherapy [9%]
6
7 E. Observation and strict surveillance [4%]
8
9
Explanation:
10
11 Various studies have suggested an important role for Helicobacter pylori infection in the
12
pathogenesis of low-grade gastric mucosa-associated lymphoid tissue (MALT) lymphoma. Such
13
14 lymphomas may regress after the eradication of H. pylori using antibiotic therapy Although
15 some recent studies have shown certain pitfalls in this strategy, antibiotic therapy still remains as
16 the most accepted and recommended management of gastric MA LT lymphoma without any
17 metastasis.
18
19 Chemotherapy typically plays a role in the management of patients with MALT lymphoma if
20 eradication of H. Pylori fails to produce regression of the lymphoma Such therapeutic regimens
21 usually include CHOP (cyclophosphamide, adriamycin, vincristine, and prednisone) or CHOP+
22 Bleomycin
23

-
24

26
27
28
29
{Choice A) In the past, radical gastrectomy with en bloc resection of the spleen and regional
nodes was performed after making the diagnosis of MALT lymphoma; however , studies have
shown that this approach is associated with a high operative mortality

(Choice D) The role of radiotherapy in the management of gastric lymphoma is controversiaL


Currently , radiotherapy is mostly used as an adjuvant to other regimens.
30
31 (Choice E) Observation is certainly not recommended in a patient with gastric lymphoma
32
33 Educational Objective:
34
Antibiotic therapy is the most accepted and recommended management fo r the eradication of H.
35
pylori in patients with gastric mucosa-associated lymphoid tissue (MA LT) lymphoma without any
36
37 metastasis.
38
39 Time Spent 2 seconds Copyright © UWorld Last updated: [04/ 18/2016]
40
2
Item: 26 of 40 ~'?Mark <J C> 61 ~ ~ , ~
0. ld : 3602 Previous Next Lab Values Notes Calculator Reverse Color Text Zoom
3
4
5
6 A 23-year-old man comes to the physician fo r frequent, foul-smelling, bulky stools and weight loss
7 over the last 6 months. He has lost about 4.5 kg ( 10 lb) over the past 3 months. He also has poor
8 energy and occasional joint pains The patient has no previous medical problems and takes no
9 medication. He does not use tobacco, alcohol, or illicit drugs Vital signs are normal.
10 Examination shows mild pallor There is no hepatosplenomegaly or lymphadenopathy
11 Hemoglobin is 10.2 g/dl and serum ferritin is 10 ng/ml. Immunoglobulin A anti-tissue
12
transglutaminase antibody screening is negative Small-bowel biopsy shows villous atrophy
13
14 W hich of the following is the most likely cause of this patient's symptoms?
15
16 6 A Aplastic anemia
17
18 e; B. Celiac disease
19 6 C. Collagenous colitis (mic roscopic colitis)
20
21 e; D. Grahn's disease
22 6 E. Irritable bowel syndrome
23
24 e; F. Lactose intolerance

-
25

27
28
29
30
Submit

31
32
33
34
35
36
37
38
39
40
2
Item: 26 of 40 ~'?Mark <] C> !I ~ ~ , ~
3
4
0. ld : 3602
. '
Previous Next Lab Values Notes Calculator Reverse Color Text Zoom

5
6 Explanation:
7
8
9
10
11
12 This patient's clinical presentation is highly suggestive of celiac disease, with multiple features of
13 malabsorption and characteristic abnormalities on biopsy of the small bowel. The diagnosis of
14
celiac disease is highly co rrelated with positive results on serological studies, primarily lgA
15
anti-tissue transglutaminase and lgA anti-endomysia! antibodies. However , many patients
16
17 with biopsy-confi rmed celiac disease will have negative results on lgA antibody testing due to an
18 associated selective lgA deficiency, which is common in celiac disease. If lgA serology is
19 negative but the suspicion for celiac disease is high, total lgA should be measured (or lgG-based
20 serologic testing should be done).
21
22 {Choice A) Aplastic anemia is an uncommon cause of anemia. In addition to symptoms of a low
23 red cell count, patients with aplastic anemia may develop bleeding (due to thrombocytopenia) and
24 infection (due to leukopenia) Malabsorption is not a characteristic of aplastic anemia, and

-
25

27
28
29
30
ferritin is usually normaL

(Choice C) Collagenous colitis is an uncommon disorder producing chronic watery diarrhea.


The colon is frequently involved, but colonoscopy shows normal mucosa. Biopsy shows mucosal
subepithelial collagen deposition
31 (Choice D) This patient has a number of features, including fatigue, diarrhea, and weight loss,
32
that could be consistent with Crohn's disease. Malabsorption is less common in Crohn's disease
33
34 and is usually due to multiple bowel resections. Patients with the disease will show inflammatory
35 signs on biopsy rather than the typical features of villous atrophy seen in this patient
36
37 (Choice E) Irritable bowel syndrome is a chronic disorder characterized by distressing
38 gastrointestinal symptoms such as diarrhea, constipation, pain, and bloating Biopsy results are
39 normal in irritable bowel syndrome, and features of malabsorption should prompt a search for an
40

~
-------------------------------------------------------------------------------------------------------------------------------
Feedback SuWend EnQock
2
Item: 26 of 40 ~'?Mark <] C> !I ~ ~ , ~
0. ld : 3602 Previous Next Lab Values Notes Calculator Reverse Color Text Zoom
3
4 (Choice A) Aplastic anemia is an uncommon cause of anemia. In addition to symptoms of a low
5 red cell count, patients with aplastic anemia may develop bleeding (due to thrombocytopenia) and
6
infection (due to leukopenia). Malabsorption is not a characteristic of aplastic anemia, and
7
ferritin is usually normaL
8
9
{Choice C) Collagenous colitis is an uncommon disorder producing chronic watery diarrhea.
10
11 The colon is frequently involved, but colonoscopy shows normal mucosa. Biopsy shows mucosal
12 subepithelial collagen deposition
13
14 (Choice D) This patient has a number of features, including fatigue, diarrhea, and weight loss,
15 that could be consistent with Crohn's disease. Malabsorption is less common in Crohn's disease
16 and is usually due to multiple bowel resections. Patients with the disease will show inflammatory
17 signs on biopsy rather than the typical features of villous atrophy seen in this patient
18
19 (Choice E) Irritable bowel syndrome is a chronic disorder characterized by distressing
20 gastrointestinal symptoms such as diarrhea, constipation, pain, and bloating Biopsy results are
21 normal in irritable bowel syndrome, and features of malabsorption should prompt a search for an
22 alternate explanation
23
24 (Choice F) Lactose intolerance is a common cause of diet-related diarrhea. Symptoms are due

-
25

27
28
29
30
to impaired digestion and absorption of lactose with resulting delive ry of a sugar load to the
colon. Malabsorption of other nutrients, however, is not typicaL

Educational objective:
Celiac disease should be suspected in any patient with malabsorption and iron deficiency
31 anemia. lgA anti-endomysia! and anti-tissue transglutaminase antibodies are highly predictive of
32 celiac disease but may be absent if there is concurrent selective lgA deficiency
33
34 References:
35
36 1. Hematologic manifestations of celiac disease.
37
38
39 Time Spent 2 seconds Copyright © UWorld Last updated: [06/ 12/2016)
40
2
Item: 27 of 40 ~'?Mark <J C> 61 ~ ~ , ~
0. ld: 2149 Previous Next Lab Values Notes Calculator Reverse Color Text Zoom
3
4
5
6 An 83-year-old woman comes to the office due to a 1-year history of progressively severe,
7 crampy abdominal pain that occurs immediately after she eats. The pain is diffuse but more
8 pronounced in the epigastric area and is associated with frequent bloating and nausea and
9 occasional diarrhea. The patient has had a 15-kg (33-lb) weight loss over the past year She
10 has lost her appetite since the pain episodes began Her other medical problems include
11
hypertension, type 2 diabetes mellitus, hypercholesterolemia, peripheral vascular disease, and
12
coronary artery disease. Three years ago, she suffered an infe rior wall myocardial infa rction,
13
14
which was treated with a drug-eluting stent The patient has a 40-pack-year history of smoking
15 but quit 10 years ago She occasionally drinks a glass of wine. Temperature is 36.8 C (98 2 F) ,
16 blood pressure is 140/92 mm Hg, pulse is 80/min, and respirations are 14/min. BMI is 24 kg/m2
17 Abdomen is soft, nontender , and nondistended. Bowel sounds are present Abdom inal x-rays
18 reveal no abnormalities. W hich of the following is the most likely cause of this patient's current
19 presentation?
20
21
22 ro A Atherosclerosis of the mesenteric arteries
23 1'0 B. Intermittent increase of intragallbladder pressure
24

...
25
26

28
29
ro C. Left ventricular aneurysm
1'0 D. Progressive pancreatic inflammation
1'0 E. Selective hypersensitization of visceral afferent nerves
1'0 F. Transmural inflammation of the small intestine
30
31
32 Submit
33
34
35
36
37
38
39
40
2
Item: 27 of 40 ~'?Mark <] C> !I ~ ~ , ~
0. ld: 2149 Previous Next Lab Values Notes Calculator Reverse Color Text Zoom
3
4
5 Explanation:
6
7
8
9
10
11 This patient with a history of vascular disease most likely has chronic mesenteric ischemia
12 (CMI) CMI commonly presents with crampy , postprandial epigastric pain (intestinal angina),
13 food aversion, and weight loss. Patients may also report nausea, early satiety, and diarrhea.
14
The anginal pain frequently starts within the first hour of eating and slowly resolves over the next 2
15
hours. The pathophysiology of the pain is most likely related to shunting of blood away from the
16
17 small intestine to meet the increased demand of the stomach. In patients with atherosclerosis,
18 the celiac or the superior mesenteric arteries may be narrowed and unable to dilate appropriately
19 to maintain adequate blood flow to the intestines.
20
21 Physical examination may show signs of malnutrition and may reveal an abdominal bruit in
22 - 50% of patients, but can be otherwise unremarkable. Although abdominal x-ray and CT scans
23 may demonstrate calcified vessels, diagnosis requires better visualization of the vessels. CT
24 angiography is the preferred choice, although Doppler ultrasonography may also be helpful

...
25
26

28
29
Treatment involves risk reduction (eg, tobacco cessation), nutritional support, and
revascularization .

(Choice B) Biliary colic occurs as the gallbladder contracts against an obstructing stone,
transiently raising intragallbladder pressure The pain is often localized to the upper right
30
quadrant and does not occur after every meal Significant weight loss is very rare.
31
32
(Choice C) Ventricular aneurysms (scarred myocardium following transmural myocardial
33
34 infarction) predispose to acute mesenteric ischemia (rather than CMI) due to thrombus formation
35 and embolization. The presentation of acute mesente ric ischemia is abrupt, with severe
36 abdominal pain out of proportion to examination findings. Both in pathophysiology and acuity,
37 acute mesenteric ischemia is analogous to acute myocardial infarction whereas CMI is similar to
38 stable cardiac angina
39
40
2
Item: 27 of 40 ~'?Mark <] C> !I ~ ~ , ~
0. ld: 2149 Previous Next Lab Values Notes Calculator Reverse Color Text Zoom
3
4
5 (Choice B) Biliary colic occurs as the gallbladder contracts against an obstructing stone,
6 transiently raising intragallbladder pressure. The pain is often localized to the upper right
7 quadrant and does not occur after every meal. Significant weight loss is very rare.
8
9 (Choice C) Ventricular aneurysms (scarred myocardium following transmural myocardial
10 infa rction) predispose to acute mesenteric ischemia (rather than CMI) due to thrombus fo rmation
11 and embolization. The presentation of acute mesenteric ischemia is abrupt, with severe
12 abdominal pain out of proportion to examination findings . Both in pathophysiology and acuity,
13 acute mesenteric ischemia is analogous to acute myocardial infa rction whereas CMI is similar to
14
stable cardiac angina
15
16 (Choices D and F) Pain accompanied by malabsorption may be due to chronic pancreatitis or
17
Crohn disease (transmural intestinal inflammation) However, these diseases frequently produce
18
abnormal CT scan and x-ray findings (eg, pancreatic calcifications). In addition, this patient
19
20 does not have risk factors fo r pancreatitis (eg, heavy alcohol use) Crohn disease generally
21 presents at an earlier age and usually causes lower right quadrant pain.
22
23 (Choice E) Irritable bowel syndrome does not usually lead to significant anorexia and weight
24 loss; if present, these features should prompt the consideration of other diagnoses

...
25
26

28
29
Educational objective:
The diagnosis of chronic mesenteric ischemia should be suspected in patients with unexplained
chronic abdominal pain, weight loss, and food aversion. Most cases are due to atherosclerotic
changes of the celiac or superior mesenteric arteries.
30
31
Refer ences:
32
33 1. Chronic ischaemic bowel diseases in the aged--go with the flow.
34
35
2. Mesenteric ischemia: Pathogenesis and challenging diagnostic and therapeutic
36 modalities.
37
38
39 Time Spent 2 seconds Copyright© UWorld Last updated: [05/31 /2016)
40
2
Item: 28 of 40 ~'?Mark <] C> !I ~ ~ , ~
0. ld : 2929 Previous Next Lab Values Notes Calculator Reverse Color Text Zoom
3
4
5
6 A 44-year-old woman comes to the emergency department with severe epigastric pain that
7 radiates to her back. The pain was sudden in onset and has remained steady for the past several
8 hours. She has associated nausea and vom iting. Past medical history is notable fo r a surgically
9 repaired femo ral hernia and appendectomy as a child. The patient takes no medications and
10 does not use tobacco, alcohol, or recreational drugs Temperature is 37.2 C (98 9 F) , blood
11 pressure is 112/70 mm Hg, pulse is 98/min, and respirations are 18/min. BMI is 35 kg/m2
12
Examination shows an agitated woman who sits leaning fo rward on the bed. Her skin has no
13
14 evidence of icterus or excoriations. Lungs are clear to auscultation and heart sounds are
15 normal. There is moderate tenderness in the epigastrium but no guarding, rebound tenderness,
16 or hepatosplenomegaly . Bowel sounds are normal. Laboratory results are as follows:
17
Serum chemistries
18
19 Albumin 4.8 g/dL
20
21 Total bilirubin 1.0 mg/dL
22
23 Alkaline phosphatase 271 U/L
24
25 Aspartate aminotransferase
133 U/L
26 (SGOT)

-
27

29
30
31
32
Alanine aminotransferase
(SGPT)

Amylase
172 U/L

2610 U/L

Lipase 3880 U/L


33
34 Triglycerides 110 mg/dL
35
36 Calcium 8.9 mg/dL
37
38
39 Complete blood count
40
2
Item: 28 of 40 ~'?Mark <J C> 61 ~ ~ , ~
0. ld : 2929 Previous Next Lab Values Notes Calculator Reverse Color Text Zoom
3 -~ - ..... - .. -
172 U/L
4 (SGPT)
5
6 Amylase 2610 U/L
7
8 Lipase 3880 U/L
9
Triglyce rides 110 mg/dl
10
11 Calcium 8.9 mg/dl
12
13
14
15 Complete blood count
16
17 Hemoglobin 12.6 g/L
18
Mean co rpuscular volume 90 1Jm3
19
20 Platelets 340,000/mm>
21
22 Leukocytes 14, 100/mm>
23
24
W hich of the following is the best next step in evaluating the underlying etiology for this patient's
25
26 acute condition?

-
27

29
30
31
32
® A CT scan of the abdomen
® B. Hepatobiliary iminodiacetic acid scan
® C. Radiograph of the abdomen
-

33 ® D. Right upper-quadrant ultrasound


34 ® E. Serologic testing fo r viral titers
35
® F. Stool elastase content
36
37
38 Submit
39 T

40
2
Item: 28 of 40 ~'?Mark <] C> !I ~ ~ , ~
0. ld : 2929 Previous Next Lab Values Notes Calculator Reverse Color Text Zoom
3
4
5
6
7
This patient has pancreatitis given her epigastric abdominal pain radiating to the back and the
8
amylase/lipase levels >3 times the upper limit of normal (ULN) Gallstone pancreatitis is most
9
10 likely as she does not use alcohol, takes no medications, has normal triglyceride and calcium
11 levels, and has had no recent major illness. Her high BMI, alanine aminotransfer ase >150 U/l ,
12 and elevated alkaline phosphatase also suggest gallstone pancreatitis
13
14 A right upper-quadrant ultrasound is advised fo r all patients with suspected gallstone pancreatitis
15 as it provides the most accurate information regarding the presence of gallstones If the
16 ultrasound is nondiagnostic and there is high clinical suspicion for common bile duct disease,
17 endoscopic retrograde cholangiopancreatography (ERCP) may be performed to better visualize
18 the biliary tree.
19
20 (Choices A and C) CT scan is not required to diagnose pancreatitis in patients (such as this
21 one) with typ ical abdominal pain and amylase/lipase levels >3 times ULN. It can be used to
22 confi rm the diagnosis and identify complications (eg, pancreatic hemorrhage or necrosis) in
23
24 patients without these classic findings (eg, atypical abdominal symptoms, amylase/lipase elevation
25 <3 times ULN). However, CT is not as sensitive as ultrasound for detecting gallstones Plain
26 radiographs also have a very low sensitivity for gallstones

-
27

29
30
31
32
{Choice B) Hepatobiliary iminodiacetic acid (HIDA) scan (cholescintigraphy) is used to
diagnose acute cholecystitis if ultrasound does not provide a clear diagnosis. It is not routinely
used to evaluate pancreatitis.

(Choice E) Viruses (eg, hepatitis B, cytomegalovirus, va ricella-zoster, HIV) are a less likely
33 cause of pancreatitis in this patient Serologic studies can be deferred unless standard testing
34 does not identify the cause.
35
36 (Choice F) Stool elastase is a marker for pancreatic exocrine function. Low levels would be
37 seen in chronic , rather than acute, pancreatitis.
38
39 Educational objective:
40
2
Item: 28 of 40 ~'?Mark <] C> !I ~ ~ , ~
0. ld : 2929 Previous Next Lab Values Notes Calculator Reverse Color Text Zoom
3 - -
4 endoscopic retrograde cholangiopancreatography (ERCP) may be performed to better visualize
5 the biliary tree.
6
7 (Choices A and C) CT scan is not required to diagnose pancreatitis in patients (such as this
8 one) with typical abdominal pain and amylase/lipase levels >3 times ULN. It can be used to
9 confirm the diagnosis and identify complications (eg, pancreatic hemorrhage or necrosis) in
10 patients without these classic findings (eg, atypical abdominal symptoms, amylase/lipase elevation
11 <3 times ULN). However, CT is not as sensitive as ultrasound for detecting gallstones Plain
12
radiographs also have a very low sensitivity for gallstones
13
14
{Choice B) Hepatobiliary iminodiacetic acid (HIDA) scan (cholescintigraphy) is used to
15
diagnose acute cholecystitis if ultrasound does not provide a clear diagnosis. It is not routinely
16
17 used to evaluate pancreatitis.
18
(Choice E) Viruses (eg, hepatitis B, cytomegalovirus, varicella-zoster , HIV) are a less likely
19
20 cause of pancreatitis in this patient. Serologic studies can be deferred unless standard testing
21 does not identify the cause.
22
23 (Choice F) Stool elastase is a marker for pancreatic exocrine function. Low levels would be
24 seen in chronic , rather than acute, pancreatitis.
25
26 Educational objective:

-
27

29
30
31
32
Gallstones and chronic alcohol abuse are the most common causes of acute pancreatitis
Abdominal ultrasound is the most sensitive and specific imaging study to detect gallstones and
should be performed in all patients with suspected gallstone-induced pancreatitis.

References:
33 1. New diagnostic criteria of acute pancreatitis.
34
35 2. Classification of acute pancreatitis--2012: revision of the Atlanta classification
36 and definitions by int ernational consensus.
37
38
39 Time Spent 5 seconds Copyright © UWorld Last updated: [09/ 12/2016)
40
2
Item : 29 of 40 ~'?Mark <J C> 61 ~ ~ , ~
0. ld : 4363 Previous Next Lab Values Notes Calculator Reverse Color Text Zoom
3
4
5
6 A 27-year-old man comes to the emergency department due to episodic abdominal pain The
7 pain is concentrated in the epigastrium and is gnawing in quality . It wakes him up during the
8 night and is promptly relieved by a glass of water and a piece of bread. The patient has no
9 associated vomiting or diarrhea but has experienced occasional "dark stools." He has no
10 significant past medical history and takes no medications. Family history is significant for biliary
11 disease in his mother. He smokes a pack of cigarettes daily and consumes a can of beer most
12
days Vital signs are with in normal limits. Phys ical examination shows mild epigastric discomfort
13
14 on deep palpation After the diagnosis is confi rmed, which of the following is the most
15 appropriate intervention to provide long-term relief of this patient's symptoms?
16
17
6 A. 4-week course of omeprazole
18
19 ® B. Abstinence from alcohol
20 e; C. Antibiotics and pantoprazole
21
22 ® D. Cholecystectomy
23
6 E. Naproxen as needed
24
25 6 F. Selective vagotomy
26

...
® G. Smoking cessation
27
28

30 Submit
31
32
33
34
35
36
37
38
39
40
2
Item : 29 of 40 ~'?Mark <] C> !I ~ ~ , ~
0. ld : 4363 Previous Next Lab Values Notes Calculator Reverse Color Text Zoom
3
4 This patient with epigastric pain and intermittent melena most likely has a duodenal ulcer ( DU)
5 The pain of DU is often worse on an empty stomach (possibly due to unopposed gastric acid
6 emptying into the duodenum) and improves with food (due to alkaline fluid secretion into the
7 duodenum) By contrast, the pain of gastric ulcers is often worse after eating due to increased
8
acid secretion.
9
10
The majority of DUs are caused by either Helicobacter pylori infection or nonsteroidal
11
12 anti-inflammatory drugs (NSAIDs) Malignant ulceration should be considered with gastric
13 ulcers but would be very unlikely in this young patient with DU. As this patient has no history of
14 NSAID use, H pylori infection is the most likely etiology and can be confi rmed with endoscopic
15 biopsy or urea breath test Management of DU due to H pylori requires the following:
16
17 • Antisecretory ther apy, preferably a proton pump inhibitor (PPI) (eg, omeprazole,
18 pantoprazole), and
19 • Antibiotic er adication (eg, amoxicillin plus clarithromycin)
20
21 (Choice A) Compared to antisecretory therapy with antibiotic eradication, antisecretory therapy
22 alone leads to reduced healing rates and a higher risk of recurrent ulcer in patients with H
23
pylori-associated DU. Antisecretory therapy without antibiotics would be recommended fo r DU
24
25 attributable to causes other than H pylori.
26

... (Choices B and G) Heavy alcohol intake can contribute to hemorrhagic gastritis, but light
27
28 alcohol intake likely does not affect the course of DU. Smoking appears to increase the risk of
peptic ulcer in patients infected with H pylori but does not increase relapse following H pylori
30 eradication.
31
32 (Choice D) Laparoscopic cholecystectomy is recommended for biliary colic (intermittent, severe
33 right upper quadrant pain with nausea) or acute cholecystitis. This patient's epigastric pain
34 (relieved by food) and melena are unlikely to be due to symptomatic gallbladder disease.
35
36 (Choice E) NSAIDs can contribute to the risk of DU. Therefore, they are contraindicated during
37 the acute management of DU and should be used with caution following confirmed ulcer
38 resolution.
39
40
2
Item : 29 of 40 ~'?Mark <] C> !I ~ ~ , ~
0. ld : 4363 Previous Next Lab Values Notes Calculator Reverse Color Text Zoom
3
4
• • • • py
5 {Choices 8 and G) Heavy alcohol intake can contribute to hemorrhagic gastritis, but light
6
alcohol intake likely does not affect the course of DU. Smoking appears to increase the risk of
7
8 peptic ulcer in patients infected with H pylori but does not increase relapse following H pylori
9 eradication.
10
11 {Choice D) Laparoscopic cholecystectomy is recommended fo r biliary col ic (intermittent, severe
12 right upper quadrant pain with nausea) or acute cholecystitis. This patient's epigastric pain
13 (relieved by food) and melena are unlikely to be due to symptomatic gallbladder disease.
14
15 {Choice E) NSAIDs can contribute to the risk of DU. Therefore, they are contraindicated during
16 the acute management of DU and should be used with caution following confirmed ulce r
17 resolution.
18
19 (Choice F) Selective vagotomy decreases gastric acid production by removing vagal input to the
20 stomach. W ith antibiotic regimens for H pylori and the advent of PPis, selective vagotomy is
21 used only in refractory cases.
22
23 Educational objective:
24 Duodenal ulcer (DU) is characterized by epigastric pain that improves with eating The majority
25 of DUs are caused by Helicobacter pylori or nonsteroidal anti-inflammatory drugs Management
26

...
of H pylori-associated ulcer requires acid suppression and antibiotic eradication therapy
27
28
References:
30
1. Helicobacter pylori and gastric or duodenal ulcer.
31
32 2. A prospective study of alcohol, smoking, caffeine, and the risk of duodenal ulcer
33 in men.
34
35
3. Does smoking predispose to peptic ulcer relapse after eradication of
36 Helicobacter pylori?
37
38
39 Time Spent 2 seconds Copyright © UWorld Last updated: [10/ 11/2016)
40
2
Item: 30 of 40 ~'?Mark <J C> 61 ~ ~ , ~
0. ld : 2920 Previous Next Lab Values Notes Calculator Reverse Color Text Zoom
3
4
5
6 A 42-year-old man comes to the office due to progressive fatigue For several years, the patient
7 has had recurrent episodes of abdominal pain and diarrhea associated with bloody stools. He
8 underwent a colonoscopy with biopsy 2 yea rs ago, wh ich demonstrated contiguous proctocolitis
9 with pseudopolyps, mucosal ulcerations, and crypt abscesses. Oral 5-aminosalicylic acid and
10 corticosteroid enemas controlled his symptoms, but he has had extreme fatigue for the past
11 several months to where he feels drained by the end of the day The patient has had no weight
12
loss. Temperature is 37 C (986 F) , blood pressure is 134/90 mm Hg, pulse is 74/min, and
13
14 respirations are 15/min. Examination shows mild left lower quadrant tenderness on deep
15 palpation There is no hepatomegaly or splenomegaly Laboratory results are as follows:
16 Hemoglobin 11 8 gm/dl
17
18 Serum albumin 3.8 g/dl
19
20 Total bilirubin 1.7 mg/dl -
21
22 Alkaline phosphatase 816 U/ L
23
Aspa rtate aminotransferase 45 U/L
24
25 Alanine aminotransferase 54 U/L
26
27
28 Erythrocyte sedimentation rate is 102 mm/hr and perinuclear antineutrophil cytoplasmic antibody

--
29

31
32
33
34
testing is positive W hich of the following is the most likely cause of this patient's current
symptoms?

6 A Bacterial cholangitis
6 B. Drug-induced hepatotoxicity
35
36
e; C. Metastatic colon cancer
37 6 D. Polyarteritis nodosa
38
6 E. Primary biliary cholangitis
39
40
2
Item: 30 of 40 ~'?Mark <J C> 61 ~ ~ , ~
0. ld : 2920 Previous Next Lab Values Notes Calculator Reverse Color Text Zoom
3
4 underwent a colonoscopy with biopsy 2 yea rs ago, which demonstrated contiguous proctocolitis
5 with pseudopolyps, mucosal ulcerations, and crypt abscesses. Oral 5-aminosalicylic acid and
6 corticosteroid enemas controlled his symptoms, but he has had extreme fatigue for the past
7 several months to where he feels drained by the end of the day The patient has had no weight
8 loss. Temperature is 37 C (98 6 F) , blood pressure is 134/90 mm Hg, pulse is 74/min, and
9 respirations are 15/min. Examination shows mild left lower quadrant tenderness on deep
10 palpation There is no hepatomegaly or splenomegaly Laboratory results are as follows
11
12 Hemoglobin 11 8 gm/dl
13
14 Serum albumin 3.8 g/dl
15
Total bilirubin 1.7 mg/dl
16
17 Alkaline phosphatase 816 U/L
18
19 Aspartate aminotransferase 45 U/L
20
21 Alanine aminotransferase 54 U/L
22
23
Erythrocyte sedimentation rate is 102 mm/hr and perinuclear antineutrophil cytoplasmic antibody
24
testing is positive W hich of the following is the most likely cause of this patient's current
25
26 symptoms?
27
28

--
® A Bacterial cholangitis
29
® B. Drug-induced hepatotoxicity
31 ® C. Metastatic colon cancer
32
33 ® D. Polyarteritis nodosa
34 ® E. Primary biliary cholangitis
35
® F. Primary sclerosing cholangitis
36
37
38 Submit
39
40
2
Item: 30 of 40 ~'?Mark <] C> !I ~ ~ , ~
0. ld : 2920 Previous Next Lab Values Notes Calculator Reverse Color Text Zoom
3
4
5 Explanation:
6
7
8
9
10
11 This patient has primary sclerosing cholangitis ( PSG), a chronic , progressive disorder of
12 unknown etiology characterized by inflammation, fibros is, and stricturing of intrahepatic and
13 extrahepatic bile ducts. Affected individuals frequently have coexisting ulcerative colitis, which
14
explains this patient's recurrent bloody diarrhea, anemia, left lower quadrant tenderness, and
15
colonoscopy findings PSG is often asymptomatic but may present with fatigue and pruritus.
16
Continued bile duct destruction leads to cholestatic complications (eg, fat-soluble vitamin
17
18 deficiencies, osteoporosis), end-stage liver disease, and portal hypertension
19
20 Laboratory tests typically show predominantly elevated alkaline phosphatase levels with smaller
21 inc reases in serum aminotransferases (cholestatic pattern) Other nonspecific findings may
22 include elevated inflammatory markers (eg, erythrocyte sedimentation rate), increased lgM, and
23 positive perinuclear antineutrophil cytoplasmic antibodies. Endoscopic retrog rade or magnetic
24 resonance cholangiopancreatography confi rms the diagnosis by showing multifocal narrowing
25 (red arrows) with intrahepatic and extrahepatic duct dilation (black arrows) Liver biopsy is
26 typically not necessary but class ically shows intrahepatic ductular obliteration with lymphocytic
27 infiltration and periductular "onion-skin" fibrosis.
28

--
29

31
32
33
34
{Choice A) Acute bacterial cholangitis usually presents with Charcot triad (ie, feve r, right upper
quadrant pain, jaundice) Although patients with PSG may experience acute cholangitis, the
absence of fever or significant jaundice makes this diagnosis less likely

{Choice B) Drug-induced hepatotoxicity can present as acute or chronic hepatitis


35 (predominately elevated serum aminotransferases), cholestasis, or a mixed pattern However,
36 oral 5-aminosalicylic acid and co rticosteroid enemas are usually not associated with
37 hepatotoxicity
38
39 (Choice C) Metastatic colon cancer to the liver with extrahepatic/intrahepatic biliary obstruction
40 - -- - - - - - -------- -- - - . -- -- - - - - - - - - - -- - - - -- - - - -- - -

~ Feedback SuWend EnQ ock


2
Item: 30 of 40 ~'?Mark <] C> !I ~ ~ , ~
0. ld : 2920 Previous Next Lab Values Notes Calculator Reverse Color Text Zoom
3 . .

4
5 (Choice B) Drug-induced hepatotoxicity can present as acute or chronic hepatitis
6 (predominately elevated serum aminotransferases), cholestasis, or a mixed pattern However,
7 oral 5-aminosalicylic acid and co rticosteroid enemas are usually not associated with
8 hepatotoxicity
9
10 {Choice C) Metastatic colon cancer to the liver with extrahepatic/ intrahepatic biliary obstruction
11 can elevate alkal ine phosphatase Although ulcerative colitis is associated with increased risk of
12 colon cancer, this patient's lack of we ight loss and hepatomegaly make metastatic cancer
13 somewhat less likely.
14
15 (Choice D) Polyarteritis nodosa is a necrotizing vasculitis that affects medium-sized arteries and
16 presents with systemic symptoms (eg, fever, malaise, weight loss), neuropathy ,
17
arthralgias/myalgias, cutaneous findings (eg, livedo reticularis), and renal disease. The
18
19
hepatobiliary system is rarely affected and there is no association with antineutrophil cytoplasmic
20 antibodies or ulcerative colitis.
21
22 (Choice E) Primary biliary cholangitis (primary biliary cirrhosis) can present similar to PSG;
23 however, it classically affects middle-aged women and has no association with ulcerative colitis.
24 Patients frequently have positive serum antimitochondrial antibodies.
25
26 Educational objective:
27 Primary sclerosing cholangitis is often asymptomatic but may present with fatigue and pruritus
28 Affected individuals frequently have coexisting ulcerative colitis, and laboratory studies usually

--
29

31
32
33
34
show elevated liver function tests in a cholestatic pattern

References:
1. Primary sclerosing cholangitis: etiopathogenesis and clinical management.

35
2. Primary sclerosing cholangitis is associated with a distinct phenotype of
36 inflammatory bowel disease.
37
38
39 Time Spent 7 seconds Copyright © UWorld Last updated: [08/ 16/2016)
40
2
Item: 3 1 of40 ~'?Mark <J C> 61 ~ ~ , ~
0. ld : 4635 Previous Next Lab Values Notes Calculator Reverse Color Text Zoom
3
4
5
6 A 52-year-old man comes to the office due to decreased libido and inability to achieve and
7 maintain erections for the past several months. He has also had fatigue, anorexia, and a 5-kg
8 (11-lb) weight loss over the same duration. The patient is married and has one child. He drinks
9 alcohol but does not use tobacco or illic it drugs Blood pressure is 110/70 mm Hg and pulse is
10 82/min. Physical examination shows bilateral gynecomastia and firm and small testes.
11 Laboratory testing shows normal TSH with decreased levels of total triiodothyronine (T3) and
12
thyroxine (T4) W hich of the following is the most likely diagnosis?
13
14
15 e A Adrenal insufficiency
16
17 e B. Chronic liver disease
18 e C. Exogenous thyroxine intake
19
20 e D. Hashimoto thyroiditis
21 e E. Penile vascular occlusion
22
23 e F. Testosterone insensitivity
24
25
26 Submit
27
28
29

--
30

32
33
34
35
36
37
38
39
40
2
Item: 3 1 of40
0. ld : 4635
~'?Mark <]
Previous
C>
Next
a
Lab Values
~
Notes
~
Calculator
,
Reverse Color
~
Text Zoom
3
4
5 Explanation:
6
7
8 Signs of liver cirrhosis
9
10
11
12
13
14
15
16
17 Portal hypertension
18
19 Esophageal va rices
20
21
22
23 Loss of sexual hair
24
25 Testicular atrophy
26
27 Anorectal - - - -- -
28 varices
29

--
30

32
33
34
35
Hepatic synthetic dysfunction

36
37
38
39
40
2
3
4
5
6
7
8
9
10
11 © UWorid
12
13 This patient's erectile dysfunction and testicular atrophy are most likely due to hypogonadism,
14
which is a common complication of chronic live r disease with cirrhosis. Cirrhosis (especially
15
due to alcoholic live r disease or hemochromatosis) can cause hypogonadism due to primary
16
17 gonadal injury or hypothalamic-pituitary dysfunction Cirrhosis is also associated with elevated
18 circulating levels of estradiol due to increased conversion from androgens Findings due to
19 excess estrogen include telangiectasias, palmar erythema, testicular atrophy , and gynecomastia
20 (usually bilateral but can be unilateral)
21
22 In addition, the liver produces serum binding proteins for thyroid hormones (eg, thy roxine-binding
23 globulin, transthyretin, albumin, lipoproteins) Cirrhosis leads to decreased synthesis of these
24 proteins, which lowers the total triiodothy ronine (T3) and thy roxine (T4) in circulation; however ,
25 free T3 and T4 levels are unchanged, and TSH will be normal, reflecting a euthyroid status.
26
27 (Choice A) Adrenal insufficiency (AI) can cause fatigue, weakness, anorexia, and weight loss.
28 In women with A I, features of hypogonadism (eg, loss of libido, decreased pubic hair) can be
29

--
seen due to decreased adrenal androgen production; however, men with A I do not develop these
30
findings as androgens are primarily produced in the testes. In addition, A I would not explain this
32 patient's gynecomastia, testicular atrophy , or thyro id hormone laboratory abnormalities.
33
34 (Choice C) Exogenous thy roid hormone intake can cause weight loss, fatigue, and anxiety.
35 However, TSH would be suppressed
36
37 {Choice D) Hashimoto thyroiditis (chronic autoimmune thy roiditis) is due to lymphocytic
38 infiltration of the thyroid causing hypothy roidism Laboratory studies show elevated TSH,
39 decreased T4, and antithy roid peroxidase antibodies.
40
2
Item: 3 1 of40 ~'?Mark <] C> !I ~ ~ , ~
0. ld : 4635 Previous Next Lab Values Notes Calculator Reverse Color Text Zoom
3 I ~ g I I • I g y I
4
5 {Choice A) Adrenal insufficiency (AI) can cause fatigue, weakness, anorexia, and weight loss.
6
In women with A I, features of hypogonadism (eg, loss of libido, decreased pubic hair) can be
7
seen due to decreased adrenal androgen production; however , men with A I do not develop these
8
9 findings as androgens are primarily produced in the testes. In addition, A I would not explain this
10 patient's gynecomastia, testicular atrophy, or thyroid hormone laboratory abnormalities.
11
12 {Choice C) Exogenous thyroid hormone intake can cause weight loss, fatigue, and anxiety
13 However, TSH would be suppressed.
14
15 (Choice D) Hashimoto thyroiditis (chronic autoimmune thyroiditis) is due to lymphocytic
16 infiltration of the thyroid causing hypothyroidism. Laboratory studies show elevated TSH,
17 decreased T4, and antithyroid peroxidase antibodies.
18
19 (Choice E) Arterial insufficiency is a common cause of erectile dysfunction. However, this
20 would not cause gynecomastia or testicular atrophy, and most patients would have other signs of
21 arterial disease.
22
23 {Choice F ) Androgen insensitivity is caused by mutations in the androgen receptor and would
24 cause primary infertility (eg, primary amenorrhea in a genetically male but phenotypically female
25 individual [complete androgen insensitivity syndrome)) It is unlikely in this patient with
26 recent-onset sexual dysfunction.
27
28 Educational objective:
29

--
Cirrhosis can cause hypogonadism due to primary gonadal injury or hypothalamic-pituitary
30
dysfunction Cirrhosis is also associated with elevated circulating levels of estradiol due to
32 inc reased conversion from androgens
33
34 References:
35
36 1. Male hypogonadism in cirrhosis and after liver transplantation.
37
38
39 Time Spent 3 seconds Copyright© UWorld Last updated: [09/20/2016)
40
2
Item: 32 of40 ~'?Mark <J C> 61 ~ ~ , ~
0. ld : 4602 Previous Next Lab Values Notes Calculator Reverse Color Text Zoom
3
4
5
6 A 53-year-old man comes to the physician with a 2-day history of right calf pain and swelling.
7 The pain is constant and worsens with knee flexion. He has a history of intravenous drug abuse,
8 bacterial endoca rditis, and embolic stroke. The patient has been wheelchair-bound for the past
9 year due to stroke-related left-sided hemiparesis His temperature is 36.7 C (98 F), blood
10 pressure is 140/90 mm Hg, and pulse is 100/min. Physical examination shows no jugular venous
11 distension or hepatojugular reflux. Lungs are clear to auscultation. His abdomen is distended
12
with shifting dullness and a fl uid wave suggesting ascites. Hepatosplenomegaly is present The
13
14 right calf is swollen and tender to palpation Which of the following is the most likely cause of this
15 patient's ascites?
16
17 e; A Atherosclerosis
18
19 e> B. Chronic liver disease
20 e; C. Nephrotic syndrome
21
22 e> D. Paradoxical embolism
23
24 e; E. Pulmonary embolism
25 e> F. Right-sided heart failure
26
27
28 Submit
29
30

-
31

33
34
35
36
37
38
39
40
2
Item: 32 of40 ~'?Mark <] C> !I ~ ~ , ~
0. ld : 4602 Previous Next Lab Values Notes Calculator Reverse Color Text Zoom
3 • • tl • • • - '

4
F. Right-sided heart failure [11 %]
5
6
7 Explanation:
8
9 This patient's history of intravenous drug use places him at high risk fo r developing hepatitis C
10 with chronic liver disease. It also likely led to bacterial endocarditis with embolic stroke. The
11
unilateral leg pain and swelling suggest deep venous thrombosis due to immobility from stroke .
12
13 Cirrhosis accounts fo r about 80% of cases of ascites in the United States and is the most likely
14 cause of ascites in this patient Alcoholic liver disease and hepatitis C are the most common
15 causes of cirrhosis.
16
17 Patients often present with anorexia, jaundice, pruritus, confusion, or new-onset ascites.
18 Phys ical examination may show stigmata of liver disease (eg, spider angiomas, palmar erythema,
19 caput medusae) , asterixis, flank or shifting dullness, a fluid wave suggestive of ascites, and/or
20 decreased breath sounds due to a pleural effusion. Cirrhotic livers are often shrunken and
21 fibrotic , but may in many cases be normal-sized or enlarged Splenomegaly occurs due to
22 congestion from portal hypertension.
23
24 Diagnosis is confirmed via cl inical, laboratory, and radiologic evaluation. Abdom inal ultrasound is
25 useful to evaluate fo r complications (eg, splenomegaly , hepatocellular ca rcinoma) Paracentesis
26
is indicated fo r all patients with new-onset ascites to determine its etiology Liver biopsy is
27
28
required to confi rm the diagnosis of cirrhosis if it is unclea r or will alter the course of
29 management
30

-
31 {Choice A) Atherosclerosis is a risk factor for ischemic heart disease, which can lead to heart
failure and ascites (due to right-sided heart fa ilure) However, this patient's normal lung
33 examination and absence of right heart failure findings (eg, jugular venous distension,
34 hepatojugular reflux) make this less likely
35
36 (Choice C) Patients with nephrotic syndrome typically develop anasarca and prominent bilateral
37 peripheral edema (unlike the unilateral leg edema in this patient) They may also have ascites
38 and are prone to thrombosis due to hype rcoagulable state.
39
40

~
-------------------------------------------------------------------------------------------------------------------------------
Feedback SuWend EnQock
2
Item: 32 of40 ~'?Mark <] C> !I ~ ~ , ~
0. ld : 4602 Previous Next Lab Values Notes Calculator Reverse Color Text Zoom
3 - -
4 useful to evaluate for complications (eg, splenomegaly, hepatocellular carcinoma) Paracentesis
5 is indicated for all patients with new-onset ascites to determine its etiology Liver biopsy is
6 required to confirm the diagnosis of cirrhosis if it is unclear or will alter the course of
7 management.
8
9 (Choice A) Atherosclerosis is a risk factor for ischemic heart disease, which can lead to heart
10 failure and ascites (due to right-sided heart failure) However, this patient's normal lung
11 examination and absence of right heart failure findings (eg, jugular venous distension,
12
hepatojugular reflux) make this less likely
13
14
{Choice C) Patients with nephrotic syndrome typically develop anasarca and prominent bilateral
15
peripheral edema (unlike the unilateral leg edema in this patient) They may also have ascites
16
17 and are prone to thrombosis due to hypercoagulable state.
18
19
(Choice D) Paradoxical embolism is due to an embolus from the venous circulation across a
20 patent foramen ovale or atrial septal defect into the systemic circulation. This can lead to a stroke
21 but usually does not cause ascites.
22
23 (Choices E and F) Chronic pulmonary emboli can lead to pulmonary hypertension and
24 subsequent right heart failure (cor pulmonale). Patients typically have right-sided heart murmurs,
25 elevated jugular venous pressure, bilateral peripheral edema, and hepatojugular reflux. This
26 patient does not have these findings
27
28 Educational objective:
29 Asc ites is most commonly caused by cirrhosis due to alcoholic liver disease or chronic viral
30 hepatitis All patients with new-onset ascites require paracentesis to determine the cause.

-
31

33
34
35
36
References:
1. Ascites: diagnosis and management.
2. Differential diagnosis of ascites.
37
38
39 Time Spent 3 seconds Copyright © UWorld Last updated: [07/ 10/2016)
40
2
Item: 33 of40 ~'?Mark <J C> 61 ~ ~ , ~
0. ld : 461 2 Previous Next Lab Values Notes Calculator Reverse Color Text Zoom
3
4
5
6 A 62-year-old man comes to the office due to fatigue and weight loss. Over the last 6 months, he
7 has lost 9 kg (20 lb) and can no longer walk long distances without sitting down to rest The
8 patient has not seen a physician in 15 years. He has smoked a pack daily for the last 35 years
9 but does not use alcohol or illicit drugs. He has not recently traveled. Temperature is 36.9 C
10 (984 F), blood pressure is 130/80 mm Hg, pulse is 88/min, and respirations are 12/min. Lungs
11 are clear to auscultation, and abdominal examination shows a liver span of 14 em with no
12
tenderness. Rectal examination reveals normal sphincter tone but a slightly enlarged prostate that
13
14
is nontender to palpation. Chest x-ray is unremarkable. CT scan of the abdomen is shown in the
15 image below.
16
17
18
19
20
21 Which of the following is most likely to establish the diagnosis in this patient?
22
23 0 A Alpha-fetoprotein measurement
24
25 0 B. Aspiration and culture of the liver lesions
26 0 C. Bone marrow biopsy
27
28 0 D. Colonoscopy
29 0 E. Prostate-specific antigen measurement
30
31
0 F. Stool test for ova and parasites

-
32

34
35
36
37
Submit

38
39
40
2
Item: 33 of40 ~'?Mark <] C> !I ~ ~ , ~
0. ld: 4612 Previous Next Lab Values Notes Calculator Reverse Color Text Zoom
3
4 p [
5
6
7 Explanation:
8
9
This patient with weight loss and fatigue has multiple liver lesions on CT scan, suggesting
10 metastatic disease to the live r rather than primary liver cancer. Colorectal cancer is the most
11 common source of liver metastases (as blood from the colon moves th rough the portal circulation
12 directly to the live r) and should be excluded in this patient, even in the absence of specific
13 symptoms (eg, alteration in bowel habits, gastro intestinal bleeding) Abdominal CT is a useful
14 screening test but can often miss primary intraluminal tumors. Colonoscopy is the most
15 appropriate next diagnostic step as it both localizes the tumor and provides a tissue diagnosis
16
17 Lung and breast cancers also metastasize to the liver , although less commonly than
18 gastrointestinal malignancies. This patient is a smoker ; however , he has a normal chest x-ray
19
and no respiratory symptoms, making metastatic lung cancer unlikely
20
21
{Choice A) Alpha-fetoprotein measurement is used along with imaging studies in the diagnosis of
22
hepatocellular carcinoma, which most commonly occurs in the setting of chronic live r disease
23
24 and presents as a solita ry mass.
25
{Choice B) W ith pyogenic liver abscesses, aspiration and culture can guide antibiotic therapy.
26
27 However, abscesses typically present acutely with feve r, right upper quadrant pain, anorexia, and
28 weight loss. Serologic studies can distinguish between pyogenic and amebic liver abscesses.
29
30 {Choice C) Bone marrow biopsy is useful in establishing a diagnosis of myelop roliferative or
31 lymphoproliferative disorders. It would be uncommon fo r these disorders to cause multiple live r

-
32

34
35
36
37
masses.

{Choice E) Prostate-specific antigen measurement is a useful marker for prostate cancer.


However, prostate cancer tends to be an indolent malignancy that commonly metastasizes to the
pelvic lymph nodes and bones rather than the liver.

38 {Choice F) Liver flukes can be diagnosed with ova and parasite stool tests. However , they tend
39 to invade the biliary tree rather than the liver parenchyma, and the patient has no history of
40

~
-------------------------------------------------------------------------------------------------------------------------------
Feedback SuWend EnQ ock
2
Item: 33 of40 ~'?Mark <] C> !I ~ ~ , ~
3
4
.
0. ld : 461 2
. . ' "'. .. . . .. . Previous
. .. Next
. ., . . . Lab Values Notes Calculator Reverse Color Text Zoom

symptoms (eg, alteration in bowel habits, gastrointestinal bleeding) Abdominal CT is a useful


5 screening test but can often miss primary intraluminal tumors. Colonoscopy is the most
6 appropriate next diagnostic step as it both localizes the tumor and provides a tissue diagnosis
7
8 Lung and breast cancers also metastasize to the liver , although less commonly than
9
gastrointestinal malignancies. This patient is a smoker ; however , he has a normal chest x-ray
10
and no respiratory symptoms, making metastatic lung cancer unlikely
11
12
{Choice A) Alpha-fetoprotein measurement is used along with imaging studies in the diagnosis of
13
14 hepatocellular carcinoma, which most commonly occurs in the setting of chronic live r disease
15 and presents as a solita ry mass.
16
17 {Choice B) W ith pyogenic liver abscesses, aspiration and culture can guide antibiotic therapy.
18 However, abscesses typically present acutely with feve r, right upper quadrant pain, anorexia, and
19 weight loss. Serologic studies can distinguish between pyogenic and amebic liver abscesses.
20
21 {Choice C) Bone marrow biopsy is useful in establishing a diagnosis of myeloproliferative or
22 lymphoproliferative disorders. It would be uncommon fo r these disorders to cause multiple live r
23 masses.
24
25 {Choice E) Prostate-specific antigen measurement is a useful marker for prostate cancer.
26 However, prostate cancer tends to be an indolent malignancy that commonly metastasizes to the
27 pelvic lymph nodes and bones rather than the liver.
28
29 {Choice F) Liver flukes can be diagnosed with ova and parasite stool tests. However , they tend
30 to invade the biliary tree rather than the liver parenchyma, and the patient has no history of
31

-
recent travel abroad, making thi s diagnosis unlikely.
32
Educational objective:
34
Multiple live r masses are much more likely to be the result of metastatic disease than infectious
35
causes or primary liver malignancy. Primary tumors of the gastrointestinal tract, lung, and breast
36
37 are the most common diseases causing liver metastases.
38
39 Time Spent 28 seconds Copyright © UWorld Last updated: [10/ 10/2016)
40
2
Item: 34 of40 ~'?Mark <J C> 61 ~ ~ , ~
0. ld : 2966 Previous Next Lab Values Notes Calculator Reverse Color Text Zoom
3
4
5
6 A 52-year-old man comes to the physician fo r a regular health check-up He is a chronic smoker
7 and has consumed 1 to 2 beers 2 to 3 times a week for the past 10 years He also drinks 6 to 7
8 cups of coffee daily The patient was diagnosed with type 2 diabetes mellitus 5 years ago and
9 takes metformin. Blood pressure is 130/80 mm Hg and body mass index is 29 kg/m2 . Random
10 blood sugar is 190 mg/dL His older brother died of pancreatic cancer at age 58, and the patient
11 is worried that he might also develop panc reatic cancer. W hich of the following inte rventions
12
would have the greatest impact on decreasing this patient's risk of pancreatic cance r?
13
14
15 <D A Achieve better glycemic control with insulin
16
17 <D B. Stop alcohol intake
18 <D C. Stop coffee intake
19
20 <D D. Stop metformin
21 <D E. Stop smoking
22
23
24
Subm it
25
26
27
28
29
30
31
32

--
33

35
36
37
38
39
40
2
3
4
5
6
7
8 Major risk factors for pancreatic cancer
9
10
11 • First-degree relative with pancreatic cancer
12 • Hereditary pancreatitis
13 Hereditary
14 • Germline mutations (eg, BRCA1, BRCA2,
15 Peutz-Jeghers syndrome)
16
17 • Cigarette smoking (most significant)
18 • Obesity, low physical activity
Environmental
19
• Nonhereditary chronic pancreatitis
20
21 @UW011d
22
23 Pancreatic cancer is the 4th leading cause of cancer deaths in the United States.
24 Adenocarcinoma is the most common tissue type. Pancreatic cancer usually occurs after age
25
45 and is more common in men and African American patients. However, it is usually diagnosed
26
relatively late in the disease course and has a high mortality rate (98% in some studies). The
27
28 major hereditary risk factors for pancreatic cancer include first-degree relatives with pancreatic
29 cancer, hereditary pancreatitis, and germline mutations (eg, BRCA1 , BRCA2, Peutz-Jeghers
30 syndrome).
31
32 Environmental risk factors include cigarette smoking, nonhereditary chronic pancreatitis, and

--
33

35
36
37
38
obesity with low physical activity Cigarette smoking is the most consistent reversible risk factor
for pancreatic cancer. The risk of cancer increases significantly with the number of cigarettes
consumed, and heavy smokers have nearly 2-3 times higher risk than nonsmokers. Some
studies estimate that smoking cessation could reduce pancreatic cancer deaths by 25%.

{Choices A and D) Long-standing diabetes is a risk factor for pancreatic cancer, and
39 conversely. new-onset diabetes is often a siqn of occult pancreatic cancer. The effect of diabetic
40
2
Item: 34 of40 ~'?Mark <] C> !I ~ ~ , ~
0. ld : 2966 Previous Next Lab Values Notes Calculator Reverse Color Text Zoom
3
4 obesity with low physical activity Cigarette smoking is the most consistent reversible risk factor
5 fo r pancreatic cancer. The risk of cancer increases significantly with the number of cigarettes
6 consumed, and heavy smokers have nearly 2-3 times higher risk than nonsmokers. Some
7 studies estimate that smoking cessation could reduce pancreatic cancer deaths by 25%.
8
9 (Choices A and D) Long-standing diabetes is a risk factor for pancreatic cance r, and
10 conversely, new-onset diabetes is often a sign of occult pancreatic cancer. The effect of diabetic
11
treatment on pancreatic cancer risk is variable; insulin and insulin secretagogues (eg,
12
13
sulfonylureas) are associated with an inc reased risk, whereas metformin is associated with a
14 lower risk. Neither discontinuing metformin nor initiating insulin would be likely to lower this
15 patient's cancer risk, regardless of improved glycemic control.
16
17 (Choices B and C) Studies have not shown a significantly decreased incidence of pancreatic
18 cancer with alcohol or caffeine reduction. Heavy alcohol use can lead to chronic panc reatitis,
19 which is associated with a slightly higher risk of pancreatic cancer. However, this is not as
20 significant as the association between smoking and pancreatic cancer. In addition, this patient is
21 not a heavy alcohol consumer.
22
23 Educational objective:
24 Pancreatic cancer can be due to hereditary (eg, first-degree relative with panc reatic cancer,
25 heredita ry pancreatitis) or environmental (eg, cigarette smoking, obesity) risk factors. Cigarette
26
smoking is the most consistent reversible risk factor for pancreatic cancer.
27
28
29 References:
30 1. Cigarette smoking and pancreatic cancer: an analysis from the International
31
Pancreatic Cancer Case-Control Consortium (PanC4).
32

--
33

35
36
37
38
2. The epidemiology of pancreatitis and pancreatic cancer.
3. Alcohol intake and pancreatic cancer: a pooled analysis from the Pancreatic
Cancer Cohort Consortium (Pan Scan).

39 Time Spent 3 seconds Copyright © UWorld Last updated [08/07/2016)


40
2
Item: 35 of 40 ~'?Mark <J C> 61 ~ ~ , ~
0. ld : 2585 Previous Next Lab Values Notes Calculator Reverse Color Text Zoom
3
4
5
6 A 35-year-old woman comes to the office for follow-up on genetic testing for Lynch syndrome.
7 She has no symptoms or medical problems and has never had surgery. A Papanicolaou test was
8 normal last month. The patient takes a daily multivitamin and no other medications. She does not
9 use contraception as she has not been sexually active since separating from her husband last
10 year The patient's mother, age 54, is receiving chemotherapy for colon carcinoma. Her
11 maternal uncle was diagnosed with colon cancer at age 49, but it is in remission; her first cousin
12
died of colon cance r at age 36. In addition to colon cancer , the patient is at greatest risk for
13
14 which of the following conditions?
15
16 e; A Breast carc inoma
17
18 e; B. Clear cell renal carcinoma
19 6 C. Endometrial carcinoma
20
21 e; D. Hepatocellular carcinoma
22 () E. Medullary thyroid carcinoma
23
24 e; F. Pancreatic adenoma
25
26 6 G. Pheochromocytoma
27
28
Submit
29
30
31
32
33

--
34

36
37
38
39
40
2
Item: 35 of 40
0. ld : 2585
~'?Mark <]
Previous
C>
Next
a
Lab Values
~
Notes
~
Calculator
,
Reverse Color
~
Text Zoom
3
4
5 Common hereditary cancer syndromes
6
7
8 Syndrome Associated neoplasms
9
10 • Colorectal cancer
11
Lynch syndrome • Endometrial cancer
12
13 • Ovarian cancer
14
15 Familial • Colorectal cancer
16 adenomatous • Desmoids & osteomas
17 polyposis • Brain tumors
18
19
20 • Hemangioblastomas
von Hippei-Lindau
21 syndrome • Clear cell renal carcinoma
22 • Pheochromocytoma
23
24 • Parathyroid adenomas
25
Multiple endocrine
neoplasia type 1 • Pituitary adenomas
26 • Pancreatic adenomas
27
28 • Medullary thyroid cancer
29 Multiple endocrine
30 neoplasia type 2 • Pheochromocytoma
31 • Parathyroid hyperplasia (type 2A)
32
33 • Breast cancer
BRCA1 & BRCA2

--
34

36
37
38
©UWorld
• Ovarian cancer

Lynch syndrome (eg, hereditary nonpolyposis colorectal cancer) is an autosomal dominant


cancer syndrome that predisposes individuals to colorectal cancer and other
39
40
rI G r t r I h ld b I li d· I r t 'th t I t ·1 h · t t

~ Feedback SuWend EnQock


2
Item: 35 of 40 ~'?Mark <] C> !I ~ ~ , ~
0. ld : 2585 Previous Next Lab Values Notes Calculator Reverse Color Text Zoom
3
4 Lynch syndrome (eg, hereditary nonpolyposis colorectal cancer) is an autosomal dominant
5 cancer syndrome that predisposes individuals to colorectal cancer and other
6 malignancies. Genetic testing should be performed in patients with a strong family history of
7 colon cancer (eg, ~3 relatives involving multiple generations) The condition is due to a germline
8 mutation in a DNA mismatch repair gene. Once the diagnosis of Lynch syndrome is established,
9 patients should undergo screening for colon cancer with colonoscopy
10
11 In addition to colon cancer, patients are at extremely high risk for endometrial
12
carcinoma. Endometrial cance r screening with annual endometrial biopsy should begin at age
13
30-35. Ovarian cancer risk is also inc reased and may present at a relatively younger
14
15 age Therefore, prophylactic hysterectomy and bilateral oophorectomy is recommended at age
16 40 or earlier if childbearing is complete
17
18 (Choice A) Breast ca rcinoma can be caused by BRCA1 and BRCA2 gene mutations, which
19 also impart an increased risk of ovarian cancer. There is no increased risk of colon cancer with
20 these mutations. It is unclear if patients with Lynch syndrome have an increased incidence of
21 breast cance r.
22
23 (Choices B and G) Clear cell renal carcinoma and pheochromocytoma can be due to von
24 Hippei-Lindau syndrome This syndrome is also autosomal dominant but is not associated with
25 colon carcinoma.
26
27 (Choice D) Important risk factors for primary hepatocellular ca rcinoma include chronic hepatitis
28 B and C infections. Most secondary liver cancers originate from the colon, and this patient would
29 be at risk for hepatic metastases if she had signs of colon cance r (eg, weight loss, bloody stools).
30
31 (Choices E, F, and G) Multiple endocrine neoplasia type 1 (MEN1) syndrome is characterized
32 by parathyroid, pituitary, and panc reatic adenomas. MEN2 syndrome is characterized by
33

--
34
medullary thyroid cancer, pheochromocytoma , and parathy roid hyperplasia Both syndromes
are autosomal dominant, but colon cancer is not a feature.
36
37
Educational Objective:
38 Lynch syndrome causes colorectal cance r and imparts an extremely high risk of endometrial
39 carcinoma for female carriers. An annual endometrial biopsy, as well as prophylactic
40 - - - - - - .- - - - .- - - - - - - - - - -- - . - .- - - -----

~ Feedback SuWend EnQ ock


2
Item: 35 of 40 ~'?Mark <] C> !I ~ ~ , ~
0. ld : 2585 Previous Next Lab Values Notes Calculator Reverse Color Text Zoom
3 - - -
4 30-35. Ovarian cancer risk is also inc reased and may present at a relatively younger
5 age Therefore, prophylactic hysterectomy and bilateral oophorectomy is recommended at age
6 40 or earlier if childbearing is complete
7
8 {Choice A) Breast carcinoma can be caused by BRCA 1 and BRCA2 gene mutations, which
9 also impart an increased risk of ovarian cancer. There is no increased risk of colon cancer with
10 these mutations. It is unclear if patients with Lynch syndrome have an increased incidence of
11 breast cancer.
12
13 (Choices B and G) Clear cell renal carc inoma and pheochromocytoma can be due to von
14
Hippei-Lindau syndrome This syndrome is also autosomal dominant but is not associated with
15
16
colon carc inoma.
17
(Choice D) Important risk factors for primary hepatocellular carcinoma include chronic hepatitis
18
19
B and C infections. Most secondary liver cancers originate from the colon, and this patient would
20 be at risk fo r hepatic metastases if she had signs of colon cancer (eg, weight loss, bloody stools)
21
22 (Choices E, F, and G) Multiple endocrine neoplasia type 1 (MEN1) syndrome is characterized
23 by parathyroid, pituitary, and pancreatic adenomas. MEN2 syndrome is characterized by
24 medullary thyroid cancer, pheochromocytoma, and parathyroid hyperplasia. Both syndromes
25 are autosomal dominant, but colon cancer is not a feature.
26
27 Educational Objective:
28 Lynch syndrome causes colo rectal cance r and imparts an extremely high risk of endometrial
29 carcinoma for female carriers. An annual endometrial biopsy, as well as prophylactic
30 hysterectomy, is recommended to prevent endometrial cancer.
31
32
33
References:

--
34

36
37
38
39
1. Lynch syndrome screening in the gynecologic tract: current state of the art.
2. ACOG Practice Bulletin No. 147: Lynch syndrome.

Time Spent 2 seconds Copyright © UWo rld Last updated: [08/22/2016)


40
2
Item: 36 of40 ~'?Mark <] C> !I ~ ~ , ~
0. ld : 4648 Previous Next Lab Values Notes Calculator Reverse Color Text Zoom
3
4
5
6 A 75-year-old man is brought to the emergency department from home after being found
7 unresponsive by his son. His past medical history is significant for coronary artery disease with
8 coronary stenting two years ago, diabetes mellitus, hypertension, and stroke. He was
9 hospitalized two months ago with pneumonia and upper Gl bleeding, during which time he
10 received IV antibiotics and four blood transfusions. In the emergency room today, his blood
11 pressure is 70/40 mm Hg, heart rate is 120/min, respiratory rate is 32/min, temperature is 39.1o
12
C, and oxygen saturation is 79% on room air. Chest x-ray reveals a new right upper lobe
13
14 infiltrate. He is treated with normal saline, antibiotics, vasopressors, and mechanical ventilation.
15 The next day, laboratory analyses reveal the following
16
17
Hemoglobin 9.5 mg/dl -
18 WBC 15,000/mm3
19
20 Platelet 120,000/mm3
21
22 Sodium 137 mEq/L
23
Potassium 4.1 mEq/L
24
25 Chloride 100 mEq/L
26
27 Bicarbonate 19 mEq/L
28
29 BUN 51 mg/dl
30
31 Creatinine 2.1 mg/dl
32 1.2 mg/dl
Bilirubin, total
33
34 AST 2,720 units/L

-
35

37
38
39
40
ALT
Alkaline phosphatase
2,250 units/L

162 mEq/L
T
2
Item: 36 of40 ~'?Mark <J C> 61 ~ ~ , ~
0. ld : 4648 Previous Next Lab Values Notes Calculator Reverse Color Text Zoom
3
~

4 Platelet 120,000/mm3
5
6 Sodium 137 mEq/L
7
Potassium 4.1 mEq/L
8
9 Chloride 100 mEq/L
10
11 Bica rbonate 19 mEq/ L
12
13 BUN 51 mg/dl
14
Creatinine 2.1 mg/dl
15
16 Bilirubin, total 1.2 mg/dl
17
18 AST 2,720 units/ L
19
20 A LT 2,250 units/ L
21
22 Alkaline phosphatase 162 mEq/L
23
24 W hich of the following most likely accounts for the abnormal live r function panel?
25
26
27 0 A Acute viral hepatitis
28 0 B. Alcoholic liver disease
29
30 0 C. Granulomatous live r disease
31 0 D. Ischemic hepatic injury
32
33 0 E. Acalculous cholecystitis
34 0 F. Iron overload

-
35

37
38
39
40
0 G. Autoimmune hepatitis

Submit
2
Item: 36 of40 ~'?Mark <] C> !I ~ ~ , ~
0. ld : 4648 Previous Next Lab Values Notes Calculator Reverse Color Text Zoom
3
4
5 Explanation:
6
7 This patient presented with septic shock and developed AST and ALT elevations one day later.
8 This is most consistent with ischemic hepatic injury, or shock liver. The hallmark of ischemic
9 hepatopathy is a rapid and massive increase in the transaminases with modest accompanying
10 elevations in total bilirubin and alkaline phosphatase. In patients who survive the underlying cause
11 of their hypotension (e.g septic shock, heart failure) , liver enzymes typically return to normal
12
within one to two weeks.
13
14
15 (Choice A) Acute hepatitis A or hepatitis B infection can present with large AST and ALT
16 elevations, typically accompanied by significant hyperbilirubinemia, nausea, and vomiting
17
18 (Choice B) Alcoholic live r disease typically causes less dramatic increases in the transaminases
19 compared to those seen above. The ASTALT ratio is usually 1.5 or greater , and the AST itself is
20 rarely greater than 300 units/L
21
22
(Choice C) Granulomatous disorders like tuberculosis and sarcoidosis rarely cause massive
23
24 transaminase elevations. Chronic hepatic damage is occasionally associated.
25
26 {Choice E) Acalculous cholecystitis is a common complication of c ritical illness that arises
27 secondary to gallbladder stasis. Patients present with symptoms of biliary obstruction. Alkaline
28 phosphatase and total bilirubin levels will be more elevated than the AST and ALT
29
30 {Choice F) Iron overload may cause chronic hepatic dysfunction with low-grade elevations in the
31 AST and A LT Four transfusions would be unlikely to cause iron overload.
32
33
34 (Choice G) Autoimmune hepatitis may cause large increases in the AST and A LT However ,

-
35 young women are more commonly affected, and the associated serum bilirubin increases are
typically more dramatic .
37
38 Educational objective:
39 Ischemic hepatic injury occurs in the setting of hypotension and manifests as acute, massive
40
2
Item: 36 of40 ~'?Mark <] C> !I ~ ~ , ~
0. ld : 4648 Previous Next Lab Values Notes Calculator Reverse Color Text Zoom
3 '"'1., 11, J ., o~l I., I ., J• •.,. '"' • ., ., ., '"' H II'"' ., I lo~ J I
4 elevations in total bilirubin and alkaline phosphatase In patients who survive the underlying cause
5
of their hypotension (e g septic shock, heart failure) , live r enzymes typically return to normal
6
7 within one to two weeks.
8
9 (Choice A) Acute hepatitis A or hepatitis B infection can present with large AST and ALT
10 elevations, typically accompanied by significant hyperbilirubinemia, nausea, and vomiting
11
12 (Choice B) Alcoholic liver disease typically causes less dramatic increases in the transaminases
13 compa red to those seen above. The ASTALT ratio is usually 1.5 or greater, and the AST itself is
14
rarely greater than 300 units/L
15
16
17 (Choice C) Granulomatous disorders like tuberculosis and sarcoidosis rarely cause massive
18 transaminase elevations. Chronic hepatic damage is occasionally associated.
19
20 (Choice E) Acalculous cholecystitis is a common complication of critical illness that arises
21 secondary to gallbladder stasis. Patients present with symptoms of biliary obstruction. Alkaline
22 phosphatase and total bilirubin levels will be more elevated than the AST and A LT
23
24 (Choice F) Iron overload may cause chronic hepatic dysfunction with low-grade elevations in the
25
AST and ALT Four transfusions would be unlikely to cause iron overload.
26
27
28 (Choice G) Autoimmune hepatitis may cause large increases in the AST and ALT However,
29 young women are more commonly affected, and the associated serum bilirubin increases are
30 typically more dramatic.
31
32 Educational objective:
33 Ischemic hepatic injury occurs in the setting of hypotension and manifests as acute, massive
34

-
inc reases in the AST and ALT with milder associated increases in the total bili rubin and alkaline
35
phosphatase In patients who survive the inciting condition, liver enzymes typically return to
37 normal within a few weeks.
38
39 Time Spent 5 seconds Copyright© UWorld Last updated [08/ 15/2016)
40
2
Item: 37 of40 ~'?Mark <J C> 61 ~ ~ , ~
0. ld : 22 17 Previous Next Lab Values Notes Calculator Reverse Color Text Zoom
3
4
5
6 A 56-year-old man is brought to the emergency department with new-onset lethargy and
7 confusion. The patient has a history of cirrhosis secondary to chronic alcoholism. His
8 medications include spironolactone and furosem ide. His temperature is 38.0 C (100.4 F) , blood
9 pressure is 123/70 mm Hg, pulse is 112/min, and respirations are 24/min. Physical examination
10 reveals a flapping tremor of his hands when they are held out The patient's breathing is labored,
11 but his lungs are clear to auscultation. Abdom inal examination reveals distension with shifting
12
dullness and diffuse tenderness to palpation There is bilateral pitting edema of his lower
13
14 extremities. Laboratory results are as follows:
15 W hite blood cells 12,000/mm'
16
17 Total bilirubin 2.5 mg/dL
18
19 Aspartate aminotransferase 110 U/L
20
21 Alanine transaminase 52 U/L
22 Alkaline phosphatase 125 U/L
23
24 Creatinine 1.4 mg/dL
25
26 Albumin 2.1 mg/dL
27
28
Chest x-ray reveals no abnormalities. Abdominal x-ray reveals gas in the small and large bowels
29
30 without air-fluid levels. W hich of the following is the most appropriate next step in management?
31
32 6 A. Barium enema
33
34 6 B. Colonoscopy
35 e; C. Diagnostic laparoscopy

-

36

38
39
40
6 D. Diagnostic paracentesis
6 E. Lumbar puncture
I I

~ Feedback SuWend EnQ ock


2
Item: 37 of40 ~'?Mark <J C> 61 ~ ~ , ~
0. ld: 2217 Previous Next Lab Values Notes Calculator Reverse Color Text Zoom
3 -·~- ---- ~-- -·- --··· -·--·--·
4 medications include spironolactone and furosem ide. His temperature is 38.0 C (1004 F) , blood
5 pressure is 123/70 mm Hg, pulse is 112/min, and respirations are 24/min. Physical examination
6 reveals a flapping tremor of his hands when they are held out The patient's breathing is labored,
7 but his lungs are clear to auscultation. Abdominal examination reveals distension with shifting
8
dullness and diffuse tenderness to palpation. There is bilateral pitting edema of his lower
9
10 extremities. Laboratory results are as follows
11 W hite blood cells 12,000/mm'
12
13 Total bilirubin 2.5 mg/dL
14
15 Aspartate aminotransferase 110 U/L
16
17 Alanine transaminase 52 U/L
18
Alkaline phosphatase 125 U/L
19
20 Creatinine 14 mg/dL
21
22 Albumin 2.1 mg/dL
23
24
Chest x-ray reveals no abnormalities. Abdom inal x-ray reveals gas in the small and large bowels
25
26 without air-fluid levels. W hich of the following is the most appropriate next step in management?
27
28 ® A Barium enema
29
30 ® B. Colonoscopy
31 ® C. Diagnostic laparoscopy
32
33 ® D. Diagnostic paracentesis
34 ® E. Lumbar puncture
35

- 36
® F. Serum alpha-fetoprotein level

38 Submit
39
• 40
2
Item: 37 of40
0. ld: 22 17
~'?Mark <]
Previous
C>
Next
a
Lab Values
~
Notes
~
Calculator
,
Reverse Color
~
Text Zoom
3
4
5 Explanation:
6
7
8
9 Spontaneous bacterial peritonitis
10
11
12 • Temperature ;::37.8 C (100 F)
13
14 Clinical
• Abdominal pain/tenderness
15 presentation • Altered mental status (abnormal connect-the-numbers test)
16
17
• Hypotension, hypothermia, paralytic ileus with severe
18 infection
19
20 PMNs =::250/mm 3
21

22 Diagnosis • Positive culture, often gram-negative organisms
23 from ascitic (eg, Escherichia coli, Klebsiella)
24 f luid
25
• Protein <1 g/dl
26 • SAAG ;::1.1 g/dl
27
28
29 • Empiric antibiotics -third-generation cephalosporins
30 Treatment (eg, cefotaxime)
31 • Fluoroquinolones for SBP prophylaxis
32
33
PMN = polymorphonuclear leukocytes; SAAG =serum-ascites albumin gradient; SBP = spontaneous bactelial
34 peritonitis.
35

-
<1>UWOI1d
36
This patient has cirrhosis and ascites accompanied by fever and lethargy, a presentation
38 concerning for spontaneous bacterial peritonitis (SBP) and hepatic encephalopathy The
39
" ... . i ,ee II l o~ ll ·ue " ... o~ ll ,11 " ll: o~lll II" ... " ... " 11•1 ... " II I I
• 40

~ Feedback SuWend EnQ ock


2
Item: 37 of40 ~'?Mark <] C> !I ~ ~ , ~
0. ld : 22 17 Previous Next Lab Values Notes Calculator Reverse Color Text Zoom
3
4
5 For making the diagnosis, par acentesis is the test of choice with the main diagnostic criteria
6 being a positive ascites fluid culture and neutrophil count of ~250/mm>. Paracentesis should be
7 done before antibiotic therapy is initiated as therapy often results in negative ascites
8 cultures. Enteric organisms such as Escherichia coli and Klebsiella are the most commonly
9 cultured organisms followed by the streptococcal species; empiric therapy usually includes a
10 third-generation cephalosporin .
11
12 (Choices A and B) There is nothing in this patient's presentation to suggest colon pathology A
13 barium enema or colonoscopy is unlikely to be helpful
14
15 (Choice C) Diagnostic laparoscopy may rarely be required in patients with peritoneal signs on
16
examination if imaging studies have not revealed a source. However, the paracentesis results
17
18 can also help distinguish between SSP and bacterial peritonitis secondary to other causes such
19 as intestinal perforation or abscess.
20
21
{Choice E) Although fever and mental status changes are common signs in meningoencephalitis,
22 a patient with ascites and abdominal pain should fi rst be evaluated for SSP as it is much more
23 prevalent in patients with cirrhosis. Lumbar puncture is much less likely to yield a diagnosis
24
25 (Choice F) Serum alpha-fetoprotein level is elevated in patients with hepatocellular carcinoma,
26 which is a possibility in this patient given his underlying cirrhosis. However , his acute changes
27 (fever and altered mental status) are more conce rning for SSP, which should be ruled out first
28
29 Educational objective:
30 Spontaneous bacterial peritonitis can have a subtle presentation and should be considered in any
31 patient with cirrhosis and ascites accompanied by either feve r or a change in mental
32 status. Paracentesis is the test of choice, with the main diagnostic criteria being a positive
33 ascites fluid culture and neutrophil count of ~250/mm>.
34
35

- 36 References:
1. Spontaneous bacterial peritonitis.
38
39 2. Spontaneous bacterial peritonitis.
• 40
2
Item: 38 of 40 ~'?Mark <] C> !I ~ ~ , ~
0. ld : 292 1 Previous Next Lab Values Notes Calculator Reverse Color Text Zoom
3
4
5
6 A 52-year-old man comes to the office due to abnormal liver chemistry results found during a life
7 insurance medical examination. He occasionally feels fatigued and has pedal edema after
8 prolonged standing but otherwise feels well. The patient has no medical history and takes no
9 medications. He drinks 1 or 2 cans of beer on weekends and does not smoke. He used
10 intravenous drugs in his 20s but has not since then. Vital signs are within normal range
11 Physical examination shows palmar erythema and multiple spider angiomas Laboratory results
12
are as follows
13
14 Complete blood count
15
16 Hemoglobin 11 .2 g/dl
17
Platelets 120,000/mm'
18
19
20
21 Serum chemistry
22
23 Creatinine 0.8 mg/dl
24
25
26 Liver function studies
27
28 Albumin 3.4 g/dl
29
30 Total bilirubin 1.3 mg/dl
31
32 Aspartate aminotransferase 62 U/L
33
Alanine aminotransferase 99 U/L
34
35
36

--

37

39
40
Coagulation studies

INR 1.4
2
Item: 38 of 40 ~'?Mark <J C> 61 ~ ~ , ~
0. ld : 292 1 Previous Next Lab Values Notes Calculator Reverse Color Text Zoom
3
4 Albumin 3.4 g/dl
5
Total bilirubin 1.3 mg/dl
6
7 Aspartate aminotransferase 62 U/L
8
9 Alanine aminotransferase 99 U/L
10
11
12
13 Coagulation studies
14
INR 1.4
15
16
17
18 Immunologic and rheumatologic studies
19
20 Hepatitis C virus antibody positive
21
Hepatitis C RNA 1 million copies/ml
22
23
24 Abdominal u~rasonog raphy reveals a coarse, nodular-appearing liver with no masses, mild
25 ascites, and splenomegaly Upper gastrointestinal endoscopy demonstrates medium-sized but
26 nonbleeding esophageal varices. He is referred for a hepatitis C evaluation with a
27 gastroenterologist W hich of the following is the best management for this patient at this time?
28
29
30 <0 A ACE inhibitors
31 <0 B. Beta-adrenergic receptor blockers
32
33 <0 C . Endoscopic sclerotherapy
34 <0 D. H2 histamine receptor blockers
35
36
<0 E. Octreotide injections

--

37

39
40
Submit
2
Item: 38 of 40 ~'?Mark <] C> !I ~ ~ , ~
0. ld : 292 1 Previous Next Lab Values Notes Calculator Reverse Color Text Zoom
3
4
5 Explanation:
6
7 This patient has evidence of cir rhosis (eg, spider angioma, nodular coa rse liver , varices), likely
8 from chronic hepatitis C. He is found to have medium-sized, nonbleeding esophageal va rices .
9 Va riceal hemorrhage develops in approximately one third of all patients with varices and is a
10 major cause of morbidity and mortality Most cirrhotic patients should undergo diagnostic upper
11
endoscopy to assess for va rices and to determine their risk of hemorrhage Those with
12
(medium- or large-sized) varices, such as this patient, generally should be started on a
13
14 nonselective beta blocker.
15
Nonselective beta blockers (eg, propranolol, nadolol) are recommended to decrease progression
16
17 to large varices and the risk of variceal hemorrhage. They are thought to act by decreasing
18 adrenergic tone in mesenteric arterioles, which results in unopposed alpha-mediated
19 vasoconstriction and decreased portal venous flow. Endoscopic variceal ligation is an alternate
20 primary preventive therapy in patients with contraindications to beta blocker therapy.
21
22 {Choice A) ACE inhibitors have not been shown to reduce variceal bleeding or improve
23 outcomes in cirrhosis, and they have no role in the management of patients with esophageal
24 va rices.
25
26 (Choice C) Endoscopic sclerotherapy is an effective treatment fo r actively bleeding esophageal
27 varices. It is not recommended for primary prophylaxis of variceal hemorrhage
28
29 (Choice D) H2 histamine receptor blockers have not been shown to reduce progression or
30 bleeding complications in patients with cirrhosis and va rices.
31
32 (Choice E) Octreotide is a long-acting somatostatin analogue that causes splanchnic
33 vasoconstriction and reduced portal blood flow by inhibiting the release of glucagon. It is used in
34
the treatment of active variceal bleeding and has no role in primary prophylaxis
35
36

--
Educational objective:
37
In most patients with nonbleeding varices, prophylactic treatment with nonselective beta blockers
(eg, propranolol, nadolol) is recommended to reduce the likelihood of progression to large varices

39
40 . .. ...... . ... ... ..
.. ... .. . .. . ... . . .
~ Feedback SuWend EnQock
2
Item: 38 of 40 ~'?Mark <] C> !I ~ ~ , ~
0. ld : 292 1 Previous Next Lab Values Notes Calculator Reverse Color Text Zoom
3
4 Nonselective beta blockers (eg, propranolol, nadolol) are recommended to decrease progression
5 to large va rices and the risk of variceal hemorrhage They are thought to act by decreasing
6
adrenergic tone in mesenteric arterioles, which results in unopposed alpha-mediated
7
vasoconstriction and decreased portal venous flow. Endoscopic variceal ligation is an alternate
8
9 primary preventive therapy in patients with contraindications to beta blocker therapy.
10
11
(Choice A) ACE inhibitors have not been shown to reduce variceal bleeding or improve
12 outcomes in cirrhosis, and they have no role in the management of patients with esophageal
13 varices.
14
15 (Choice C) Endoscopic sclerotherapy is an effective treatment for actively bleeding esophageal
16 varices. It is not recommended fo r primary prophylaxis of va riceal hemorrhage.
17
18 (Choice D) H2 histamine receptor blockers have not been shown to reduce progression or
19 bleeding complications in patients with cirrhosis and va rices.
20
21 (Choice E) Octreotide is a long-acting somatostatin analogue that causes splanchnic
22 vasoconstriction and reduced portal blood flow by inhibiting the release of glucagon It is used in
23 the treatment of active variceal bleeding and has no role in primary prophylaxis
24
25 Educational objective:
26 In most patients with nonbleeding varices, prophylactic treatment with nonselective beta blockers
27 (eg, propranolol, nadolol) is recommended to reduce the likelihood of progression to large varices
28 as well as the risk of variceal hemorrhage. Endoscopic variceal ligation can be used as an
29
alternate primary preventive therapy in patients with contra indications to beta blocker therapy
30
31
32 References:
33
1. Expanding consensus in portal hypertension: report of the Baveno VI
34
35
Consensus Workshop: stratifying risk and individualizing care for portal
36 hypertension.

--

37

39
40
Time Spent 5 seconds Copyright © UWorld Last updated: [10/ 10/2016)
2
Item: 39 of 40 ~'?Mark <] C> !I ~ ~ , ~
0. ld : 2936 Previous Next Lab Values Notes Calculator Reverse Color Text Zoom
3
4
5
6 A 44-year-old alcoholic man comes to the clinic with vague fatigue for the past 6 months. His
7 other medical problems include depression, dental abscess, and onychomycosis He takes no
8 medications. Examination shows a thin man with spider angiomas on the chest and dullness in
9 the flanks. Abdominal ultrasound reveals splenomegaly , mild ascitic fl uid, and an echogenic ,
10 shrunken liver without obvious masses. Laboratory results are as follows
11
12 Live r function studies
13
14 Total bilirubin 2.1 mg/dL
15
Alkaline phosphatase 130 U/L
16
17 Aspartate
18 282 U/L
aminotransferase
19
20 Alanine aminotransferase 120 U/L
21
22
23
24 Complete blood count
25
Hemoglobin 11 0 g/L
26
27 Mean co rpuscular volume 104 fl
28
29 Platelets 105,000/IJL
30
31 Leukocytes 4,100/IJL
32
33
34
Coagulation studies
35
36 Prothrombin time 12 sec
37

--

38

40
Partial thromboplastin time 35 sec
2
Item: 39 of 40 ~'?Mark <J C> 61 ~ ~ , ~
0. ld : 2936 Previous Next Lab Values Notes Calculator Reverse Color Text Zoom
3
4
Alanine aminotransferase 120 U/L
5
6
7
8 Complete blood count
9
10 Hemoglobin 11 0 g/L
11
12 Mean corpuscular volume 104 fl
13
Platelets 105,000/J.!L
14
15 Leukocytes 4,100/J.!L
16
17
18
19 Coagulation studies
20
Prothrombin time 12 sec
21
22 Partial thromboplastin time 35 sec
23
24
25 Testing for hepatitis B and C infection is negative The patient appears concerned about his
26 health and is willing to follow medical advice. W hich of the following is the best next step in
27 management of his condition?
28
29
30 <0 A Abdominal computed tomography scan
31 <0 B. Antimitochondrial antibodies
32
33 <0 C . Esophageal endoscopy
34 <0 D. Liver biopsy
35
36
<0 E. Serum ammonia levels
37

--

38

40
Submit
2
Item: 39 of 40
0. ld : 2936
~'?Mark <]
Previous
C>
Next
a
Lab Values
~
Notes
~
Calculator
,
Reverse Color
~
Text Zoom
3
4
5 Explanation:
6
7
8
9 Management of Cirrhosis
10
11 Periodic surveillance of liver • U~rasoun<l surveillance for
functions tests Compensated hepatocellular carcinoma :1:
12
(eg, International Normalized alpha-fetoprotein every 6
13 Ratio, albumin, bBirubin) months
14 • EGO varices surveillance
15
16 Decompensated
17
18
19
20
21 Assess complications
22
23
24
25
26
Variceal hemorrhage Ascites Hepatic encephalopathy
27
Start non-selec live beta Dietary sodium restriction, Identify underlying cause (eg,
28 diuretics, paracentesis, infection, gastrointestinal
blockers, repeaI EGO every year
29 abstain from alcohol bleeding), lactulose therapy
30
©UWorld
31
32
This patient's presentation is most consistent with cirrhosis due to chronic alcohol abuse.
33
34 Patients with compensated cirrhosis typically are either asymptomatic or complain of vague
35 symptoms (eg, anorexia, weakness, fatigue). Those with decompensated cirrhosis may present
36 with jaundice, pruritus, upper gastrointestinal bleeding, abdominal distension due to ascites, or
37 confusion due to hepatic encephalopathy

--

38

40
Management goals in c irrhotic patients include identifying and treating reversible factors and
2
Item: 39 of 40 ~'?Mark <] C> !I ~ ~ , ~
0. ld : 2936 Previous Next Lab Values Notes Calculator Reverse Color Text Zoom
3
confusion due to hepatic encephalopathy
4
5
Management goals in c irrhotic patients include identifying and treating reversible factors and
6
7 potential complications (eg, variceal hemorrhage, hepatocellular carcinoma, hepatic
8 encephalopathy). Esophageal varices are the major cause of morbidity and mortality and can
9 occur in up to 50% of patients As a result, all patients with cirrhosis should undergo diagnostic
10 endoscopy to exclude varices, determine risk of va riceal hemorrhage, and indicate strategies (eg,
11 nonselective beta blockers) for primary prevention of variceal hemorrhage
12
13 {Choice A) This patient's ultrasound showed no liver masses suggestive of hepatocellular
14 ca rcinoma. As a result, he does not require triple-phase computed tomography scan of the
15 abdomen to visualize potential masses. However , all patients with cirrhosis should have an
16 ultrasound every 6 months to identify potential live r masses.
17
18 {Choice B) Antimitochondrial antibodies are usually positive in patients with primary biliary
19 cirrhosis ( PBC) Although PBC can cause cirrhosis, it usually is more common in women and
20
presents with significantly elevated alkal ine phosphatase out of proportion to aspartate and
21
22 alanine aminotransferase levels. This patient's lower alkaline phosphatase level makes PBC less
23 likely
24
25 {Choice D) Although live r biopsy is the gold standard for diagnosing cirrhosis, this patient's
26 alcohol history, comb ined with the laboratory and imaging findings, strongly suggests the
27 diagnosis without the need for biopsy.
28
29 {Choice E) Serum ammonia levels are usually checked in a cirrhotic patient with suspected
30 hepatic encephalopathy However , encephalopathy usually presents with confusion, neurologic
31 impairment (eg, asterixis), and/or sleep disturbances. This patient does not exhibit these
32 symptoms
33
34 Educational objective:
35 Patients with cirrhosis should undergo screening endoscopy to exclude varices, indicate the risk
36 of variceal hemorrhage, and determine strategies (eg, nonselective beta blockers) for primary
37

--
prevention of va riceal hemorrhage All patients with cirrhosis, regardless of etiology, should also
38
undergo surveillance for hepatocellular carcinoma with ultrasound every 6 months.
• 40
2
Item: 40 of 40 ~'?Mark <] C> !I ~ ~ , ~
0. ld: 41 50 Previous Next Lab Values Notes Calculator Reverse Color Text Zoom
3
4
5
6 A 23-year-old man comes to the physician because of a two-month history of loose stools,
7 decreased appetite, and weight loss. He has no history of medical problems He takes no
8 medications. His temperature is 36.7° C (98° F), blood pressure is 120/76 mmHg, pulse is
9 90/min, and respirations are 16/min. Laboratory studies show
10
11 Hemoglobin 11 .2 g/dL
12
MCV 80 fl
13
14 Leukocyte count 9,500/cmm
15
16 Segmented Neutrophils 65%
17
18 Bands 3%
19
Eosinophils 1%
20
21 Basophils 0%
22
23 Lymphocytes 25%
24
25 Monocytes 6%
26
27 Platelets 550,000/cmm
28
ESR 50 mm/hr
29
30 Serum sodium 145 mEq/L
31
32 Serum potassium 4.0 mEq/L
33
34
Test of the stool for occult blood is positive Which of the following is the most likely type of
35
36 diarrhea in this patient?
37
38

--
ID A Inflammatory
39
2
Item: 40 of 40 ~'?Mark <] C> !I ~ ~ , ~
0. ld: 41 50 Previous Next Lab Values Notes Calculator Reverse Color Text Zoom
3
~

4 MCV 80 fl
5
6 Leukocyte count 9,500/cmm
7
Segmented Neutrophils 65%
8
9 Bands 3%
10
11 Eosinophils 1%
12
13 Basophils 0%
14
Lymphocytes 25%
15
16 Monocytes 6%
17
18 Platelets 550,000/cmm
19
20 ESR 50 mm/hr
21
22 Serum sodium 145 mEq/ L
23
Serum potassium 4.0 mEq/L
24
25 -
26 Test of the stool fo r occult blood is positive Which of the following is the most likely type of
27 diarrhea in this patient?
28
29
30 <0 A Inflammatory
31 <0 B. Secretory
32
33 <0 C. Osmotic
34 <0 D. Motor
35
36
<0 E. F actitial
37
38

--
Submit
39
2
Item: 40 of 40 ~'?Mark <J C> 61 ~ ~ , ~
0. ld: 41 5 0 Previous Next Lab Values Notes Calculator Reverse Color Text Zoom
3
4 ., A Inflammatory [85%)
5
6 B. Secretory [6%)
7 C. Osmotic [6%)
8
D. Motor [1%)
9
10 E. Factitial [2%)
11
12
13 Explanation:
14
15 The clinical scenario described is characteristic fo r inflammatory diarrhea. The typical cause is
16 idiopathic inflammatory bowel disease (IBD). Infectious causes are less likely with chronic
17 diarrhea (duration > 4 weeks) than with acute diarrhea. In this case, the clues to the correct
18 diagnosis include weight loss, anemia, elevated ESR, and reactive thrombocytosis. The positive
19 occult blood test is also a very important clue. Furthermore, inflammatory changes in the blood
20 (e g , anemia, elevated ESR, acute phase reactants, reactive thrombocytosis) and
21 blood/leukocyte positive stool are usually absent in the other types of diarrhea mentioned.
22
23 (Choice B) Secretory diarrhea is usually the result of some medication use or hormonal
24
disturbances.
25
26 {Choice C) Osmotic diarrhea is caused by the ingestion of osmotically active, poorly absorbable
27
substances. Lactose intolerance is a classic example.
28
29 (Choice D) Motor diarrhea is exemplified by hyperthyroidism
30
31 (Choice E) Factitial diarrhea is typically associated with psychiatric disturbances.
32
33 Educational Objective:
34
Chronic inflammatory diarrhea is typically associated with inflammatory changes in the blood
35
(anemia, elevated ESR, acute phase reactants, reactive thrombocytosis)
36
37 Blood/leukocyte-positive stool is another important finding.
38

--
39

~
Time Spent 5 seconds Copyright © UWorld Last updated: [08/ 15/2016)

-------------------------------------------------------------------------------------------------------------------------------
Feedback SuWend EnQ ock
-
2
3
4
5
Item: 1 of40
0. ld: 4747
~'?Mark <]
Previous
C>
Next
!I
Lab Values
~
Notes
~
Calculator
,
Reverse Color
~
Text Zoom

6 A 44-year-old man comes to the emergency department for exertional dyspnea, abdominal
7 distention, and abdominal discomfort. He has a history of alcohol abuse, peptic ulcer disease,
8 and acute pancreatitis. Physical examination shows a distended abdomen with shifting dullness
9 and 1+ ankle edema. A diagnostic paracentesis is performed and shows the following:
10
11 Albumin 2.5 g/dL
12
13 Neutrophils 200/pL
14
Lactate dehydrogenase 40 units/L
15
16 Glucose 80 mg/dL
17
-
18 Amylase 40 units/L
19
20
Blood studies show the following
21
22 Albumin 3.8 g/dL
23
24 Potassium 3.2 mEq/L
25
26 Sodium 130 mEq/L
27
Glucose 120 mg/dL
28
29 Aspartate aminotransferase
30 50 units/L
(AST)
31
32 Alanine aminotransferase
33 40 units/L
(ALT)
34
35 100
36 Lactate dehydrogenase
units/ L
37
38 Amylase 80 units/L
39
40
-
2
3
4
5
Item: 1 of40
0. ld: 4747
~'?Mark <J
Previous
C>
Next
61
Lab Values
~
Notes
~
Calculator
,
Reverse Color
~
Text Zoom

Blood studies show the following:


6
7 Albumin 3.8 g/dl
8
9 Potassium 3.2 mEq/ L
10
11 Sodium 130 mEq/L
12 Glucose 120 mg/dl
13
14 Aspartate aminotransferase
15 50 units/L
(AST)
16
17 Alanine aminotransferase
18 40 units/L
(ALT)
19
20 100
Lactate dehydrogenase
21 units/L
22
23 Amylase 80 units/L
24
25 Creatinine 1.2 mg/dl
26
27 Which of the following is the most likely cause of this patient's ascites?
28
29
30 <0 A Decreased liver synthetic activity
31 <0 B. Increased capillary hydrostatic pressure
32
33 <0 C. Increased capillary permeability
34 <0 D. Inc reased glomerular filtration pressure
35
36
<0 E. Primary hyperaldosteronism
37
38 Submit
39
40
-
2
3
4
5
Item: 1 of40
0. ld: 4747

Explanation:
~'?Mark <]
Previous
C>
Next
a
Lab Values
~
Notes
~
Calculator
,
Reverse Color
~
Text Zoom

6
7
8
9 Ascites fluid characteristics
10
11
12 • Bloody: Trauma, malignancy, TB (rarely)
13
14 • Milky: Chylous, pancreatic
Color
15 • Turbid: Possible infection
16
• Straw color: Likely more benign causes
17
18
19 • <250/mm3 : No peritonitis
20 Neutrophils
21 • :2:250/mm 3 : Peritonitis (secondary or spontaneous bacterial)
22
23
• :2:2.5 g/dl (high-protein ascites)
24
25 o CHF, constrictive pericarditis, peritoneal carcinomatosis,
26 Total TB, Budd-Chiari syndrome, fungal (eg, coccidioidomycosis)
27 protein
• <2.5 g/dl (low-protein ascites)
28
29 o Cirrhosis, nephrotic syndrome
30
31
32 • :2:1.1 g/dl (indicates portal hypertension)
33 o Cardiac ascites, cirrhosis, Budd-Chiari syndrome
34 SAAG
35
• <1 .1 g/dl (absence of portal hypertension)
36 o TB, peritoneal carcinomatosis, pancreatic ascites,
37 nephrotic syndrome
38
39 CHF = congestive heart failure; SAAG = serum-to-ascites albumin gradient; TB = tuberculosis.
40
-
2
3
4
5
Item: 1 of40
0. ld: 4747
~'?Mark <]
Previous
C>
Next

Ascites can be due to portal hypertensive (eg, cardiac ascites, cirrhosis) or non-portal
!I
Lab Values
~
Notes
~
Calculator
,
Reverse Color
~
Text Zoom

hypertensive (eg, malignancy, pancreatitis, nephrotic syndrome, tuberculosis)


6
7 causes. Assessment of color , neutrophil count, total protein, and the serum-to-ascites albumin
8 gr adient (SAAG) in ascitic fluid analysis can help to diagnose the etiology
9
10 SAAG is calculated by subtracting the peritoneal fl uid albumin concentration from the serum
11 albumin concentration and is useful to differentiate between portal and non-portal hypertensive
12 etiologies A SAAG ~1.1 g/dl indicates portal hype rtension while a SAAG <1 .1 g/dl suggests
13 other causes. This patient's SAAG of 1.3 g/dl (3 8- 2.5 g/dl) is consistent with portal
14 hypertension (eg, cirrhosis) with increased hyd rostatic pressure within hepatic capilla ry beds.
15
16 {Choice A) Decreased liver synthetic function leads to decreased serum albumin concentration
17 and inc reased prothrombin time (INR). Hypoalbuminemia may contribute to edema in end-stage
18 live r disease, but this patient's normal serum albumin makes this less likely
19
20 (Choice C) Inc reased capillary permeability (eg, malignant ascites) typically leads to non-portal
21 hype rtensive ascites with a SAAG <1 .1 g/dl.
22
23 {Choice D) Increased glomerular filtration pressure usually increases the glomerular filtration
24 rate and decreases serum c reatinine. However, it typically does not cause ascites.
25
26 (Choice E) Although primary hyperaldosteronism causes sodium and water retention, it typically
27
does not cause significant ascites or edema due to spontaneous diuresis (aldosterone escape
28
phenomena)
29
30
Educational objective:
31
32 In evaluating ascites, a serum-to-ascites albumin gradient (SAAG) ~1 . 1 g/dl indicates portal
33 hypertensive etiologies (eg, ca rdiac ascites, cirrhosis) while a SAAG <1.1 suggests non-portal
34 hype rtensive etiologies (eg, malignancy, pancreatitis, nephrotic syndrome, tuberculosis)
35
36 References:
37
38 1. Ascites: diagnosis and management.
39 2. Serum-ascites albumin gradient in differential diagnosis of ascites.
40
-
3
4
5
6
1
Item: 2 of 40
0. ld : 4547
~'?Mark <J
Previous
C>
Next

A 33-year-old man is admitted to the hospital after an episode of tonic-clonic seizures. He is a


61
Lab Values
~
Notes
~
Calculator
,
Reverse Color
~
Text Zoom

7 known intravenous drug abuser. He admits using intravenous cocaine and heroin prior to the
8 seizure. He has never had seizures before. Following the seizure, he complains of muscle pain
9 but otherwise feels normal. His temperature is 37.2°C (98 9°F) , blood pressure is 156/90 mm Hg,
10 and respirations are 14/min. Laboratory analyses reveal the following:
11
12 Creatine phosphokinase 11,200
13 (CPK) U/L
14
15 AST 545 U/L
16
17 ALT 560 U/L
18
Troponin T normal
19
20 Anti-HAV antibodies negative -
21
22 HB,Ag negative
23
24 Anti-HB.Ag negative
25
26 lgM Anti-HB,Ag negative
27
28
Anti-HCV antibodies positive
29
30 He reports that a recent HIV test was negative W hich of the following is the most appropriate
31 management for this patient?
32
33
34 ~ A Fluid restriction
35 6 B. Metoprolol
36
37 ~ C. Hepatitis B vaccination
38 e; D. Lifelong phenytoin therapy
39
40
-
3
4
5
6
1
Item: 2 of 40
0. ld : 4547
-
~'?Mark
-
<J
Previous
C>
Next

seizure. He has never had seizures before. Following the seizure, he complains of muscle pain
but otherwise feels normal. His temperature is 37.2°C (98 9°F) , blood pressure is 156/90 mm Hg,
and respirations are 14/min. Laboratory analyses reveal the following:
61
Lab Values
~
Notes
~
Calculator
,
Reverse Color
~
Text Zoom

7
8 Creatine phosphokinase 11 ,200
9 (CPK) U/L
10
11 AST 545 U/L
12
ALT 560 U/L
13
14 Troponin T normal
15
16 Anti-HAV antibodies negative
17
18 HB.Ag negative
19
20 Anti-HB.Ag negative
21
22 lgM Anti-HB.Ag negative
23
24
Anti-HCV antibodies positive -
25
26 He reports that a recent HIV test was negative W hich of the following is the most appropriate
27 management fo r this patient?
28
29
30 <0 A Fluid restriction
31 <0 B. Metoprolol
32
33 <0 C. Hepatitis B vaccination
34 <0 D. Lifelong phenytoin therapy
35
36
<0 E. No intervention
37
38 Submit
39
40
-
3
4
5
6
1
Item: 2 of 40
0. ld : 4547
~'?Mark <]
Previous
C>
Next

This patient presents with evidence of rhabdomyolysis and a first seizure following simultaneous
use of cocaine and heroin (i.e , speedball) Cocaine and opiate intoxication both predispose to
!I
Lab Values
~
Notes
~
Calculator
,
Reverse Color
~
Text Zoom

seizures, and cocaine can also lead to rhabdomyolysis. The muscle breakdown can lead to
7
significant myoglobinuria and eventual renal fa ilure. As a result, this patient should initially
8
9
acutely be treated with aggressive fluid resuscitation to treat the rhabdomyolysis and prevent
10 renal fa ilure.
11
12 This patient also has elevated transaminases and positive anti- hepatitis C virus (HCV)
13 antibodies. Hepatitis C is transmitted by exposure to infected blood, intravenous drug use,
14 tattoos, and blood transfusions. Hepatitis B is also transmitted by infected blood (as well as by
15 sexual contact), and patients with risk factors for HCV are also at high risk for hepatitis B virus
16 (HBV) This patient is negative for both hepatitis B surface antigen (HB.Ag) and anti-HB.Ag,
17 indicating that he is neither infected with nor vaccinated against HBV. In addition to treating his
18
acute condition, hepatitis B vacc ination is important to prevent future HBV infection. Both HCV
19
and HBV can lead to chronic infection, cirrhosis, and an increased risk of liver cancer.
20
21
{Choice A) This patient's elevated serum creatine phosphokinase level indicates muscle damage,
22
23 with evidence of rhabdomyolysis from the tonic-clonic seizure and/or coca ine use. Aggressive
24 fluid resuscitation rather than restriction is required to prevent renal failure from myoglobinuria
25
26 (Choice B) Beta-blockers should be avoided in patients with coca ine intoxication because
27 unopposed alpha activation can worsen hypertension and lead to coronary vasospasm.
28
29 (Choice D) This patient's seizure was most likely drug induced. While central nervous system
30 imaging studies should be done to identify a possible mass lesion, chronic anticonvulsant therapy
31 is not indicated following a first episode of seizures.
32
33 {Choice E) This patient requires hospitalization with intravenous fluid resuscitation and
34 vaccination against hepatitis B.
35
36 Educational objective:
37 Patients with risk factors for hepatitis B virus should be vaccinated, which is especially important
38 in patients already infected with hepatitis C virus who are continuing their high-risk behaviors.
39
40
~'?Mark C> !I ~ ~ , ~

--
Item: 3 of 40 <]
2 0. ld : 4386 Previous Next Lab Values Notes Calculator Reverse Color Text Zoom

4
5
6 A 54-year-old man is brought to the emergency department after an episode of hematemesis and
7 lightheadedness He has a history of anemia, intravenous drug use, hepatitis C infection with
8 cirrhosis, and alcohol use. The patient says he has had no alcohol in about 5 days He takes no
9 medications. His temperatu re is 36.7 C (98 F) , blood pressure is 105/60 mm Hg, pulse is
10 110/min, and respirations are 16/min. He has scleral icterus. Examination shows blood in the
11 oropharynx. The lungs are clear to auscultation. The abdomen is distended with a fluid wave.
12
Laboratory results are as follows
13
14 Leukocytes 9,500/mm>
15
16 Hemoglobin 10.2 g/dL
17
18 Platelets 72,000/mm>
19
20
21 Serum sodium 140 mEq/L
22
23 Serum potassium 3.5 mEq/L
24
25 Bicarbonate 27 mEq/L
26
27 Blood urea nitrogen 34 mg/dL
28 0.8 mg/dL
Serum creatinine
29
30
31
32 International Normalized Ratio 1.8 (normal 0 8-1 1)
33
34 Partial thromboplastin time 40 sec
35
36 The patient receives normal saline through 2 large-bore intravenous lines and is started on
37 ceftriaxone. Which of the following is the most appropriate next step in management of this
38
patient?
39
40
~'?Mark <J C> 61 ~ ~ , ~

--
Item: 3 of 40
2 0. ld : 4386 Previous Next Lab Values Notes Calculator Reverse Color Text Zoom

4
Hemoglobin 10.2 g/dl
5
6 Platelets 72,000/mm>
7
8
9
10 Serum sodium 140 mEq/L
11
12 Serum potassium 3.5 mEq/L
13
Bicarbonate 27 mEq/L
14
15 Blood urea nitrogen 34 mg/dl
16
17 Serum creatinine 0.8 mg/dl
18
19
20
International Normalized Ratio 1.8 (normal 0 8-1 1)
21
22 Partial thromboplastin time 40 sec
23
24
25 The patient receives normal saline through 2 large-bore intravenous lines and is started on
26 ceftriaxone. W hich of the following is the most appropriate next step in management of this
27 patient?
28
29
30 <0 A Factor VIla transfusion
31 <0 B. Octreotide infusion
32
33 <0 C. Packed red blood cell transfusion
34 <0 D. Platelet transfusion
35
36
<0 E. Sengstaken-Biakemore balloon tamponade
37
38 Submit
39
40
~'?Mark C> a ~ ~ , ~

--
Item: 3 of 40 <]
2 0. ld : 4386 Previous Next Lab Values Notes Calculator Reverse Color Text Zoom

4
5
Suspected variceal hemorrh age I
6 ~
7
8 Place 2 large-bore IV catheters I
9
10
11 • Volume resuscitation
12
• IV octreotide
13
14 · Antibiotics
15
16 !
17 IUrgent endoscopic therapy of esophageal varices I
18
19
20
21
No further bleeding Continued bleeding [ Early rebleeding [
22
23
24
25 Initiate secondary Balloon tamponade Repeat endoscopic
26 prophylaxis (temporary) therapy
27
28
29
30 Beta blocker+
!
[ Recurrent hemorrhage
31 endoscopic band
32 ligation 1-2 weeks
33 later
34 +-I TIPS or shunt surg ery I•
35
36 IV = intravenous; TIPS = transjugular intrahepatic portosystemic shunt
37 euworld
38
39 This patient with hepatitis C and alcoholic liver disease has an upper gastrointestinal hemorrhage,
40
~'?Mark C> !I ~ ~ , ~

--
Item: 3 of 40 <]
2 0. ld : 4386 Previous Next Lab Values Notes Calculator Reverse Color Text Zoom

4 This patient with hepatitis C and alcoholic live r disease has an upper gastrointestinal hemorrhage,
5 most likely due to esophageal va rices. Initial treatment of suspected va riceal bleeding includes
6 volume resuscitation , through 2 to 3 large-bore peripheral intravenous lines. Prophylactic
7 antibiotics (eg, ceftriaxone) should be given to cirrhotic patients with gastrointestinal bleeding to
8
decrease infectious compl ications, recurrent bleeding, and mortality. Somatostatin analogues
9
10 (eg, octreotide) inhibit the release of vasodilator hormones, which leads indirectly to splanchnic
11 vasoconstriction and decreased portal flow.
12
13 Urgent endoscopy (within 12 hours) can diagnose and treat (eg, endoscopic band ligation,
14 sclerotherapy) active bleeding Patients with uncontrollable bleeding require temporary balloon
15 tamponade (eg, Sengstaken-Biakemore, Minnesota, Linton-Nachlas tubes) as a short-term
16 measure until more definitive therapy, including transjugular intrahepatic portosystemic shunt
17 (TIPS) or shunt surgery (Choice E) Patients without further bleeding after endoscopy can be
18 monitored and receive secondary prophylaxis (beta blocker) with repeat endoscopic band
19 ligation 1-2 weeks later.
20
21 (Choice A) Recombinant factor VIla is effective in treating some types of hemophilia However,
22 current studies have not shown significant benefit fo r co rrecting coagulopathy in active variceal
23
bleeding Fresh frozen plasma is commonly used for correcting coagulopathy of live r disease,
24
25 but it increases the risk of volume overload and may not adequately co rrect the coagulopathy
26
(Choice C) Current guidelines suggest keeping the hemoglobin >9 g/dl in va riceal hemorrhage.
27
28 This patient should have serial blood counts drawn and may require transfusion if the hemoglobin
29 decreases to <9 g/dl.
30
31 (Choice D) Platelet transfusions are generally reserved for patients with active bleeding and
32 platelet count <50,000/mm'. This patient's platelet count of 72,000/mm' does not require platelet
33 transfusion at this time.
34
35 Educational objective:
36 Treatment of actively bleeding esophageal varices involves hemodynamic support, pharmacologic
37 treatment (eg, octreotide), endoscopic therapy, and prophylactic antibiotics. Coagulopathy ,
38 anemia, and thrombocytopenia are common compl ications and may also require correction.
39
40
2
Item: 4 of40 ~'?Mark <] C> !J ~ ~ , ~

-
0. ld : 2199 Previous Next Lab Values Notes Calculator Reverse Color Text Zoom
3

5
6 A 62-year-old man comes to the office due to difficulty swallowing both solids and liquids. The
7 patient's symptoms have progressively worsened over the last 3 months. He also has occasional
8 regurgitation of undigested food and a nighttime cough that disturbs his sleep. He has never had
9 similar symptoms before. The patient has lost 4.5 kg (10 lb) during this period but has no other
10 medical problems and takes no medications. He has a 20-pack-year smoking history but quit 10
11 years ago He drinks wine on the weekends. Vital signs are normal, and his physical
12
examination is unremarkable. Chest x-ray reveals a widened mediastinum, and barium studies
13
14 show a dilated esophagus with smooth tapering of the distal esophagus Which of the following is
15 the best next step in management of this patient?
16
17 C) A. CT scan of the chest
18
19 C) B. Endoscopic evaluation
20
C) C. Esophageal pH monitoring
21
22 C) D. Laparoscopic myotomy
23
24 C) E. Reassurance and calcium channel blocker
25 C) F. Trial of proton pump inhibitor
26
27
28 Submit
29
30
31
32
33
34
35
36
37
38
39
40
2
Item: 4 of40 ~'?Mark <] C> !I ~ ~ , ~

-
0. ld : 2199 Previous Next Lab Values Notes Calculator Reverse Color Text Zoom
3
This patient's presentation, with dysphagia to solids and liquids and a dilated esophagus with
5
6 smooth tapering of the distal esophagus, suggests either primary achalasia (ie, loss of peristalsis
7 in the distal esophagus with lack of lower esophageal sphincte r relaxation) or pseudoachalasia
8 due to esophageal cance r. Several clues point to pseudoachalasia (eg, narrowing of distal
9 esophagus not due to denervation) caused by malignancy
10
11 Tobacco use is a major risk factor for esophageal adenocarcinoma and squamous cell
12 carc inoma; alcohol use is also an important risk factor for esophageal squamous cell ca rcinoma.
13 Significant weight loss, rapid symptom onset (<6 months), and presentation at age >60 all
14 increase the likelihood of malignancy (by comparison, patients with achalasia have symptoms for
15 approximately 5 years before receiving a diagnosis, and they typically only have mild weight
16
loss) Tumor metastasis (eg, mediastinal lymph nodes) or local involvement may gave a
17
18 radiologic appearance similar to that seen with a widened mediastinum. Endoscopic evaluation
19 can diffe rentiate between achalasia and pseudoachalasia. In achalasia, this evaluation usually
20 shows normal-appearing esophageal mucosa and a dilated esophagus with possible residual
21 material; in addition, it is generally possible to easily pass the endoscope through the lower
22 esophageal sphincter (unlike in malignancy)
23
24 (Choice A) If endoscopy shows a malignancy, aCT scan can be performed for staging. CT
25 scan can also be obtained if endoscopy is non revealing and there is still concern for malignancy
26
27 (Choices C and F) Esophageal pH monitoring is typically used to confirm gastroesophageal
28 reflux disease (GERD) in patients who have symptoms consistent with GERD but who fail a trial of
29 proton pump inhibitor (PPI) therapy. Although GERD can present with food regurgitation and
30 nighttime cough, it would not explain the patient's weight loss and dysphagia or the barium
31
esophagram findings In addition, PPis do not significantly improve the symptoms of achalasia.
32
33
(Choices D and E) Laparoscopic myotomy and pneumatic balloon dilation are the treatments of
34
choice for patients with achalasia who are at low surgical risk. Options for patients at high
35
36 surgical risk include botulinum toxin injection, nitrates, and calcium channel blockers. However ,
37 all these treatments should be considered only after malignancy is excluded and the diagnosis of
38 achalasia is confi rmed.
39
40
2
Item: 4 of40 ~'?Mark <] C> !I ~ ~ , ~

-
0. ld : 2199 Previous Next Lab Values Notes Calculator Reverse Color Text Zoom
3
can d1ffe rent1ate between achalasia and pseudoachalas1a. In achalasia, th1s evaluation usually
5 shows normal-appearing esophageal mucosa and a dilated esophagus with possible residual
6 material; in addition, it is generally possible to easily pass the endoscope through the lower
7 esophageal sphincter (unlike in malignancy)
8
9 (Choice A) If endoscopy shows a malignancy, aCT scan can be performed for staging. CT
10 scan can also be obtained if endoscopy is non revealing and there is still concern for malignancy
11
12 (Choices C and F) Esophageal pH monitoring is typically used to confirm gastroesophageal
13 reflux disease (GERD) in patients who have symptoms consistent with GERD but who fail a trial of
14 proton pump inhibitor (PPI) therapy. Although GERD can present with food regurgitation and
15 nighttime cough, it would not explain the patient's weight loss and dysphagia or the barium
16 esophagram findings In addition, PPis do not significantly improve the symptoms of achalasia.
17
18 (Choices D and E) Laparoscopic myotomy and pneumatic balloon dilation are the treatments of
19
choice for patients with achalasia who are at low surgical risk. Options for patients at high
20
surgical risk include botulinum toxin injection, nitrates, and calcium channel blockers. However ,
21
22 all these treatments should be considered only after malignancy is excluded and the diagnosis of
23 achalasia is confi rmed.
24
25 Educational objective:
26 Pseudoachalasia, which is due to narrowing of the distal esophagus secondary to causes other
27 than denervation (eg, esophageal cancer) , can closely mimic achalasia. Clues pointing to
28 pseudoachalasia include significant weight loss, rapid symptom onset, and presentation at age
29 >60. Consequently, endoscopy is recommended to exclude malignancy in all patients with
30 suspected achalasia.
31
32
References:
33
34 1. Achalasia.
35
36 2. Achalasia: A review of clinical diagnosis, epidemiology, treatment and outcomes.
37
38
39 Time Spent 3 seconds Copyright © UWorld Last updated: [05/23/2016)
40
2
Item: 5of40 ~'?Mark <] C> !J ~ ~ , ~
0. ld : 4752 Previous Next Lab Values Notes Calculator Reverse Color Text Zoom
3

-
4

6
7
8
9
A 45-year-old man with known cirrhosis due to hepatitis C is admitted to the hospital for
abdominal discomfort and confusion. His blood pressure is 95/60 mm Hg and his pulse is
100/min. Physical examination shows a distended abdomen, leg edema, and deep yellow
discoloration of the skin and sclerae. Bibasilar crackles are heard on chest auscultation. His
10 serum sodium level is 127 mEq/L, potassium is 2.9 mEq/L, and creatinine is 1.6 mg/dL On day
11 3 of his hospitalization, the serum creatinine is 3.5 mg/dL Urinalysis results are as follows
12
13 Protein Negative
14
15 Glucose Negative
16
17 W hite blood cells 4-5/hpf
18
Red blood cells 0-1 /hpf
19
20
21 Renal ultrasound is normal and the post-void residual urinary volume is <50 mL He is given
22 normal saline and albumin intravenously with no change in his serum creatinine concentration.
23 This patient's kidney dysfunction can be best co rrected by which of the following?
24
25
26 e') A. Angiotensin-converting enzyme inhibitors
27 6 B. Broad-spectrum antibiotics
28
29 6 C. Corticosteroids
30 6 D. High-dose spironolactone
31
e') E. Kidney transplantation
32
33 6 F. Liver transplantation
34
6 G. Low-dose dopamine
35
36 6 H. Pegylated interferon
37
38
39 Submit
40
2
Item: 5of40 ~'?Mark <J C> 61 ~ ~ , ~
0. ld : 4752 Previous Next Lab Values Notes Calculator Reverse Color Text Zoom
3

-
4 ., F. Liver transplantation [59%I
G. Low-dose dopamine [6%]
6
7 H. Pegylated interferon [4%]
8
9
Explanation:
10
11 Hepatorenal syndrome (HRS) is one of the most dangerous complications of end-stage liver
12
disease, occurring in up to 10% of patients with cirrhosis. HRS is characterized by decreased
13
14 glomerular filtration in the absence of shock, proteinuria, or other clear cause of renal
15 dysfunction, and a fa ilure to respond to a 1.5 L normal saline bolus. It is thought to result from
16 renal vasoconstriction in response to decreased total renal blood flow and vasodilatory substance
17 synthesis There are 2 subtypes of HRS. Type 1 is rapidly progressive; most patients die within
18 10 weeks without treatment Type 2 progresses more slowly , with an average survival of 3-6
19 months. The most common causes of death are infection and hemorrhage Unfortunately , no
20 medication has consistently proven beneficial in HRS and the mortality for these patients placed
21 on dialysis is very high Liver transplantation is the only intervention with established benefit
22
23 (Choice A) Unfortunately, angiotensin-converting enzyme inhibitors are of no benefit to patients
24
with HRS.
25
26 {Choice D) Diuretics may worsen HRS.
27
28 (Choice E) The kidneys of HRS patients are essentially normal on biopsy and renal function
29 improves with liver transplantation
30
31 (Choice G) Low-dose dopamine increases renal vasodilation but has not proven beneficial in
32
treating HRS in a number of clinical trials.
33
34
Educational objective:
35
Hepatorenal syndrome is a deadly complication of cirrhosis characterized by renal failure that
36
37 does not respond to volume resuscitation. It is best treated with liver transplantation
38
39 Time Spent 4 seconds Copyright© UWorld Last updated: [05/12/2016]
40
2
Item: 6 of 40 ~'?Mark <J C> 61 ~ ~ , ~
0. ld : 4389 Previous Next Lab Values Notes Calculator Reverse Color Text Zoom
3
4

-
5

7
8
9
10
A 63-year-old man comes to the physician after several weeks of vague abdominal pain and
increased fatigability His past medical history is not significant The patient smokes a pack of
cigarettes a day and consumes 5-6 cans of beer on weekends. His mother died of a stroke , and
his father died of a myocardial infa rction. Physical examination shows a mildly distended, soft
abdomen. The liver edge is hard and is palpated 5 em below the costal margin. Hepatojugular
11 reflux is not appreciated. Trace ankle edema is present Laboratory results are as follows :
12
13 Hemoglobin 9.0 g/L
14
15 Mean co rpuscular volume 70 fL
16
17 Platelets 200,000/1-fL
18
Leukocytes 4,500/1-fL -
19
20 Blood urea nitrogen ( BUN) 19 mg/dL
21
22 Serum creatin ine 0.8 mg/dL
23
24 Aspa rtate aminotransferase 48 U/L
25
Alanine aminotransferase 56 U/L
26
27 Alkaline phosphatase 320 U/ L
28
29
30 Coagulation studies are within normal limits. Chest x-ray reveals a small left-sided pleural
31 effusion. Fecal occult blood test is positive W hich of the following is the most likely cause of
32 hepatomegaly in this patient?
33
34
35 e') A Alcoholic cirrhosis
36 6 B. Autoimmune hepatitis
37
38 6 C. Hemochromatosis
39 6 D. Left ventricular fa ilure
40
2
Item: 6 of 40 ~'?Mark <J C> 61 ~ ~ , ~
0. ld : 4389 Previous Next Lab Values Notes Calculator Reverse Color Text Zoom
3
4 Hemoglobin 9.0 g/L ~

-
5

7
8
9
10
Mean corpuscular volume

Platelets
Leukocytes
70 fL

200,000/IJL

4,500/IJL

11 Blood urea nitrogen ( BUN) 19 mg/dL


12
13 Serum creatinine 0.8 mg/dL
14
Aspa rtate aminotransferase 48 U/L
15
16 Alanine aminotransferase 56 U/L
17
18 Alkaline phosphatase 320 U/L
19
20
Coagulation studies are within normal limits. Chest x-ray reveals a small left-s ided pleural
21
22 effusion. Fecal occult blood test is positive. W hich of the following is the most likely cause of
23 hepatomegaly in this patient?
24
25
ID A Alcoholic cirrhosis
26
27 ID B. Autoimmune hepatitis
28 ID C. Hemochromatosis
29
30 ID D. Left ventricular failure
31 ID E. Metastatic disease
32
33 ID F. Nonalcoholic steatohepatitis
34 ID G. Portal hypertension
35
ID H. Pulmonary hypertension
36
37
38 Submit
39
40
2
Item: 6 of 40 ~'?Mark <] C> !I ~ ~ , ~
0. ld : 4389 Previous Next Lab Values Notes Calculator Reverse Color Text Zoom
3
4 This patient- with abdominal pain, mic rocytic anemia, positive fecal occult blood, and

-
5

7
8
9
10
hepatomegaly with a hard edge on liver palpation- has typical features of gastrointestinal
malignancy , likely colon cancer , metastatic to the liver. The liver is the most common site of
metastasis in colon cancer. The moderate abnormalities in hepatic markers in this case (elevated
alkaline phosphatase and slightly elevated aspartate aminotransferase [AST] and alanine
aminotransferase [ALT]) are more suggestive of infiltrative or cholestatic disease than of
11 hepatocellular injury. The patient also has a small pleural effusion on the left that may be
12 malignant. (Hepatic hydrothorax due to cirrhosis usually occurs on the right.) This patient should
13 have a CT of the abdomen with intravenous contrast to evaluate for malignancy
14
15 (Choice A) In alcoholic cirrhosis, the liver is often shrunken and the edge is not palpable Also ,
16 in hepatocellular injury due to alcohol, AST is characteristically greater than ALT, usually by a
17 multiple of 1.5-2.0.
18
19 {Choice B ) Autoimmune hepatitis is typically seen in young to middle-aged women and may
20 present as acute or chronic hepatitis. There is significant hepatocellular injury, with
21 transaminases often above 1,000 U/L
22
23 {Choice C) Hemochromatosis is an autosomal recessive disease resulting in multiorgan
24 dysfunction from systemic iron overload. The most common sequelae include cirrhosis, heart
25
failure, diabetes, hypogonadism, and arthritis. It is typically associated with significant elevations
26
27 in transaminases.
28
(Choices 0 and H ) Left ventricular fa ilure does not directly cause hepatomegaly , though it may
29
30 eventually lead to right heart fa ilure. Pulmonary hypertension and right heart failure may lead to
31 congestive hepatopathy In such cases, other signs of right ventricular failure will be present:
32 peripheral edema, elevated jugular venous pressure, positive hepatojugular reflux, and an S3 on
33 cardiac exam.
34
35 (Choice F) Nonalcoholic steatohepatitis (NASH) is often asymptomatic and manifests as
36 hepatomegaly with elevated transaminases. Imaging will reveal fatty infiltration of the liver. The
37 leading causes of NASH are obesity, diabetes mellitus, and hype rtriglyceridemia This patient's
38 hard hepatomegaly , anemia, and positive fecal occult blood in the setting of minimal transaminase
39 elevations are more consistent with metastatic disease.
40
2
Item: 6 of 40 ~'?Mark <] C> !I ~ ~ , ~
0. ld : 4389 Previous Next Lab Values Notes Calculator Reverse Color Text Zoom
3
4 in hepatocellular injury due to alcohol, AST is characteristically greater than ALT, usually by a

-
5

7
8
9
10
multiple of 1.5-2.0.

(Choice B) Autoimmune hepatitis is typically seen in young to middle-aged women and may
present as acute or chronic hepatitis There is significant hepatocellular injury, with
transaminases often above 1,000 U/l.

11 {Choice C) Hemochromatosis is an autosomal recessive disease resulting in multiorgan


12 dysfunction from systemic iron overload. The most common sequelae include cirrhosis, heart
13 failure, diabetes, hypogonadism, and arthritis. It is typically associated with significant elevations
14
in transaminases.
15
16 (Choices 0 and H) Left ventricular failure does not directly cause hepatomegaly , though it may
17
eventually lead to right heart failure. Pulmonary hypertension and right heart failure may lead to
18
19 congestive hepatopathy In such cases, other signs of right ventricular fail ure will be present:
20 peripheral edema, elevated jugular venous pressure, positive hepatojugular reflux, and an S3 on
21 cardiac exam.
22
23 (Choice F) Nonalcoholic steatohepatitis (NASH) is often asymptomatic and manifests as
24 hepatomegaly with elevated transaminases. Imaging will reveal fatty infiltration of the live r. The
25 leading causes of NASH are obesity, diabetes mellitus, and hype rtriglyceridemia This patient's
26 hard hepatomegaly , anemia, and positive fecal occult blood in the setting of minimal transaminase
27 elevations are more consistent with metastatic disease.
28
29 (Choice G) Cirrhotic and noncirrhotic portal hypertension may result in ascites and significant
30 peripheral edema. Other signs of portal hypertension include esophageal varices, spider nevi,
31 palmar erythema, and caput medusa. Thrombocytopenia and coagulopathy are often seen.
32
33 Educational objective:
34
The most common site of colon cancer metastasis is the liver . Liver metastases manifest as right
35
upper quadrant pain, mildly elevated liver enzymes, and firm hepatomegaly The diagnosis is
36
37 confi rmed by CT of the abdomen.
38
39 Time Spent: 5 seconds Copyright © UWorld Last updated: [05/28/2016)
40
2
Item: 7of40 ~'?Mark <J C> 61 ~ ~ , ~
0. ld : 2198 Previous Next Lab Values Notes Calculator Reverse Color Text Zoom
3
4

...
5
6

8
9
A 45-year-old man comes to the clinic fo r evaluation of chronic diarrhea. He has lost almost 7 kg
(15 lb) over the past year He has no blood in the stool. A 24-hour stool collection shows fecal
fat content of 10 g/day (normal <6 g/day) Stool microscopy shows no pathogens and no
leukocytes Serum electrolytes and renal function are within normal limits. The patient is given
10 25 g oral 0-xylose solution, and his urinary excretion of 0-xylose at 5 hours is 1.2 g (normal
11 4.5-7.5 g). After 4 weeks of treatment with rifaximin, the 0-xylose test is repeated, and the
12
urinary excretion at 5 hours is 1.3 g Based on these findings, which of the following is the most
13
14 likely diagnosis in this patient?
15
16 e A Bacterial overgrowth
17
18 e B. Celiac disease
19
20
e C. Lactose intolerance
21 e 0 . Pancreatic insufficiency
22
23
e E. Terminal ileal disease
24
25
26 Submit
27
28
29
30
31
32
33
34
35
36
37
38
39
40
2
Item: 7of40
0. ld : 2198
~'?Mark <]
Previous
C>
Next
a
Lab Values
~
Notes
~
Calculator
,
Reverse Color
~
Text Zoom
3
4

...
5
6

8
9
Explanation:

0 -xylose test of proximal small intestinal absorption

10
11
12 t Intestinal ~ Intestinal
13 absorption of absorption of
14 d-xylose into the d-xylose into the
15 blood (shown via blood (shown via
16 high serum levels) low serum levels)
17
18
19
20
21
22
23
24
25
26
27
28
29 Minimal fecal High fecal
excretion excretion
30
31
32
t Excretion of ' ~ Excretion of
33
d·xylose in d·xylosein
34 urine (shown via urine (shown via
35 Normal high urine levels) Abnormal low urine levels)
36 ©UWortd
37
38 This patient has chronic diarrhea, steatorrhea, and weight loss suggesting malabsorption.
39 c r d. t 1b Ir d· h t · db t Ih t· t r
40

~ Feedback SuWend EnQock


2
Item: 7of40 ~'?Mark <] C> !I ~ ~ , ~
3
4
0. ld : 2198
.., .. '
Previous Next Lab Values Notes Calculator Reverse Color Text Zoom

...
5
6

8
9
This patient has chronic diarrhea, steatorrhea, and weight loss suggesting malabsor ption.
Celiac disease is a common cause of malabsorption and is characterized by atrophy of intestinal
villi in the proximal small bowel due to exposure to gluten-conta ining wheat products It is most
common in patients of Northern European descent, and prevalence increases with age
10 D-xylose is a monosaccharide that can be absorbed in the proximal small intestine without
11 degradation by panc reatic or brush border enzymes It is subsequently excreted in the urine. In
12
the 0-xylose test, the patient is given an oral dose of 0-xylose, with subsequent assay of urine
13
14 and venous blood. Patients with proximal small intestinal mucosal disease (eg, celiac disease)
15 cannot absorb the 0-xylose in the intestine, and urinary and venous 0-xylose levels will be low.
16 By contrast, patients with malabsorption due to enzyme deficiencies (eg, chronic pancreatitis) will
17 have normal absorption of 0-xylose (Choice D) A false-positive 0-xylose test (ie, decreased
18 urinary excretion of 0-xylose despite normal mucosal absorption) can be seen in patients with
19 delayed gastric emptying or impaired glomerular filtration. This patient has low urinary excretion
20 (ie, low mucosal absorption) of 0-xylose consistent with celiac disease.
21
22 (Choice A) Small intestinal bacterial overgrowth (SIBO) is characterized by alterations in the
23 flora of the small intestine, usually due to abnormal intestinal anatomy or motility . A false-positive
24
0-xylose test can be seen in SIBO due to bacterial fermentation of the ingested 0-xylose dose
25
before it can be absorbed. However , SIBO is unlikely in this case as treatment with rifaximin did
26
27 not significantly change 0-xylose absorption
28
29 (Choice C) Intestinal lactase deficiency causes diarrhea, abdominal pain, and flatulence after
30 consuming milk or milk-containing products. However , fecal fat and absorption of 0-xylose will be
31 normal.
32
33 (Choice E) Crohn disease is characterized by transmural inflammation of the gastrointestinal
34 tract, most commonly in the distal ileum. Patients may have steatorrhea due to abnormal
35 absorption of bile salts in the distal ileum, but 0-xylose is absorbed in the proximal small bowel
36 and unlikely to be affected.
37
38 Educational objective
39 0-xylose is a monosaccharide that is absorbed in the proximal small intestine without degradation
40
2
Item: 7of40 ~'?Mark <] C> !I ~ ~ , ~
0. ld : 2198 Previous Next Lab Values Notes Calculator Reverse Color Text Zoom
3
4

...
D-xylose is a monosaccharide that can be absorbed in the proximal small intestine without
5
degradation by pancreatic or brush border enzymes It is subsequently excreted in the urine. In
6
the 0-xylose test. the patient is given an oral dose of 0 -xylose. with subsequent assay of urine
8 and venous blood. Patients with proximal small intestinal mucosal disease (eg, celiac disease)
9 cannot absorb the 0-xylose in the intestine, and urinary and venous 0-xylose levels will be low.
10 By contrast, patients with malabsorption due to enzyme deficiencies (eg, chronic panc reatitis) will
11 have normal absorption of 0-xylose (Choice D) A false-positive 0-xylose test (ie, decreased
12 urinary exc retion of 0 -xylose despite normal mucosal absorption) can be seen in patients with
13 delayed gastric emptying or impaired glomerular filtration. This patient has low urinary excretion
14
(ie, low mucosal absorption) of 0 -xylose consistent with celiac disease.
15
16 (Choice A) Small intestinal bacterial overgrowth (SIBO) is characterized by alterations in the
17
flora of the small intestine, usually due to abnormal intestinal anatomy or motility A false-positive
18
19 0-xylose test can be seen in SIBO due to bacterial fermentation of the ingested 0 -xylose dose
20 before it can be absorbed. However , SIBO is unlikely in this case as treatment with rifaximin did
21 not significantly change 0 -xylose absorption
22
23 (Choice C) Intestinal lactase deficiency causes diarrhea, abdominal pain, and flatulence after
24 consuming milk or milk-containing products However , fecal fat and absorption of 0-xylose will be
25 normaL
26
27 (Choice E) Crohn disease is characterized by transmural inflammation of the gastrointestinal
28 tract, most commonly in the distal ileum. Patients may have steatorrhea due to abnormal
29 absorption of bile salts in the distal ileum. but 0-xylose is absorbed in the proximal small bowel
30 and unlikely to be affected.
31
32 Educational objective:
33 0 -xylose is a monosaccharide that is absorbed in the proximal small intestine without degradation
34
by pancreatic or brush border enzymes. Patients with small intestinal mucosal disease will have
35
impaired absorption of 0 -xylose. Patients with malabsorption due to enzyme deficiencies will
36
37 have normal absorption of 0 -xylose
38
39 Time Spent 3 seconds Copyright © UWo rld Last updated: [08/22/2016)
40
2
Item: 8of40 ~'?Mark <J C> 61 ~ ~ , ~
0. ld : 4303 Previous Next Lab Values Notes Calculator Reverse Color Text Zoom
3
4
5
6 A 47-year-old man comes to the emergency department after an episode of coffee ground

-
7

9
10
11
12
emesis. He has had upper abdominal discomfort for several months, which he describes as
"burning and fullness" that is relieved by food. Over the last week the patient has had black, tarry
stools associated with weakness and fatigue. His medical history is unremarkable. He takes no
prescription or over-the-counter medications. The patient smokes a pack of cigarettes a day and
occasionally drinks alcohol. His blood pressure is 124/82 mm Hg while supine and 110/70 mm
Hg while standing. Pulse is 102/min. On examination, the conjunctivae and palmar c reases
13
14 appear pale. Abdominal examination is unremarkable. Which of the following is most likely
15 increased in this patient?
16
17 e; A Aldosterone/renin ratio
18
19 e> B. Alkaline phosphatase
20 e; C. Blood urea nitrogen/c reatin ine ratio
21
22 e> D. Prothrombin time
23
24 e; E. Urine sodium excretion
25
26
S ubm it
27
28
29
30
31
32
33
34
35
36
37
38
39
40
2
Item: 8of40 ~'?Mark <] C> !I ~ ~ , ~
0. ld : 4303 Previous Next Lab Values Notes Calculator Reverse Color Text Zoom
3
D. Prothrombin time [9%)
4
5 E. Urine sodium excretion [2%)
6

-
7

9
10
11
12
Explanation:

This patient has abdominal pain, hematemesis, and melena, likely due to upper gastrointestinal
(GI) bleeding from a peptic ulcer. His tachyca rdia suggests at least mild volume depletion.
Patients with upper (but not lower) Gl bleeding often have an elevated blood urea nitrogen (BUN)
13 and elevated BUN/cr eatinine ratio. Possible causes include increased urea production from
14 intestinal breakdown of hemoglobin and increased urea reabsorption in the proximal tubule due to
15 associated hypovolemia
16
17 (Choice A) The aldosterone/renin ratio is elevated in primary hyperaldosteronism, in which
18
autonomous secretion of aldosterone suppresses renin release. Patients with acute Gl bleeding
19
will usually have proportionate increases in both plasma renin and aldosterone in response to
20
21 hypovolemia
22
(Choice B) Alkaline phosphatase is increased in biliary obstruction and skeletal disease with
23
24 increased osteoblast activity (eg, Paget disease). Mild elevations can be seen in inflammatory
25 bowel disease or intra-abdominal infections.
26
27 {Choice D) Prolonged prothrombin time can be seen with warfarin use, vitamin K deficiency,
28 certain hereditary coagulation disorders, antibiotic use, and liver disease.
29
30 {Choice E) The urine sodium (ie, fractional excretion of sodium [FENa)) is often low in patients
31 who are volume depleted as the kidney attempts to retain sodium to restore circulatory vol ume.
32 Inc reased FENa is seen in intrinsic renal disease.
33
34 Educational objective:
35 Patients with upper gastro intestinal bleeding often have an elevated blood urea nitrogen (BUN)
36 and elevated BUN/creatinine ratio, possibly due to increased urea production (from intestinal
37 breakdown of hemoglobin) and increased urea reabsorption (due to hypovolemia)
38
39
40
2
Item: 9 of40 ~'?Mark <] C> !I ~ ~ , ~
0. ld : 2982 Previous Next Lab Values Notes Calculator Reverse Color Text Zoom
3
4
5
6 A 67-year-old man comes to the hospital with acute-onset upper abdominal pain associated with
7 nausea and vomiting. The patient was discharged 24 hours earlier after he underwent elective

-
8

10
11
12
13
coronary angiography and stent placement due to left circumflex artery stenosis. He has been
taking his prescribed medications and reports no chest pain, shortness of breath, or flank pain.
His other medical problems include type 2 diabetes, hypertension, hypercholesterolemia, and
peripheral vascular disease. The patient does not use tobacco, alcohol, or illicit drugs His
temperature is 37.7 C (99 9 F) , blood pressure is 134/88 mm Hg, pulse is 92/min, and
respirations are 18/min. The abdomen is tender to palpation in the epigastric area. Bowel
14
sounds are decreased. The skin over the lower extremities and toes appears mottled,
15
16 reddish-blue, and reticulated but blanches on pressure Laboratory results are as follows
17 Complete blood count
18
19 Hemoglobin 13.1 g/dL
20
21 Platelets 206,000/mm'
22
23 Leukocytes 12,300/mm'
24
25
26 Serum chemistry
27
28 Creatinine 1.5 mg/dL
29
30
31
32 Liver function studies
33 Total bilirubin 1.0 mg/dL
34
35 Alkaline phosphatase 125 U/L
36
37 Aspartate aminotransfe rase 30 U/L
38
39 Alanine aminotransferase 28 U/L
40
2
Item: 9 of40 ~'?Mark <J C> 61 ~ ~ , ~
0. ld : 2982 Previous Next Lab Values Notes Calculator Reverse Color Text Zoom
3 II
4
5
6
7 Serum chemistry

-
8

10
11
12
13
Creatinine

Liver function studies


1.5 mg/dl

14 Total bilirubin 1.0 mg/dl


15
16 Alkaline phosphatase 125 U/L
17
18 Aspartate aminotransferase 30 U/L
19
Alanine aminotransferase 28 U/L
20
21 Lipase 1,841 U/L
22
23
24 CT scan of the abdomen reveals diffuse enlargement of the pancreas. Abdom inal ultrasonogram
25 reveals normal-sized gallbladder and common bile duct without gallstones. The patient's pain is
26 improved with a dose of intravenous morphine W hich of the following is the best therapy for this
27 patient?
28
29
30 <0 A Elective cholecystectomy
31 <0 B. Endoscopic retrograde cholangiopancreatography
32
33 <0 C . Intravenous antibiotics
34 <0 D. Intravenous corticosteroids
35
<0 E. Intravenous fluids and supportive ca re only
36
37
38 Submit
39
40
2
Item: 9 of40
0. ld : 2982
~'?Mark <]
Previous
C>
Next
a
Lab Values
~
Notes
~
Calculator
,
Reverse Color
~
Text Zoom
3
4 Explanation:
5
6
7

-
8

10
11
12
13



Acute pancreatitis

Chronic alcohol use (-40%)


Gallstones (-40%)
Hypertriglyceridemia
14 Etiology
• Drugs (eg, azathioprine, valproic acid, thiazides)
15 • Infections (eg, CMV, Legionella, Aspergillus)
16
• Iatrogenic (post-ERCP, ischemic/atheroembolic)
17
18
19 Diagnosis (requires 2 of the following)
20 • Acute epigastric pain radiating to the back
21 • t Amylase or lipase >3 times normal limit
22 • Abnormalities on imaging consistent with pancreatitis
23 Clinical
24 presentation Other findings
25
• ALT level >150 U/L suggests biliary pancreatitis
26
27 • Severe disease: Fever, tachypnea, hypoxemia,
28 hypotension
29
30 AlT =alanine aminotransferase; CMV = cytomegalovirus;
ERCP = endoscopic retrograde cholangiopancteatography.
31
32 @UWorld
33
34 This patient's abdominal pain, nausea, and elevated lipase are consistent with acute
35 pancreatitis He has no history of alcohol use, and his ultrasound reveals no gallstones. His
36 pancreatitis is likely due to cholesterol emboli. Patients with risk factors for aortic
37 atherosclerosis (eg, hypercholesterolemia, diabetes, peripheral vascular disease) who undergo
38 cardiac catheterization or a vascular procedure are at increased risk for cholesterol emboli as a
39 result of vascular manipulation These emboli can occlude blood vessels and cause the following
40
2
Item: 9 of40 ~'?Mark <] C> !I ~ ~ , ~
0. ld : 2982 Previous Next Lab Values Notes Calculator Reverse Color Text Zoom
3
4 atherosclerosis (eg, hypercholesterolemia, diabetes, peripheral vascular disease) who undergo
5 cardiac catheter ization or a vascular procedure are at increased risk for cholesterol emboli as a
6 result of vascular manipulation These emboli can occlude blood vessels and cause the following
7

-
8 • Skin manifestations Livedo reticularis (reticulated, mottled, discolored skin), blue toe
syndrome
10 • Kidney manifestations Acute kidney injury
11 • Gastrointestinal manifestations: Pancreatitis, mesenteric ischemia
12
13
14
Supportive care (eg, pain control, intravenous fl uids, bowel rest) is recommended for
uncorrectable causes of acute pancreatitis (eg, hypotension, ischemia, viruses,
15
16 atheroembolism). Most acute pancreatitis attacks are self-limiting and improve in 4-7 days with
17 conservative management This patient should receive nothing by mouth except essential
18 medications (ie, antiplatelet therapy to prevent stent thrombosis).
19
20 (Choices A and B) Elective cholecystectomy is generally indicated for patients with gallstones
21 and biliary colic or cholecystitis. Endoscopic retrog rade cholangiopancreatography is indicated
22 fo r sphincterotomy and stone removal in patients with gallstone pancreatitis and cholangitis or in
23 those who have high surgical risk for cholecystectomy This patient has no gallstones or common
24 bile duct dilation on ultrasound.
25
26 {Choice C) Prophylactic antibiotics are not routinely used in patients with acute pancreatitis
27 unless there is evidence of necrotizing panc reatitis with local infection (which is not seen on this
28
patient's CT scan)
29
30 {Choice D) Glucoco rticoids have no role in management of uncomplicated acute pancreatitis.
31
32 Educational objective:
33
Cholesterol embolism can occur after a vascular procedure (eg, ca rdiac catheterization) and may
34
35
lead to skin (eg, livedo reticularis, blue toe) or gastrointestinal (eg, mesenteric ischemia, acute
36 pancreatitis) complications Acute pancreatitis from uncorrectable causes (eg, ischemia,
37 atheroembolism) can be conservatively managed with analgesics and intravenous fluids.
38
39
40
2
Item : 10of40 ~'?Mark <J C> 61 ~ ~ , ~
0. ld: 41 65 Previous Next Lab Values Notes Calculator Reverse Color Text Zoom
3
4
5
6 A 65-year-old man comes to the physician with a 4-week history of weakness and vague
7 postprandial epigastric pain His past medical history is insignificant He does not take any
8 medications. The patient smokes 1 pack of cigarettes daily and drinks alcohol occasionally

--
9

11
12
13
14
Fecal occult blood test is positive. Gastroduodenoscopy shows an antral ulcer . Four of seven
biopsies taken from the margins of the ulcer are consistent with adenocarcinoma. W hich of the
following is the most appropriate next step in management?

<D A CTscan
15
16 <D B. Exploratory laparotomy
17
18
<D C. Helicobacter py lori testing
19 e; D. Laparoscopy
20
21 <D E. Serologic marke rs
22
23
24 Submit
25
26
27
28
29
30
31
32
33
34
35
36
37
38
39
40
2
Item : 10 of40
0. ld: 41 65
~'?Mark <]
Previous
C>
Next
a
Lab Values
~
Notes
~
Calculator
,
Reverse Color
~
Text Zoom
3
4
5
6
7 Staging of gastric adenocarcinoma
8

--
9

11
12
13
14
15
16
17
18
19 PET/CT, endoscopic ultrasound
20 Laparoscopy, CT chest
21 +/-Paracentesis/peritoneal lavage
22
23
24
Limited stage
25
26
27
28 Chemotherapy
Surgical resection
29 +/- Palliative surgery
30
© UWorld
31
32 For adenocarcinoma of the stomach, treatment options and prognosis are determined primarily
33
by the disease stage at the time of diagnosis Surgical removal of the affected tissues is the
34
35 mainstay of therapy, therefore early detection of gastric cance r improves the chances of
36 successful management However , almost 90% of patients with gastric cancer are diagnosed at
37 advanced stages (III-IV), at which point radical resection is very complicated or impossible For
38 this reason, evaluation of the extent of the cancer is the most important objective following initial
39 histologic diagnosis
40
2
Item : 10of40 ~'?Mark <] C> !I ~ ~ , ~
0. ld: 41 65 Previous Next Lab Values Notes Calculator Reverse Color Text Zoom
3
advanced stages {III-IV), at which point radical resection is very complicated or impossible For
4
5 this reason, evaluation of the extent of the cancer is the most important objective following initial
6 histologic diagnosis
7
8 A CT scan of the abdomen and pelvis is commonly employed fo r disease staging and is sensitive

--
9 for revealing metastases (especially in the liver) Depending on CT findings , additional staging
procedures, such as laparoscopy (Choice D), endoscopic ultrasound, chest CT, or positron
11 emission tomography/CT (PET/CT) may be necessary Patients diagnosed with limited-stage
12 disease may be considered for curative resection, but those with more advanced disease staging
13 are generally referred for palliative interventions (Choice B) Laparotomy is eventually required
14 fo r curative or palliative management in most patients, but initial CT imaging is needed to plan the
15
appropriate next steps in evaluation.
16
17 (Choice C) Helicobacter pylori infection is a significant risk factor for gastric adenocarcinoma
18
and lymphoma (mucosa-associated lymphoid tissue [MALT] lymphoma) Eradication is
19
20 recommended fo r patients with resectable disease to reduce the risk of developing a second
21 cancer. Eradication of H pylori causes remission in some patients with gastric MALT lymphoma,
22 but it is not curative fo r adenocarcinoma.
23
24 (Choice E) A number of serologic markers have been proposed for use in gastric cancer
25 staging, including ca rcinoembryonic antigen and cancer antigen (CA) 125. These tumor markers
26 are often elevated in patients with advanced disease, but studies have not found them to have any
27 significant clinical utility
28
29 Educational objective:
30 In gastric adenocarcinoma, tumor stage at the time of diagnosis determines prognosis and
31 treatment options A CT scan is the initial staging modality
32
33
34 References:
35 1. Modern surgical considerations for gastric cancer.
36
37 2. Modern oncological approaches to gastric adenocarcinoma.
38 3. Preoperative serum tumor marker levels in gastric cancer
39
40
2
Item : 11 of40 ~'?Mark <] C> !I ~ ~ , ~
0. ld : 2910 Previous Next Lab Values Notes Calculator Reverse Color Text Zoom
3
4
5
6 A 47-year-old woman comes to the office for follow-up of abnormal liver function test results. She
7 has a 3-year history of hypercholesterolemia treated with a stalin medication, but the medication
8 was discontinued 6 months ago when a moderate elevation in hepatic transaminases was noted.
9 The patient has no jaundice, abdominal pain, or nausea. Medical history is also notable for type

--
10

12
13
14
15
2 diabetes mellitus with stable diabetic nephropathy Current medications include metformin and
lisinopril, both of which she has taken for several years. She drinks 1 to 2 glasses of wine on
weekends and does not use tobacco or illicit drugs. Vital signs are normal. BMI is 33 kg/m'
Examination shows mild hepatomegaly. Laboratory results are as follows:
Albumin 4.0 g/dl
16
17 Total bilirubin 1.0 mg/dl
18
19 Alkaline phosphatase 121 U/L
20
21 Aspa rtate aminotransferase (SGOT) 82 U/L
22 Alanine aminotransferase (SGPT) 93 U/L
23
24
25
26 Antinuclear antibody negative
27
28 Smooth muscle antibody negative
29
30
31 HBsAg negative
32
33 Anti-HBs positive
34
35 Anti-HBc negative
36
37 Anti-HCV negative
38
39 Iron studies are normal. Which of the following is the most likely diagnosis?
40
2
Item : 11 of40 ~'?Mark <] C> !I ~ ~ , ~
0. ld : 2910 Previous Next Lab Values Notes Calculator Reverse Color Text Zoom
3 • I I ~ I I I
4
5 Total bilirubin 1.0 mg/dl
6
7 Alkaline phosphatase 121 U/L
8
Aspa rtate aminotransferase (SGOT) 82 U/L
9

--
10

12
13
14
15
Alanine aminotransferase (SGPT)

Antinuclear antibody
93 U/L

negative

16 Smooth muscle antibody negative


17
18
19 HBsAg negative
20
21 Anti-HBs positive
22
23 Anti-HBc negative
24
Anti-HCV negative
25
26
27 Iron studies are normal. W hich of the following is the most likely diagnosis?
28
29
30 <0 A Alcoholic liver disease
31 <0 B. Autoimmune hepatitis
32
33 <0 C . Chronic hepatitis B infection
34 <0 D. Nonalcoholic fatty liver disease
35
<0 E. Primary biliary cholangitis
36
37
38 Submit
39
40
2
Item : 11 of40
0. ld : 2910
~'?Mark <]
Previous
C>
Next
a
Lab Values
~
Notes
~
Calculator
,
Reverse Color
~
Text Zoom
3
4 " D. Nonalcoholic fatty liver di sease [87%]
5 E. Primary biliary cholangitis [7%)
6
7
8 Explanation:
9

--
10

12
13
14
15
Nonalcoholic fatty liver disease

• Hepatic steatosis on imaging or biopsy


16 Definition • Exclusion of significant alcohol use
17 • Exclusion of other causes of fatty liver
18
19
• Mostly asymptomatic
20
21 Clinical • Metabolic syndrome
22 features • +/- Steatohepatitis (AST/ALT ratio <1)
23 • Hyperechoic texture on ultrasound
24
25 • Diet & exercise
26 Treatment • Consider bariatric surgery if BMI ;:::35
27
28
AST = aspartate aminotransferase; AlT =alanine aminotransferase.
29
30 ©UWo ~d

31
32 This patient with abnormal liver function tests and hepatomegaly is obese and has additional
33 findings of metabolic syndrome, including diabetes mellitus and hyperlipidemia She does not
34 drink alcohol excessively and has normal iron studies and negative autoimmune and hepatitis
35 markers. This presentation is suggestive of nonalcoholic fatty liver disease (NAFLD), defined
36 as hepatic steatosis in the absence of other causes of secondary hepatic fat accumulation (eg,
37 alcohol) The incidence of NAFLD has risen along with the increase in obesity, with nearly half
38 of obese patients demonstrating some degree of steatosis on live r biopsy
39
40
2
Item : 11 of40 ~'?Mark <] C> !I ~ ~ , ~
0. ld : 2910 Previous Next Lab Values Notes Calculator Reverse Color Text Zoom
3
4 alcohol). The incidence of NAFLD has risen along with the increase in obesity, with nearly half
5 of obese patients demonstrating some degree of steatosis on liver biopsy
6
7 Most patients with NAFLD have hepatomegaly, but other stigmata of chronic liver disease are
8 rarely seen. Live r marke rs typically show mild elevations in aspartate aminotransferase (AST)
9 and alanine aminotransferase (A LT), with an AST/Al T r atio <1. Diagnosis is often made based

--
10

12
13
14
15
on laboratory and imaging findings (eg, hyperechoic texture on ultrasound), but it can be
confirmed with liver biopsy Hepatic fibrosis develops in 40% of individuals with NAFLD, and
cirrhosis in 10%-15%. The foundation of management includes weight loss and control of
metabolic risk factors. It is generally safe to continue statin therapy in patients with NAFLD.

16 (Choice A) The hepatic steatosis in NAFLD resembles that found in alcoholic liver disease.
17 However, alcoholic liver disease is characterized by AST predominance (AST/ALT ratio 2:1) in
18 contrast to the parallel rise in NAFLD. In addition, alcoholic live r disease is unlikely with light to
19 moderate alcohol intake (<15 drinks/wk for men, <10/wk for women) A standard drink is
20 equivalent to 12 oz of beer, 5 oz of wine, or 1.5 oz of 80-proof spirits (1 oz = 30 ml )
21
22 (Choice B) Antinuclear antibody titers are a sensitive marke r for autoimmune hepatitis, and
23 antismooth muscle titers are relatively sensitive and specific This patient has negative titers for
24 both autoantibodies.
25
26 (Choice C) This patient has a typical serologic pattern of hepatitis B immunization, with a
27 positive anti-HBs titer and negative markers for hepatitis B surface antigen and core antibody
28
29 (Choice E) Primary biliary cholangitis (cirrhosis) is characterized by destruction of the
30 intrahepatic bile ducts leading to bile stasis and cirrhosis. Clinical features include pruritus,
31
fatigue, elevated alkaline phosphatase levels, and positive antimitochondrial antibodies.
32
33 Educational objective:
34
Nonalcoholic fatty liver disease is characterized by hepatomegaly with mild elevations in liver
35
36 transaminases and alkaline phosphatase in the absence of other causes of secondary hepatic fat
37 accumulation (eg, alcohol). It is most common in patients with obesity and diabetes.
38 Management includes weight loss and control of metabolic risk factors.
39
40
2
Item : 12 of40 ~'?Mark <J C> 61 ~ ~ , ~
0. ld : 3 585 Previous Next Lab Values Notes Calculator Reverse Color Text Zoom
3
4
5
6 A 54-year-old woman comes to the physician due to worsening epigastric pain over the last 2
7 months. The pain, which is worse at night, was initially intermittent but is now somewhat constant
8 and radiates to the back. She has lost 6.8 kg (15 lb) over the past 3 months. The patient reports
9 no change in the color of stools or urine. Two years ago, she was diagnosed with idiopathic
10 chronic pancreatitis and started on pancreatic enzyme replacement therapy. The patient initially

-
11

13
14
15
16
had diarrhea, epigastric discomfort, and flatulence that improved with pancreatic enzymes She
does not use tobacco, alcohol, or illicit drugs Vital signs are within normal limits. Her BMI is 21
kg/m2 . There is no jaundice. Abdom inal examination demonstrates mild epigastric tenderness on
deep palpation. Laboratory results are as follows
Albumin 3.3 g/dl
17
18 Total bilirubin 0.9 mg/dl -
19
20 Direct bilirubin 0.4 mg/dl
21
22 Alkaline phosphatase 54 U/L
23
24 Aspa rtate
28 U/L
25 aminotransferase
26 Alanine aminotransferase 25 U/L
27
28 Amylase 45 U/L
29
30 31 U/L (0-40
Lipase
31 U/L)
32
33
34 W hich of the following is the most appropriate next step in management of this patient?
35
36 6 A Cancer-associated antigen testing
37
38 6 B. CT scan of the abdomen with contrast
39 e; C. Endoscopic retrog rade cholangiopancreatography
40
2
Item : 12 of40 ~'?Mark <] C> !J ~ ~ , ~
3
4
.. . . .... . .. . . .
0. ld : 3 585
. .. . .. .
Previous
.. "'
Next
. ' . Lab Values Notes Calculator Reverse Color Text Zoom

kg/m2 There is no jaundice Abdominal examination demonstrates mild epigastric tenderness on


5 deep palpation Laboratory results are as follows
6
7 Albumin 3.3 g/dl
8
9 Total bilirubin 0.9 mg/dl
10

-
Direct bilirubin 0.4 mg/dl
11
Alkaline phosphatase 54 U/L
13
14 Aspartate
15 28 U/L
aminotransferase
16
17 Alanine aminotransferase 25 U/L
18
19 Amylase 45 U/L
20
31 U/L (0-40
21 Lipase
22 U/ L)
23
24 W hich of the following is the most appropriate next step in management of this patient?
25
26
27 0 A Cancer-associated antigen testing
28 0 B. CT scan of the abdomen with contrast
29
30 0 C. Endoscopic retrograde cholangiopancreatography
31 0 D. Intensification of pancreatic enzyme replacement therapy
32
33 0 E. Plain abdominal radiography
34 0 F. Reassurance and opioid therapy
35
0 G. Secretin panc reatic function testing
36
37
38 Submit
39
40
2
Item : 12 of40
0. ld : 3 585
~'?Mark <]
Previous
C>
Next
a
Lab Values
~
Notes
~
Calculator
,
Reverse Color
~
Text Zoom
3
4
5 Explanation:
6
7
8
9 Pancreatic adenocarcinoma
10

-
11

13
14
15
16
Risk factors




Smoking
Hereditary pancreatitis
Nonhereditary chronic pancreatitis
Obesity & lack of physical activity
17
18 • Systemic symptoms (eg, weight loss, anorexia) (>85%)
19 • Abdominal pain/back pain (80%)
20
21
Clinical • Jaundice (56%)
22 presentation • Recent-onset atypical diabetes mellitus
23 • Unexplained migratory superficial thrombophlebitis
24 • Hepatomegaly & ascites with metastasis
25
26
27 • Cholestasis (I alkaline phosphatase & direct bilirubin)
Laboratory
28
studies • t Cancer-associated antigen 19-9 (not as a screening test)
29 • Abdominal ultrasound (if jaundiced) or CT scan (if no jaundice)
30
31 @UWorld
32
33 This patient's worsening epigastric pain and weight loss in the setting of idiopathic chronic
34
pancreatitis suggest pancreatic cancer. The presentation and workup vary depending on tumor
35
location. Classic manifestations include weight loss and insidious onset of gnawing abdominal
36
37 pain (usually epigastric and radiating to the back), which is worse at night, with eating, or when
38 lying supine
39 I I I
40

~ Feedback SuWend EnQ ock


2
Item : 12 of40 ~'?Mark <] C> !I ~ ~ , ~
0. ld : 3 585 Previous Next Lab Values Notes Calculator Reverse Color Text Zoom
3 • • I I ,. J ... I I ,. ., I ., I .. I I .. I .. ' H '' I ... .. I H ... .. I I H ...
4 lying supine
5
6 • Cancers in the head of the pancreas (60%-70%) typically present with jaundice (common
7 bile duct obstruction, elevated alkaline phosphatase and bilirubin) and steatorrhea
8 (pancreatic exocrine insufficiency or pancreatic duct blockage). In patients with these
9
findings , abdominal ultrasound is preferred for detecting pancreatic head tumors and
10

-
excluding other potential causes of biliary obstruction (eg, choledocholithiasis)
11
• Cancers in the body and tail usually do not present with obstructive jaundice. Abdominal
13 CT scan is preferred (more sensitive and specific) and helps exclude other conditions.
14 Ultrasound is less sensitive fo r visualizing the pancreatic body and tail (due to overlying
15 bowel gas) and for detecting smaller (<3 em) tumors.
16
17
(Choice A) Cancer-associated antigen 19-9, a tumor marker associated with pancreatic cance r,
18
19
is not recommended for screening due to va riable sensitivity and low specificity. Normal levels do
20 not rule out pancreatic cancer.
21
22 (Choice C) Endoscopic retrograde cholangiopancreatography is more invasive than CT scan,
23 and is reserved for patients with cholestasis (from the tumor compressing the biliary system) who
24 may require an intervention (eg, stenting)
25
26 {Choice D) Pancreatic enzyme replacement therapy is typically titrated based on the patient's
27 clinical symptoms and stool fat content However, this patient does not need a dose adjustment
28 as her symptoms (eg, bloating, diarrhea) improved with initial therapy
29
30 {Choice E) Plain abdominal radiographs may show calcifications in chronic pancreatitis, but are
31 not useful ·for diagnosing panc reatic cancer. In addition, calcifications are rarely seen in
32 idiopathic pancreatitis
33
34 (Choice F) Opioid therapy may be necessary in some patients with chronic pancreatitis
35 However, reassurance and symptomatic management are not appropriate in this patient with
36 suspected pancreatic cancer.
37
38 {Choice G) The secretin test directly measures the ability of pancreatic ductal cells to produce
39 b. b t It . f I . d. I I h I tT b t t h I I f I . I f I I 'bl
40

~ Feedback SuWend EnQ ock


2
Item : 12 of40 ~'?Mark <] C> !I ~ ~ , ~
0. ld : 3 585 Previous Next Lab Values Notes Calculator Reverse Color Text Zoom
3
4
5 {Choice D) Pancreatic enzyme replacement therapy is typically titrated based on the patient's
6
clinical symptoms and stool fat content However, this patient does not need a dose adjustment
7
8 as her symptoms (eg, bloating, diarrhea) improved with initial therapy
9
10
(Choice E) Plain abdominal radiographs may show calc ifications in chronic panc reatitis, but are

-
11 not useful for diagnosing pancreatic cancer. In addition, calcifications are rarely seen in
idiopathic pancreatitis
13
14 (Choice F) Opioid therapy may be necessary in some patients with chronic pancreatitis
15 However, reassurance and symptomatic management are not appropriate in this patient with
16 suspected pancreatic cance r.
17
18 (Choice G) The secretin test directly measures the ability of pancreatic ductal cells to produce
19 bica rbonate. It is useful in diagnosing chronic panc reatitis, but not helpful in evaluating possible
20 pancreatic cance r.
21
22 Educational objective:
23 Panc reatic cance r should be suspected in patients with a history of chronic pancreatitis who
24 develop abdominal pain and we ight loss. Patients with jaundice should undergo abdominal
25 ultrasound to rule out cancer in the head of the pancreas. Patients without jaundice are likely to
26
have cancer in the body and tail of the pancreas, and should undergo abdominal CT scan.
27
28
29 References:
30
1. Endoscopic ultrasonography for pancreatic cancer: Current and future
31
32
perspectives.
33 2. Pancreatitis and pancreatic cancer risk: A pooled analysis in the International
34 Pancreatic Cancer Case-Control Consortium (PanC4).
35
36 3. Imaging of pancreatic cancer: An overview.
37
38
39 Time Spent 4 seconds Copyright © UWorld Last updated: [04/ 18/2016)
40
2
Item: 13 of40 ~'?Mark <J C> 61 ~ ~ , ~
0. ld : 2965 Previous Next Lab Values Notes Calculator Reverse Color Text Zoom
3
4
5
6 A 34-year-old man is admitted to the hospital with sudden onset of severe epigastric pain and
7 vomiting He has no significant medical history. The patient smokes half a pack of cigarettes a
8 day He takes no medications and does not use alcohol or illicit drugs His father died of a heart
9 attack at age 42. The patient is married but has no children. His blood pressure is 144/92 mm
10 Hg and pulse is 11 0/min. On examination, there is marked epigastric tenderness. Bowel sounds
11

-
are decreased. Crops of yellow-red papules are seen on the extensor surfaces of his arms and
12
shoulders. Laboratory results are as follows:
14 Total bilirubin 0.7 mg/dl
15
16 Aspartate aminotransferase
33 U/L
17 (SGOT)
18
19 Alanine aminotransferase (SPOT) 28 U/L
20
21 Alkal ine phosphatase 101 U/L
22 3,500 U/L (0-160
23 Lipase
U/L)
24
25
26 An abdominal ultrasound reveals no evidence of gallstones The common bile duct appears
27 normal. W hich of the following is most likely to reveal the cause of this patient's current condition?
28
29
30 e> A. CFTR genetic testing
31 6 B. Endoscopic retrograde cholangiopancreatography
32
33 6 C. Fasting lipid profile
34 6 D. Serology fo r viral titers
35
36
e> E. Urine toxicology screen
37
38
Submit
39
40
2
Item: 13 of40
0. ld : 2965
~'?Mark <]
Previous
C>
Next
a
Lab Values
~
Notes
~
Calculator
,
Reverse Color
~
Text Zoom
3
4
5 Explanation:
6
7
8
9 Acute pancreatitis
10
11

-
12 • Chronic alcohol use (-40%)
• Gallstones (-40%)
14 • Hypertriglyceridemia
15 Etiology
• Drugs (eg, azathioprine, valproic acid, thiazides)
16 • Infections (eg, CMV, Legionella, Aspergillus)
17
18 • Iatrogenic (post-ERCP, ischemic/atheroembolic)
19
20 Diagnosis (requ ires 2 of the following)
21 • Acute epigastric pain radiating to the back
22 • t Amylase or lipase >3 times normal limit
23 • Abnormalities on imaging consistent with pancreatitis
24 Clinical
25 presentation Other findings
26
27 • ALT level >150 U/L suggests biliary pancreatitis
28 • Severe disease: Fever, tachypnea, hypoxemia,
29 hypotension
30
31 ALT = alanine aminotransferase; CMV = cytomegalovirus;
32 ERCP = endoscopic retrograde cholangiopancreatography.
33 @UWorld
34
35 This patient's epigastric pain, vomiting, and markedly elevated lipase are consistent with acute
36 pancreatitis The most common causes of acute pancreatitis include alcohol (which this patient
37 does not use) and gallstones (which were not seen on the ultrasound). Other possible causes
38 include hypertriglyceridemia (third most common), medications (eg, azathioprine, valproic acid,
39 thiaz ide diuretics), infections (eg, cytomegalovirus [CMV)), recent endoscopic retrograde
40
2
Item: 13 of40 ~'?Mark <] C> !I ~ ~ , ~
0. ld : 2965 Previous Next Lab Values Notes Calculator Reverse Color Text Zoom
3
p y I p p p I I gg p
4
5
xanthomas, which are due to subcutaneous fat deposition Eruptive xanthomas are usually
6 associated with marked hypertriglyceridemia (>1 ,000 mg/dl), typically due to familial
7 hypertriglyce ridemia (which may also explain the patient's father's myocardial infarction at age
8 42). A fasting serum lipid profile can determine if hypertriglyceridemia is the underlying cause
9 of this patient's xanthomas and pancreatitis
10
11 (Choice A) Abnormalities in the CFTR gene lead to cystic fib rosis (CF), which is a potential

-
12

14
15
16
17
cause of pancreatitis. However , this patient likely would have had pulmonary manifestations of
CF long before age 34. In addition, CF does not typically cause eruptive xanthomas.

(Choice B) ERCP is usually done to evaluate biliary panc reatitis. Although patients with biliary
panc reatitis may sometimes have a normal ultrasound (especially if the stone is passed), they
typically have an elevated alanine aminotransferase (ALT) level (>150 U/L) This patient's normal
18
ALT level makes biliary pancreatitis less likely ERCP should also be considered in patients with
19
>1 episode of acute pancreatitis of unknown cause.
20
21
{Choice D) Acute panc reatitis has been associated with viruses (eg, mumps, hepatitis B, HIV,
22
23 coxsackievirus, CMV, herpes simplex virus), but patients often develop other characteristic
24 findings of particular viral infections. Viral serology is generally not recommended as the
25 significance of a positive result is unknown, and there is no potential treatment for most of these
26 viruses.
27
28 (Choice E) Cannabis use has been associated with case reports of acute pancreatitis, but it
29 would not cause eruptive xanthomas. Other illicit drugs usually do not cause acute pancreatitis.
30
31 Educational objective:
32 Acute pancreatitis is most commonly due to chronic alcohol use and gallstones Marked
33 hypertriglyceridemia (>1 ,000 mg/dl) can lead to acute panc reatitis and can cause eruptive
34 xanthomas. The diagnosis can be confirmed with a fasting serum lipid profile.
35
36
37 References:
38 1. Acute pancreatitis: etiology, clinical presentation, diagnosis, and therapy.
39
40
. . . .. . . . . . . . . . .. . . .. . .. .. . .

~ Feedback SuWend EnQ ock


2
Item : 14 of 40 ~'?Mark <J C> 61 ~ ~ , ~
0. ld : 2335 Previous Next Lab Values Notes Calculator Reverse Color Text Zoom
3
4
5
6 A 42-year-old man is evaluated for a 3-month history of burning substernal chest pain after every
7 meal. He has tried several over-the-counte r antacids with partial relief. He undergoes upper
8 gastrointestinal endoscopy, which shows mucosal irregularity and ulce ration of the
9 squamocolumnar junction above the lower esophageal sphincter . Multiple biopsies are taken.
10 Four hours after the procedure, the patient develops worsening substernal and left-sided pain
11 radiating to the back, along with mild shortness of breath. His temperature is 37.1 C (98 9 F) ,
12

-
blood pressure is 110/70 mm Hg, pulse is 120/min, and respirations are 34/min. Chest x-ray
13
reveals a small, left pleural effusion that was not present on a chest radiograph taken 2 weeks
15 ago. ECG shows sinus tachycardia but is otherwise unremarkable. W hich of the following is the
16 most appropriate test to confirm the diagnosis?
17
18
® A. Repeat upper Gl endoscopy
19
20 ® B. Serum amylase and lipase levels
21
22 ® C. Thoracocentesis
23 ® D. Transthoracic echocardiogram
24
25 ® E. Water-soluble contrast esophagogram
26
27
28 Submit
29
30
31
32
33
34
35
36
37
38
39
40
2
Item : 14 of 40
0. ld : 2335
~'?Mark <]
Previous
C>
Next
a
Lab Values
~
Notes
~
Calculator
,
Reverse Color
~
Text Zoom
3
4
5 Clinical features of esophageal perforation
6
7 • Spontaneous rupture (Boerhaave syndrome)
8
9
• Instrumentation (eg, endoscopy)
Etiology
10 • Esophagitis (infectious/pills/caustic)
11
12
• Esophageal ulcer

-
13

15
16
17
18
Clinical
presentation
• Chest & abdominal pain, systemic findings (eg, fever)
• Subcutaneous emphysema in the neck
• Hamman sign (crunching sound on chest auscultation)
19
20 • CXR or CT scan: Wide mediastinum, pneumomediastinum,
21 pneumothorax, air around paraspinal muscles, pleural
22 effusion (late)
23 Diagnosis
24 • CT scan: Esophageal wall thickening, mediastinal air fluid level
25 • Water-soluble contrast esophagogram: Leak at
26 perforation site
27
28
29 • Antibiotics & supportive care for all patients
30 Management • Surgical repair for significant leakage with systemic
31
inflammatory response
32
33
34 CXR = chest x-ray.
35 © UWond
36
37 This patient has features of acute esophageal rupture, including acute chest pain, tachypnea,
38 and pleural effusion (often left-sided) Patients also frequently have subcutaneous emphysema.
39 Endoscopy is the most common cause of esophageal rupture (especially when performed with
40
2
Item : 14 of 40 ~'?Mark <] C> !I ~ ~ , ~
0. ld : 2335 Previous Next Lab Values Notes Calculator Reverse Color Text Zoom
3
4
p pg p • g p yp
5
and pleural effusion (often left-sided) Patients also frequently have subcutaneous emphysema.
6 Endoscopy is the most common cause of esophageal rupture (especially when performed with
7 additional interventions, such as dilation of strictures), although rupture can also be due to severe
8 retching ( Boerhaave syndrome) or penetrating trauma.
9
10 Esophageal rupture is a life-threatening condition that requires emergent evaluation. A contrast
11 esophagram is recommended to confirm the diagnosis and can demonstrate contrast
12 extravasation through the tear. Water -soluble contrast is preferred (less inflammatory to

-
13

15
16
17
18
tissues), but a barium study has higher sensitivity and would be advised if the initial water-soluble
contrast study is nondiagnostic . If perforation is confi rmed, primary closure of esophagus and
drainage of mediastinum must be attempted urgently to prevent the development of mediastinitis.

{Choice A) Endoscopy is avoided in most cases if there is concern for esophageal rupture
because the insufflated air or the tip of the endoscope could extend the rupture
19
20 (Choice B) Serum amylase and lipase levels are elevated in acute pancreatitis (which sometimes
21
occurs following endoscopic retrograde cholangiopancreatography) Pancreatitis can cause a
22
23 left pleural effusion but is less likely in this patient given his lack of typical epigastric pain and his
24 symptoms that abruptly worsened following endoscopy Pleural fluid (rather than serum) amylase
25 would be elevated in esophageal rupture
26
27 (Choice C) Thoracocentesis is not usually performed as part of the workup for esophageal
28 rupture, especially with a small pleural effusion. Salivary amylase in the pleural fluid could
29 confi rm the diagnosis but would be less sensitive than a contrast esophagram and less useful in
30 locating the tear in preparation for potential surgery.
31
32 {Choice D) Urgent transthoracic echocardiogram is indicated to evaluate suspected ca rdiac
33 tamponade Signs of tamponade would include hypotension, jugular venous distension,
34 low-voltage QRS on ECG, and enlarged cardiac silhouette on chest x-ray
35
36 Educational objective:
37 Esophageal perforation can be due to endoscopy, severe retching, or penetrating trauma.
38
Characteristic clinical findings include acute chest pain, subcutaneous emphysema, and
39
'"'I 1'"'1 I ... ., '"'I II ! II ., .. 111 ., 1 ., u ., 1 II II I'"' I .. t i l
40

~ Feedback SuWend EnQ ock


2
Item : 15of40 ~'?Mark <J C> 61 ~ ~ , ~
0. ld : 3 588 Previous Next Lab Values Notes Calculator Reverse Color Text Zoom
3
4
5
6 A 40-year-old woman comes to the physician with a 4-week history of abdominal symptoms. She
7 complains of upper abdominal discomfort and occasional dull epigastric pain accompanied by
8 nausea. Her symptoms are often worse after eating, especially with large meals. She sometimes
9 wakes up at night to have a snack but does not awaken with pain or have significant nocturnal
10 symptoms. The patient has no unexpected weight loss, heartburn, dysphagia, gross blood in the
11 stool, or melena. She lives with her mother, who has a history of gastric ulcer. Her vital signs are
12
within normal limits. Examination shows no abnormalities. Test of the stool for occult blood is
13

--
14 negative Laboratory results are as follows:
Hemoglobin 13.5 g/dl
16
17 0.6
18 Total bilirubin
mg/dl
19
20 Alkaline phosphatase 72 U/L -
21
22 Aspartate
24 U/ L
23 aminotransferase
24
25 Alanine
22 U/L
26 aminotransferase
27
125
28 Ferritin , serum
29 ng/ml
30
31 W hich of the following is the most appropriate next step in management of this patient?
32
33
34 e') A Abdom inal ultrasound
35 6 B. A ntacids and metoclopramide
36
37 6 C. Barium esophagram
38 6 D. Helicobacter pylori stool antigen testing
39
40
2
Item : 15of40 ~'?Mark <J C> 61 ~ ~ , ~
0. ld : 3 588 Previous Next Lab Values Notes Calculator Reverse Color Text Zoom
3
4 nausea. Her symptoms are often worse after eating, especially with large meals. She sometimes
5 wakes up at night to have a snack but does not awaken with pain or have significant nocturnal
6 symptoms. The patient has no unexpected weight loss, heartburn, dysphagia, gross blood in the
7 stool, or melena. She lives with her mother, who has a history of gastric ulce r. Her vital signs are
8 within normal limits. Examination shows no abnormalities. Test of the stool for occult blood is
9
negative Laboratory results are as follows:
10
11 Hemoglobin 135 g/dl
12
13 0.6

--
14 Total bilirubin
mg/dl

16 Alkaline phosphatase 72 U/L


17
18 Aspa rtate
24 U/L
19 aminotransferase
20
21 Alanine
22 U/ L
22 aminotransferase
23
24 Ferritin , serum
125 -
ng/ml
25
26
27 W hich of the following is the most appropriate next step in management of this patient?
28
29
30 <0 A Abdominal ultrasound
31 <0 B. Antacids and metoclopramide
32
33 <0 C . Barium esophagram
34 <0 D. Helicobacter pylori stool antigen testing
35
36
<0 E. Upper gastrointestinal endoscopy
37
38 Submit
39
40
2
Item : 15 of40
0. ld : 3 588
~'?Mark <]
Previous
C>
Next
a
Lab Values
~
Notes
~
Calculator
,
Reverse Color
~
Text Zoom
3
4
5 Explanation:
6
7
8 Dyspepsia
9
10
11 Alarm
12 .---------------,symptoms
13 Typical GERD N$AID/COX·2 NoGERD or .---------,

--
Endoscopy
14 symptoms inhibitor use NSAID/COX·2 inhibitor use

16
17
18
19
I Acid suppression I Discontinue agent or
add acid suppression
20
21
22
23 Yes
24
25
26
Posit ive
27 H. pylori testing J Treat for H. pylori
28
29 Negative
30
31
32
33
IPPI trial 4-6 weeks I
USMlEWorld.llC (S) 2011
34
35 This patient has epigastric fullness or discomfort after eating, features typical of
36 dyspepsia Dyspepsia is often associated with nausea and symptoms of gastroesophageal reflux
37 disease (GERD) such as heartburn. In addition to GERD, common causes of dyspepsia include
38
nonsteroidal anti-inflammatory drugs, gastric or esophageal cancer , functional dyspepsia, and
39
Ill 111 ., •'"' If.,. .. I I I .. II o I I t II 1'"'1 .. I• I ... I .,111 I
40

~ Feedback SuWend EnQock


2
Item : 15of40 ~'?Mark <] C> !I ~ ~ , ~
0. ld : 3 588 Previous Next Lab Values Notes Calculator Reverse Color Text Zoom
3 - ------ - --------- -------- -------- -- -- --- ----- ---- --
4 disease (GERD) such as heartburn. In addition to GERD, common causes of dyspepsia include
5 nonsteroidal anti-inflammatory drugs, gastric or esophageal cancer, functional dyspepsia, and
6 symptomatic Helicobacter pylori infection (with or without peptic ulcer) Definitive diagnosis of
7 dyspepsia often requires upper gastrointestinal endoscopy, but this test is invasive and relatively
8
expensive, especially given the benign nature of most of the potential causes. Patients who
9
should be considered for endoscopy include those age >55, as well as those with alar m
10
11 symptoms, including weight loss, gross or occult bleeding, anemia, dysphagia, persistent
12 vomiting, or early satiety (Choice E)
13

--
14 Most younger patients without alarm symptoms can be managed with a trial of antisecretory
therapy, either a proton pump inhibitor (PPI) or H2 blocker. A PPI is preferred fo r patients with
16 dyspepsia associated with use of nonsteroidal anti-inflammatory drugs. Patients with an
17 increased risk fo r H pylori, such as those from regions with high prevalence or who have
18 household exposure, may elect to pursue a "test-and-treat" strategy A test fo r active H pylori
19 infection (breath test or stool assay, but not serology) is administered, and patients receive
20 appropriate antibiotic therapy if positive Regardless of alarm symptoms, patients who fail to
21
improve after initial measures should undergo endoscopy.
22
23
(Choice A) Abdominal ultrasound is an effective imaging modality for biliary disease. Patients
24
with symptomatic gallstones typically report intermittent, cramping, right upper-quadrant
25
26 pain Atypical cases may present with epigastric discomfort, and ultrasound may be appropriate
27 fo r this patient if evaluation of dyspepsia is not diagnostic
28
29 (Choice B) Patients who have resolution of symptoms with PPI therapy may change to step-
30 down therapy with antacids. However, empiric treatment with antacids and metoclopramide is not
31 as effective as a PPI for patients with dyspepsia.
32
33 {Choice C) Barium esophagram is inferior to upper endoscopy for evaluation of dyspepsia and
34 is not generally recommended fo r the initial investigation.
35
36 Educational objective:
37 Patients with dyspepsia who have risk factors fo r gastric or esophageal cancer- age >55, weight
38 loss, gross or occult bleeding, anemia, dysphagia, or early satiety- should be evaluated with
39 upper gastrointestinal endoscopy Patients with nonsteroidal anti-inflammatory drug-induced
40
2
Item : 15of40 ~'?Mark <] C> !I ~ ~ , ~
0. ld : 3 588 Previous Next Lab Values Notes Calculator Reverse Color Text Zoom
3
4 inc reased risk fo r H pylori, such as those from regions with high prevalence or who have
5 household exposure, may elect to pursue a "test-and-treat" strategy A test fo r active H pylori
6 infection (breath test or stool assay, but not serology) is administered, and patients receive
7 appropriate antibiotic therapy if positive Regardless of alarm symptoms, patients who fail to
8 improve after initial measures should undergo endoscopy .
9
10 (Choice A) Abdominal ultrasound is an effective imaging modality for biliary disease. Patients
11 with symptomatic gallstones typically report intermittent, cramping, right upper-quadrant
12 pain Atypical cases may present with epigastric discomfort, and ultrasound may be appropriate
13

--
fo r this patient if evaluation of dyspepsia is not diagnostic
14
(Choice B) Patients who have resolution of symptoms with PPI therapy may change to step-
16
down therapy with antacids. However, empiric treatment with antacids and metoclopramide is not
17
18 as effective as a PPI for patients with dyspepsia.
19
(Choice C) Barium esophagram is inferior to upper endoscopy for evaluation of dyspepsia and
20
21 is not generally recommended fo r the initial investigation.
22
23 Educational objective:
24 Patients with dyspepsia who have risk factors fo r gastric or esophageal cancer - age >55, weight
25 loss, gross or occult bleeding, anemia, dysphagia, or early satiety - should be evaluated with
26 upper gastro intestinal endoscopy Patients with nonsteroidal anti-inflammato ry drug-induced
27 dyspepsia should receive a proton pump inhibitor. Patients from high-prevalence areas or with
28 possible exposure to Helicobacter pylori may have testing fo r active infection and treatment as an
29 initial step
30
31
32 References:
33 1. Evaluation and treatment of dyspepsia.
34
35 2. Helicobacter pylori " test-and-treat" strategy for management of dyspepsia: a
36 comprehensive review.
37
38
39 Time Spent 5 seconds Copyright © UWorld Last updated [06/ 11/2016]
40
2
Item: 16 of40 ~'?Mark <J C> 61 ~ ~ , ~
0. ld : 3086 Previous Next Lab Values Notes Calculator Reverse Color Text Zoom
3
4
5
6 A 65-year-old male complains of fatigue for the past one month. He has been living alone since
7 the death of his wife one year ago He does not like to cook, and his diet consists mostly of
8 canned food , hot tea, and toast He drinks one glass of white wine nightly with dinner.
9 Examination shows pallor of the skin, nail beds and conjunctiva. Rectal exam is heme-negative.
10 His physical examination is otherwise unremarkable. Adm inistration of which of the following
11 would be most helpful in treating his condition?
12
13
14 e A Vitamin 812

-
15

17
18
19
20
e B. Folic acid
e C. Vitamin C
e D. Vitamin D
e E. Vitamin E
21
22 e F. Vitamin A
23
24
e G. Selenium
25 e H. Copper
26
27
28 Submit
29
30
31
32
33
34
35
36
37
38
39
40
2
Item: 16 of40 ~'?Mark <J C> 61 ~ ~ , ~
0. ld : 3086 Previous Next Lab Values Notes Calculator Reverse Color Text Zoom
3
4 D. V itamin D [1%)
5
6 E. Vitamin E [1%)
7 F. Vitamin A [1%)
8
G. Selenium [1%)
9
10 H. Copper [0%)
11
12
13 Explanation:
14

-
15 This patient's pale skin, nail beds and conjunctiva suggest anemia. His diet shows an obvious
lack of folate, a nutrient typically found in foods like fresh green leafy vegetables and liver.
17 Furthermore, folate is heat-sensitive, so cooked foods are typically lacking In the setting of
18 dietary deficiency, folic acid stores can become depleted within 4-5 months, leading to
19 decreased R8C production and macrocytic anemia.
20
21 (Choice A) Strict vegetarians (vegans) may become deficient in vitamin 812. Vitamin 812
22 deficiency can cause anemia accompanied by neurologic deficits (in contrast to folate
23 deficiency, which causes anemia alone) The body has 812 stores sufficient to last at least 3-4
24
years.
25
26 {Choice C) Vitamin C deficiency can cause scurvy Patients present with perifoll icular
27
hemorrhage, swollen gums, and poor wound healing
28
29 (Choice D) Vitamin D deficiency causes osteomalacia and hypocalcemic tetany in adults.
30
31 (Choice E) Vitamin E deficiency can cause R8C fragility, hyporeflexia, muscle weakness, and
32
blindness.
33
34
Educational objective:
35
A tea and toast type of diet is associated with folic acid deficiency Folic acid is heat sensitive.
36
37 Folic acid deficiency causes macrocytic anemia.
38
39 Time Spent 2 seconds Copyright © UWorld Last updated: [06/24/2016)
40
2
Item : 17 of 40 ~'?Mark <] C> !I ~ ~ , ~
0. ld : 2975 Previous Next Lab Values Notes Calculator Reverse Color Text Zoom
3
4
5
6 A 42-year-old man comes to the physician complaining of fatigue and dark urine. His past
7 medical history is insignificant and he takes no medications. He used intravenous drugs in the
8 remote past The patient is an ex-smoker with 25-pack-year history. His father died of colon
9 cancer at age 52. His temperature is 37° C (98 6° F), blood pressure is 122/70 mm Hg, and
10 heart rate is 80/min. His body mass index is 30 kg/m2 The sclera and skin are icteric . The
11 abdomen is nondistended and nontender. Laboratory results are as follows:
12
13 Live r function studies
14
15 Albumin 3.8 g/dl

-
16

18
19
20
21
Total bilirubin

Direct bili rubin

Alkaline phosphatase
11.0 mg/dl

8.0 mg/dl

822 U/L
-

22 Aspa rtate aminotransferase


55 U/L
23 (SGOT)
24
25 Alanine aminotransferase (SGPT) 40 U/L
26
27
28 Coagulation studies
29
30 11
31 Prothrombin time
seconds
32
33
34 W hich of the following is the best next step in establishing the diagnosis?
35
36
e') A Endoscopic retrograde cholangiopancreatogram
37
38 6 B. Per ipheral smear examination
39 6 C Serum iron transferrin saturation and ferritin levels
40
2
Item : 17 of 40 ~'?Mark <J C> 61 ~ ~ , ~
0. ld : 2975 Previous Next Lab Values Notes Calculator Reverse Color Text Zoom
3
heart rate is 80/min. His body mass index is 30 kg/m The sclera and skin are icteric. The
2.
4
5 abdomen is nondistended and nontender. Laboratory results are as follows
6 Liver function studies
7
8 Albumin 3.8 g/dl
9
10 Total bilirubin 11 0 mg/dl
11
12 Direct bilirubin 8.0 mg/dl
13
Alkaline phosphatase 822 U/L
14
15 Aspartate aminotransferase

-
16

18
19
20
21
(SGOT)
Alanine aminotransferase (SGPT)
55 U/L

40 U/L

22 Coagulation studies
23
24 11
Prothrombin time
25 seconds
26
27 Which of the following is the best next step in establishing the diagnosis?
28
29
30 <0 A Endoscopic retrograde cholangiopancreatogram
31 <0 B. Peripheral smear examination
32
33 <0 C. Serum iron, transferrin saturation, and fe rritin levels
34 <0 D. Ultrasonography of the abdomen
35
<0 E. Viral hepatitis panel and inflammatory markers
36
37
38 Submit
39
40
2
Item : 17of40
0. ld : 2975
~'?M k
ar
<]
Previous
C>
Next
a
Lab Values
~
Notes
~
Calculator
,
Reverse Color
~
Text Zoom
3 o yper •
4
• pp o c • •
5
Possible causes
6
7 Overproduction (eg, hemolysis)
8 Mainly Reduced uptake (eg, drugs,
9 L-H_Y_P_er_b_il,
ir_ub_i_n_e_
m_ia_i-un-c-
on-J-ug~a-tro
___. portosystemic shunt)
10 Mainly Conjugation defect (eg,
11 conjugated Gilbert's syndrome)
12
13
14 Evaluate liver
15 enzyme pattern

-
16

18
19
20
21
Predominantly elevated Normal AST, ALT, alkaline Predominantly elevated
AST&ALT phosphatase alkaline phosphatase
22
23 • Viral hepatitis . Dubin·Johnson syndrome • Cholestasis of pregnancy
24 . Autoimmune hepatitis • Rotor's syndrome • Malignancy
25 • Toxin/drug-related (eg, pancreas, ampullary)
26 hepatitis . Cholangiocarcinoma
27 . Hemochromatosis . Primary biliary cirrhosis
28 . Ischemic hepatitis . Primary sclerosing cholangitis
29
30
. Alcoholic hepatitis . Choledocholithiasis
31
32
33
34 Abdominal imaging
35 (ultrasound or CT)
36 AlT = alanine transaminase: AST =aspartate transaminase. Antimltochondrial antibody
37 © UWorld
38
This patient has elevated conjugated (direct) hyperbilirubinemia along with elevated levels of
39
40 . .
.. .. .. ... .. .
.. .. .. ... .. .. ... .. ... ..
.. .. .. .
.. .. .. .. ... .. . . . ..
~ Feedback SuWend EnQ ock
2
Item : 17 of 40 ~'?Mark <] C> !I ~ ~ , ~
0. ld : 2975 Previous Next Lab Values Notes Calculator Reverse Color Text Zoom
3
4 @ - - -

5 This patient has elevated conjugated (direct) hyperbilirubinemia along with elevated levels of
6
alkaline phosphatase. This pattern of abnormal live r function is consistent with cholestasis in the
7
setting of extrahepatic or intrahepatic biliary obstruction. The next appropriate step in
8
9 management for patients with predominantly cholestatic pattern is to obtain an abdominal
10 ultrasound to assess hepatic parenchyma and biliary ducts. The presence of biliary dilatation is
11 suggestive of extrahepatic cholestasis; the absence of biliary dilatation suggests intrahepatic
12 cholestasis.
13
14 {Choice A) Endoscopic retrograde cholangiopancreatogram (ERCP) is usually performed in
15 patients when initial ultrasonography or computed tomography scan suggests the presence of

-
16

18
19
20
21
obstruction due to cholelithiasis or malignancy. ERCP in these settings can be both diagnostic
and therapeutic by relieving obstruction and facilitating biliary drainage

(Choice B) Examination of the peripheral blood smear is useful in the evaluation of patients with
suspected hemolytic anemias. However, patients with hemolysis have elevated unconjugated
(indirect) hyperbilirubinemia
22
23 (Choice C) Measurement of serum iron, transferrin saturation, and ferritin levels is useful in the
24
diagnosis of iron overload syndromes, including hereditary hemochromatosis. Patients with
25
26 hereditary hemochromatosis have progressive iron deposition in the liver and elevated serum
27 aminotransferases (AST and ALT)
28
29 (Choice E) Patients with acute viral hepatitis typically have marked elevation of serum aspartate
30 and alanine aminotransferase levels. Chronic viral hepatitis can be associated with mild to
31 moderate elevation of serum aminotransferases. This patient has normal aminotransferase levels
32 and is unlikely to have viral hepatitis as a cause of hyperbilirubinemia
33
34 Educational objective:
35 Elevated serum alkaline phosphatase levels are indicative of cholestasis. These patients (with or
36 without hyperbilirubinemia) should be evaluated with right upper-quadrant ultrasound to assess fo r
37 intrahepatic or extrahepatic causes of biliary obstruction.
38
39
40
..... • ... , • ... • • • •• • , @ :a. • .. .... ... a: 1 1 •

~ Feedback SuWend EnQ ock


2
Item : 18 of40 ~'?Mark <J C> 61 ~ ~ , ~
0. ld : 2581 Previous Next Lab Values Notes Calculator Reverse Color Text Zoom
3
4
5
6 A 64-year-old man comes to the emergency department due to diarrhea and intermittent
7 abdominal cramps for the last 6 weeks. He has had 4 or 5 bowel movements every day , and the
8 stool contains blood and mucus. He also reports poor appetite and fatigue and has lost 5.9 kg
9 (13 lb) over this period The patient has intermittent right knee swelling and tenderness that
10 responds to ibuprofen. He has not taken any antibiotics recently . He has no other medical
11 problems The patient last saw a physician 2 years ago, and his last colonoscopy 4 years ago
12
was normaL Family history is positive for colon cance r in his mother. Temperature is 37.3 C
13
14 (99 1 F) , blood pressure is 142/84 mm Hg, pulse is 98/min, and respirations are 16/min.
15 Physical examination shows tenderness in the left lower quadrant without rebound or guarding
16 Digital rectal examination is positive fo r blood. Laboratory results are as follows:

--
17

19
20
21
22
Leukocytes

Hemoglobin

Platelets
13,600 mm'
9.6 mg/dL

31 4,000/mm>

23 Erythrocyte sedimentation
126 mm/hr
24 rate
25
26
Flexible sigmoidoscopy reveals erythematous friable mucosa extending from the rectum to the
27
28
sigmoid colon. Small shallow ulce rs are also seen. W hich of the following is the most likely
29 cause of this patient's current condition?
30
31 6 A Amebic dysentery
32
33 6 B. Clostridium difficile colitis
34 e; C. Colon cance r
35
36 6 D. Ischemic colitis
37 6 E. Small-vessel vasculitis
38
39 6 F. Ulcerative colitis
40
2
Item : 18 of40
0. ld : 2581
~'?Mark <]
Previous
C>
Next
a
Lab Values
~
Notes
~
Calculator
,
Reverse Color
~
Text Zoom
3
4
5 Explanation:
6
7
8
9 Ulcerative colitis
10
11
12 • Bloody diarrhea
Symptoms
13 • Weight loss, fever
14
15
16 • Erythema, friable mucosa

--
17

19
20
21
22
Endoscopic
findings



Pseudopolyps
Involvement of rectosigmoid
Continuous colonic involvement (no skip lesions)

Biopsy • Mucosal & submucosal inflammation


23 • Crypt abscesses
24
25
26 • Toxic megacolon
27 • Primary sclerosing cholangitis
28 Complications • Colorectal cancer
29
• Erythema nodosum, pyoderma gangrenosum
30
31 • Spondyloarthritis
32 @UWorld
33
34
This patient's symptoms are concern ing for ulcerative colitis (UC) Similar to Crohn disease
35
(CD), most patients first develop UC at age 15-40; however, there appears to be a bimodal
36
37 distribution, with a second peak at age 50-80. UC is characterized by chronic idiopathic
38 inflammation of the colon and rectum with intermittent exacerbations. Patients commonly have
39 abdominal pain, bloody diarrhea, tenesmus, fecal incontinence, and, in more severe cases,
40
2
Item : 18 of40 ~'?Mark <] C> !I ~ ~ , ~
0. ld : 2581 Previous Next Lab Values Notes Calculator Reverse Color Text Zoom
3 I II I H ., ... I I t • ., ' ., -.~ 1 '"' I il I ., ., .. '"'Itt I I 11 ..
4 inflammation of the colon and rectum with intermittent exacerbations. Patients commonly have
5
abdominal pain, bloody diarr hea, tenesmus, fecal incontinence, and, in more severe cases,
6
7 weight loss. UC may be complicated by toxic megacolon The most common extraintestinal
8 manifestations include arthritis (seen in this patient), uveitis, episcleritis, erythema nodosum, and
9 primary sclerosing cholangitis.
10
11 Laboratory findings can include leukocytos is, iron deficiency anemia, reactive thrombocytosis,
12 and elevated inflammatory markers (eg, erythrocyte sedimentation rate [ESR)). Endoscopy may
13 demonstrate erythematous and friable mucosa with ulcers, as seen in this patient Exudates,
14 edema, and spontaneous bleeding may also be evident The lesions are usually continuous and
15 circumferential (unlike in CD). Biopsy shows inflammation limited to the mucosa and submucosa
16

--
(compared to transmural inflammation in CD) with crypt abscesses and c rypt distortion.
17
(Choice A) Amebic colitis also commonly presents with bloody diarrhea, abdominal pain, and
19 elevated inflammatory markers but is typically less acute. In addition, endoscopy commonly
20
shows skip lesions as opposed to contiguous areas of inflammation and ulcers.
21
22 (Choice B) Clostridium difficile infection is classically associated with pseudomembranes on
23
24 endoscopy It usually causes a pancolitis, presents with watery diarrhea, and occurs most
25 commonly in patients with a history of recent antibiotic use.
26
27 (Choice C) Although this patient's age, weight loss, and anemia are concerning for colon
28 cancer, his recent negative colonoscopy , lack of masses on endoscopy, and elevated ESR make
29 this much less likely Moreover, diarrhea is not a common presentation of colon cance r.
30
31 (Choices D and E) Patients with chronic colonic ischemia or small-vessel vasculitis can also
32 present with bloody diarrhea and weight loss and may be initially misdiagnosed with UC.
33 However , the significant elevation in ESR and the involvement of the rectum, which has a dual
34 blood supply , make ischemia due to atherosclerosis less likely Moreover , this patient's lack of
35 other findings of small-vessel inflammation (eg, skin, pulmonary, or renal involvement) reduces
36 the likelihood of vasculitis.
37
38 Educational objective:
39 Th t b b. d I d. t b t IT 'th
I I f I d I k t I 50 80
40

~ Feedback SuWend EnQock


2
Item : 18 of40 ~'?Mark <] C> !I ~ ~ , ~
0. ld : 2581 Previous Next Lab Values Notes Calculator Reverse Color Text Zoom
3 - . - ,. ,. ••• ,. - ., • -- . • ,. • ,. •• - - , . - - - • ,. - I ,.., • • '•' <I e
4 circumferential (unlike in CD). Biopsy shows inflammation limited to the mucosa and submucosa
5 (compa red to transmural inflammation in CD) with crypt abscesses and crypt distortion.
6
7 {Choice A) Amebic colitis also commonly presents with bloody diarrhea, abdominal pain, and
8 elevated inflammatory markers but is typically less acute. In addition, endoscopy commonly
9
shows skip lesions as opposed to contiguous areas of inflammation and ulcers.
10
11
(Choice B) Clostridium difficile infection is classically associated with pseudomembranes on
12
endoscopy. It usually causes a pancolitis, presents with watery diarrhea, and occurs most
13
14 commonly in patients with a history of recent antibiotic use.
15
16 (Choice C) Although this patient's age, weight loss, and anemia are concerning for colon

--
17 cancer , his recent negative colonoscopy, lack of masses on endoscopy, and elevated ESR make
this much less likely Moreover , diarrhea is not a common presentation of colon cance r.
19
20 (Choices D and E) Patients with chronic colonic ischemia or small-vessel vasculitis can also
21 present with bloody diarrhea and weight loss and may be initially misdiagnosed with UC.
22 However, the significant elevation in ESR and the involvement of the rectum, which has a dual
23 blood supply , make ischemia due to atherosclerosis less likely Moreover, this patient's lack of
24 other findings of small-vessel inflammation (eg, skin, pulmonary, or renal involvement) reduces
25 the likelihood of vasculitis.
26
27 Educational objective:
28
There appears to be a bimodal distribution of ulcerative colitis with a second peak at age 50-80.
29
Patients commonly experience abdominal pain, bloody diarrhea, tenesmus, and fecal
30
31 incontinence. Endoscopy demonstrates continuous involvement with erythematous and friable
32 mucosa and ulcers.
33
34 References:
35
36 1. Ulcerative colitis.
37
38
39 Time Spent 4 seconds Copyright © UWorld Last updated: [09/23/2016)
40
2
Item : 19 of40 ~'?Mark <J C> 61 ~ ~ , ~
0. ld : 2977 Previous Next Lab Values Notes Calculator Reverse Color Text Zoom
3
4
5
6 A 20-year-old man comes to the physician due to jaundice and dark urine. The patient is
7 otherwise asymptomatic . He recently immigrated to the United States and has not been under the
8 ca re of a phys ician fo r some time. The patient has no known medical problems. He occasionally
9 takes acetaminophen for headache. The patient has no siblings. His father had diabetes and
10 heart fa ilure and died of live r problems due to alcohol. Physical examination shows scleral icterus
11 and jaundiced skin. Urine dipstick is positive for bilirubin and negative for urobilinogen Serum
12
alanine aminotransferase, aspartate aminotransferase, and alkaline phosphatase levels are
13
14 normal. W hich of the following is also likely to be present in this patient?
15
16 ® A. Defect in hepatic secretion of bilirubin

...
17
18

20
21
® B. Intravascular hemolytic anemia
® C. Iron overload in the liver
® D. Obstructive gallstones and biliary sludge
22
® E. Positive acute viral hepatitis serologies
23
24
25
Submit
26
27
28
29
30
31
32
33
34
35
36
37
38
39
40
2
Item : 19 of40
0. ld : 2977
~'?M
ar
k <]
Previous
C>
Next
a
Lab Values
~
Notes
~
Calculator
,
Reverse Color
~
Text Zoom
3
4 Approach to hyperbilirubinemia In adults
5
6 Possible causes
7 Overproduction (eg, hemolysis)
8
Mainly Reduced uptake (eg, drugs,
9 Hyperbilirubinemia 1 - - - - ' -- -
10 L---- - - - - , - - -_J unconjugated portosystemic shunt)
11 Mainly Conjugation defect (eg,
12 conjugated Gilbert's syndrome)
13
14
Evaluate liver
15
enzyme pattern
16

...
17
18

20
21 Predominantly elevated Normal AST, ALT, alkaline Predominantly elevated
22 AST&ALT phosphatase al kallne p hosphatase
23
• Viral hepatitis . Dubin·Johnson syndrome • Cholestasis of pregnancy
24
25
. Autoimmune hepatitis • Rotor's syndrome • Malignancy
26 . Toxin/drug-related (eg, pancreas, ampullary)
27 hepatitis . Cholangiocarcinoma
28 . Hemochromatosis . Primary biliary cirrhosis
29 . Ischemic hepatitis . Primary sclerosing cholangitis
30
31
. Alcoholic hepatitis . Choledocholithiasis
32
33
34
Abdominal imaging
35
36 (ultrasound or CT)
AlT =alanine transaminase: AST =aspartate transarnilase. Antimitochondrial antibody
37 © UWorld
38
39 d. t bT b. . t t 'th
40 • h •
~ Feedback SuWend EnQ ock
2
Item : 19 of40 ~'?Mark <J C> 61 ~ ~ , ~
0. ld : 2977 Previous Next Lab Values Notes Calculator Reverse Color Text Zoom
3 •\ •\.: • • • I

4 © UWorld
5
6 This patient has jaundice and icterus with a positive urine bilirubin assay, consistent with a
7 buildup of conjugated bilirubin. The etiology is not likely due to hepatic injury (normal
8 transaminases) or biliary duct obstruction (normal alkaline phosphatase) He likely has Rotor's
9 syndr ome, a rare, benign hereditary condition with chronic or fluctuating conjugated
10
hyperbilirubinemia due to a defect in hepatic secretion of conjugated bilirubin into the biliary
11
12 system It is typically discovered in childhood but sometimes goes undiagnosed Live r function
13 tests are otherwise normal, and treatment is unnecessary.
14
15 An understanding of the bilirubin metabolism pathway clarifies how a positive ur ine bilirubin
16 assay reflects a buildup of conjugated bilirubin. Following hemoglobin breakdown, the majority

...
17
18

20
21
(95%) of bilirubin is in the unconjugated form, which undergoes hepatic conjugation Conjugated
bilirubin then goes into the intestines, where it is predominantly deconjugated and recycled
(enterohepatic circulation) to be reconjugated in the liver. Alternatively , some of its metabolism
products are excreted in feces as urobilinogen, with - 1% of urob ilinogen being renally excreted.

22 Unconjugated bilirubin (tightly bound to albumin) is highly insoluble, so it cannot be excreted in


23 urine. Excess unconjugated bilirubin (eg, hemolysis) undergoes the regular conjugation pathway
24 with eventual production of urobilinogen, and the resultant excess urobilinogen is primarily
25
recycled; however, a higher than normal amount of urobilinogen is excreted in feces or urine,
26
leading to a positive urine urobilinogen assay (not seen in this patient). On the other hand,
27
28 conjugated bilir ubin (loosely bound to albumin) is water soluble, so it can be excreted in
29 urine. Despite this, no bilirubin is normally seen on urinalysis (negative urine bilirubin assay)
30 because conjugated bilirubin is usually degraded in the intestines. However , when there is
31 hepatic dysfunction, biliary obstruction, or a defect in hepatic bilirubin secretion (as in this
32 patient), there is plasma buildup of conjugated bilirubin, which leaks into urine. This results in
33 dark urine and a positive urine bilirubin assay
34
35 (Choice B) Hemolysis would lead to unconjugated hype rbilirubinemia and a positive urobilinogen
36 assay, unlike in this patient Additionally , the urine assay for bilirubin is typically negative in
37 hemolytic anemia Even though the inc rease in unconjugated bilirubin produces large amounts of
38
conjugated bilirubin, this conjugated bilirubin is metabolized through the regular pathway and
39
. . . . . . . . . . . . ... ..... .. ...... . . . . . . . .. . . ........... 1 11 ••
40

~ Feedback SuWend EnQ ock


2
Item : 19 of40 ~'?Mark <] C> !I ~ ~ , ~
0. ld : 2977 Previous Next Lab Values Notes Calculator Reverse Color Text Zoom
3
4 because conjugated bilirubin is usually degraded in the intestines. However, when there is
5 hepatic dysfunction, biliary obstruction, or a defect in hepatic bilirubin secretion (as in this
6 patient), there is plasma buildup of conjugated bilirubin, which leaks into urine. This results in
7 dark urine and a positive urine bilirubin assay
8
9 {Choice B) Hemolysis would lead to unconjugated hyperbilirubinemia and a positive urobilinogen
10 assay, unlike in this patient Additionally, the urine assay for bilirubin is typically negative in
11 hemolytic anemia: Even though the increase in unconjugated bilirubin produces large amounts of
12 conjugated bilirubin, this conjugated bilirubin is metabolized through the regular pathway and
13 does not build up in plasma, as there is no hepatic dysfunction (no problem with bilirubin
14
reprocessing), biliary duct obstruction, or storage defects.
15
16

...
(Choices C, D, and E) Iron overload is commonly seen in hemochromatosis; however , most
17
(75%) patients have elevated liver function tests, and jaundice is not a common manifestation.
18
Skin pigmentation, diabetes, heart failure, and cirrhosis occur late in the disease. Jaundice due
20 to gallstones would likely be caused by biliary obstruction, which would be associated with
21 elevated alkaline phosphatase levels. Although viral hepatitis can present with jaundice, this
22 diagnosis is unlikely given the normal aminotransferase levels; typically, both urobilinogen and
23 bilirubin urine assays would be positive
24
25 Educational objective:
26 A positive urine bilirubin assay is typically indicative of a buildup of conjugated bilirubin.
27
28
References:
29
30 1. Hyperbilirubinemia syndromes (Gilbert-Meulengracht, Crigler-Najjar,
31 Dubin-Johnson, and Rotor syndrome).
32
33 2. The patient presenting with isolated hyperbilirubinemia.
34 3. Management of a patient with colon cancer and rotor syndrome: A case report.
35
36 4. A case with Rotor syndrome in hyperbilirubinemic family [abstract].
37
38
39 Time Spent 2 seconds Copyright© UWorld Last updated: [07/ 19/2016)
40
2
Item : 20of40 ~'?Mark <J C> 61 ~ ~ , ~
0. ld : 2926 Previous Next Lab Values Notes Calculator Reverse Color Text Zoom
3
4
5
6 A 34-year-old woman who recently immigrated to the United States from China has laboratory
7 tests performed prior to entering nursing school. The patient has no known past medical history
8 and takes no medications. She is currently in a monogamous relationship and does not use illicit
9 drugs. The patient believes she is up-to-date with all of her immunizations. She brings the
10 following report with her to the clinic .
11
12 HBsAg Negative
13
14 Anti-HBs Positive
15
Anti-HBc Positive
16
17 HBeAg Negative
18

-
19

21
22
23
24
Anti-HBe Negative

Which of the following is the appropriate interpretation of these serological test results?

6 A Acute hepatitis B infection


25
26 <0 B. Chronic hepatitis B infection
27 6 C. Recovery phase of hepatitis B infection
28
29 ® D. Resolved hepatitis B infection
30 e; E. Vaccination against hepatitis B infection
31
32
33 Subm it
34
35
36
37
38
39
40
2
Item : 20of40
0. ld : 2926
~'?Mark <]
Previous
C>
Next
a
Lab Values
~
Notes
~
Calculator
,
Reverse Color
~
Text Zoom
3
lgM lgG Anti- Anti- HBV
4 HBsAg HBeAg
5 anti-HBc anti-HBc HBs HBe DNA
6
7 Acute HBV
8
9
10 Early phase + + + +++
11
12
Window + +
13 phase
14
15
16 Recovery
+ + + Likely+
17 phase
18

-
19

21
22
23
24
Chronic HBV
carrier

Acute flare
of chronic
+

+ Likely+ +
+

+ +
25 HBV
26
27 Vaccinated
28 +
for HBV
29
30
31 Immune due
32 to natural + +
33 HBV
34 infection
35
36 HBV= hepatitis B virus.
37 @UWorid
38
39 This patient's serologic test results are consistent with resolved hepatitis B virus (HBV)
40
2
Item : 20of40 ~'?Mark <J C> 61 ~ ~ , ~
0. ld : 2926 Previous Next Lab Values Notes Calculator Reverse Color Text Zoom
3
©UWorid
4
5
This patient's serologic test results are consistent with resolved hepatitis 8 virus (HBV)
6
7 infection. Individuals who are immune to HBV due to natural infection are positive for anti-HBs
8 and negative fo r HBsAg They are also positive for lgG anti-HBc because they form antibodies
9 directed against the HBV core antigen The HBV vaccine contains HBsAg, which stimulates
10 production of anti-HBs and confe rs immunity in the host However , the vaccine does not contain
11 the core antigen so antibodies are not made against it and patients are consequently anti-HBc
12 negative {Choice E)
13
14 Serological markers for the hepatitis B virus include the following
15
16 HBsAg The fi rst serologic marker detected in the serum after inoculation. It precedes the onset
17 of clinical symptoms and elevation of serum aminotransferases. It also remains detectable during
18

-
the entire symptomatic phase of acute hepatitis B and suggests infectivity
19
Anti-HBs: Appears in the serum after either successful HBV vaccination or clearance of HBsAg ,
21 and remains detectable for life in most patients. It indicates non-infectivity and immunity
22 However, there is a time lag between the disappearance of HBsAg and the appearance of
23
24 anti-HBs; this is termed the "window period."
25
HBcAg This marke r is not detectable in serum as it is normally sequestered within the HBsAg
26
27 coat
28
Anti-HBc lgM appears shortly after the emergence of HBsAg and may be the only diagnostic
29
30 marke r for acute HBV infection during the "window period " lgG remains detectable during
31 recovery from acute HBV infection or progression to chronic infection.
32
33 HBeAg This antigen is detectable shortly after the appearance of HBsAg and indicates active
34 viral replication/infectivity It is associated with the presence of HBV DNA HBeAg tends to
35 disappear shortly after aminotransferase levels peak and before HBsAg is eliminated; it is
36 followed by the appearance of anti-HBe.
37
38 Anti-HBe: This marke r suggests cessation of active viral replication and low infectivity.
39
40
2
Item : 20of40 ~'?Mark <] C> !I ~ ~ , ~
0. ld : 2926 Previous Next Lab Values Notes Calculator Reverse Color Text Zoom
3 " -; . . .. , - n , , n , . . ,,
4
5 HBcAg This marker is not detectable in serum as it is normally sequestered within the HBsAg
6 coat
7
8 Anti-HBc: lgM appears shortly after the emergence of HBsAg and may be the only diagnostic
9
marker for acute HBV infection during the "window period " lgG remains detectable during
10
recovery from acute HBV infection or progression to chronic infection.
11
12
HBeAg This antigen is detectable shortly after the appearance of HBsAg and indicates active
13
14 viral replication/ infectivity It is associated with the presence of HBV DNA HBeAg tends to
15 disappear shortly after aminotransferase levels peak and before HBsAg is eliminated; it is
16 followed by the appearance of anti-HBe.
17
18 Anti-HBe This marke r suggests cessation of active viral replication and low infectivity

-
19

21
22
23
24
(Choices A and B) Acute hepatitis B infection is characterized by the presence of HBsAg and
lgM anti-HBc. In contrast, chronic hepatitis B infection is defined by the presence of HBsAg in
the serum for >6 months.

(Choice C) The recovery phase of hepatitis B infection is characterized by the presence of


25 anti-HBs, anti-HBc (predominantly lgG), and anti-HBe.
26
27 Educational objective:
28 Individuals with resolved hepatitis B infection are positive for anti-HBs and anti-HBc but negative
29 fo r HBsAg Individuals who are immunized with the hepatitis B vacc ine are positive fo r anti-HBs
30 but negative fo r HBsAg and anti-HBc.
31
32
33
References:
34 1. Hepatitis B: diagnosis and treatment.
35
36 2. Hepatitis B virus infection.
37
38
39 Time Spent 2 seconds Copyright © UWorld Last updated: [10/ 11/2016)
40
2
Item : 2 1 of40 ~'?Mark <J C> 61 ~ ~ , ~
0. ld : 3 5 93 Previous Next Lab Values Notes Calculator Reverse Color Text Zoom
3
4
5
6 A 45-year-old nurse practitioner presents to the emergency department due to painful abdominal
7 cramps and watery diarrhea. She has about 10 to 20 bowel movements a day. She also has
8 nocturnal bowel movements. She has had multiple hospitalizations in the past for similar problems
9 without a definite diagnosis A lower Gl endoscopy during a previous hospitalization showed dark
10 brown discoloration of the colon with lymph follicles shining through as pale patches W hich of
11 the following is the most likely diagnosis?
12
13
14 ® A Factitious diarrhea
15
16 ® B. Irritable bowel syndrome
17 ® C. Celiac disease
18
19 e> D. Infectious diarrhea

-
20

22
23
24
25
® E. Non-Hodgkin's lymphoma

Sub mit

26
27
28
29
30
31
32
33
34
35
36
37
38
39
40
2
Item : 2 1 of40 ~'?Mark <J C> 61 ~ ~ , ~
0. ld : 3 5 93 Previous Next Lab Values Notes Calculator Reverse Color Text Zoom
3
4 C. Celiac disease I11 % I
5
6 D. Infectious diarrhea [8%]
7 E. Non-Hodgkin's lymphoma [16%]
8
9
10 Explanation:
11
12 The above case is a classic description of factitious diarrhea, specifically laxative abuse. The
13 diarrhea associated with laxative abuse is typically described as watery, with an increase in both
14 frequency and volume of stooL The bowel movement is generally profuse, with approximately 10
15 to 20 bowel movements occurring daily Nocturnal bowel movement and painful abdominal
16 cramps may be accompanying symptoms As in this case, the patient is usually employed in the
17
medical field, comes from a higher socioeconomic class, and generally has a history of multiple
18
19 hospital admissions in an effort to establish the cause of the diarrhea. The diagnosis can be

-
20 confi rmed with the characteristic biopsy finding of dark brown discoloration of the colon with
lymph foll icles shining through as pale patches (melanosis coli); this is typically seen in those
22 using or abusing anthraquinone-containing laxatives (e g., bisacodyl) It generally develops within
23 four months of the onset of laxative ingestion, and can disappear in the same amount of time if
24 laxative use is discontinued. In addition, an alternative means of diagnosis is histological
25 evidence of pigment in the macrophages of the lamina propria
26
27 (Choice B) Nocturnal bowel movement is not characteristic of functional diarrhea, such as
28 irritable bowel syndrome.
29
30 {Choices C, D and E) Melanosis coli is not characteristic for celiac disease, infectious
31 diarrhea, or non-Hodgkin's lymphoma.
32
33 Educational Objective:
34
Laxative abuse is characterized by very frequent, watery, nocturnal diarrhea. The diagnosis can
35
be confirmed with the characteristic biopsy finding of dark brown discoloration of the colon with
36
37 lymph follicles shining through as pale patches (melanosis coli)
38
39 Time Spent 3 seconds Copyright © UWorld Last updated: [08/ 15/2016]
40
2
Item: 22 of 40 ~'?Mark <J C> 61 ~ ~ , ~
0. ld : 2905 Previous Next Lab Values Notes Calculator Reverse Color Text Zoom
3
4
5
6 A 22-year-old Caucasian male college student presents to his primary care physician
7 complaining of a two-day history of right upper abdominal pain and "not feeling well." He also
8 admits to having unusually dark urine, feeling fatigued and anorexic , and developing a sudden
9 aversion to cigarette smoking He returned from a trip to Mexico with his family approximately
10 three weeks ago. His sister has similar complaints but has not yet sought medical treatment
11 Physical examination reveals yellow sclera, right upper quadrant tenderness, and hepatomegaly
12
Total bilirubin is measured at 6.0 mg/dl W hich of the following is the most likely cause of
13
14 hepatitis in this patient?
15
16 6 A Hepatitis D
17
18 e; B. Hepatitis B
19 6 C. Hepatitis C
20

-
21

23
24
25
26
e; D. Hepatitis A
6 E. Infectious mononucleos is

Submit

27
28
29
30
31
32
33
34
35
36
37
38
39
40
2
Item: 22 of 40 ~'?Mark <] C> !I ~ ~ , ~
0. ld : 2905 Previous Next Lab Values Notes Calculator Reverse Color Text Zoom
3
4
5 Explanation:
6
7 The clinical picture of viral hepatitis can be divided into three phases: the prodromal phase, the
8 icteric phase, and the convalescent phase Given this patient's jaundice and worsening prodromal
9 symptoms, he is considered to be in the icte ric phase of acute hepatitis Infection with the
10 hepatitis A virus is strongly suggested by his recent travel to an endemic country and his sister's
11 similar clinical complaints
12
13 The pathogen responsible fo r hepatitis A is an RNA picornavirus with an average incubation
14 period of 30 days. Transmission occurs through the fecal-oral route and is common in areas with
15 overcrowding and poor sanitation. Outbreaks frequently result from contaminated water or food.
16
Onset is acute, and symptoms can include malaise, fatigue, anorexia, nausea, vomiting, mild
17
18 abdominal pain, and an aversion to smoking. Hepatomegaly is commonly seen. AST and ALT
19 spike early in the illness, followed by increases in bilirubin and alkaline phosphatase. Fortunately ,
20 hepatitis A infection is a self-limiting disease and does not progress to chronic hepatitis,

-
21

23
24
25
26
cirrhosis, or hepatocellular carcinoma. The mortality rate is less than 0.2% , although a
significantly prolonged prothrombin time correlates with increased mortality.

Treatment of hepatitis A infection is largely supportive, with complete recovery expected in 3-6
weeks. Close contacts of individuals with hepatitis A should promptly be given immune globulin
People considered at high risk (eg, those living in or traveling to endemic areas, those with
27 chronic liver disease or clotting-factor disorders, men who have sex with men) should be given
28 the hepatitis A vaccine as prophylaxis.
29
30 {Choice B) Hepatitis B is a DNA virus with an incubation period of 30-180 days Onset is
31
typically insidious, and transmission can occur sexually, parenterally, or vertically Chronic
32
hepatitis develops in 1-2% of immunocompetent adults and 90% of newborns.
33
34
{Choice C) Hepatitis C is an RNA virus with an average incubation period of 40-50 days Clinical
35
36 illness is mild and patients are often asymptomatic Infection with hepatitis C virus primarily
37 occurs parenterally, but sexual and ve rtical transmission are also possible. Chronic hepatitis
38 develops in >80% of patients, who are then at inc reased risk of cirrhosis or hepatocellular
39 ca rcinoma.
40
2
Item: 22 of 40 ~'?Mark <] C> !I ~ ~ , ~
0. ld : 2905 Previous Next Lab Values Notes Calculator Reverse Color Text Zoom
3
4 cirrhosis, or hepatocellular carcinoma. The mortality rate is less than 0.2% , although a
5 significantly prolonged prothromb in time correlates with increased mortality
6
7 Treatment of hepatitis A infection is largely supportive, with complete recovery expected in 3-6
8 weeks. Close contacts of individuals with hepatitis A should promptly be given immune globulin.
9 People considered at high risk (eg, those living in or traveling to endemic areas, those with
10 chronic liver disease or clotting-factor disorders, men who have sex with men) should be given
11 the hepatitis A vaccine as prophylaxis.
12
13 {Choice B) Hepatitis 8 is a DNA virus with an incubation period of 30-180 days Onset is
14 typically insidious, and transmission can occur sexually , parenterally, or vertically Chronic
15 hepatitis develops in 1-2% of immunocompetent adults and 90% of newborns.
16
17 (Choice C) Hepatitis C is an RNA virus with an average incubation period of 40-50 days Clinical
18 illness is mild and patients are often asymptomatic Infection with hepatitis C virus primarily
19
occurs parenterally, but sexual and vertical transmission are also possible. Chronic hepatitis
20

-
develops in >80% of patients, who are then at increased risk of cirrhosis or hepatocellular
21
carcinoma.
23
24 (Choice A) Hepatitis D is an incomplete RNA virus that causes hepatitis only in association with
25 hepatitis 8 infection. It is transmitted by percutaneous, sexual, and perinatal routes.
26 Superinfection with HDV in a patient with chronic hepatitis 8 can result in fulminant hepatitis or
27 severe chronic hepatitis that quickly progresses to cirrhosis.
28
29 {Choice E) Infectious mononucleosis can lead to hepatitis in adolescents and young adults.
30 Signs and symptoms typically include sore throat, fever, lymphadenopathy , rash, and
31 splenomegaly
32
33 Educational objective:
34
Epidemiological differences (eg, incubation period, route of transmission, risk factors) offer some
35
of the best means of identifying the virus responsible for a patient's hepatitis Confirmation can be
36
37 established with serological testing.
38
39 Time Spent 2 seconds Copyright © UWorld Last updated: [06/24/2016)
40
2
Item: 23 of 40 ~'?Mark <J C> 61 ~ ~ , ~
0. ld : 4654 Previous Next Lab Values Notes Calculator Reverse Color Text Zoom
3
4
5
6 A 57-year-old man comes to the physician with intermittent arthralgias fo r the past 8 years
7 During this time, he has had transiently elevated hepatic transaminase levels. He had an episode
8 of mild jaundice 2 years ago after having his mitral valve replaced. The patient currently takes
9 daily warfarin . His vital signs are within normal limits. Physical examination shows vesicles and
10 erosions on the dorsum of both hands. The remainder of his physical examination is within
11 normal limits. Laboratory results are as follows
12
13 Liver function studies
14
15 Albumin 3.5 g/dL
16
17 1.2
Total bilirubin
18 mg/dL
19
Aspa rtate
20 35 U/L
21 aminotransferase

--
22

24
25
26
27
Alanine aminotransferase

Coagulation studies
42 U/L

28 International Normalized
3.0
29 Ratio
30
31 W hich of the following is the most likely disorder associated with this patient's underlying
32
condition?
33
34
35 e> A. Erythema nodosum
36
6 B. Essential mixed cryoglobulinemia
37
38 6 C. Lambert-Eaton syndrome
39
40
2
Item: 23 of 40 ~'?Mark <J C> 61 ~ ~ , ~
0. ld : 4654 Previous Next Lab Values Notes Calculator Reverse Color Text Zoom
3
4 daily warfarin . His vital signs are within normal limits. Physical examination shows vesicles and
5 erosions on the dorsum of both hands. The remainder of his physical examination is within
6 normal limits. Laboratory results are as follows
7
8 Liver function studies
9
Albumin 3.5 g/dL
10
11 1.2
12 Total bilirubin
mg/dL
13
14 Aspartate
15 35 U/L
aminotransferase
16
17 Alanine aminotransferase 42 U/L
18
19
20
Coagulation studies
21

--
22

24
25
26
27
International Normalized
Ratio
3.0

W hich of the following is the most likely disorder associated with this patient's underlying
condition?
28
29
30 <0 A Erythema nodosum
31 <0 B. Essential mixed cryoglobulinemia
32
33 <0 C. Lambert-Eaton syndrome
34 <0 D. Polyarteritis nodosa
35
<0 E. Rap idly progressive glomerulonephritis
36
37
38 Submit
39
40
2
Item: 23 of 40
0. ld : 4654
~'?Mark <]
Previous
C>
Next
a
Lab Values
~
Notes
~
Calculator
,
Reverse Color
~
Text Zoom
3
4
5 Explanation:
6
7
8
9 Clinical features of chronic hepatitis C
10
11
12 • Can be asymptomatic or develop fatigue (most common)
13
14 • Other nonspecific symptoms (eg, nausea, anorexia,
15 myalgia, arthralgia, weakness, weight loss)
Clinical
16 • Serum transaminases can be elevated or normal (up to
17 presentation
1/3 of patients)
18
19 • Can progress to cirrhosis in up to 20% of patients
20 • Increased risk of hepatocellular carcinoma
21

--
22

24
25
26
27
Extrahepatic
manifestations
• Heme: Essential mixed cryoglobulinemia
• Renal: Membranoproliferative glomerulonephritis
• Skin: Porphyria cutanea tarda, lichen planus
• Endocrine: Increased risk of diabetes
28
29 ©UWorld
30
31 This patient has a number of features suggesting chronic Hepatitis C (HCV) including
32 intermittent elevations of transaminases and skin findings consistent with porphyria cutanea
33 tarda (PCT) -fragile skin, photosensitivity, and vesicles and erosions on the dorsum of the
34
hands. Hepatitis C (HCV) has a strong correlation with PCT, and all patients with PCT should be
35
screened for HCV. HCV also has a strong correlation with essential mixed cryoglobulinemia;
36
37 nearly 90% of patients with this condition have HCV, and nearly 50% of patients with HCV have
38 cryoglobulinemia Essential mixed c ryoglobulinemia is due to circulating immune complexes that
39 deposit in small to medium vessels and may be associated with low serum complement levels.
40
2
Item: 23 of 40 ~'?Mark <] C> !I ~ ~ , ~
0. ld : 4654 Previous Next Lab Values Notes Calculator Reverse Color Text Zoom
3 y •1 • p I "" • J. Iy I'
p • J.
4
cryoglobulinemia Essential mixed c ryoglobulinemia is due to circulating immune complexes that
5
deposit in small to medium vessels and may be associated with low serum complement levels.
6
7 Patients can develop palpable purpura. arthralgias and renal complications (usually
8 membranoproliferative glomerulonephritis).
9
10 In general, patients with HCV can be asymptomatic or have only nonspecific symptoms such as
11 fatigue, nausea, anorexia, myalgia, arthralgia, weakness or weight loss. Some studies have
12 shown that nearly 20% of HCV patients progress to cirrhosis, and there is an increased risk of
13 hepatocellular ca rcinoma (approximately 3% annual risk) In addition to PCT and
14 c ryoglobulinemia, extrahepatic sequelae of HCV include lichen planus. and leukocytoclastic
15 vasculitis.
16
17 (Choice A) Erythema nodosum presents as painful nodules on the anterior legs that can
18 progress to appear as bruises with eventual scarring Erythema nodosum can be associated with
19
infections (eg, streptococcal, tuberculosis) or sarcoidosis or can be idiopathic. However . it is not
20
known to be associated with HCV.
21

--
22

24
25
26
27
(Choice C) Lambert-Eaton syndrome is a neuromuscular junction disorder that presents most
commonly with muscle weakness and dry mouth. It is most often associated with small cell lung
cancer or lymphoproliferative disorder s (eg, Hodgkin lymphoma)

(Choice D) Polyarteritis nodosa is more commonly associated with hepatitis 8 rather than HCV.
28 Polyarteritis nodosa is a necrotizing vasculitis in small and medium vessels likely due to the
29 deposition of circulating antigen-antibody immune complexes in blood vessel walls.
30
31 (Choice E) Hepatitis C is most commonly associated with membranoproliferative
32 glomerulonephritis but not rapidly progressive glomerulonephritis.
33
34 Educational objective:
35 Chronic hepatitis C can be asymptomatic or present with fatigue and nonspecific symptoms (eg,
36 arthralgias, myalgias) Patients usually have waxing and waning elevations in transaminase
37 levels. Extrahepatic sequelae can include essential mixed cryoglobulinemia, porphyria cutanea
38 ta rda, and membranoproliferative glomerulonephritis
39
40 -----------------------------------------------------------------------
~ Feedback SuWend EnQ ock
2
Item: 24 of 40 ~'?Mark <] C> !J ~ ~ , ~
0. ld : 2586 Previous Next Lab Values Notes Calculator Reverse Color Text Zoom
3
4
5
6 A 19-year-old man comes to the office and says, "Doctor ! I have been having a peculiar problem
7 lately My stool has a funny color. First, it was black and almost sticky. Yesterday, it became
8 maroon. In fact, today I saw some bright red blood." He denies any associated pain or fever. He
9 is adopted, and his family history is unknown. His vital signs are stable. Physical examination is
10 normal, but his stool is positive for occult blood. Laboratory studies reveal a hematocrit of 29%
11 and hemoglobin concentration of 9.6 g/dl. Colonoscopy reveals hundreds of colonic polyps,
12
which are identified as adenomatous polyps with the biopsy . W hat is the appropriate
13
14 recommendation for this patient at this point?
15
16 e A. Reassure the patient as the polyps are most probably benign and have no long-term
17 complications.
18
19 ® B. Perform regular colonoscopy and biopsy every three years from now on to check for
20 any malignant change
21
0 C. Start regular colonoscopy and biopsy eight years from now.
22

--
23

25
26
27
28
@ D. The patient needs elective procto-colectomy

o E. The patient needs close surveillance with r egular FOBT and CEA levels.

Subm it
29
30
31
32
33
34
35
36
37
38
39
40
2
Item: 24 of 40 ~'?Mark <] C> !J ~ ~ , ~
0. ld : 2586 Previous Next Lab Values Notes Calculator Reverse Color Text Zoom
3 ~--- -- - - - - - - - - -- --- -- -- --
4 normal, but his stool is positive fo r occult blood. Laboratory studies reveal a hematocrit of 29%
5 and hemoglobin concentration of 9.6 g/dL Colonoscopy reveals hundreds of colonic polyps,
6 which are identified as adenomatous polyps with the biopsy What is the appropriate
7 recommendation for this patient at this point?
8
9
10 A Reassure the patient as the polyps are most probably benign and have no long-term
11 complications. [2%)
12
B. Perform regular colonoscopy and biopsy every three years from now on to check fo r
13
14 any malignant change [19%)
15 C. Start regular colonoscopy and biopsy eight years from now. [2%)
16
., D. The patient needs elective procto-coJectomy. [73%)
17
18 E. The patient needs close surveillance with regular FOBT and CEA levels. [3%)
19
20
21 Explanation:
22

--
23 Familial colonic polyposis (FAP) is an autosomal dominant disease caused by mutations in the
adenomatous polyposis coli (APC) gene FAP occurs in approximately 1/ 10,000to 1/30,000 live
25 births. It affects both sexes equally, and has a worldwide distribution. It accounts for less than 1
26 %of the total colon cancer risk in the United States. A patient with FAP and hundreds of
27 adenomas in the colon has a 100% risk of cancer , if he is not treated appropriately with a
28 proctocolectomy.
29
30 {Choices A, B, C and E) There is no consensus on the regime for surveillance in such patients
31 All the different methods have been tried in various trials, but have not proved to be very
32
encouraging
33
34
Educational Objective:
35
A patient with familial colonic polyposis has a 100% risk of cancer , if not treated appropriately
36
37 with a proctocolectomy at the time of diagnosis
38
39 Time Spent 2 seconds Copyright© UWorld Last updated: [04/ 18/2016)
40
2
Item: 25of40 ~'?Mark <J C> 61 ~ ~ , ~
0. ld : 3695 Previous Next Lab Values Notes Calculator Reverse Color Text Zoom
3
4
5
6 A 45-year-old Caucasian male presents with a 2-year history of progressive heartburn which is
7 most severe while supine Over-the-counter antacids have not relieved his symptoms.
8 Endoscopy shows a hiatal hernia. The patient is reluctant to accept any treatment W hich of the
9 following is he at risk for if his condition is left untreated?
10
11
12 6 A Peptic ulceration
13 10 B. Squamous cell ca rcinoma of esophagus
14
15 6 C. Aspiration pneumonia
16
17 10 D. Mallory Weiss syndrome
18 e') E. Adenocarc inoma of esophagus
19
20
21 Submit
22
23

-
24

26
27
28
29
30
31
32
33
34
35
36
37
38
39
40
2
Item: 25of40 ~'?Mark <J C> 61 ~ ~ , ~
0. ld : 3695 Previous Next Lab Values Notes Calculator Reverse Color Text Zoom
3 • • •
4
5 B. Squamous cell ca rcinoma of esophagus [7%]
6 C. Aspiration pneumonia [7%]
7
8 D. Mallory Weiss syndrome [1%]
9 v E. Adenoca rcinoma of esophagus [82%]
10
11
12 Explanation:
13
14 Esophageal cancer is the ninth most common malignancy on a worldwide basis. The two major
15 histological types are squamous cell ca rcinoma (SCC) and adenocarcinoma. In the past two
16 decades, the incidence of adenocarcinoma of the esophagus has been increasing in the western
17 world. Today, there are equal numbers of cases of adenocarcinoma and squamous cell
18 carcinoma of the esophagus The risk factors for adenocarcinoma include Barrett's esophagus,
19
obesity, high dietary calorie and fat intake, smoking, medications that promote GERD, etc. On
20
the other hand, the major risk factors for squamous cell cancer of the esophagus are smoking,
21
22 alcohol, dietary deficiency of beta-carotene, vitamin B-1 , zinc , selenium, environmental viral
23 infections, toxin producing fungi, hot food and beverages, pickled vegetables and food rich in

-
24

26
27
28
29
N-nitroso compounds, etc.

In this case, the patient has symptomatic GERD due to a hiatal hernia. Chronic GERD can lead
to metaplastic changes in the lower end of the esophagus, thus replacing the normal squamous
epithelium with columnar epithelium This is called Barrett's esophagus, which as mentioned
above, is a risk factor for developing adenocarcinoma of the esophagus Patients with Barett's
30 esophagus have a 1% per yea r chance of developing subsequent esophageal adenocarcinoma.
31
32 Educational Objective:
33 Chronic GERD and Barrett's esophagus are risk factors fo r adenocarc inoma of the esophagus.
34
Patients with Barrett's esophagus have a 1% per year risk of developing adenocarc inoma of the
35
esophagus. On the other hand, the major risk facto rs for squamous cell cancer of the esophagus
36
37 are smoking and alcohol.
38
39 Time Spent 3 seconds Copyright © UWorld Last updated: [10/ 13/2016]
40
2
Item: 26 of 40 ~'?Mark <J C> 61 ~ ~ , ~
0. ld : 2901 Previous Next Lab Values Notes Calculator Reverse Color Text Zoom
3
4
5
6 A 52-year-old man with a history of alcoholic liver disease is admitted to the hospital due to
7 fatigue and abdominal discomfort He has had no nausea, vomiting, constipation, or diarrhea.
8 The patient has chronic ascites treated with spironolactone and furosem ide. He had an inguinal
9 hernia repaired 20 years ago His temperature is 37.9 C (100 2 F) , blood pressure is 102/68 mm
10 Hg, and pulse is 106/min. The patient is awake, alert, and oriented to time, place, and person but
11 fails to perform a timed connect-the-numbers test Auscultation of the heart and lungs is normal.
12
Abdom inal examination shows moderate ascites with diffuse discomfort to palpation. There is no
13
14 rigidity or rebound. Bowel sounds are decreased. The rectal examination is normal, with brown
15 stool negative for occult blood. Abdominal x-ray shows dilated loops of large bowel with air in the
16 colon and rectum. W hich of the following is the most likely diagnosis in this patient?
17
18
e> A Acute pancreatitis
19
20 e; B. Alcoholic hepatitis
21
(:) C. Peptic ulcer perforation
22
23 e; D. Small bowel obstruction
24

-
25 (:) E. Spontaneous bacterial peritonitis

27
28 Submit
29
30
31
32
33
34
35
36
37
38
39
40
2
Item: 26 of 40
0. ld : 2901
~'?Mark <]
Previous
C>
Next
a
Lab Values
~
Notes
~
Calculator
,
Reverse Color
~
Text Zoom
3
4
5
Spontaneous bacterial peritonitis
6
7
8
• Temperature ~37.8 C (100 F)
9
Clinical
• Abdominal pain/tenderness
10
11 presentation • Altered mental status (abnormal connect-the-numbers test)
12 • Hypotension, hypothermia, paralytic ileus with severe
13 infection
14
15
16 • PMNs =::250/mm 3
17
18
Diagnosis • Positive culture, often gram-negative organisms
from ascitic (eg, Escherichia coli, Klebsiella)
19
fluid • Protein <1 g/dl
20
21 • SAAG ~1 . 1 g/dl
22
23
24 • Empiric antibiotics - third-generation cephalosporins

-
25

27
28
29
30
Treatment

(eg, cefotaxime)
Fluoroquinolones for SBP prophylaxis

PMN = polymorphonuclear leukocytes; SAAG = serum-ascites albumin gradient; SBP = spontaneous bacterial
peritonttis.
31 @UWOI!d
32
33 This patient's presentation is consistent with spontaneous bacterial peritonitis (SBP), an ascitic
34 fluid infection without an obvious intraabdominal surgical etiology SBP is most likely due to
35 either intestinal bacterial translocation directly into the ascitic fl uid or hematogenous spread to the
36 liver and ascitic fluid (due to other bacterial infections).
37
38 Patients with cirrhosis are often relatively hypothermic, and any temperature ~37.8 C (100 F)
39 warrants investigation Other manifestations of SBP may include diffuse abdominal
40
2
Item: 26 of 40 ~'?Mark <] C> !I ~ ~ , ~
0. ld :2901 Previous Next Lab Values Notes Calculator Reverse Color Text Zoom
3
4
numbers test similar to children's connect-the-dots pictures, is excellent for use in detecting
5 subtle mental status changes sometimes present in patients with cirrhosis and
6 SBP. Hypotension, hypothermia, or paralytic ileus (dilated loops of bowel on x-ray) indicate
7 severe SBP. As the vast majority of cases of SBP are associated with cirrhosis, the serum-
8 ascites albumin gradient (SAAG) is usually ~1 . 1 g/dl (a SAAG <1 .1 g/dl makes SBP
9 unlikely Ascitic fluid with a polymorphonuclear leukocyte ( PMN) count ~50/mm• and positive
10 peritoneal fluid culture confirm the diagnosis.
11
12 {Choice A) Patients with chronic alcohol use can develop acute pancreatitis with diffuse
13 abdominal pain and ileus, but nausea and vomiting are common in these patients and the pain
14
frequently radiates to the back.
15
16 (Choice B) Alcoholic hepatitis can also present similarly to SBP with fever, right upper quadrant
17
abdominal pain, and peripheral leukocytosis However, alcoholic hepatitis does not usually cause
18
diffuse abdominal pain and decreased bowel sounds. Encephalopathy is seen only in severe
19
20 cases.
21
22 (Choice C) The absence of free air under the diaphragm on x-ray makes bowel perforation
23 less likely in this patient Ascites separates the visceral and parietal peritoneal surfaces and
24 prevents development of a rigid abdomen, even with organ perforation. As a result, presentation

-
25

27
28
29
30
of secondary peritonitis (ascitic infection due to a surgically treatable intraabdominal source such
as perforated peptic ulcer) can be difficult to distinguish from SBP.

{Choice D) Small bowel obstruction (SBO) may present with diffuse abdominal pain and dilated
loops of small bowel, but nausea and vom iting are much more common and bowel sounds tend to
be high-pitched initially Dilated loops of large bowel would not be expected with SBO (although
31 air in the colon can sometimes be seen with partial SBO)
32
33 Educational objective:
34
Spontaneous bacterial peritonitis (SBP) should be suspected in any patient with cirrhosis and
35
ascites who presents with low-grade feve r, abdominal discomfort, or altered mental status. The
36
37 Reitan trail test, a timed connect-the-numbers test, helps detect subtle mental status changes
38 sometimes present in patients with SBP.
39
40
2
Item: 27 of 40 ~'?Mark <] C> !I ~ ~ , ~
0. ld : 2953 Previous Next Lab Values Notes Calculator Reverse Color Text Zoom
3
4
5
6 A 60-year-old man is brought to the emergency department due to progressive lethargy The
7 patient has a history of alcoholic cirrhosis and was recently seen in his primary care provider's
8 office for increasing abdominal distension and leg swelling His wife says that the swelling
9 improved slightly after furosemide and spironolactone were started, but for the last 2 days he has
10 been confused and irritable. The patient has slept most of the time and has not eaten.
11 Temperature is 36.7 C (98 F), blood pressure is 96/50 mm Hg, and pulse is 96/min. On
12
examination, the patient appears lethargic but is arousable. He is not very cooperative and at
13
14 times is agitated Mucous membranes are dry His outstretched hands exhibit a flapping tremor.
15 The abdomen is nontender and distended, and shifting dullness is present Bilateral lower
16 extremity pitting edema is present Stool occult blood is negative Laboratory results are as
17 follows:
18 Leukocytes 8,000/mm3
19
20 Platelets 120,000/mm3
21
22
23
24 Serum chemistry

...
25
26

28
29
Sodium
Potassium

Bicarbonate
134 mEq/L

3.0 mEq/L

30 mEq/L
30
31 Blood urea nitrogen 28 mg/dL
32
Creatinine 1.1 mg/dL
33
34 80 mg/dL
Glucose
35
36
37 Wh ich of the following is the best initial treatment for this patient?
38
39
40
2
Item: 27 of 40 ~'?Mark <J C> 61 ~ ~ , ~
0. ld : 2953 Previous Next Lab Values Notes Calculator Reverse Color Text Zoom
3 • w ~ - - -- - ----- - --- •• - -- --- - - - ·----- -- --

4 times is agitated. Mucous membranes are dry His outstretched hands exhibit a flapp ing tremor.
5 The abdomen is nontender and distended, and shifting dullness is present Bilateral lower
6 extremity pitting edema is present Stool occult blood is negative Laboratory results are as
7
follows
8
9 Leukocytes 8,000/mm'
10
11 Platelets 120,000/mm'
12
13
14 Serum chemistry
15
16 Sodium 134 mEq/L
17
18 Potassium 3.0 mEq/L
19
Bicarbonate 30 mEq/L
20
21 Blood urea nitrogen 28 mg/dL
22
23 Creatinine 1.1 mg/dL
24

...
25
26

28
29
Glucose

W hich of the following is the best initial treatment for this patient?

<0 A Dietary protein restriction


80 mg/dL -

30
31 <0 B. Low-dose lorazepam
32
33 <0 C. Oral neomycin
34 <0 D. Repletion of serum potassium
35
<0 E. Transjugular intrahepatic portosystemic shunt
36
37
38 Submit
39
40
2
Item : 27 of 40 ~'?Mark <] C> a ~ ~ , ~
3
4
0. ld : 2953
. "' ' .. . . Previous Next Lab Values Notes Calculator Reverse Color Text Zoom

B. Low-dose lorazepam [7%)


5
6 C. Oral neomycin [15%)
7 v D. Repletion of serum potassium [53%)
8
9 E. Transjugular intrahepatic portosystemic shunt [14%)
10
11
12 Explanation:
13
14
15
16 Hepatic encephalopathy
17
18
19 • Drugs (eg, sedatives, narcotics)
20 • Hypovolemia (eg, diarrhea)
21 Precipitating • Electrolyte changes (eg, hypokalemia)
22 factors • 1 nitrogen load (eg, Gl bleeding)
23 • Infection (eg, pneumonia, UTI, SBP)
24

...
• Portosystemic shunting (eg, TIPS)
25
26
• Sleep pattern changes
28 Clinical • Altered mental status
29 presentation • Ataxia
30
31 • Asterixis
32
33
• Correct precipitating causes
34
(eg, fluids, antibiotics)
35 Treatment
36 • l blood ammonia concentration
37 (eg, lactulose, rifaximin)
38
39 Gl =gastrointestinal; SBP = spontaneous bacterial peritonitis;
40
2
3
4
5
6 Gl =gastrointestinal; SBP = spontaneous bacterial peritonitis;
TIPS = transjugular intrahepatic portosystemic shun~ UTI = urinary tract infection.
7
8 @UWorid
9
10 This patient with cirrhosis has lethargy, confusion, and asterixis (flapping tremor with
11 outstretched hands) suggestive of hepatic encephalopathy (HE) HE refers to impaired central
12 nervous system (CNS) function in patients with cirrhosis and is due in part to the neurotoxicity
13 from ammonia (NHJ in the setting of impaired liver function.
14
15 In this patient, HE was likely triggered by the recent initiation of diuretic therapy This, along with
16 poor oral intake, led to low intravascular volume (hypotension, dry mucous membranes) with:
17
18 • Hypokalemia, which can exacerbate HE as the resultant intracellular acidosis (excreted
19 intracellular potassium replaced by hydrogen ions to maintain electroneutrality) causes
20 increased NH, production (glutamine conversion) in renal tubular cells
21 • Metabolic alkalosis (elevated bicarbonate), which can also exacerbate HE as it promotes
22
conversion of ammonium (NH;), which cannot enter the CNS, to NH,, which can
23
24

...
25
26

28
29
As a result, patients with HE and hypokalemia require prompt potassium repletion in addition to
intravascular volume r epletion Disaccharides (eg, lactulose, lactitol) are also administered to
lower NH, Ievels.

{Choice A) Patients with cirrhosis tend to be malnourished; therefore , dietary protein restriction
30
is generally not recommended. Protein-free diets can cause a negative nitrogen balance and
31
increase mortality Protein restriction is generally limited to patients who have required
32
33 transjugular intrahepatic portosystemic shunting (TIPS)
34
35 (Choice B) Lorazepam can be used to treat alcohol withdrawal; however , this patient does not
36 have typical manifestations of withdrawal (eg, tachycardia, hypertension, tremor) .
37
38 (Choice C) Neomycin is a nonabsorbable antibiotic used to treat HE in patients unresponsive to
39 lactulose and those unable to tolerate rifaximin.
40
2
Item: 27 of 40 ~'?Mark <] C> !I ~ ~ , ~
0. ld : 2953 Previous Next Lab Values Notes Calculator Reverse Color Text Zoom
3
4 As a result, patients with HE and hypokalemia require prompt potassium repletion in addition to
5 intravascular volume repletion Disaccharides (eg, lactulose, lactitol) are also administered to
6 lower NH, levels.
7
8 (Choice A) Patients with cirrhosis tend to be malnourished; therefore, dietary protein restriction
9
is generally not recommended. Protein-free diets can cause a negative nitrogen balance and
10
inc rease mortality Protein restriction is generally limited to patients who have required
11
12 transjugular intrahepatic portosystemic shunting ( TIPS).
13
14 (Choice B) Lorazepam can be used to treat alcohol withdrawal; however , this patient does not
15 have typ ical manifestations of withdrawal (eg, tachycardia, hypertension, tremor)
16
17 {Choice C) Neomycin is a nonabsorbable antibiotic used to treat HE in patients unresponsive to
18 lactulose and those unable to tolerate rifaximin.
19
20 (Choice E) TIPS is performed when a patient has ascites that does not respond to medical
21 therapy (eg, diuretics) or has ongoing active or recurrent va riceal bleeding even after appropriate
22 treatment with upper endoscopy TIPS is associated with HE in up to 35% of patients due to
23 (NH,-rich) blood bypassing the liver.
24

...
25
26

28
29
Educational objective:
Patients with hepatic encephalopathy on diuretics can develop low intravascular volume despite
having total volume overload, leading to a metabolic alkalosis with associated hypokalemia
Treatment includes volume resuscitation and repletion of hypokalemia in addition to serum
ammonia-lowering medications (eg, lactulose)
30
31
32 References:
33
1. Hepatic encephalopathy in chronic liver disease: 2014 Pr actice Guideline by the
34
American Association for the Study of Liver Diseases and the European
35
36 Association for the Study of the Liver.
37
38
39 Time Spent 4 seconds Copyright © UWorld Last updated: [10/ 12/2016)
40
2
Item : 28 of 40 ~'?Mark <J C> 61 ~ ~ , ~
0. ld : 391 8 Previous Next Lab Values Notes Calculator Reverse Color Text Zoom
3
4
5
6 A 52-year-old Caucasian male comes to you fo r a routine health check-up You decide to do
7 age-appropriate screening in this patient and order a colonoscopy. The colonoscopy is normal,
8 except for a 1.5cm polyp in the left descending colon. A colonoscopic polypectomy is done and
9 the biopsy results are pending The patient is anxious to know the expected biopsy results and
10 the risk of cance r. W hich of the following types of polyps is considered to be most premalignant?
11
12
13 (:) A. Hamartomatous polyp
14
15 10 B. Hyperplastic polyp
16 e> C. Villous adenoma
17
18 0 D. Tubulovillous adenoma
19
20 e E. Tubular adenoma
21
22
Submit
23
24
25
26

-
27

29
30
31
32
33
34
35
36
37
38
39
40
2
Item : 28 of 40 ~'?Mark <] C> !I ~ ~ , ~
0. ld : 391 8 Previous Next Lab Values Notes Calculator Reverse Color Text Zoom
3
4 D. Tubulovillous adenoma [8%)
5 E. Tubular adenoma [4%]
6
7
8 Explanation:
9
10 A polyp is a grossly visible protrusion from the flat mucosal surface of the intestine. Most polyps
11 are benign Polyps can be classified as follows
12
13 1. Hyperplastic polyps: These are the most common non-neoplastic polyps in the colon.
14 These arise from hyperplastic mucosal proliferation No further work-up is needed.
15 2. Hamartomatous polyps: These include juvenile polyp (a non-malignant lesion, generally
16 removed due to the risk of bleeding) and Peutz Jeghers polyp (generally non-malignant).
17 3. Adenoma This is the most common type of polyp found in the colon. It is present in
18
approximately 30-50 % of elderly people These polyps are potentially premalignant;
19
however, <1 % of such polyps become malignant Most polyps are asymptomatic ; less
20
21 than 5% of patients have positive occult stool tests.
22
23 The probability of an adenomatous polyp progressing into cancer can be judged clinically by the
24 lesion's gross appearance, histology, and size.
25
26 1. Adenoma can be sessile or stalked (pedunculated). Cancer is seen more commonly in

-
27

29
30
31
32
sessile polyps.
2. Histologically, adenoma is divided into tubular , tubulovillous, and the villous variety. Villous
adenomas, which are most commonly sessile, are most likely (three times more likely than
tubular adenoma fo r malignant transformation) to become malignant among all three
va rieties. Second on the list is tubulovillous, followed by tubular adenoma with the least risk
of malignant transformation. Therefore, as the villus component increases, the risk of
33
34 malignancy increases.
35 3. The likelihood of an adenomatous lesion containing invasive cancer also depends on the
36 size of the polyp The risk is negligible (<2%) with < 1.5 em polyp, intermediate (2-10%)
37 with 1.5-2.5 em size polyp, and substantial (10%) with polyps >2.5 em in size.
38
39
• • • I . e• . .
40

~
-------------------------------------------------------------------------------------------------------------------------------
Feedback SuWend EnQ ock
2
Item: 28 of 40 ~'?Mark <] C> !I ~ ~ , ~
0. ld : 3918 Previous Next Lab Values Notes Calculator Reverse Color Text Zoom
3
4 1. Hyperplastic polyps: These are the most common non-neoplastic polyps in the colon.
5 These arise from hype rplastic mucosal proliferation No further work-up is needed.
6 2. Hamartomatous polyps These include juvenile polyp (a non-malignant lesion, generally
7 removed due to the risk of bleeding) and Peutz Jeghers polyp (generally non-malignant)
8 3. Adenoma This is the most common type of polyp found in the colon. It is present in
9 approximately 30-50 % of elderly people These polyps are potentially premalignant;
10 however , <1 % of such polyps become malignant Most polyps are asymptomatic ; less
11 than 5% of patients have positive occult stool tests.
12
13
14 The probability of an adenomatous polyp progressing into cancer can be judged clinically by the
15 lesion's gross appearance, histology, and size.
16
17 1. Adenoma can be sessile or stalked (pedunculated) Cance r is seen more commonly in
18 sessile polyps.
19 2. Histologically, adenoma is divided into tubular , tubulovillous, and the villous variety. Villous
20 adenomas, which are most commonty sessile, are most likely (three times more likely than
21 tubular adenoma fo r malignant transformation) to become malignant among all three
22 va rieties. Second on the list is tubulovillous, followed by tubular adenoma with the least risk
23 of malignant transformation. Therefore, as the villus component increases, the risk of
24
malignancy increases.
25
3. The likelihood of an adenomatous lesion containing invasive cancer also depends on the
26

-
27 size of the polyp. The risk is negligible ( <2%) with < 1.5 em polyp, intermediate (2-10%)
with 1.5-2.5 em size polyp, and substantial (10%) with polyps >2.5 em in size.
29
30 Educational Objective:
31 Most colon cancers develop from polyps The risk factors fo r a polyp progressing into
32
malignancy are villous adenoma, sessile adenoma, and size >2.5 em. Only adenomatous polyps
33
34 are clearly premalignant, but <1% of such lesions progress to malignancy Hyperplastic polyps
35 are non-neoplastic and do not require further work-up.
36
37 • Extremely high yield question fo r the USMLE!!!
38
39 Time Spent 3 seconds Copyright © UWorld Last updated: [08/ 15/2016)
40

~
-------------------------------------------------------------------------------------------------------------------------------
Feedback SuWend EnQ ock
2
Item : 29 of 40 ~'?Mark <J C> 61 ~ ~ , ~
0. ld : 2930 Previous Next Lab Values Notes Calculator Reverse Color Text Zoom
3
4
5
6 A 48-year-old woman comes to the emergency department due to sudden-onset right flank pain
7 fo r 6 hours. She has never had abdominal pain before. Her only other medical problem is
8 psoriasis at both knees, for which she uses topical vitamin 0 and beclomethasone. Vital signs
9 are normaL The abdomen is soft with no tenderness. Bowel sounds are present Abdominal
10 imaging reveals a 3-mm stone in the right upper ureter and a few gallstones in the gallbladder.
11 The liver and common bile duct are normal, the gallbladder is not distended, and there is no
12
pericholecystic fluid or tenderness to ultrasound probe pressure. The patient is given analgesics
13
14 and intravenous fl uids; in a few hours she passes the ureteral stone in the urine with complete
15 pain relief. Which of the following is the best next step in management of her gallstones?
16
17 e; A Laparoscopic cholecystectomy
18
19 e> B. Lithotripsy
20 e; C. No treatment at this time
21
22 e> D. Open cholecystectomy
23
24 e; E. Ursodeoxycholic acid
25
26

...
27
28

30
31
Submit

32
33
34
35
36
37
38
39
40
2
Item : 29 of 40
0. ld : 2930
~'?Mark <]
Previous
C>
Next
a
Lab Values
~
Notes
~
Calculator
,
Reverse Color
~
Text Zoom
3 •••••• .I •• ,

4
5
6 Explanation:
7
8
9
10 Management of gallstones
11
12
13 Gallstones without
symptoms • No treatment necessary in most patients
14
15
16
17 • Elective laparoscopic cholecystectomy
Gallstones with typical
18 • Possible ursodeoxycholic acid in poor
b iliary colic symptoms
19 surgical candidates
20
21
22 Complicated gallstone
23 disease
24 (acute cholecystitis, • Cholecystectomy within 72 hours
25 choledocholithiasis,
26 gallstone pancreatitis)

...
27
28

30
31
©UWorld

This patient had acute symptoms due to a small kidney stone, which has now passed However,
she also had an inc idental finding of asymptomatic gallstones. Only 20% of patients with
32 asymptomatic gallstones will develop symptoms within 15 years In light of this, no treatment is
33 typically required fo r such patients Exceptions to this rule include patients who are at increased
34
risk for developing gallbladder carc inoma (eg, porcelain gallbladder)
35
36
{Choices A and D) laparoscopic cholecystectomy is often performed in patients with biliary
37
38
colic (intermittent right upper quadrant pain) or acute cholecystitis. Open cholecystectomy is
performed in patients who cannot tolerate a laparoscopic procedure or who have another
39
40 . . . .
.. .. .... .. ... .. .. .. . ..
~
-------------------------------------------------------------------------------------------------------------------------------
Feedback SuWend EnQ ock
Item : 29 of 40 ~'?Mark <J C> 61 ~ ~ , ~
2
3
4
0. ld : 2930
. -.. . "'
'
gallstone pancreatitis)
Previous Next Lab Values Notes Calculator Reverse Color Text Zoom

5
6 ©UWorld
7
8 This patient had acute symptoms due to a small kidney stone, which has now passed However,
9 she also had an incidental finding of asymptomatic gallstones Only 20% of patients with
10 asymptomatic gallstones will develop symptoms within 15 years. In light of this, no treatment is
11 typically required for such patients Exceptions to this rule include patients who are at increased
12
risk for developing gallbladder carcinoma (eg, porcelain gallbladder)
13
14
{Choices A and D) Laparoscopic cholecystectomy is often performed in patients with biliary
15
colic (intermittent right upper quadrant pain) or acute cholecystitis Open cholecystectomy is
16
17 performed in patients who cannot tolerate a laparoscopic procedure or who have another
18 intraabdominal process requiring surgery
19
20 (Choice B) Lithotripsy (laser or mechanical fragmentation) is sometimes used fo r gallstones in
21 the bile duct that cannot be removed endoscopically.
22
23 (Choice E) Oral ursodeoxycholic acid supplementation reduces symptoms from gallstones but
24 does not lead to complete dissolution. It is used to treat symptomatic gallstones in individuals who
25 are poor surgical candidates, but would be of no benefit for asymptomatic stones.
26

...
27
28

30
31
Educational objective:
Asymptomatic gallstones should not be treated. Laparoscopic cholecystectomy is the treatment
of choice for symptomatic gallstone disease.

Refer ences:
32
33 1. Asymptomatic cholelithiasis: is cholecystectomy really needed? A critical
34 reappraisal 15 years after the introduction of laparoscopic cholecystectomy.
35
36 2. Surgical and nonsurgical management of gallstones.
37
38
39 Time Spent 2 seconds Copyright© UWorld Last updated: [09/08/2016)
40
2
Item: 30 of 40 ~'?Mark <J C> 61 ~ ~ , ~
0. ld : 4626 Previous Next Lab Values Notes Calculator Reverse Color Text Zoom
3
4
5
6 A 43-year-old nurse presents to her primary care physician complaining of occasional
7 heartburn. On physical examination she is found to have hepatomegaly. Her spleen is not
8 palpable She denies any history of jaundice , rash, joint pain, vom iting, or black stool Serum
9 analysis reveals the following:
10
11 HBsAg negative
12
13 Anti-HBsAg antibodies positive
14
Anti-HAV antibodies negative
15
16 Anti-HCV antibodies positive
17
18 AST 70 units/L
19
20 110
ALT
21 units/L
22
23 Bilirubin 1.2 mg/dl
24
Creatinine 0.9 mg/dl
25
26 PT/ INR 12 0/ 1.0
27
28

--
29 W hich of the following is the best recommendation for this patient?

31 6 A. Prednisone
32
33 6 B. Lamivudine
34 e; C . Hepatitis A vaccination
35
36 6 D. Hepatitis B vaccination
37 6 E. Low-salt diet
38
39 6 F. Furosemide
40
2
Item: 30 of 40 ~'?Mark <] C> !I ~ ~ , ~
0. ld : 4626 Previous Next Lab Values Notes Calculator Reverse Color Text Zoom
3
4
5 Explanation:
6
7 This woman is infected with the hepatitis C virus but does not have evidence of cirrhosis on
8 physical examination or by laboratory data. Her transaminases and bilirubin are mildly elevated
9 but her INR is within normal range All patients with chronic liver disease should be immunized
10 against hepatitis A and B unless they are already immune, as they are at high risk for acute
11 hepatic failure or cirrhosis upon infection with viral hepatitis This woman is positive for anti-
12
hepatitis B surface antibodies (anti-HBsAg), indicating immunity against hepatitis B. She is not
13
14 immune against hepatitis A , however, and should be vaccinated.
15
16 (Choice A) Prednisone is used to treat severe alcoholic hepatitis. This disease typically presents
17 with fever, abdominal pain, jaundice, nausea, and vom iting.
18
19 (Choice B) Lamivudine (3TC) is a reverse transcriptase inhibitor used to treat HIV and chronic
20 hepatitis B. This woman's hepatitis B surface antigen (HBsAg) is negative, arguing against active
21 infection with hepatitis B.
22
23
24 (Choice D) Anti-HBsAg antibodies convey immunity against the hepatitis B virus. She is anti-
25 HBsAg antibody positive and does not require vaccination against hepatitis B.
26
27 (Choice E) Patients with cirrhosis are intravascularly depleted and impressive salt retainers. A
28 low-salt diet is beneficial in decreasing ascites and peripheral edema by decreasing the total

--
29

31
32
33
34
amount of sodium that can be retained. This patient does not have cirrhosis, so a low-salt diet is
not necessary

(Choice F) Furosemide is a loop diuretic used to help prevent hype rvolemia in cirrhosis. This
woman does not have cirrhosis and will not benefit from a loop diuretic .
35
36 Educational objective:
37 All patients with chronic live r disease should be immunized against hepatitis A and B.
38
39 Time Spent 3 seconds Copyright © UWorld Last updated [08/ 15/2016]
40
2
Item: 3 1 of40 ~'?Mark <J C> 61 ~ ~ , ~
0. ld : 2341 Previous Next Lab Values Notes Calculator Reverse Color Text Zoom
3
4
5
6 A 70-year-old man is brought to the emergency department by his wife due to rectal bleeding.
7 The patient initially had a bowel movement consisting of a large volume of bright red blood without
8 associated abdominal pain Since then, he has had persistent lightheadedness along with several
9 urges to defecate in which blood is produced. The patient has no prior history of gastrointestinal
10 bleeding Past medical history is notable for hypertension and chronic constipation Current
11 medications include chlorthalidone and low-dose aspirin Temperature is 36.5 C (97 8 F), blood
12
pressure is 85/45 mm Hg, pulse is 120/min, and respirations are 20/min. The abdomen is soft,
13
14
nondistended, and nontender; bowel sounds are normal. No masses or organomegaly are
15 palpable Rectal examination shows bright red blood. Nasogastric aspiration returns nonbilious
16 stomach contents without blood. A plain radiograph of the abdomen is normal. Which of the
17 following is the most likely cause of this patient's bleeding?
18
19
<0 A Angiodysplasia
20
21 <0 B. Colon cancer
22
23
<0 C. Diverticulosis
24 e; D. Hemorrhoids
25
<0 E. Ischemic colitis
26
27 <0 F. Mesenteric thrombosis
28
29 <0 G. Peptic ulcer disease

--
30

32
33
34
35
Submit

36
37
38
39
40
2
Item: 3 1 of40 ~'?Mark <] C> !I ~ ~ , ~
0. ld : 2341 Previous Next Lab Values Notes Calculator Reverse Color Text Zoom
3
4 Explanation:
5
6 This patient has large-volume rectal bleeding most likely due to dive rticulosis. Diverticulosis is
7 the most common cause of gross lower gastrointestinal (GI) bleeding in adults. Diverticula are
8
outpouchings of the colonic wall that form at points of weakness. The deformation in the colonic
9
10 wall can cause weakness in the associated arterial supply and lead to bleeding into the
11 diverticular lumen. Dive rticulosis is most common in the sigmoid colon, but dive rticular bleeding
12 is more common in the right colon.
13
14 Dive rticular bleeding is typ ically painless, but large-volume bleeding may be associated with
15 lightheadedness and hemodynamic instability . Low- or moderate-volume bleeding from the right
16 colon will mix with stool and pass as dark or maroon-colored hematochezia. Large-volume
17 hemorrhage can lead to passage of frank red blood. The diagnosis is confi rmed on
18 colonoscopy. Most cases of dive rticular hemorrhage will resolve spontaneously, but a
19 minority of patients will require endoscopic or surgical intervention.
20
21 (Choice A) Colonic angiodysplasia can cause painless bleeding in the right colon. However , it is
22 significantly less common than diverticular hemorrhage. Angiodysplasia usually also causes
23
low-volume (venous) bleeding, whereas dive rticulosis can cause large-volume arterial hemorrhage.
24
25
(Choice B) Colon cancer tends to cause chronic occult blood loss, with abdominal pain, altered
26
passage of stool, and weight loss. Gross bleeding is less likely
27
28
(Choice D) Hemorrhoids cause painless rectal bleeding, with bright red blood in the toilet bowl or
29

--
30 on the paper Hemorrhoids rarely cause massive lower Gl bleeding

(Choice E) Ischemic colitis causes sudden onset of abdominal pain and tenderness, followed by
32
33 rectal bleeding It can occur due to inadequate perfusion of "watershed" areas of the colon (eg,
34 splenic flexure) in the setting of nonocclusive ischemia or surgical or endovascular interventions.
35
36 (Choice F) Acute mesenteric thrombosis presents with abdominal pain out of proportion to
37 physical findings, nausea/vomiting, and bloody diarrhea due to mucosal sloughing Patients have
38 numerous atherosclerotic risk factors.
39
40
2
Item: 3 1 of40 ~'?Mark <] C> !I ~ ~ , ~
0. ld : 2341 Previous Next Lab Values Notes Calculator Reverse Color Text Zoom
3 • I I I •
4
significantly less common than dive rticular hemorrhage. Angiodysplasia usually also causes
5
6
low-volume (venous) bleeding, whereas diverticulosis can cause large-volume arterial hemorrhage
7
(Choice B) Colon cancer tends to cause chronic occult blood loss, with abdominal pain, altered
8
9
passage of stool, and weight loss. Gross bleeding is less likely
10
(Choice D) Hemorrhoids cause painless rectal bleeding, with bright red blood in the toilet bowl or
11
12 on the paper Hemorrhoids rarely cause massive lower Gl bleeding.
13
14 (Choice E) Ischemic colitis causes sudden onset of abdominal pain and tenderness, followed by
15 rectal bleeding It can occur due to inadequate perfusion of "watershed" areas of the colon (eg,
16 splenic flexure) in the setting of nonocclusive ischemia or surgical or endovascular interventions.
17
18 (Choice F) Acute mesente ric thrombosis presents with abdominal pain out of proportion to
19 phys ical findings, nausea/vomiting, and bloody diarrhea due to mucosal sloughing Patients have
20 numerous atherosclerotic risk factors.
21
22 (Choice G) Brisk upper Gl bleeding (eg, due to peptic ulce r disease) can present with bright red
23 blood per rectum; however, most patients with upper hemorrhage of that degree usually have
24 hematemesis as well This patient's nonbloody gastric aspirate makes an upper Gl source less
25 likely
26
27 Educational objective:
28 Dive rticulosis is the most common cause of gross lower gastrointestinal bleeding in adults.
29

--
Bleeding is typically painless but may be associated with lightheadedness and hemodynamic
30
instability Dive rticular hemorrhage usually resolves spontaneously but occasionally requires
32 endoscopic or surgical intervention.
33
34 Refer ences:
35
36 1. Bleeding colonic diverticula.
37
38
39 Time Spent 3 seconds Copyright© UWorld Last updated [09/08/2016)
40
2
Item: 32 of40 ~'?Mark <J C> 61 ~ ~ , ~
0. ld : 4934 Previous Next Lab Values Notes Calculator Reverse Color Text Zoom
3
4
5
6 A 40-year-old man comes to the office due to a 2-day history of retrosternal chest pain, pain with
7 swallowing, and epigastric burning. The pain is severe and the patient is afraid to swallow food.
8 He has never had similar symptoms before. The patient has no associated shortness of breath,
9 vomiting, melena, or blood in the stool. Past medical history is notable for nonischemic
10 ca rdiomyopathy , for which he takes furosemide, carvedilol, spironolactone, lisinopril, and
11 potassium chloride. Temperature is 37.2 C (99 F), blood pressure is 110/65 mm Hg, and pulse is
12
67/min. Examination shows a soft holosystolic murmur at the ca rdiac apex, similar to that in prior
13
14 examinations. The rest of the examination shows no abnormalities. The patient's ECG shows
15 sinus rhythm with no acute ischemic changes Endoscopy reveals circumferential deep
16 ulceration with relatively normal surrounding mucosa at the middle third of the esophagus W hich
17 of the following is most likely diagnosis?
18
19
20 <0 A Candida esophagitis
21 <0 B. D iffuse esophageal spasm
22
23
<0 C. Esophageal ca rcinoma
24 e; 0 . Gastroesophageal reflux disease
25
<0 E. Pill-induced esophagitis
26
27 <0 F. Viral esophagitis
28
29
30

-
Submit
31

33
34
35
36
37
38
39
40
2
Item: 32 of40 ~'?Mark <] C> !I ~ ~ , ~
0. ld : 4934 Previous Next Lab Values Notes Calculator Reverse Color Text Zoom
3
4 This patient has abrupt-onset retrosternal pain and severe odynophagia, suggestive of
5 medication-induced esophagitis (pill esophagitis) Pill esophagitis is due to a direct effect of
6 certain medications on esophageal mucosa. Mucosal injury in pill esophagitis can be due to acid
7 effect (eg, tetracyclines), osmotic tissue injury (eg, potassium chloride), or disruption of normal
8 gastroesophageal protection (eg, nonsteroidal anti-inflammatory drugs). Patients usually do not
9
have prior esophageal disease, although pill esophagitis can be worse in those with concurrent
10
11 gastroesophageal reflux.
12
Typical symptoms of pill esophagitis include sudden-onset odynophagia and retrosternal pain
13
14 that can sometimes cause difficulty swallowing. It is most common in the mid-esophagus due to
15 compression by the aortic arch or an enlarged left atrium. The diagnosis is usually made
16 clinically but can be confi rmed on endoscopy, which shows discrete ulce rs with relatively
17 normal-appearing surrounding mucosa. Treatment includes primarily stopping the offending
18 medication to prevent future injury
19
20 (Choices A and F) Endoscopy in Candida esophagitis is characterized by white plaques, and
21 most patients will also have oral thrush. Ulcerating lesions are seen in esophagitis due to herpes
22 simplex (vesicles and round/ovoid ulce rs) or cytomegalovirus (large linear ulcers) These
23
conditions are most common in immunocompromised patients
24
25 {Choice B) Diffuse esophageal spasm is characterized by uncoo rdinated, simultaneous
26 contractions of the lower esophagus. Patients typically have recurrent episodes of liquid/solid
27
dysphagia and chest pain
28
29
{Choice C) Esophageal cancer typically presents with progressive solid food dysphagia and
30

-
31 unintentional weight loss. Risk factors fo r squamous cell ca rc inoma include alcohol and tobacco
use, whereas factors for adenocarcinoma include Barrett esophagus, gastroesophageal reflux
33 disease, smoking, and obesity
34
35 (Choice D) Gastroesophageal reflux disease typically causes recurrent or persistent burning
36 pain in the upper abdomen and chest Symptoms are worse following large meals or ce rtain
37 foods (eg, chocolate, peppermint) or when lying down. Symptoms are usually subacute to
38 chronic rather than abrupt
39
40
2
Item: 32 of40 ~'?Mark <] C> !I ~ ~ , ~
0. ld : 4934 Previous Next Lab Values Notes Calculator Reverse Color Text Zoom
3 <I ' .. <I . - • - •• - •• •• • " • " I -- I - " -<I ,-'

4 normal-appearing surrounding mucosa. Treatment includes primarily stopping the offending


5 medication to prevent future injury.
6
7 {Choices A and F) Endoscopy in Candida esophagitis is characterized by white plaques, and
8 most patients will also have oral thrush. Ulcerating lesions are seen in esophagitis due to herpes
9
simplex (vesicles and round/ovoid ulcers) or cytomegalovirus (large linear ulcers) These
10
11 conditions are most common in immunocompromised patients.
12
13
(Choice B) Diffuse esophageal spasm is characterized by uncoordinated, simultaneous
14 contractions of the lower esophagus Patients typically have recurrent episodes of liquid/solid
15 dysphagia and chest pain.
16
17 (Choice C) Esophageal cancer typically presents with progressive solid food dysphagia and
18 unintentional weight loss. Risk factors for squamous cell carcinoma include alcohol and tobacco
19 use, whereas factors for adenocarcinoma include Barrett esophagus, gastroesophageal reflux
20 disease, smoking, and obesity
21
22 (Choice D) Gastroesophageal reflux disease typically causes recurrent or persistent burning
23 pain in the upper abdomen and chest Symptoms are worse following large meals or certain
24 foods (eg, chocolate, peppermint) or when lying down. Symptoms are usually subacute to
25 chronic rather than abrupt
26
27 Educational objective:
28 Pill esophagitis is due to a direct effect of certain medications on esophageal mucosa.
29
Tetracyclines, potassium chloride, bisphosphonates, and nonsteroidal anti-inflammatory drugs
30

-
31 are common causes. Patients experience sudden-onset odynophagia and retrosternal pain that
can sometimes cause difficulty swallowing.
33
34 References:
35
36 1. Drug-induced esophagitis.
37
38
39 Time Spent 2 seconds Copyright © UWorld Last updated: [08/22/2016)
40
2
Item: 33 of40 ~'?Mark <J C> 61 ~ ~ , ~
0. ld : 2904 Previous Next Lab Values Notes Calculator Reverse Color Text Zoom
3
4
5
6 A 42-year-old obese woman with a history of chronic alcoholism presents with right upper
7 quadrant abdominal pain that began shortly after she had dinner at a local steakhouse. The pain
8 is sharp, rated 7/ 10 in severity , and radiates to her right scapula. She also complains of nausea
9 and vomiting Her medical records indicate she has a history of asymptomatic gallstones. She
10 drinks approximately 6 beers a day fo r the past 20 years but denies tobacco or illicit drug use.
11 Her temperature is 38.5 C (101 3 F), blood pressure is 110/60 mm Hg, pulse is 96/min, and
12
respirations are 16/min. On physical examination, there is right upper quadrant tenderness with
13
14 deep inspiration The rest of her examination is within normal limits. Laboratory studies are
15 shown below.
16
1.2
17 Total bilirubin
mg/dl
18
19 0.8
20 Direct bilirubin
mg/dl
21
22 Alkaline phosphatase 110 U/L
23
24 Aspa rtate aminotransferase
48 U/L
25 (SGOT)
26
27 Alanine aminotransferase
65 U/L
28 (SGPT)
29
30 Amylase 125 U/L
31

-
32

34
35
36
37
Complete blood count shows leukocytosis with mild left shift. W hich of the following is the most
likely initial event leading to this patient's symptoms?

e> A. Impaction of gallstone in cystic duct


6 B. Infection of gallbladder
38
39 6 C. Common bile duct obstruction with stone
40
2
Item: 33 of40 ~'?Mark <J C> 61 ~ ~ , ~
0. ld : 2904 Previous Next Lab Values Notes Calculator Reverse Color Text Zoom
3
4 drinks approximately 6 beers a day for the past 20 years but denies tobacco or illicit drug use.
5 Her temperature is 38.5 C (101 .3 F), blood pressure is 110/60 mm Hg, pulse is 96/min, and
6 respirations are 16/min. On physical examination, there is right upper quadrant tenderness with
7 deep inspiration The rest of her examination is within normal limits. Laboratory studies are
8
shown below.
9
10 1.2
11 Total bilirubin
mg/dl
12
13 0.8
14 Direct bilirubin
mg/dl
15
16 Alkaline phosphatase 110 U/L
17
18 Aspartate aminotransferase
48 U/L
19 (SGOT)
20
Alanine aminotransferase
21 65 U/L
22 (SGPT)
23
Amylase 125 U/L
24
25
26 Complete blood count shows leukocytosis with mild left shift. W hich of the following is the most
27 likely initial event leading to this patient's symptoms?
28
29
30 <0 A Impaction of gallstone in cystic duct
31 <0 B. Infection of gallbladder

-
32

34
35
36
37
<0 C. Common bile duct obstruction with stone
<0 D. Carcinoma of pancreatic head
<0 E. Alcohol toxicity

38 Submit
39
40
2
Item: 33 of40 ~'?Mark <] C> !I ~ ~ , ~
0. ld : 2904 Previous Next Lab Values Notes Calculator Reverse Color Text Zoom
3
4
5 Explanation:
6
7 This patient presents with findings consistent with acute cholecystitis, which usually presents with
8 sudden onset of steady epigastric or right upper quadrant pain after a large or fatty meal. The
9 pain may radiate to the right scapula or be accompanied by Murphy's sign, described as
10 worsening of right upper quadrant pain with inspiration that sometimes causes the patient to
11 suddenly hold their breath. Other classical findings include fever , vomiting, and leukocytosis.
12
Uncompl icated cholecystitis can cause mild elevations in transaminases, total serum bilirubin from
13
14 1-4 mg/dl, and serum amylase without obvious common bile duct or pancreatic disease. In
15 these cases, the laboratory elevations are usually due to passage of sludge or pus in the common
16 bile duct However, alkaline phosphatase is usually not elevated without associated cholangitis or
17 choledocholithiasis.
18
19 Acute cholecystitis is secondary to gallstone fo rmation in over 90% of cases, most commonly
20 arising when a gallstone impacts in the cystic duct Ingestion of fatty foods then stimulates the
21 contraction of the gallbladder against the obstructed cystic duct, and leads to severe colicky
22 pain. The tissue behind the duct obstruction becomes inflamed , from stasis leading to bacterial
23 overgrowth. The subsequent ischemic changes most commonly lead to gangrene and
24
perforation, generalized peritonitis, or a well-circumscribed abscess. Other potential
25
complications include cholangitis and chronic cholecystitis.
26
27
Treatment fo r acute cholecystitis includes supportive care (nothing by mouth, intravenous
28
29 antibiotics, and analgesics). Laparoscopic cholecystectomy is recommended shortly after
30 hospitalization and should be performed immediately in cases of perforation or gangrene
31

-
32 {Choice B) Although infection of the gallbladder is present in 50-70% of acute cholecystitis
cases, the original inciting event is gallstone impaction in the cystic duct Therefore, infection
34 would not be the primary mechanism fo r this patient's symptoms.
35
36 {Choices C and D) Common bile duct obstruction with stone or carcinoma would normally cause
37 obstructive jaundice characterized by severe icterus and very high alkaline phosphatase levels.
38
39 {Choice E) Alcohol toxicity can cause acute pancreatitis but is not usually associated with acute
40
2
Item: 33 of40 ~'?Mark <] C> !I ~ ~ , ~
0. ld : 2904 Previous Next Lab Values Notes Calculator Reverse Color Text Zoom
3
4 Acute cholecystitis is secondary to gallstone fo rmation in over 90% of cases, most commonly
5 arising when a gallstone impacts in the cystic duct Ingestion of fatty foods then stimulates the
6
contraction of the gallbladder against the obstructed cystic duct, and leads to severe colicky
7
pain. The tissue behind the duct obstruction becomes inflamed , from stasis leading to bacterial
8
9 overgrowth. The subsequent ischemic changes most commonly lead to gangrene and
10 perforation, generalized peritonitis, or a well-circumscribed abscess. Other potential
11 complications include cholangitis and chronic cholecystitis.
12
13 Treatment for acute cholecystitis includes supportive care (nothing by mouth, intravenous
14 antibiotics, and analgesics). Laparoscopic cholecystectomy is recommended shortly after
15 hospitalization and should be performed immediately in cases of perforation or gangrene
16
17 {Choice B) Although infection of the gallbladder is present in 50-70% of acute cholecystitis
18 cases, the original inciting event is gallstone impaction in the cystic duct Therefore, infection
19 would not be the primary mechanism fo r this patient's symptoms.
20
21 {Choices C and D) Common bile duct obstruction with stone or carcinoma would normally cause
22 obstructive jaundice characterized by severe icte rus and very high alkaline phosphatase levels.
23
24 {Choice E) Alcohol toxicity can cause acute pancreatitis but is not usually associated with acute
25 cholecystitis.
26
27 Educational objective:
28 Acute cholecystitis usually presents with sudden onset of right upper quadrant abdominal pain,
29
fever, vomiting, and leukocytosis The primary inciting event is a gallstone obstructing the cystic
30
31 duct with subsequent inflammation and infection

-
32

34
35
36
37
References:
1. Hyperbilirubinemia without common bile duct abnormalities and
hyperamylasemia without pancreatitis in patients with gallbladder disease.

38
39 Time Spent 5 seconds Copyright © UWorld Last updated: [07/07/2016)
40
2
Item: 34 of40 ~'?Mark <J C> 61 ~ ~ , ~
0. ld : 2900 Previous Next Lab Values Notes Calculator Reverse Color Text Zoom
3
4
5
6 A 35-year-old woman is brought to the emergency department due to 3 days of progressive
7 nausea, anorexia, malaise, abdominal pain, and lethargy The patient is a chronic carrier of
8 hepatitis B virus and has not received any treatment. She has a history of injection drug use and
9 completed a rehabilitation program 2 years ago, but recently started using drugs again She
10 drinks alcohol but does not use tobacco. Her temperature is 37.8 C (100 2 F) , blood pressure is
11 106/64 mm Hg, and pulse is 11 4/min. Mild tender hepatomegaly is present. Laboratory testing
12
shows markedly elevated serum aminotransferase levels and a positive serum hepatitis D
13
14 antigen Acute live r failure due to hepatitis D virus superinfection is suspected Which of the
15 following is required to make a diagnosis of acute liver failure in this patient?
16
17 C) A. Decreased urine output
18
19 C) B. Elevated portal venous pressure
20
C) C. Features of liver cirrhosis
21
22 C) D. Severe hyperbilirubinemia
23
24 C) E. Signs of hepatic encephalopathy
25
26
27 Subm it
28
29
30
31
32

--
33

35
36
37
38
39
40
2
Item: 34 of40
0. ld : 2900
~'?Mark <]
Previous
C>
Next
a
Lab Values
~
Notes
~
Calculator
,
Reverse Color
~
Text Zoom
3 &' • • .. • .. • • .. • '

4
5
6 Explanation:
7
8
9
10 Acute liver failure
11
12
13 • Viral hepatitis (eg, HSV; CMV; hepatitis A, 8 , D & E)
14 • Drug toxicity (eg, acetaminophen overdose, idiosyncratic)
15 • Ischemia (eg, shock liver, Budd-Chiari syndrome)
16 Etiology
• Autoimmune hepatitis
17
• Wilson disease
18
19
• Malignant infiltration
20
21
22 • Generalized symptoms (eg, fatigue, lethargy, anorexia, nausea)
23 • Right upper quadrant abdominal pain
24 Clinical • Pruritus & jaundice due to hyperbilirubinemia
25 presentation • Renal insufficiency
26 • Thrombocytopenia
27 • Hypoglycemia
28
29
30 • Severe acute liver injury (ALT & AST often >1000 U/L)
31
Diagnostic
req uirements • Signs of hepatic encephalopathy (eg, confusion, asterixis)
32

--
• Synthetic liver dysfunction (INR ;::1.5)
33
ALT =alanine aminotransferase; AST =aspartate aminotransferase; CMV =cytomegalovirus;
35 HSV = herpes simplex virus.
36 IDUWOI1d
37
38 Acute liver failure (ALF) is a serious condition characterized by severe acute liver injury in a
39 patient without cirrhosis or underlying liver disease. The diagnosis requires
40
2
Item: 34 of40 ~'?Mark <] C> !I ~ ~ , ~
0. ld : 2900 Previous Next Lab Values Notes Calculator Reverse Color Text Zoom
3
4
5 Acute liver failure (ALF) is a serious condition characterized by severe acute live r injury in a
6 patient without cirrhosis or underlying live r disease. The diagnosis requires
7
• Severe acute live r injury as evidenced by elevated aminotransfer ases (often >1000 U/L)
8
9 • Signs of hepatic encephalopathy (HE)
10 • Impaired hepatic synthetic function (defined as INR 2:1.5)
11
12 The presence of HE differentiates ALF from acute hepatitis, which has a much better prognosis
13 than ALF. In addition to varying degrees of HE, other common manifestations of ALF include
14
fatigue, lethargy, nausea, vomiting, jaundice, pruritus, and right upper quadrant pain The most
15
common causes of ALF are drug toxicity (eg, acetaminophen overdose) and acute viral
16
17 hepatitis (eg, hepatitis A virus, hepatitis B virus [HBV)) Other causes include autoimmune
18 hepatitis, ischemia, W ilson disease, and malignant infiltration of the liver.
19
20 Acute superinfection with hepatitis D vi rus carries a high risk of ALF development in
21 intravenous drug users with chronic HBV. On further evaluation of this patient, clinical signs of
22 HE and laboratory findings of INR <::1.5 would be diagnostic of ALF.
23
24 (Choices A and B) Decreased urine output (due to intravascular volume depletion and
25 decreased renal perfusion) and elevated portal venous pressure (due to increased resistance to
26 blood flow through the inflamed live r) are both common in ALF. However, neither is specific for
27 ALF or part of the diagnostic criteria.
28
29 (Choice C) Features of liver cirrhosis include fluid retention, jaundice, caput medusae, palmar
30 erythema, muscle wasting, and gynecomastia ALF includes some of these features (eg,
31 jaundice, fl uid retention), but it is characterized by acute live r injury in a patient without cirrhosis.
32

--
33

35
36
37
38
{Choice D) Severe hyperbilirubinemia is common in ALF and results from a combination of
impaired conjugation and impaired excretion of bilirubin. However, it is not a requirement for the
diagnosis of A LF.

Educational objective:
39 Acute live r failure is defined as acute onset of severe live r injury with encephalopathy and
••• ... • ... • ......... ~ !t • • ... •• • .......... ..
40

~ Feedback SuWend EnQock


2
Item: 34 of40 ~'?Mark <] C> !I ~ ~ , ~
0. ld : 2900 Previous Next Lab Values Notes Calculator Reverse Color Text Zoom
3 .i. : •:~ I I
4
5
hepatitis, ischemia, Wilson disease, and malignant infiltration of the liver.
6
Acute superinfection with hepatitis D virus ca rries a high risk of A LF development in
7
8 intravenous drug users with chronic HBV. On further evaluation of this patient, clinical signs of
9 HE and laboratory findings of INR ~1 . 5 would be diagnostic of A LF.
10
11 {Choices A and B) Decreased urine output (due to intravascular volume depletion and
12 decreased renal perfusion) and elevated portal venous pressure (due to increased resistance to
13 blood flow through the inflamed liver) are both common in ALF. However, neither is specific for
14 ALF or part of the diagnostic criteria.
15
16 (Choice C) Features of liver cirrhosis include fluid retention, jaundice, caput medusae, palmar
17 erythema, muscle wasting, and gynecomastia ALF includes some of these features (eg,
18 jaundice, fluid retention), but it is characterized by acute live r injury in a patient without cirrhosis.
19
20 (Choice D) Severe hyperbilirubinemia is common in ALF and results from a combination of
21 impaired conjugation and impaired excretion of bilirubin. However, it is not a requirement for the
22 diagnosis of ALF.
23
24 Educational objective:
25 Acute live r failure is defined as acute onset of severe liver injury with encephalopathy and
26
impaired synthetic function (defined as INR ~1 5) in a patient without cirrhosis or underlying live r
27
28
disease. Drug toxicity and acute viral hepatitis are the most common causes.
29
30 References:
31
32
1. Acute liver failure .

--
33

35
36
37
38
2. Outcomes in adults with acute liver failure between 1998 and 2013: an
observational cohort study.
3. The pathology of acute liver failure.

39 Time Spent 3 seconds Copyright © UWorld Last updated: [08/08/2016)


40
2
Item: 35 of 40 ~'?Mark <] C> !I ~ ~ , ~
0. ld : 4603 Previous Next Lab Values Notes Calculator Reverse Color Text Zoom
3
4
5
6 A 42-year-old man comes to the hospital after a day of severe abdominal pain and vomiting He
7 describes the pain as constant and nagging, localizing to the upper abdomen, and radiating to the
8 back. The patient has not been to a physician in years and has no previous medical conditions
9 or surgeries. He takes no medications. He drinks large amounts of alcohol, but does not use
10 tobacco or illicit drugs. His temperature is 36.r C (98° F), blood pressure is 110/80 mm Hg,
11 pulse is 90/min, and respirations are 14/min. The patient is admitted to the hospital and given
12
opioid analgesics and intravenous fl uids. On day 2 of his hospitalization, his blood pressure
13
14 drops to 80/60 mm Hg and his pulse increases to 120/min and regular His oxygen saturation is
15 92% on 2 L nasal canula oxygen Examination shows normal jugular venous pressure Bilateral
16 crackles are heard on lung auscultation. The patient's abdomen is mildly distended, and there is
17 tenderness in the epigastrium His urine output is 8 mUhr He has received a total of 4 L IV
18 fl uids over the last 24 hours. Laboratory results are as follows
19
20 Hemoglobin 14.9 g/dL
21
Leukocytes 16,500/mm'
22
23 Platelets 120,000/mm'
24
25
26
27 Sodium 138 mEq/L
28
29 Potassium 4.9 mEq/L
30 Chloride 97 mEq/L
31
32 Bicarbonate 21 mEq/L
33

--
34

36
37
38
39
Glucose

Creatinine
Blood urea nitrogen
210 mg/dL

1.9 mg/dL

45 mg/dL

40
2
Item: 35 of 40 ~'?Mark <J C> 61 ~ ~ , ~
0. ld : 4603 Previous Next Lab Values Notes Calculator Reverse Color Text Zoom
3
4 fluids over the last 24 hours. Laboratory results are as follows:
5 Hemoglobin 14.9 g/dL
6
7 Leukocytes 16,500/mm'
8
9 Platelets 120,000/mm'
10
11
12
Sodium 138 mEq/L
13
14 Potassium 4.9 mEq/L
15
16 Chloride 97 mEq/L
17
18 Bicarbonate 21 mEq/L
19
Glucose 210 mg/dL
20
21 Creatinine 1.9 mg/dL
22
23 Blood urea nitrogen 45 mg/dL
24
25
26 Which of the following is the most likely cause of this patient's hypotension?
27
28 ® A Increased vascular permeability
29
30 ® B. Myocardial infarction
31 ® C. Opioid overdose
32
33 ® D. Occult bleeding

--
34

36
37
38
39
® E. Pericardia! effusion
® F. Pseudocyst formation

Submit

40
2
Item: 35 of 40
0. ld : 4603
~ t'Mark ~
Prevoous Next
C> a
Lab Values
~
Notes
~
Calc ·
,
Reverse Color
~
Text Zoom
3
4
5 Clinical features of severe pancreatitis
6
7
• Fever, tachycardia, hypotension
8
9 • Dyspnea, tachypnea &/or basilar cracKles
10 • Abdominal tenderness &/or distension
11 Clinical
12 presentation • Cullen sign: Periumbilical bluish coloration indicating
13 hemoperitoneum
14 • Grey-Turner sign: Reddish-brown coloration around flanKs
15 indicating retroperitoneal bleed
16
17
• Age >75
18
19 • Obesity
20 • Alcoholism
21
22 Associated w ith • C-reactive protein >150 mgfdl at 48 hours after presentation
23 I risk of severe • Rising blood urea nitrogen & creatinine in the first 48 hours
24 pancreatitis
• Chest x-ray with pulmonary infiltrates or pleural effusion
25
26 • Computed tomography scantmagnetic resonance
27 cholangiopancreatography with pancreatic necrosis
28 & extrapancreatic inflammation
29
30 • Pseudocyst
31
32 • Peripancreatic fluid collection
33 • Necrotizing pancreatitis

--
34 Complications
• Acute respiratory distress syndrome
36 • Acute renal failure
37 • Gastrointestinal bleeding
38
39 @UWorid
40
2
3
4
5
@UWorid
6
7
This patient's presentation is consistent with acute severe pancreatitis that has progressed to
8
9
multisystem organ dysfunction (eg, shock, renal failure, early respiratory failure) Most patients
10 with acute pancreatitis have mild disease and recover with conservative management (eg, fluids,
11 bowel rest, pain medication) in 3-5 days However, nearly 15%-20% of patients can develop
12 severe acute pancreatitis, defined as pancreatitis with failure of at least 1 organ Clinical marke rs
13 associated with increased risk fo r severe pancreatitis include age >75, alcoholism, obesity,
14 C-reactive protein >150 mg/dl at 48 hours, and inc reased blood urea nitrogen (BUN)/creatinine
15 in the first 48 hours. Abdominal imaging (computed tomography scan or magnetic resonance
16 cholangiopanc reatography) is indicated for suspected severe pancreatitis to look for panc reatic
17
necrosis and extrapancreatic inflammation, which also indicate severe acute pancreatitis
18
19
Severe pancreatitis causes local release of activated pancreatic enzymes that enter the vascular
20
system and increase vascular permeability within and around the pancreas This leads to large
21
22 volumes of fluid migrating from the vascular system to the surrounding retroperitoneum Systemic
23 inflammation also ensues as the inflammatory mediators enter the vascular system The net
24 effect is widespread vasodilation, capillary leak, shock, and associated end-organ damage
25 Treatment usually involves supportive care with several liters of IV fl uid to replace the lost
26 intravascular volume.
27
28 (Choice B) Hypotensive episodes seen in severe pancreatitis could incite an underlying
29 myocardial infarction. However, this patient has no signs or symptoms (eg, chest pain)
30 suggesting myocardial infa rction.
31
32 (Choice C) Narcotics (eg, meperidine) are often given fo r analgesia in acute panc reatitis
33

--
Narcotic overdose can cause respiratory depression, hypotension, and bradycardia. This
34
patient's tachycardia, crackles, abdominal distention, worsening BUN, and low urine output make
severe pancreatitis with early multiorgan failure more likely
36
37
{Choice D) Occult bleeding is unlikely to cause significant hypotension without resulting in
38
39 anemia.
40
2
Item: 35 of 40 ~'?Mark <] C> !I ~ ~ , ~
0. ld : 4603 Previous Next Lab Values Notes Calculator Reverse Color Text Zoom
3
4 (Choice B) Hypotensive episodes seen in severe pancreatitis could incite an underlying
5 myocardial infarction. However , this patient has no signs or symptoms (eg, chest pain)
6 suggesting myocardial infarction.
7
8 {Choice C) Narcotics (eg, meperidine) are often given for analgesia in acute pancreatitis
9 Narcotic overdose can cause respiratory depression, hypotension, and bradycardia. This
10 patient's tachyca rdia, crackles, abdominal distention, worsening BUN, and low urine output make
11 severe pancreatitis with early multiorgan failure more likely.
12
13 (Choice D) Occult bleeding is unlikely to cause significant hypotension without resulting in
14
anemia.
15
16 (Choice E) Perica rdia! effusion can be a compl ication of pancreatitis, but tamponade severe
17
enough to cause cardiac failure would cause jugular venous distention (not seen in this patient's
18
19
physical examination).
20
{Choice F) Acute panc reatitis can be compl icated by a peripancreatic pseudocyst, a fluid
21
22 collection (containing pancreatic enzymes, blood, fl uid, and tissue debris) surrounded by a
23 necrotic or fibrous capsule. Pseudocysts typically take 3-4 weeks to develop after acute
24 pancreatitis and would be a less likely cause of acute hypotension in this patient
25
26 Educational objective:
27 Acute pancreatitis compl icated by hypotension is thought to arise from intravascular volume loss
28 secondary to local and systemic vascular endothelial injury. This causes vasodilation, increased
29 vascular permeability, and plasma leak into the retroperitoneum, resulting in systemic hypotension
30
31
References:
32
33 1. Elevated serum creatinine as a marker of pancreatic necrosis in acute

--
34

36
37
38
39
pancreatitis
2. Management of fluid collections and necrosis in acute pancreatitis

Time Spent 6 seconds Copyright © UWorld Last updated: [08/ 10/2016)


40
2
Item: 36 of40 ~'?Mark <J C> 61 ~ ~ , ~
0. ld : 4694 Previous Next Lab Values Notes Calculator Reverse Color Text Zoom
3
4
5
6 A 51-year-old man comes to the physician with difficulty swallowing solids but not liquids. His
7 medical history is significant for gastroesophageal reflux disease for the last 12 years He
8 underwent endoscopy and was diagnosed with Barrett's esophagus 6 months ago His heartburn
9 resolved 3 months after this diagnosis The patient's vital signs are within normal limits. His body
10 mass index is 38 kg/m2 Barium swallow now shows an area of symmetric , circumferential
11 narrowing affecting the distal esophagus W hich of the following best explains this finding?
12
13
14 e') A Achalasia
15
16 ® B. Esophageal adenocarcinoma
17
® C. Esophageal stricture
18
19 ® D. Hiatal hernia
20
21 ® E. Vascular ring
22
23
24 Submit
25
26
27
28
29
30
31
32
33
34

-
35

37
38
39
40
2
Item: 36 of40 ~'?Mark <] C> !I ~ ~ , ~
0. ld : 4694 Previous Next Lab Values Notes Calculator Reverse Color Text Zoom
3
4
This patient's presentation - chronic gastroesophageal reflux with new dysphagia and symmetric
5
6 lower esophageal narrowing - suggests esophageal (peptic) stricture. Chronic gastroesophageal
7 reflux disease (GERD) predisposes to Barrett's esophagus (intestinal metaplasia of the lower
8 esophagus) and esophageal strictures. Both conditions are consequences of the body's
9 reparative response to chronic gastric acid exposure and can occur simultaneously. Benign
10 strictures affect 5%- 15% of patients with GERD. Other causes of peptic strictures include
11 radiation, systemic sclerosis, and caustic ingestions
12
13 Strictures typically cause slowly progressive dysphagia to solid foods without anorexia or weight
14 loss. As they progress, they can actually block reflux, leading to improvement of heartburn
15 symptoms (as seen in this patient) Strictures tend to appear as symmetric , c ircumferential
16
narrowing on barium swallow. Nonetheless, in any case of stricture in the setting of Barrett's
17
18 esophagus, biopsy is necessary to rule out adenocarcinoma. This is usually accomplished via
19 endoscopy which may be diagnostic and therapeutic (dilation is performed if no malignancy is
20 detected)
21
22 (Choice A) Achalasia is an esophageal motility disorder that presents with dysphagia (both
23 solids and liquids) and regurgitation of undigested food or saliva. Barium swallow typically shows
24 aperistalsis, poor emptying of barium, dilation of the proximal esophagus (caused by retention of
25 food), and narrowing in a "bird beak" pattern at the gastroesophageal junction
26
27 (Choice B) Adenocarcinoma typically occurs in patients who have had GERD symptoms for >20
28 years Early symptoms of esophageal cancer are subtle and incl ude retrosternal discomfort, mild
29 dysphagia to solid foods , and/or a burning sensation. Barium swallow generally shows
30 asymmetric narrowing of the esophageal lumen.
31
32 (Choice D) A hiatal hernia is a protrusion of the stomach above the diaphragm These patients
33
may have GERD and are also at risk for Barrett's esophagus, peptic strictures, and
34

-
adenocarc inoma. Barium swallow shows gastric folds protruding above the diaphragm
35
(Choice E) Vascular rings are uncommon congenital anomalies in which aortic arch vessels
37
encircle the trachea and/or esophagus Patients may present in infancy with symptoms of airway
38
39 obstruction, though some patients do not present until adulthood. In adult cases, dysphagia is
• • .. .. .. • • • .. ... • • •• .. • .. .. .. • .. .. .. • • • • .. ! ••
40

~ Feedback SuWend EnQ ock


2
Item: 36 of40 ~'?Mark <] C> !I ~ ~ , ~
0. ld : 4694 Previous Next Lab Values Notes Calculator Reverse Color Text Zoom
3
4
solids and liquids) and regurgitation of undigested food or saliva. Barium swallow typically shows
5 aperistalsis, poor emptying of barium, dilation of the proximal esophagus (caused by retention of
6 food), and narrowing in a "bird beak" pattern at the gastroesophageal junction
7
8 {Choice B) Adenocarcinoma typically occurs in patients who have had GERD symptoms fo r >20
9 years. Early symptoms of esophageal cancer are subtle and include retrosternal discomfort, mild
10 dysphagia to solid foods, and/or a burning sensation. Barium swallow generally shows
11 asymmetric narrowing of the esophageal lumen.
12
13 (Choice D) A hiatal hernia is a protrusion of the stomach above the diaphragm. These patients
14 may have GERD and are also at risk for Barrett's esophagus, peptic strictures, and
15 adenocarcinoma. Barium swallow shows gastric folds protruding above the diaphragm
16
17 (Choice E) Vascular rings are uncommon congenital anomalies in which aortic arch vessels
18 encircle the trachea and/or esophagus Patients may present in infancy with symptoms of airway
19
obstruction, though some patients do not present until adulthood. In adult cases, dysphagia is
20
usually the presenting complaint. There is not a strong association with GERD or Barrett's
21
22 esophagus
23
24 Educational objective:
25 Gastroesophageal reflux disease (GERD) predisposes to Barrett's esophagus, erosive
26 esophagitis, and esophageal (peptic) stricture formation. Peptic strictures cause symmetric and
27 circumferential narrowing of the involved esophagus with dysphagia for solids but typically no
28 weight loss. Other causes of peptic strictures include radiation, systemic sclerosis, and caustic
29 ingestions
30
31
References:
32
33 1. Incidence of adenocarcinoma among patients with Barrett's esophagus.
34

-
35
2. Predictive value of symptoms and demographics in diagnosing malignancy or
peptic stricture.
37
38
39 Time Spent 2 seconds Copyright© UWorld Last updated: [09/05/2016)
40
2
Item: 37 of40 ~'?Mark <J C> 61 ~ ~ , ~
0. ld : 4433 Previous Next Lab Values Notes Calculator Reverse Color Text Zoom
3
4
5
6 A 36-year-old woman comes to the emergency department after having severe epigastric and
7 right shoulder pain for 2 hours. The patient also had an episode of emesis. W hen asked about
8 her diet, she explains that she unintentionally fasted all day and had a cheeseburger 2 hours
9 ago. The patient had a similar episode last month but did not seek medical attention as it was less
10 severe and resolved spontaneously in a few hours. Her past medical history is significant for
11 heartburn, and she takes ranitidine. On examination, the patient's abdomen is soft and
12
nontender. About 4 hours after the pain began, it resolves completely W hich of the following
13
14 best explains this episode?
15
16 6 A Acid hypersecretion and mucosal irritation
17
18 e; B. Fat necrosis
19 6 C. Hollow organ contraction and outlet obstruction
20
21 e; D. Outlet obstruction and mucosal inflammation
22 6 E. Peritoneal irritation
23
24 e; F. Vascular obstruction
25
26
27 Submit
28
29
30
31
32
33
34
35

-

36

38
39
40
2
Item: 37 of40 ~'?Mark <] C> !I ~ ~ , ~
0. ld : 4433 Previous Next Lab Values Notes Calculator Reverse Color Text Zoom
3 J, I o
4
5
6 Explanation:
7
8 This is the classic presentation of biliary colic secondary to gallstones. Ingestion of a large
9 (usually fatty) meal stimulates gallbladder contraction. The intra-gallbladder pressure increases
10 on gallbladder contraction against an obstructed cystic duct and causes pain Subsequent
11
gallbladder relaxation allows the stone to fall back from the duct with subsequent pain resolution.
12
13 The pain is in the right upper quadrant or epigastric region, often constant (rather than colicky),
14 and accompanied by nausea, vom iting, and right-sided shoulder or subscapular discomfort
15 (referred pain) Patients may have recurrent symptoms that resolve between episodes Features
16 that distinguish biliary colic from cholecystitis are pain resolution within 4-6 hours and absence of
17 abdominal tenderness, feve r, and leukocytosis.
18
19 (Choice A) Epigastric discomfort is accompanied by heartburn secondary to gastric acid
20 hypersecretion and mucosal irritation. However , pain is usually described as a burning sensation
21 and does not radiate to the right shoulder. Dyspepsia provoked by meals is often associated with
22 belching and early satiety. Vomiting may occur but is uncommon.
23
24 (Choice B) Fat necrosis may accompany acute pancreatitis Symptoms include nausea,
25 vom iting, and constant epigastric pain that radiates to the back. Pain usually does not resolve
26
spontaneously within a few hours.
27
28
(Choice D) In acute cholecystitis, the gallbladder mucosa is inflamed. The cystic duct is
29
frequently obstructed with gallstones or sludge Although the pain is similar to biliary colic pain in
30
31 location, radiation, and associated symptoms, it usually lasts longer than 6 hours. Furthermore,
32 patients frequently have fever, leukocytosis, and tenderness to palpation in the midaxillary line
33 under the right costal margin (Murphy's sign)
34
35 (Choice E) Rebound tenderness, abdominal guarding, and decreased bowel sounds are found

-

36

38
39
40
on abdominal examination in patients with peritoneal irritation. This patient's abdominal
examination is benign

(Choice F) Bowel ischemia from vascular obstruction causes severe abdominal pain out of

~
-------------------------------------------------------------------------------------------------------------------------------
Feedback SuWend EnQ ock
Item: 37 of40 ~'?Mark <] C> !I ~ ~ , ~
2
3
4
. ...
0. ld : 4433
- . .. . .. - . . ... ...- . . .... -...
- -
Previous

belching and early satiety. Vomiting may occur but is uncommon.


Next
-... . - ... . -.
... "
Lab Values Notes Calculator Reverse Color Text Zoom

5
6 (Choice B) Fat necrosis may accompany acute pancreatitis Symptoms include nausea,
7 vomiting, and constant epigastric pain that radiates to the back. Pain usually does not resolve
8 spontaneously within a few hours.
9
10 (Choice D) In acute cholecystitis, the gallbladder mucosa is inflamed. The cystic duct is
11
frequently obstructed with gallstones or sludge Although the pain is similar to biliary col ic pain in
12
13 location, radiation, and associated symptoms, it usually lasts longer than 6 hours. Furthermore,
14 patients frequently have fever , leukocytosis, and tenderness to palpation in the midaxillary line
15 under the right costal margin (Murphy's sign).
16
17 (Choice E) Rebound tenderness, abdominal guarding, and decreased bowel sounds are found
18 on abdominal examination in patients with peritoneal irritation. This patient's abdominal
19 examination is benign.
20
21 (Choice F) Bowel ischemia from vascular obstruction causes severe abdominal pain out of
22 proportion to findings on physical examination. The pain is often periumbilical, lasts longer than
23 that of biliary colic , does not resolve spontaneously, and is frequently accompanied by nausea,
24 vomiting, and fecal blood.
25
26 Educational objective:
27 Biliary colic occurs due to increased intra-gallbladder pressure that is created when the
28 gallbladder contracts against an obstructed cystic duct The pain is exacerbated by fatty meals,
29
usually lasts less than 6 hours, and resolves completely between episodes. There is no feve r,
30
31 abdominal tenderness on palpation, or leukocytosis
32
33 References:
34
35
1. Natural history of gallstone disease: Expectant management or active

- 36 treatment? Results fr om a population-based cohort study .

38
39 Time Spent 3 seconds Copyright © UWorld Last updated: [05/06/2016)
• 40

~
-------------------------------------------------------------------------------------------------------------------------------
Feedback SuWend EnQock
2
Item: 38 of 40 ~'?Mark <J C> 61 ~ ~ , ~
0. ld : 2895 Previous Next Lab Values Notes Calculator Reverse Color Text Zoom
3
4
5
6 A 27-year-old Caucasian female presents to her primary care physician complaining of
7 significant malaise, anorexia, and nausea for the past few weeks, which has limited her ability to
8 work as a waitress. Further inquiry reveals that she has also noticed her urine to be ve ry dark
9 yellow and that her boyfriend believes she is "more moody than normal." She has no history of
10 medical problems, takes no medications, and has no known drug allergies. She has smoked one
11 pack of cigarettes per day for the past eight years and drinks one to two beers daily . Phys ical
12
examination reveals yellow sclera, spider nevi on her upper torso, rigidity, and tremor at rest
13
14 Slit-lamp examination demonstrates greenish brown deposits around both corneas. Palpation of
15 the abdomen reveals the liver to be enlarged with an irregular edge Laboratory work-up returns
16 with the following results:
17
18 Live r studies
19 Total bilirubin 4.5 mg/dl
20 Direct bilirubin 3.2 mg/dl
21 Alkaline phosphatase 40 U/L
22 Aspa rtate aminotransferase (AST, SGOT) 276 U/L
23
24 Alanine aminotransferase (ALT, SGPT) 180 U/L
25 Prothrombin time 12.0 sec
26
27 W hich of the following is the most likely diagnosis?
28
29
0 A Hemochromatosis
30
31 0 B. Hepatolenticular degeneration
32
0 C. Acute viral hepatitis
33
34 0 D. Alcoholic hepatitis
35 0 E. Autoimmune hepatitis
36

-- 37 0 F. Primary biliary cirrhosis


0 G. Sclerosing cholangitis
39
• 40
2
Item: 38 of 40 ~'?Mark <] C> !I ~ ~ , ~
0. ld : 2895 Previous Next Lab Values Notes Calculator Reverse Color Text Zoom
3
4
5 Explanation:
6
7 This patient most likely has compensated cirrhosis. The combination of cirrhosis,
8 neuropsychiatric symptoms and Kayser-Fleischer rings (brownish or gray-green rings of fine
9 granular copper deposits in the cornea) in a young adult is highly suggestive of W ilson's disease,
10 also called hepatolenticular degeneration
11
12 W ilson's disease is a rare, autosomal recessive disease most often identified in younger
13 individuals aged 5 to 40 yea rs. The genetic mutations associated with W ilson's disease hinder
14
copper metabolism by reducing the formation and secretion of ceruloplasmin and by decreasing
15
the secretion of copper into the biliary system Copper is a pro-oxidant, and as it accumulates in
16
17 greater quantities within the liver , it causes damage to the hepatic tissue through the generation of
18 free radicals. Eventually coppe r leaks from injured hepatocytes into the blood to be deposited in
19 various tissues, including the basal ganglia (hepatolenticular degeneration) and cornea.
20
21 Live r involvement in W ilson's disease may present as asymptomatic live r function abnormalities,
22 chronic hepatitis, fulminant hepatitis, portal hype rtension, or macronodular cirrhosis.
23 Neuropsychiatric symptoms can include Parkinsonian-like tremor , rigidity , ataxia, slurred speech,
24 drooling, personality changes, depression, paranoia, and catatonia. W ilson's disease is also
25 associated with Fanconi syndrome, hemolytic anemia, and neuropathy The gold standard for
26 diagnosis is live r biopsy that demonstrates a quantitative hepatic coppe r level >250 meg/gram dry
27
weight More commonly, diagnosis is confirmed by the presence of low serum ce ruloplasmin
28
29 (particularly <20 mg/dl) in conjunction with increased urinary coppe r excretion or
30 Kayse r-Fleischer rings
31
32 Treatment must be adhered to for the patient's lifetime and focuses on removing accumulated
33 coppe r in the tissues and preventing re-accumulation. First-line medications include copper
34 chelators like d-penicillamine or trientine. Oral zinc is also recommended as it prevents coppe r
35 absorption Liver transplantation may be the only option for those with fulminant hepatic fa ilure or
36 decompensated liver disease that does not respond to pharmacotherapy

--

37

39
40
(Choice A) Hemochromatosis is an autosomal recessive disease characterized by abnormally
high iron absorption that results in iron overload. Common clinical manifestations include liver
2
Item: 38 of 40 ~'?Mark <] C> !I ~ ~ , ~
0. ld : 2895 Previous Next Lab Values Notes Calculator Reverse Color Text Zoom
3
4 chronic hepatitis, fulminant hepatitis, portal hypertension, or macronodular cirrhosis.
5 Neuropsychiatric symptoms can include Parkinsonian-like tremor, rigidity, ataxia, slurred speech,
6 drooling, personality changes, depression, paranoia, and catatonia. Wilson's disease is also
7 associated with Fanconi syndrome, hemolytic anemia, and neuropathy The gold standard for
8 diagnosis is liver biopsy that demonstrates a quantitative hepatic copper level >250 meg/gram dry
9 weight More commonly , diagnosis is confirmed by the presence of low serum ceruloplasmin
10 (particularly <20 mg/dl) in conjunction with increased urinary copper excretion or
11 Kayser-Fleischer rings.
12
13 Treatment must be adhered to for the patient's lifetime and focuses on removing accumulated
14
copper in the tissues and preventing re-accumulation. First-line medications include copper
15
chelators like d-penicillamine or trientine. Oral zinc is also recommended as it prevents copper
16
17 absorption Liver transplantation may be the only option for those with fulminant hepatic failure or
18 decompensated liver disease that does not respond to pharmacotherapy
19
20 {Choice A) Hemochromatosis is an autosomal recessive disease characterized by abnormally
21 high iron absorption that results in iron overload. Common clinical manifestations include liver
22 disease, hyperpigmentation, diabetes mellitus ("bronze diabetes"), impotence, arthropathy, and
23 cardiac enlargement
24
25 (Choice C) W hile viral hepatitis can eventually cause cirrhosis of the liver , acute viral hepatitis is
26 not associated with neuropsychiatric symptoms or the Kayser-Fleischer ring Moreover, serum
27 transaminases are typically much higher in patients diagnosed with viral hepatitis
28
29 (Choice D) Alcoholic hepatitis is typically associated with a more significant drinking history and
30 is commonly characterized by an ASTALT ratio greater than 2:1 . The Kayse r-Fleischer ring is
31
not associated with alcoholic hepatitis.
32
33 Educational Objective:
34
Wilson's disease is the likely diagnosis in a patient younger than 30 years old with unexplained
35
chronic hepatitis. The presence of low serum ceruloplasmin and increased urinary coppe r
36

-- 37 excretion or Kayser-Fleischer rings confirms the diagnosis

39 Time Spent 3 seconds Copyright © UWorld Last updated: [04/ 18/2016)


• 40
2
Item: 39 of 40 ~'?Mark <J C> 61 ~ ~ , ~
0. ld : 3605 Previous Next Lab Values Notes Calculator Reverse Color Text Zoom
3
4
5
6 A 45-year-old Asian-American female comes to the physician due to bloating, flatulence,
7 abdominal cramps and explosive watery diarrhea. These symptoms occur after ingesting dairy
8 products. She has not had any weight loss. She has not had bone pain or easy bruising
9 Physical examination shows abdominal distention and borborygmi You decide to investigate the
10 patient further. W hich of the following test results is most likely to be observed?
11
12
13 6 A. Positive urine test for reducing substances
14
@ B. Decreased stool osmotic gap
15
16 6 C. Positive hyd rogen breath test
17
18 @ D. Alkaline stool pH
19
6 E. Positive acid steatocrit test
20
21
22
Submit
23
24
25
26
27
28
29
30
31
32
33
34
35
36
37

--

38

40
~'?Mark 61 ~ ~ , ~
2
3
4
Item: 39 of 40
0. ld : 3605
. . . ...
.. ... - .
E. Positive acid steatocrit test [11 %)
<J
Previous
C>
Next Lab Values Notes Calculator Reverse Color Text Zoom

5
6
7
Explanation:
8
9
Lactase is a brush border enzy me that hydrolyses lactose. Its concentration declines steadily as
10
one ages into adulthood, especially in people of non-European ancestry. W hen there is an
11
12 inability to absorb lactose found in milk and dairy products, the condition is called lactose
13 intolerance. This is most commonly seen in Asian-Americans (90%). Patients typically manifest
14 with osmotic diarrhea, abdominal cramps, bloating and flatulence after ingestion of such products
15
16 Previously , the lactose tolerance test was used to aid in the diagnosis of patients. This test is
17 based on measurement of the blood glucose level after oral lactose administration. The lactose
18 tolerance test is cumbersome and time consuming Currently , the lactose hydrogen breath test
19 has largely replaced the lactose tolerance test A positive hydrogen breath test is characterized
20 by a rise in the measured breath hydrogen level after the ingestion of lactose, thus indicating
21 bacterial carbohydrate metabolism.
22
23 (Choice A) Urine test fo r reducing substances is positive in patients with glucosuria,
24
galactosuria, etc. Those are not characteristic features of lactose intolerance.
25
26 {Choice B) The diarrhea secondary to lactase deficiency has a high osmotic gap, due to the
27
unmetabolized lactose and organic acids. The osmotic gap is calculated as 290- [2 (stool Na +
28
29 stool K)] and is greater than 50 mOsm/kg in all fo rms of osmotic diarrhea.
30
(Choice D) The stool pH is acidic in lactase deficiency due to the fermentation products
31
32
(Choice E) Acid steatocrit is a test for fat malabsorption, not carbohydrate malabsorption
33
34
Educational objective:
35
Lactose intolerance is characterized by a positive hyd rogen breath test, positive stool test fo r
36
37 reducing substances, low stool pH and increased stool osmotic gap. There is no steatorrhea.

--

38

40
Time Spent 2 seconds Copyright © UWorld Last updated: [06/24/2016)
2
Item: 40 of 40 ~'?Mark <] C> !I ~ ~ , ~
0. ld : 2935 Previous Next Lab Values Notes Calculator Reverse Color Text Zoom
3
4
5
6 A 62-year-old male presents complaining of anorexia, fatigue, and a 9 kg (20 lb) weight loss over
7 the past six months. He rarely drinks alcohol and does not use illicit drugs, though he has smoked
8 one pack of cigarettes per day fo r the past 40 yea rs He has previously been diagnosed with
9 hypertension, diverticulosis, external hemorrhoids, and osteoarthritis. His current medications are
10 atenolol and aspirin, and he has no known drug allergies. Vital signs include temperature of 37.0°
11 C (98 6° F) , blood pressure of 132/88 mm Hg, pulse of 70/min, and respirations of 12/min.
12
Physical examination reveals a soft abdomen that is mildly tender to deep palpation in the
13
14 mid-epigastric region. The sclerae are icteric and his skin is jaundiced. Laboratory evaluation
15 returns with the following:
16
17 Liver studies
18 Albumin 3.9 g/dL
19
20 Total bilirubin 6.7 mg/dL
21
22 Alkaline phosphatase 647 U/L
23
24 Aspa rtate aminotransferase (SGOT) 110 U/L
25
Alanine aminotransferase (SGPT) 102 U/L
26
27
28 Complete blood count
29
30 Hemoglobin 13.5 g/ L
31
Platelets 220,000/mm3
32
33 Leukocyte count 5,100/mm3
34
35
36 Anti-mitochondrial antibodies are negative An abdominal ultrasound demonstrates mild dilation of
37 the common bile duct There are no stones present in the gallbladder and no evidence of
38 gallbladder wall thickening W hat is the next best step in the management of this patient's

--
39 condition?
2
Item: 40 of 40 ~'?Mark <J C> 61 ~ ~ , ~
0. ld : 2935 Previous Next Lab Values Notes Calculator Reverse Color Text Zoom
3
4
5 Liver studies
6 Albumin 3.9 g/dl
7
8 Total bilirubin 6.7 mg/dl
9
10 Alkaline phosphatase 647 U/ L
11
12 Aspartate aminotransferase (SGOT) 110 U/ L
13 Alanine aminotransferase (SGPT) 102 U/ L
14
15
16 Complete blood count
17
18 Hemoglobin 13.5 g/L
19
Platelets 220,000/mm3
20
21 Leukocyte count 5,100/mm3
22
23
24 Anti-mitochondrial antibodies are negative An abdominal ultrasound demonstrates mild dilation of
25 the common bile duct There are no stones present in the gallbladder and no evidence of
26 gallbladder wall thickening What is the next best step in the management of this patient's
27 condition?
28
29
30 <0 A Abdominal CT scan
31 <0 B. Endoscopic retrograde pancreatography (ERCP)
32
33 <0 C. Percutaneous transhepatic cholangiogram (PTC)
34 <0 D. Serum amylase and lipase
35
36
<0 E. Serum CA 19-9
37
38

--
Submit
39
2
Item: 40 of 40 ~'?Mark <] C> !I ~ ~ , ~
0. ld : 2935 Previous Next Lab Values Notes Calculator Reverse Color Text Zoom
3
4
5 Explanation:
6
7 In the United States, pancreatic cance r is the fourth leading cause of cancer-related death. It
8 typically presents late and has a poor prognosis, even in those with a potentially resectable mass.
9 The five-year survival rate ranges from 10-30%, depending on nodal status. Incidence is higher
10 in men and in blacks, with the disease developing most often after the age of 45. Major risk
11 factors include chronic pancreatitis, smoking, diabetes mellitus, and hereditary predisposition.
12
13 Pancreatic carcinoma most commonly presents with a dull upper abdominal pain that radiates to
14
the back, weight loss, or jaundice. Those tumors located in the pancreatic body or tail are most
15
likely to present with pain and weight loss, while those tumors located in the pancreatic head are
16
17 most likely to present with steatorrhea, weight loss, and jaundice Physical examination is often
18 unremarkable aside from jaundice, with an abdominal mass or ascites present in only 20% of
19 patients with pancreatic cancer. Classic findings include a nontender but palpable gallbladder at
20 the right costal margin in a jaundiced patient (Courvoisier's sign) or left supraclavicular
21 adenopathy (Virchow's node) in a patient with metastatic disease.
22
23 Laboratory evaluation of those with panc reatic cance r typically demonstrates increased serum
24 bilirubin and alkaline phosphatase in conjunction with a mild anemia.
25
26 Abdominal ultrasound is usually the initial imaging performed on patients with jaundice W hen the
27 ultrasound is nondiagnostic (as in this case) , the next step is to obtain an abdominal CT scan.
28 This imaging modality has a sensitivity and specificity of 85-90% and 90-95%, respectively , for
29 the imaging of pancreatic cancer. Commonly , CT scan is effective in detecting bile and
30
31 pancreatic duct dilation, mass lesions within the pancreas, and indications of extrahepatic spread
32 (eg, metastases or ascites).
33
34 (Choice B) Endoscopic retrograde pancreatography ( ERCP) is an excellent tool in the diagnosis
35 of pancreatic cancer , with a sensitivity and specificity of 90-95% . Because it is an invasive
36 procedure, however, it is most commonly reserved for those patients who have already
37 undergone a nondiagnostic ultrasound and a nondiagnostic CT scan.
38

--
39 (Choice C) Percutaneous transhepatic cholangiogram (PTC) is an invasive procedure in which a
2
Item: 40 of 40 ~'?Mark <] C> !I ~ ~ , ~
0. ld : 2935 Previous Next Lab Values Notes Calculator Reverse Color Text Zoom
3
4
5 Abdominal ultrasound is usually the initial imaging performed on patients with jaundice W hen the
6 ultrasound is nondiagnostic (as in this case) , the next step is to obtain an abdominal CT scan.
7 This imaging modality has a sensitivity and specificity of 85-90% and 90-95%, respectively , for
8 the imaging of pancreatic cancer. Commonly, CT scan is effective in detecting bile and
9 pancreatic duct dilation, mass lesions within the pancreas, and indications of extrahepatic spread
10 (eg, metastases or ascites).
11
12 (Choice B) Endoscopic retrograde pancreatography (ERCP) is an excellent tool in the diagnosis
13 of panc reatic cancer , with a sensitivity and specificity of 90-95%. Because it is an invasive
14
procedure, however, it is most commonly reserved for those patients who have already
15
undergone a nondiagnostic ultrasound and a nondiagnostic CT scan.
16
17
{Choice C) Percutaneous transhepatic cholangiogram (PTC) is an invasive procedure in which a
18
19 needle is inserted into a dilated bile duct and contrast material is injected for bile duct
20 opacification PTC is used in the evaluation of patients who have previously identified biliary tract
21 dilation but are not candidates for ERCP. PTC also allows for certain therapeutic interventions,
22 including the drainage of infected bile (in patients with cholangitis) , extraction of stones in the
23 biliary tract, dilation of benign biliary strictures, or stent placement across malignant strictures.
24
25 (Choice D) Serum amylase and lipase are appropriate to obtain in patients with suspected
26 pancreatitis They are not helpful in diagnosing panc reatic ca rcinoma.
27
28 (Choice E) The most commonly used tumor marker for pancreatic cancer is the cance r
29 associated antigen (CA) 19-9, which has a sensitivity and specificity of 80-90% . CA 19-9 can be
30 elevated in patients with jaundice but no pancreatic cancer, however , which reduces its utility as
31 a screening tool. Postoperative monitoring of pancreatic cancer with CA 19-9 may be helpful in
32
evaluating the tumor response to chemotherapy
33
34
Educational Objective:
35
Abdom inal CT scan is a very sensitive and specific tool used in the diagnosis of pancreatic
36
37 carcinoma.
38

--
39 Time Spent 5 seconds Copyright © UWorld Last updated: [10/08/2016)
-
2
3
4
5
Item : 1 of40
0. ld: 22 14
~'?Mark <J
Previous
C>
Next
61
Lab Values
~
Notes
~
Calculator
,
Reverse Color
~
Text Zoom

6 A 76-year-old man with multi-infarct dementia is being evaluated for cough and low-grade fever.
7 He was treated for pneumonia twice in the last year. For the past 6 months, he has had difficulty
8 swallowing and occasionally regurgitates undigested food. The patient has a long history of
9 hypertension and chronic atrial fibrillation. His body mass index is 22 kg/m2 His temperature is
10 38.5 C (101 3 F) , blood pressure is 150/95 mm Hg, pulse is 102/min, and respirations are
11 16/min. Physical examination is notable fo r foul-smelling breath and a fl uctuant mass in the left
12
neck. Auscultation shows crackles in the right lung base. Chest x-ray shows an infiltrate without
13
14
cavitation in the right lower lung field. The patient is admitted, sputum and blood cultures are
15 sent, and antibiotics are started. W hich of the following is the most appropriate next step in his
16 management?
17
18
6 A. Bronchoscopy
19
20 <0 B. Contrast esophagram
21
6 C. Gastric feeding tube placement
22
23 D. Neck mass biopsy
24 <0
25
26
27 e; E. Upper gastro intestinal endoscopy
28
29
30 Submit
31
32
33
34
35
36
37
38
39
40
-
2
3
4
5
Item : 1 of40
0. ld: 22 14
~'?Mark <]
Previous
C>
Next

The most likely underlying diagnosis in this patient is Zenker's diverticulum (ZD) ZD is most
!I
Lab Values
~
Notes
~
Calculator
,
Reverse Color
~
Text Zoom

6 common in elderly patients, particularly men. It occurs in the posterior lower ce rvical esophagus
7 near the cricopharyngeus muscle. Patients may complain of dysphagia and
8 r egurgitation They often have foul-smelling breath (halitosis) secondary to pooling of material
9 in the diverticulum. If particularly large, the diverticula may be palpable Patients with ZD are at
10 risk for aspiration pneumonia, as occurred in this patient
11
12
A contrast esophagram, which will clearly show the dive rticulum, is the diagnostic test of
13
choice. Oral contrast administration in patients with a history of aspiration is associated with a
14
risk of pneumonitis. However, the risk can be reduced by a number of positioning techniques
15
16 and is indicated to diagnose ZD unless the patient has severe swallowing difficulty. Moreover ,
17 other options to visualize the esophagus, including endoscopy, are associated with very serious
18 risks of complications such as esophageal perforation ZD is usually repaired surgically
19
20 (Choice A) Bronchoscopy is generally indicated if there is concern for an obstructing mass that
21 may be causing recurrent pneumonias in the same anatomical location. Even then, CT scan of
22 the chest is the preferred initial study
23
24 {Choice C) Gastric feeding tube placement in this patient would be premature since surgical
25 treatment of ZD will likely solve his problems with aspiration
26
27 (Choice D) Neck mass biopsy would not be advisable in this patient, as the mass palpated on
28 physical examination actually represents an outpouching of the esophagus Biopsy could
29 therefore lead to esophageal perforation.
30
31 (Choice E) Upper gastrointestinal endoscopy is not the test of choice when attempting to
32 diagnose ZD as it can be confusing or even risky if the diverticulum itself is inadvertently
33
cannulated during the procedure.
34
35 Educational objective:
36
Zenke r's diverticulum is most common in elderly men and presents with dysphagia, regurgitation,
37
38 foul-smelling breath, aspiration, and occasionally a palpable mass. Contrast esophagram is the
39 test of choice fo r confi rming the diagnosis Treatment is generally surgicaL
40
-
3
4
5
6
1
Item: 2 of 40
0. ld : 2209
~'?Mark <J
Previous
C>
Next

A 58-year-old man comes to the office due to skin discoloration, anorexia, and unintentional
61
Lab Values
~
Notes
~
Calculator
,
Reverse Color
~
Text Zoom

7 weight loss of 6 kg (13.2 lb) over the past 3 months. He also reports dark urine and pale stools.
8 The patient has no fever, abdominal pain, constipation, or diarrhea. He has a history of
9 hypertension and hyperlipidemia The patient does not use tobacco, alcohol, or illic it drugs His
10 vital signs are within normal limits. BMI is 32 kg/m2 Scleral icterus is present An enlarged,
11 nontender gallbladder is palpated below the right costal margin There is no ascites. Abdom inal
12
imaging in this patient would most likely reveal which of the following?
13
14
15 e') A Clot in the portal vein
16
17 e> B. Gallstone obstructing the cystic duct
18 e') C. Intra- and extrahepatic biliary tract dilation
19
20 e> D. Intrahepatic biliary cyst
21 e') E. Panc reatic calcifications
22
23
24
Submit
25
26
27
28
29
30
31
32
33
34
35
36
37
38
39
40
-
3
4
5
6
1

7 Pancreatic and biliary duct system


8
Common hepatic duct
9
10 Liver
11
12
13
14
15
16
17
18 Gallbladder
19
20 Pancreas
21
22 t.lSpiral fold
C YSIC
23
24 duct
Smooth part
25
26
27
Accessory
28
29 pancreatic duct
Minor (of Santorini)
30
duodenal
31
papilla
32 Major
33 duodenal
34 papilla Hepatopancreatic
35
36 ampulla
Small intestine
37 @UWorld

..
38
39
40
... ... ....... ... .. . .. ...
This patient's most likely diagnosis is pancreatic cancer. The presentation can vary depending
. .. .. . ... .. .. ... .. .. .. ... . .....
~ Feed back SuWend EnQock
-
3
4
5
6
1
Item: 2 of 40
0. ld : 2209
<DUWorld
~'?Mark <J
Previous
C>
Next

This patient's most likely diagnosis is pancreatic cancer. The presentation can va ry depending
61
Lab Values
~
Notes
~
Calculator
,
Reverse Color
~
Text Zoom

on the tumor's location. Most (60%-70%) cancers occur in the head of the pancreas As these
7
tumors expand, they comp ress the pancreatic duct and common bile duct (leading to painless
8
9 jaundice), sometimes seen on imaging as the double duct sign
10
11
Subsequent backup of bile leads to intra- and extrahepatic biliary duct dilation and a
12 nontender , distended gallbladder at the right costal margin (Courvoisier sign) The jaundice
13 can also lead to pruritus, pale stools, and dark urine. In contrast, cancers in the body or tail of
14 the pancreas usually present with abdominal pain but without jaundice
15
16 (Choice A) Portal-vein thrombosis ( PVT) is most often associated with cirrhosis. Acute PVT can
17 present with gradual or sudden-onset abdominal pain, whereas chr onic PVT most often presents
18 as va riceal bleeding. This patient's 2-month history of we ight loss and jaundice is more consistent
19 with pancreatic cancer.
20
21 (Choice B) Gallstones intermittently blocking the cystic duct could lead to intermittent biliary colic
22 symptoms but are less likely to cause icterus, weight loss, and enlarged gallbladder.
23
24 (Choice D) Intrahepatic biliary cysts are an uncommon cause of abdominal pain and jaundice.
25 These cysts tend to occur more frequently in women (up to 50% of cases in Asian countries) and
26 children. In addition, intrahepatic biliary cysts usually do not cause an enlarged gallbladder.
27
28 (Choice E) Chronic pancreatitis is most often due to chronic alcohol use and can present with
29
abdominal pain radiating to the back, as well as nausea and vomiting Abdom inal imaging
30
31 typically reveals panc reatic calcifications. The patient's obstructive jaundice, absence of alcohol
32 use, and lack of abdominal pain make chronic panc reatitis unlikely
33
34 Educational objective:
35 Tumors in the head of the pancreas can present with weight loss, jaundice, and a nontender,
36 distended gallbladder on examination. Characteristic findings on imaging include intra- and
37 extrahepatic biliary tract dilation.
38
39
40
~'?Mark <J C> 61 ~ ~ , ~

--
Item: 3 of 40
2 0. ld : 2582 Previous Next Lab Values Notes Calculator Reverse Color Text Zoom

4
5
6 A 35-year-old man comes to the office for follow up of ulcerative colitis. He was diagnosed 8
7 years ago, and had involvement of the colon from the hepatic flexure to the rectum at that time.
8 The patient is maintained on daily 5-aminosalicylic acid and feels well; however , he had an
9 exacerbation of colitis 6 months ago that required a brief course of treatment with antibiotics and
10 oral glucocorticoids He has no nausea, vomiting, blood in his stool, or weight loss. Medical
11 history is otherwise unremarkable. The patient smokes a pack of cigarettes daily and does not
12
use alcohol or illicit drugs. Family history is negative for inflammatory bowel disease and
13
14 cancer. Physical examination is normal. W hich of the following is the most appropriate
15 management at this time?
16
17 e; A Offer colonoscopy now and every 1-2 years thereafter
18
19 6 B. Offer colonoscopy now and every 5 years thereafter
20 e; C. Recommend prophylactic colectomy with ileal pouch
21
22 6 D. Recommend surveillance colonoscopy starting at age 45
23
24 e; E. Surveillance colonoscopy is not needed as his disease is well controlled
25
26
Submit
27
28
29
30
31
32
33
34
35
36
37
38
39
40
~'?Mark C> a ~ ~ , ~

--
Item: 3 of 40 <]
2

4
0. ld : 2582
.. .~ ...... . . . ., Previous
. Next
., ... - . Lab Values Notes Calculator Reverse Color Text Zoom

5
6 Explanation:
7
8
9
10 Colon cancer screening in high-risk patients
11
12
13 Family history of adenomatous • Colonoscopy at age 40 or 10 years before
14 the age of diagnosis in the relative
polyps or CRC
15 (whichever comes first)
16 • First-degree relative at age <60
17 • Repeat every 3-5 years
18
19 Inflammatory bowel disease • Begin 8 years post diagnosis
20 (12-15 years if disease only in left colon)
21 • Ulcerative colitis
22 • Crohn colitis • Colonoscopy with biopsies every 1-2 years
23
24 • Begin at age 10-12
25 Familial adenomatous polyposis
• Colonoscopy every year
26
27
28 Hereditary nonpolyposis CRC • Begin at age 20-25
29 (Lynch syndrome) • Colonoscopy every 1-2 years
30
31 CRC = colorectal cancer.
32
33 @UWorld
34
35 Patients with ulcerative colitis have an increased risk of colorectal cancer (CRC), and the risk
36 is proportionate to the duration and extent of disease. CRC risk is also likely elevated in patients
37 with Crohn disease involving the colon (Crohn colitis)
38
39 CRC screening with colonoscopy and mucosal sampling should be offered to patients with
40
~'?Mark C> !J ~ ~ , ~

--
Item: 3 of 40 <]
2 0. ld : 2582 Previous Next Lab Values Notes Calculator Reverse Color Text Zoom

4 Hereditary nonpolyposis CRC • Begin at age 20-25


5 (Lynch syndrome) • Colonoscopy every 1-2 years
6
7 CRC = colorectal cancer.
8 @UWorfd
9
10 Patients with ulcerative colitis have an increased risk of colorectal cancer (CRC), and the risk
11
is proportionate to the duration and extent of disease. CRC risk is also likely elevated in patients
12
13
with Crohn disease involving the colon (Crohn colitis).
14
CRC screening with colonoscopy and mucosal sampling should be offered to patients with
15
16 ulce rative colitis, beginning 8 years after the initial diagnosis (Choices 0 and E) (patients with
17 disease limited to the rectum and left colon may begin 12-15 years post diagnosis). Repeat
18 colonoscopy should be performed every 1-2 years thereafter. Colonic dysplasia is associated
19 with progression to adenocarcinoma, and prophylactic colectomy is advised if dysplasia is
20 identified (Choice C)
21
22 (Choice B) A screening colonoscopy interval of 5 years is appropriate for patients without
23 inflammatory bowel disease who are found to have 1 or 2 small adenomatous polyps. Five yea rs
24 is also an appropriate interval for patients with a first-degree family history of CRC or
25 adenomatous polyps
26
27 Educational objective:
28 Patients with ulce rative colitis are at inc reased risk for colorectal cancer. Screening colonoscopy
29
with mucosal sampling should be offered to patients with ulce rative colitis beginning 8 years after
30
31 the initial diagnosis, and repeated every 1-2 years thereafter.
32
33 References:
34
35
1. AGA medical position statement on the diagnosis and management of colorectal
36 neoplasia in inflammatory bowel disease.
37
38
39 Time Spent 3 seconds Copyright © UWo rld Last updated: [09/06/2016)
40
2
Item: 4 of40 ~'?Mark <J C> 61 ~ ~ , ~

-
0. ld : 2898 Previous Next Lab Values Notes Calculator Reverse Color Text Zoom
3

5
6 A 52-year-old woman comes to the office due to intense itching and fatigue She is unable to
7 specify when her symptoms started as they developed gradually . Past medical history is
8 significant for hypothyroidism and carpal tunnel syndrome Current medications include
9 levothyroxine. The patient lives with her husband and 3 children. She does not smoke and drinks
10 wine on social occasions. Vital signs are normal. Cardiopulmonary examination shows no
11 abnormalities. The abdomen is soft with normal bowel sounds. Hepatomegaly is present There
12
is no scleral icterus or jaundice , but bilateral xanthelasma and skin excoriations are evident
13
14 Laboratory results are as follows
15 Serum creatin ine 1.0 mg/dl
16
17 Total cholesterol 503 mg/dl
18
19
20
21 Total bilirubin 1.5 mg/dl
22 Alkal ine phosphatase 410 U/L
23
24 Aspa rtate aminotransferase (AST, SGOT) 42 U/L
25
26 Alanine aminotransferase (ALT, SGPT) 44 U/L
27
28
A right upper quadrant ultrasound shows a normal common bile duct W hich of the following is
29
30 the most appropriate next step in management of this patient?
31
32 6 A Check anti-m itochondrial antibodies
33
34 6 B. Check anti-smooth muscle antibodies
35 e; C . Discontinue levothyroxine
36
37 6 D. Obtain MRI scan of the abdomen
38 6 E. Prescribe oral glucoco rticoids
39
40
2
Item: 4 of40 ~'?Mark <] C> !I ~ ~ , ~

-
0. ld : 2898 Previous Next Lab Values Notes Calculator Reverse Color Text Zoom
3

5 Explanation:
6
7 This patient has several features of cholestasis (impaired biliary flow), including fatigue, pruritus,
8 and elevated alkaline phosphatase A right upper quadrant ultrasound distinguishes intrahepatic
9 (no biliary tract dilation) from extrahepatic (biliary tract dilation; eg, due to gallstones)
10 cholestasis. If ultrasound suggests intrahepatic cholestasis (as with this patient), the next step is
11 to obtain serum anti-mitochondrial antibody titers, which have high sensitivity and specificity
12
for pr imary biliary cholangitis (PBC, previously termed primary biliary c irrhosis)
13
14
PBC is a chronic liver disease characterized by autoimmune destruction of the intrahepatic bile
15
ducts with resulting cholestasis. It presents most commonly in middle-aged women and is
16
17 insidious in onset. As the disease progresses, jaundice, hepatomegaly, steatorrhea, and portal
18 hype rtension may develop Additional complications can include severe hyperlipidemia (with
19 xanthelasma) and metabolic bone disease. PBC is often associated with other autoimmune
20 disorders (eg, autoimmune thyr oid disease)
21
22 {Choices 8 and E) Autoimmune hepatitis is associated with elevated titers of antinuclear
23 antibodies and anti-smooth muscle antibodies. It is characterized by fluctuating hepatocellular
24 injury (ie, elevated transaminases) rather than cholestasis. First-line treatment includes oral
25 glucocorticoids
26
27 (Choice C) A number of medications can cause intrahepatic cholestasis, including certain
28 antibiotics (eg, macrolides), anabolic steroids, and oral contraceptives. Levothyroxine is not
29
associated with cholestasis.
30
31 (Choice D) Abdominal MRI is primarily used to evaluate abdominal masses or clarify nonspecific
32
abnormalities on other imaging tests. It is not needed in the evaluation of PBC.
33
34
Educational objective:
35
Primary biliary cholangitis (previously termed primary biliary cirrhosis) is a chronic liver disease
36
37 characterized by intrahepatic cholestasis due to autoimmune destruction of small bile ducts. It
38 presents in middle-aged women with fatigue, pruritus, hepatomegaly , and elevated alkaline
39 phosphatase The diagnosis is confi rmed with serum anti-mitochondrial antibody titers.
40
2
Item: 5of40 ~'?Mark <] C> !I ~ ~ , ~
0. ld : 2951 Previous Next Lab Values Notes Calculator Reverse Color Text Zoom
3

-
4

6
7
8
9
A 63-year-old man comes to the office due to anorexia and weight loss for the past 2 months. He
has no chest pain, abdominal pain, cough, or fever. His medical problems include hypertension
and latent tuberculosis treated 30 years ago He does not use tobacco, alcohol, or illicit drugs.
Temperature is 37.5 C (99.5 F) , blood pressure is 124/80 mm Hg, pulse is 78/min, and
10 respirations are 14/min. Physical examination reveals mucosal pallor and mild hepatomegaly
11
The abdomen is soft, nondistended, and nontender. Cardiopulmonary examination is normal.
12
Fecal occult blood testing is positive Laboratory results are as follows
13
14 Complete blood count
15
16 Hemoglobin 9.8 g/dL
17
18 Mean corpuscular volume 72 ~m'
19
Platelets 276,000/mm'
20
21 Leukocytes 6,700/mm'
22
23
24
25 Live r function studies
26
27 Total bilirubin 1.3 mg/dL
28
Alkaline phosphatase 190 U/L
29
30 Aspartate aminotransferase (SGOT) 32 U/L
31
32 Alanine aminotransferase (SGPT) 38 U/L
33
34
Ultrasound reveals a solitary liver lesion measuring 2x3 em. W hich is the most likely diagnosis?
35
36
37 6 A. Angiosarcoma of the liver
38
39 6 B. Cholangiocarcinoma
40
2
Item: 5of40 ~'?Mark <J C> 61 ~ ~ , ~
0. ld : 2951 Previous Next Lab Values Notes Calculator Reverse Color Text Zoom
3

-
4

6
7
8
9
Complete blood count

Hemoglobin

Mean corpuscular volume


9.8 g/dl

72 1Jm3

Platelets 276,000/mm'
10
11 Leukocytes 6,700/mm'
12
13
14
15
Liver function studies
16 Total bilirubin 1.3 mg/dl
17
18 Alkaline phosphatase 190 U/L
19
20 Aspa rtate aminotransferase (SGOT) 32 U/L
21
22 Alanine aminotransferase (SGPT) 38 U/L
23
24 Ultrasound reveals a solitary live r lesion measuring 2x3 em. W hich is the most likely diagnosis?
25
26
27 0 A Angiosarcoma of the live r
28 0 B. Cholangiocarcinoma
29
30 0 C. Focal nodular hyperplasia
31 0 D. Hepatic adenoma
32
33 0 E. Hepatic tuberculosis
34 0 F. Hepatocellular carc inoma
35
36
0 G. Metastatic liver disease
37
38 Submi t
39
40
2
Item: 5of40
0. ld : 2951
~'?Mark <]
Previous
C>
Next
a
Lab Values
~
Notes
~
Calculator
,
Reverse Color
~
Text Zoom
3

-
4

6
7
8
9
Explanation:

Solid liver masses


10
11
12 Focal nodular • Associated with anomalous arteries
13 hyperplasia • Arterial flow & central scar on imaging
14
15
16 Hepatic • Women on long-term oral contraceptives
17
18
adenoma • Possible hemorrhage or malignant transformation
19
20 Regenerative
21 • Acute or chronic liver injury (eg, cirrhosis)
22 nodules
23
24
Hepatocellular • Systemic symptoms
25
26 • Chronic hepatitis or cirrhosis
carcinoma
27 • Elevated alpha fetoprotein
28
29
Liver • Single/multiple lesions
30
31 metastasis • Known extrahepatic malignancy
32
33 ©UWorld
34
35 Metastatic disease is the most common cause of liver mass and is much more common than
36 primary liver cancer. Undiagnosed colorectal cancer (CRC) is likely in this patient with a
37 solitary liver mass, probable iron deficiency anemia (low mean corpuscular volume), and positive
38 fecal occult blood screen. Gastrointestinal malignancies, such as colorectal or pancreatic
39 cancer, are the most frequent source of liver metastases as their venous drainage is through the
40
2
Item: 5of40 ~'?Mark <] C> !I ~ ~ , ~
0. ld : 2951 Previous Next Lab Values Notes Calculator Reverse Color Text Zoom
3

-
solita ry liver mass, probable iron deficiency anemia (low mean corpuscular volume), and positive
4
fecal occult blood screen. Gastrointestinal malignancies, such as colorectal or pancreatic
6 cancer, are the most frequent source of liver metastases as their venous drainage is through the
7 portal system directly to the liver . Lung, breast, and skin cancers (melanoma) often also spread
8 to the liver. The liver is a common site of metastatic disease due to its dual blood supply
9 (systemic and portal) and hepatic sinusoidal fenestrations allowing for easier metastatic
10 deposition.
11
12 Liver metastases are often clinically silent unless pressure on the live r capsule or obstruction of
13 the biliary tree causes pain or jaundice Laboratory findings may be normal or mildly elevated,
14
even in the setting of heavy tumor burden. Multiple hepatic nodules are typically seen in
15
metastatic disease; however, solitary lesions are not uncommon. If a primary tumor is identified
16
17 (eg, with colonoscopy), liver biopsy is usually not needed.
18
(Choice A) Hepatic angiosarcoma is a rare liver neoplasm more common in older men who have
19
20 been exposed to toxins (eg, vinyl chloride gas, inorganic arsenic compounds, thorium dioxide).
21
22 (Choice B) Cholangiocarcinoma typically presents with symptoms of biliary obstruction -
23 jaundice, pruritus, light-colored stools, and dark urine. The main risk factor is a history of
24 primary sclerosing cholangitis
25
26 (Choices C and D) Focal nodular hyperplasia (FNH) and hepatic adenoma are usually benign
27 liver tumors seen in young women. FNH is typically asymptomatic ; hepatic adenoma may cause
28 right upper quadrant pain and is associated with oral contraceptive use.
29
30 (Choice E) Primary hepatic tuberculosis is rare due to the low oxygen tension in the liver. Even
31 with disseminated tuberculosis, a solitary liver mass would be unlikely
32
33 (Choice F) Hepatocellular carcinoma (HCC) typically emerges from a chronically inflamed liver
34 (eg, chronic hepatitis B or C infection). HCC is the most common type of primary live r cance r
35 but is a far less common cause of hepatic malignancy than metastas.is from another source.
36 Microcytic anemia and a positive fecal occult blood screen make CRC more likely in this patient
37
38 Educational objective:
39 t t . f
40
Th t 1· • f th I' th •
~ Feedback SuWend EnQock
2
Item: 5of40 ~'?Mark <] C> !I ~ ~ , ~
0. ld : 2951 Previous Next Lab Values Notes Calculator Reverse Color Text Zoom
3

-
4

6
7
8
9
Liver metastases are often clinically silent unless pressure on the liver capsule or obstruction of
the biliary tree causes pain or jaundice Laboratory findings may be normal or mildly elevated,
even in the setting of heavy tumor burden. Multiple hepatic nodules are typically seen in
metastatic disease; however, solitary lesions are not uncommon. If a primary tumor is identified
(eg, with colonoscopy), live r biopsy is usually not needed.
10
11 (Choice A) Hepatic angiosarcoma is a rare liver neoplasm more common in older men who have
12 been exposed to toxins (eg, vinyl chloride gas, inorganic arsenic compounds, thorium dioxide)
13
14 {Choice B) Cholangiocarcinoma typically presents with symptoms of biliary obstruction -
15 jaundice, pruritus, light-colored stools, and dark urine. The main risk factor is a history of
16 primary sclerosing cholangitis
17
18 (Choices C and D) Focal nodular hyperplasia (FNH) and hepatic adenoma are usually benign
19 liver tumors seen in young women. FNH is typically asymptomatic ; hepatic adenoma may cause
20 right upper quadrant pain and is associated with oral contraceptive use.
21
22 {Choice E) Primary hepatic tuberculosis is rare due to the low oxygen tension in the live r. Even
23 with disseminated tuberculosis, a solitary liver mass would be unlikely .
24
25 {Choice F) Hepatocellular carcinoma (HCC) typ ically emerges from a chronically inflamed liver
26 (eg, chronic hepatitis B or C infection). HCC is the most common type of primary live r cance r
27
but is a far less common cause of hepatic malignancy than metastasis from another source.
28
29 Microcytic anemia and a positive fecal occult blood screen make CRC more likely in this patient
30
31 Educational objective:
32 The most common malignancy of the live r is metastasis from another primary source.
33
34 References:
35
36 1. Epidemiology of primary and secondary liver cancers.
37
38
39 Time Spent 5 seconds Copyright © UWorld Last updated: [10/06/2016)
40
2
Item: 6 of 40 ~'?Mark <J C> 61 ~ ~ , ~
0. ld : 2207 Previous Next Lab Values Notes Calculator Reverse Color Text Zoom
3
4

-
5

7
8
9
10
A 32-year-old woman comes to the office due to intermittent abdominal pain and nonbloody
diarrhea for the past 3-4 months. She describes the pain as crampy and located in the
mid-abdomen and right lower quadrant. The patient thinks she has lost some weight during this
period. She modified her diet several times in an attempt to decrease the symptoms, believing
that heavy meals exacerbate the pain and diarrhea. The patient underwent appendectomy with
11 abscess debridement one year ago. She has no other medical issues and takes no medication
12
other than oral contraceptives. She has not traveled outside the United States and does not use
13
14 tobacco, alcohol, or recreational drugs. Temperature is 36.7 C (98 F), blood pressure is 120/70
15 mm Hg, pulse is 85/min, and respirations are 14/min. A few shallow ulcers are present in her
16 mouth. Abdom inal examination shows mild tenderness in the right lower quadrant without
17 rebound. Laboratory results are as follows
18
19 Hemoglobin 10.2 g/dL
20 Leukocytes 14,500/mm' -
21
22 Platelets 530,000/mm'
23
24 Eryth rocyte sedimentation
48 mm/hr
25 rate
26
27
28
W hich of the following is the most likely diagnosis for this patient?
29
30 e') A. Celiac disease
31
32 6 B. Grahn disease
33 6 C. Giardia infection
34
35 6 D. Irritable bowel syndrome
36 e') E. Lactose intolerance
37
6 F. Ulcerative col itis
38
39 6 G. Yersinia infection
40 -------------------------------

~ Feedback SuWend EnQock


2
Item : 6 of 40
0 . l d : 2207
~'?Mark <]
Previous
C>
Next
a
Lab Values
~
Notes
~
Calculator
,
Reverse Color
~
Text Zoom
3
4

-
5

7
8
9
10
Explanation:

Crohn disease
11
12 • Gl: Abdominal pain, nonbloody diarrhea, oral ulcers,
13 malabsorption, weight loss, fistula/abscess formation
14 Clinical
15 find ings • Extraintestinal: MSK (arthritis). eye (eg, uveitis, scleritis,
16 episcleritis), skin (eg, erythema nodosum, pyoderma
17 gangrenosum)
18
19
20 • t WBC, iron deficiency anemia, t inflammatory markers
21
Diagnosis • Endoscopy: Focal ulcerations adjacent to normal mucosa
22 (cobblestoning), skip areas of disease
23
24 • Radiography: Strictu res, bowel wall thickening
25
26 • 5-ASA drugs, corticosteroids, antibiotics
27
28 Treatment • Azathioprine
29 • Anti-TNF therapies
30
31
A SA = aminosalicylic acid; Gl = gastrointestinal; MSK = musculoskeletal;
32 TNF = tumor necrosis factor, WBC = white blood cell.
33
©UWond
34
35 This patient with chronic abdominal pain , diarrhea, weight loss, and evidence of inflammation
36 (elevated erythrocyte sedimentation rate [ESR)) likely has Cr ohn disease. Other frequent
37
gastrointestinal manifestations of Crohn disease include microscopic bleeding, fistula and
38
39 stricture formation, abdominal abscesses, and malabsorption It can involve any part of the
I ~ I I., t ~ ~ •tt Ill I '"' I ~ •t •• . I ~ t 1111111 '"'4 ~ I., t ~
40

~ Feedback SuWend EnQ ock


2
Item: 6 of 40 ~'?Mark <] C> !I ~ ~ , ~
3
4
0. ld : 2207
. . ... . .. . . - Previous
. - Next
.. . . ... .. Lab Values Notes Calculator Reverse Color Text Zoom

gastrointestinal manifestations of Crohn disease incl ude microscopic bleeding, fistula and

-
5

7
8
9
10
stricture formation, abdominal abscesses, and malabsorption It can involve any part of the
gastrointestinal tract from mouth (eg, aphthous ulcers) to anus. Common extraintestinal
symptoms include arthritis, uveitis, scleritis, and erythema nodosum. Laboratory findings are
similar to other chronic inflammatory diseases and can include leukocytosis, anemia, reactive
thrombocytos is, and elevated inflammatory markers (eg, ESR). Diagnosis is confi rmed with
11 endoscopic or r adiogr aphic studies, and treatment involves immunosuppressive therapy.
12
13 (Choice A) Although cel iac disease can also present with chronic diarrhea, abdominal pain,
14 weight loss, and anemia from malabsorption, elevated inflammatory markers are uncommon, as
15 are oral manifestations.
16
17 {Choice C) Chronic giardiasis commonly presents with loose, greasy stools, significant weight
18 loss, and abdominal cramping; however , oral involvement, anemia, and elevated inflammatory
19 markers are not typ ical. Moreover, this patient does not have clear risk factors for
20 Giardia infection (eg, international travel, hiking)
21
22 (Choices 0 and E) Irritable bowel syndrome and lactose intolerance are not commonly
23 associated with weight loss, anemia, leukocytos is, or elevated inflammatory marke rs. Such
24 findings should prompt evaluation for other causes of gastrointestinal complaints
25
26 (Choice F) Ulcerative colitis more frequently presents as bloody (rather than nonbloody)
27
diarrhea with tenesmus and incontinence, and does not involve the oral mucosa.
28
29 {Choice G) Infectious col itis due to a number of bacteria (including Yersinia) , parasites, or
30
amoebae can closely mimic Crohn disease, especially if it presents with ileitis. However ,
31
32 infectious colitis is a more acute process and does not involve the oral mucosa.
33
34 Educational objective:
35 Crohn disease is characterized by chronic abdominal pain, diarrhea, weight loss, and evidence
36 of inflammation (eg, anemia, elevated inflammatory markers). It can involve any part of the
37 gastrointestinal tract from mouth to anus.
38
39
40
2
Item: 7of40 ~'?Mark <] C> !J ~ ~ , ~
0. ld : 3834 Previous Next Lab Values Notes Calculator Reverse Color Text Zoom
3
4

...
5
6

8
9
A 44-year-old woman comes to the physician with 2 months of low-grade fever, abdominal pain,
and bloody diarrhea. Over the past 2 days, her symptoms have increased. She takes no
medications and has no allergies She has lost 4.5 kg (10 lb) over the past 4 weeks. Her
temperature is 38.9 C (102 F), blood pressure is 102/70 mm Hg, pulse is 118/min, and
10 respirations are 22/min. Examination shows pale and dry mucous membranes. Abdom inal
11 examination shows diffuse tenderness and distension. Laboratory results are as follows
12
13 Hemoglobin 9.5 g/dl
14
15 W hite blood cells 16 ,000/~L
16
17 Sodium 145 mEq/L
18
Potassium 3.0 mEq/L
19
20
21 Her HIV test is negative. An x-ray of the abdomen is shown below.
22
23
24
25
W hich of the following is the most likely diagnosis in this patient?
26
27
28 0 A Campylobacter colitis
29
30 0 B. Cytomegalovirus colitis
31 0 C. Ischemic colitis
32
33
0 D. Obstructed colon cancer
34 0 E. Pseudomembranous colitis
35 0 F. Staphylococcus aureus gastroenteritis
36
37 0 G. Toxic megacolon
38
39
40
2
3
4
5

-
6

8
9
10
11
12
13
14
15
16 Which of the following is the most likely diagnosis in this patient?
17
18
A. Campy/obacter colitis (4%]
19
20 B. Cytomegalovirus colitis (1%)
21 C. Ischemic colitis (3%)
22
23 D. Obstructed colon cancer (11%)
24 E. Pseudomembranous colitis (2%)
25
26 F. Staphylococcus aureus gastroenteritis (0%)
27 .; G. Toxic megacolon [80%)
28
29
30 Explanation:
31
32
33
34
35
36
37
38
39
40
Item : 7of40 ~t'Mark ~ C> !I ~ ~ , ~
2
3
4
0. ld : 3834
. . .
. .
Prevoous Next · · .. olues Notes Calculator Reverse Color Text Zoom

...
5
6

8
9
10
11
12
13
14
15
16
17
18
19
20
21
22
23
24
25
26
27
28
29
30
31
32
33
34
35 This patient's presentation is consistent with likely inflammatory bowel disease {lBO) Her fever
36 and 2 months of bloody diarrhea suggest ulcerative colitis (UC). Patients with UC typically
37 develop multiple bloody bowel movements (sometimes >10-15/day) with systemic findings
38 including fever and we ight loss. UC patients usually have inflammation limited to the colonic
39
II t" •• • · .. " I ... t• ... II l i., uu., II ....( ... 1 1 II t 1· 1111 I II ... " ...... , , It
40

~ Feedback SuWend EnQock


2
3
4

...
5
6

8
9
This patient's presentation is consistent with likely inflammatory bowel disease {lBO) Her fever
and 2 months of bloody diarrhea suggest ulcerative colitis (UC). Patients with UC typically
10
11 develop multiple bloody bowel movements (sometimes >10-15/day) with systemic findings
12 including fever and we ight loss. UC patients usually have inflammation limited to the colonic
13 mucosa. However, a subset develops inflammation extending to the smooth muscle layer , leading
14 to muscle paralysis and colonic dilation.
15
16 Toxic megacolon typ ically presents with total or segmental nonobstructive colonic dilation, severe
17 bloody diarrhea, and systemic findings (eg, fever , tachycardia) lBO patients are at the highest
18 risk of developing toxic megacolon early in the disease, within 3 years of diagnosis in most
19 cases. Other causes of toxic megacolon include ischemic colitis, volvulus, diverticulitis, infections
20 (eg, Clostridia difficile), and obstructive colon cancer (less common). Diagnosis is confi rmed by
21
plain abdominal x-rays and 2:3 of the following fever >38 C (100.4 F), pulse >120/min, white
22
23 blood cells >10,500/iJL, and anemia. Other findings can include volume depletion, altered mental
24 status, hypotension, and electrolyte abnormalities. Plain films usually reveal dilated right or
25 transverse colon (>6 em), possible multiple air-fluid levels, and thick haustral markings that do not
26 extend across the entire lumen (white arrows in above image).
27
28 Toxic megacolon is a medical emergency that can progress rapidly and result in colonic
29 perforation Treatment includes intravenous fl uids, broad-spectrum antibiotics, and bowel
30 rest Intravenous corticosteroids are preferred for treating ISO-induced toxic
31 megacolon Emergency surgery (subtotal colectomy with end-ileostomy as the procedure of
32 choice) may be required if the colitis does not resolve.
33
34 {Choice A) Campylobacter, Shigella, and Salmonella infections are rarely complicated by toxic
35 megacolon.
36
37 (Choice B) Cytomegalovirus colitis can cause toxic megacolon more commonly in HIV
38 patients. This patient's negative HIV test makes this less likely
39
40
2
Item: 7of40 ~'?Mark <] C> !I ~ ~ , ~
0. ld : 3834 Previous Next Lab Values Notes Calculator Reverse Color Text Zoom
3
4 (Choice C) This patient has no risk factors (eg, aortoiliac surgery, ca rdiopulmonary bypass,

...
5
6

8
9
myocardial infa rction, hemodialysis) for ischemic colitis. Even if the patient had these factors,
her presentation would be acute rather than after 2 months of symptoms .

(Choice D) Right-sided colon cancer tends to present with anemia, whereas left-sided tumors
present with bowel obstruction. Both types are usually associated with a change in bowel
10 habits. Left-sided tumors can present with abdominal pain and bowel obstruction. However ,
11
these patients tend to have constipation rather than bloody diarrhea. In addition, obstructive
12
colon cancer causes toxic megacolon less commonly than lBO.
13
14
(Choice E) Pseudomembranous colitis is usually due to Clostridium difficile col itis that is most
15
16 commonly due to recent antibiotic use (eg, clindamycin, fluoroquinolones). Patients typically
17 develop diarrhea, abdominal pain, and findings of col itis in the sigmoid/rectal area. This patient's
18 absence of recent antibiotic use makes this less likely.
19
20 (Choice F) Staphylococcus aureus gastroenteritis is due to ingestion of preformed
21 enterotoxin. It has a short incubation period and short duration. Patients usually develop vomiting
22 and profuse watery diarrhea. However, it is not usually associated with the bloody diarrhea seen
23 in this patient
24
25 Educational objective:
26 Toxic megacolon typ ically presents with total or segmental nonobstructive colonic dilation, severe
27 bloody diarrhea, and systemic findings (eg, fever, tachycardia) Patients with inflammatory bowel
28 disease are at higher risk of developing toxic megacolon. Diagnosis is confirmed by plain
29
abdominal x-rays and <::3 of the following fever >38 C (1004 F), pulse >120/min, white blood cell
30
31 count >10,500/IJL, and anemia. Toxic megacolon is a medical emergency that requires prompt
32 intravenous steroids, nasogastric decompression, antibiotics, and fluid management
33
34 References:
35
36 1. Toxic megacolon.
37
38
39 Time Spent 3 seconds Copyright © UWorld Last updated: [06/ 11/2016)
40
2
Item: 8of40 ~'?Mark <J C> 61 ~ ~ , ~
0. ld : 4085 Previous Next Lab Values Notes Calculator Reverse Color Text Zoom
3
4
5
6 A 65-year-old woman comes to the office due to gastro intestinal bleeding. She has had 3 or 4

-
7

9
10
11
12
episodes of dark maroon-colored stools in the last 2 weeks, with normal intervening bowel
movements. The patient has no associated abdominal or rectal pain, nausea, or vomiting She
has a history of hypertension, type 2 diabetes mellitus, and hypercholesterolemia Her
temperature is 36.7 C (98 F) , blood pressure is 140/80 mm Hg, pulse is 95/min, and respirations
are 16/min. Physical examination reveals a 3/6 systolic ejection murmur in the right second
intercostal space Carotid pulses are delayed on palpation bilaterally Abdominal and rectal
13
14 examinations are benign Laboratory results are as follows
15 Hemoglobin 11 .1 g/dl
16
17 Mean co rpuscular
18 90 11m'
volume
19
20 Blood urea nitrogen 34 mg/dl
21 -
22 1.6
Creatinine
23 mg/dl
24
25 Colonoscopy 6 months earlier was unremarkable but was somewhat limited in the ascending
26
colon due to suboptimal bowel preparation W hich of the following is the most likely cause of this
27
28
patient's symptoms?
29
30 e') A Angiodysplasia
31
32 6 B. Colon cancer
33 6 C. Diverticulosis
34
35 6 D. Hemorrhoids
36 e') E. Ischemic col itis
37
38
39 Submit
40
2
Item: 8of40 ~'?Mark <] C> !I ~ ~ , ~
0. ld : 4085 Previous Next Lab Values Notes Calculator Reverse Color Text Zoom
3
4
5
Explanation:
6

-
7

9
10
11
12
13
14
15
16
17
18
19
20
21
22
23
24
25
26
27
28
29
30
31
32
33
34
35
36
37
38
39
40
2
3
4
5
6

-
7

9
10
11
12
This clinical presentation of episodic painless gastrointestinal (GI) bleeding suggests
angiodysplasia Angiodysplasia is characterized by dilated submucosal veins and arteriovenous
malformations, and has an increased incidence after age 60. It may occur anywhere in the Gl
tract but is most common in the right colon. Angiodysplasia is more frequently diagnosed in
patients with advanced renal disease and von W illebrand (vW) disease, possibly due to the
13 bleeding tendency associated with these disorders. Angiodysplasia may also be more common
14 in patients with aortic stenosis (AS), possibly due to acquired vW factor deficiency (from
15 disruption of the vW multimers as they traverse the turbulent valve space induced by AS)
16 Angiodysplastic bleeding has been reported to remit following aortic valve replacement
17
18 Diagnosis of angiodysplasia is usually made on endoscopic evaluation (eg, upper Gl endoscopy,
19
colonoscopy). However , it is not uncommon for angiodysplasia to be missed on colonoscopy due
20
to poor bowel preparation or location behind a haustral fold. Asymptomatic patients do not
21
22 require treatment Patients with anemia or gross or occult bleeding can be treated
23 endoscopically, usually with cautery.
24
25 (Choice B) Colon cancer can cause painless chronic bleeding However, a cancer capable of
26 causing gross bleeding (as seen in this patient) is unlikely to have been missed on colonoscopy
27 and would likely have led to microcytic anemia (with mean corpuscular volume <80fl.tm') It is
28 more likely that angiodysplasia rather than colon cancer would be missed on colonoscopy
29
30 (Choice C) Diverticulosis is also unlikely to have been missed on colonoscopy. In addition,
31 bleeding from diverticula is frequently arterial, and typ ically results in passage of bright red
32 blood. Maroon-colored stools are more characteristic of right colonic angiodysplasia.
33
34 (Choice D) Hemorrhoids cause bright red rectal bleeding, with blood on the surface of the stool
35 or dripping into the toilet They are usually apparent on rectal examination or during colonoscopy
36
37 (Choice E) Ischemic colitis usually presents with sudden onset of abdominal pain and
38 tenderness followed by rectal bleeding or bloody diarrhea within 24 hours. This patient has no
39 bd . I I .
40

~ Feedback SuWend EnQock


2
Item: 8of40 ~'?Mark <] C> !I ~ ~ , ~
0. ld : 4085 Previous Next Lab Values Notes Calculator Reverse Color Text Zoom
3
4 require treatment Patients with anemia or gross or occult bleeding can be treated
5 endoscopically, usually with cautery
6

-
7

9
10
11
12
(Choice B) Colon cancer can cause painless chronic bleeding. However, a cance r capable of
causing gross bleeding (as seen in this patient) is unlikely to have been missed on colonoscopy
and would likely have led to microcytic anemia (with mean corpuscular volume <80fl.lm') It is
more likely that angiodysplasia rather than colon cance r would be missed on colonoscopy

{Choice C) Diverticulosis is also unlikely to have been missed on colonoscopy. In addition,


13 bleeding from diverticula is frequently arterial, and typically results in passage of bright red
14
blood. Maroon-colored stools are more characteristic of right colonic angiodysplasia
15
16 (Choice D) Hemorrhoids cause bright red rectal bleeding, with blood on the surface of the stool
17
or dripping into the toilet They are usually apparent on rectal examination or during colonoscopy.
18
19 (Choice E) Ischemic colitis usually presents with sudden onset of abdominal pain and
20
tenderness followed by rectal bleeding or bloody diarrhea within 24 hours. This patient has no
21
22 abdominal pain
23
24 Educational objective:
25 Angiodysplasia is characterized by dilated submucosal veins and arteriovenous malformations. It
26 is a common cause of recurrent, painless gastrointestinal bleeding. Diagnosis is made on
27 colonoscopy, although it is frequently missed. Asymptomatic patients do not require treatment
28 Those with anemia or bleeding can be treated with cautery
29
30 References:
31
32 1. Factors that contr ibute to blood loss in patients with colonic angiodysplasia
33 fr om a population-based study.
34
35
2. Clinical features and endoscopic findings in patients with actively bleeding
36 colonic angiodysplasia.
37
38
39 Time Spent 3 seconds Copyright© UWorld Last updated: [09/27/2016)
40
2
Item : 9 of40 ~'?Mark <J C> 61 ~ ~ , ~
0. ld : 4919 Previous Next Lab Values Notes Calculator Reverse Color Text Zoom
3
4
5
6 A 54-year-old man comes to the office after a year of progressive fatigue and frequent loose
7 stools. He describes voluminous, foul-smelling stools that float and are difficult to fl ush. The

-
8

10
11
12
13
patient has lost about 9 kg (20 lbs) in the past 6 months. He was hospitalized on multiple
occasions several years ago for epigastric pain radiating to the back, nausea, and vomiting.
Lately, the patient has had intermittent episodes of similar pain lasting for 15-30 minutes after
meals. He has had no hematemesis, hematochezia, or melena. The patient stopped drinking
alcohol several years ago and does not use tobacco or recreational drugs. Temperature is 37 C
14 (986 F) , blood pressure is 11 8/80 mm Hg, and pulse is 78/min. BMI is 19.5 kg/m2 Abdominal
15 examination shows epigastric tenderness on deep palpation. Bowel sounds are normaL Which
16 of the following is most likely to improve this patient's symptoms?
17
18
0 A Antimicrobial therapy
19
20 0 B. Cholecystectomy and stone removal
21
0 C. Gluten-free diet
22
23 6 D. Helicobacter pylori eradication
24 0 E. Mesenteric angioplasty
25
26 0 F. Pancreatic enzyme supplementation
27 ® G. Restriction of dairy products
28
29
30
Submit
31
32
33
34
35
36
37
38
39
40
2
Item : 9 of40 ~'?Mark <] C> a ~ ~ , ~
3
4
. .
a. ld : 4919
. .
Previous Next I Ah V')lues Notes Calculator Reverse Color Text Zoom

5
6
7

-
8
Common causes of steatorrhea

10 • Chroni c pancreatitis due to alcohol abuse, cystic


11 Pancreatic fibrosis, or autoimmune/hereditary pancreatitis
12 insufficiency
13 • Pancreatic cancer
14
15
16 • Small-bowel Crohn disease
17 • Bacterial overgrowth
18
Bile salt-related • Primary biliary cirrhosis
19
20 • Primary sclerosing cholangitis
21
• Surgical resection of ileum (at least 60-100 em)
22
23
24 • Celiac disease
25
Impaired
26 intestinal surface • AIDS enteropathy
27 epithelium • Giardiasis
28
29
30 • Whipple disease
31 Other rare
• Zollinger-EIIison syndrome
32 causes
33 • Medication-induced
34
35 © UWOI1d
36
37 This patient's presentation of weight loss and loose, greasy, malodorous stools that float in the
38 toilet and are difficult to flush is consistent with fat malabsorption (steatorrhea), which can be
39 caused by various conditions.
40
2
Item : 9 of40 ~'?Mark <] C> !I ~ ~ , ~
0. ld : 4919 Previous Next Lab Values Notes Calculator Reverse Color Text Zoom
3 p p g I 'g y, o
4
toilet and are difficult to flush is consistent with fat malabsorption (steatorr hea), which can be
5
6
caused by various conditions.
7

-
This patient's steatorrhea is most likely due to chr onic pancreatitis related to long-standing
8
alcohol abuse. His previous hospitalizations for vomiting and epigastric pain that radiates to the
10 back raise suspic ion for recurrent episodes of acute alcoholic pancreatitis Repeated pancreatic
11 injury causes progressive inflammation and fib rosis and results in impaired endocrine and
12 exocrine (loss of digestive enzymes) functions. Patients with chronic pancreatitis may initially be
13 asymptomatic , but disease progression ultimately leads to malabsorption, steatorrhea, and
14 postprandial epigastric pain that can become continuous. Symptomatic improvement can be
15 achieved with alcohol cessation and pancreatic enzyme supplementation
16
17 (Choice A) Antimicrob ial therapy (eg, rifaximin) should be considered in patients with small
18 intestinal bacterial overgrowth syndrome, which can present with abdominal pain/bloating and
19
symptoms of malabsorption However, this condition is typically associated with anatomical
20
abnormalities (eg, surgical blind loop) or motility disorders (eg, scleroderma) of the
21
22 gastrointestinal tract
23
24 (Choice B) Cholecystectomy and stone removal should be considered in patients with biliary
25 colic , which typically presents with postprandial right upper quadrant/epigastric pain associated
26 with nausea/vomiting Steatorrhea and malabsorption are not characteristic .
27
28 (Choice C) A gluten-free diet is appropriate fo r individuals with celiac disease, which can
29 present with diarrhea, malabsorption, and nutritional deficiencies. However, this patient's history
30 of alcohol use and epigastric abdominal pain that radiates to the back is more consistent with
31 chronic panc reatitis.
32
33 (Choice D) Helicobacter pylori infection is a common cause of peptic ulcer disease, which can
34 present with postprandial epigastric pain, nausea/vomiting, and weight loss due to food aversion.
35 However , peptic ulce r disease does not usually cause steatorrhea.
36
37 (Choice E) Mesenteric angioplasty should be considered in patients with chronic mesenteric
38 ischemia, which can present with postprandial abdominal pain (intestinal angina) and weight loss
39
due to food aversion. However. steatorrhea is not characteristic .
40
2
Item : 9 of40 ~'?Mark <] C> !I ~ ~ , ~
0. ld : 4919 Previous Next Lab Values Notes Calculator Reverse Color Text Zoom
3
4 colic, which typically presents with postprandial right upper quadrant/epigastric pain associated
5 with nausea/vomiting Steatorrhea and malabsorption are not characteristic.
6
7 (Choice C) A gluten-free diet is appropriate fo r individuals with celiac disease, which can

-
8

10
11
12
13
present with diarrhea, malabsorption, and nutritional deficiencies. However, this patient's history
of alcohol use and epigastric abdominal pain that radiates to the back is more consistent with
chronic pancreatitis.

{Choice D) Helicobacter pylori infection is a common cause of peptic ulcer disease, which can
present with postprandial epigastric pain, nausea/vomiting, and weight loss due to food aversion.
14
However, peptic ulcer disease does not usually cause steatorrhea.
15
16 (Choice E) Mesenteric angioplasty should be considered in patients with chronic mesenteric
17
ischemia, which can present with postprandial abdominal pain (intestinal angina) and weight loss
18
19 due to food aversion. However, steatorrhea is not characteristic.
20
{Choice G) Restriction of dairy products may be considered in patients with lactose intolerance,
21
22 which typically presents with abdominal pain/c ramping, bloating, flatulence, and frothy/watery
23 diarrhea after dairy product consumption. However, steatorrhea and malabsorption are not
24 characteristic.
25
26 Educational objective:
27 Steatorrhea occurs due to fat malabsorption and generally presents with voluminous, greasy, and
28 foul-smelling stools that are difficult to flush. It is commonly caused by pancreatic exocrine
29 insufficiency (loss of digestive enzymes) in patients with chronic alcoholic pancreatitis Alcohol
30 cessation and pancreatic enzyme supplementation can improve symptoms in such patients.
31
32
Refer ences:
33
34 1. Chronic pancreatitis.
35
36 2. Diagnosis and treatment of pancreatic exocrine insufficiency.
37
38
39 Time Spent 2 seconds Copyright© UWorld Last updated: [08/ 11/2016)
40
2
Item : 10of40 ~'?Mark <J C> 61 ~ ~ , ~
0. ld : 49 27 Previous Next Lab Values Notes Calculator Reverse Color Text Zoom
3
4
5
6 A 43-year-old Caucasian female is brought to the emergency room (ER) with bloody vom iting
7 Her past medical history is significant for idiopathic cirrhosis. She was hospitalized five months
8 ago fo r variceal band ligation. Upon initial evaluation, she is somnolent and barely arousable.

--
9

11
12
13
14
Her blood pressure is 94/52 mmHg and her heart rate is 121/min. Dried blood is seen around
her mouth. The abdomen is distended with a positive fluid shift. There is 1+ peripheral edema.
Her hemoglobin is 8.2 g/dl and her platelet count is 59,000/mm' . Her INR is 1.9. Two large-bore
IV lines are placed and two liters of normal saline are given wide open W hile in the ER, she has
another episode of bloody vomiting with clots. W hich of the following is the best initial step in
15 managing this patient?
16
17 e; A piagnostic paracentesis
18
19 e> B. Endotracheal intubation
20 e; C. Abdominal x-ray
21
22 e> D. Nasogastric tube placement
23
24 e; E. Upper Gl endoscopy
25
26
Sub mit
27
28
29
30
31
32
33
34
35
36
37
38
39
40
2
Item : 10of40 ~'?Mark <] C> !I ~ ~ , ~
0. ld : 49 27 Previous Next Lab Values Notes Calculator Reverse Color Text Zoom
3 • W. • W. • • ' • I

4
5
6 Explanation:
7
8 This patient has upper gastro intestinal bleeding with ongoing hematemesis. Given her history of

--
9

11
12
13
14
cirrhosis and va riceal band ligation, esophageal va riceal hemorrhage is the most likely cause.
She is developing hemodynamic instability and a depressed level of consciousness and could
rapidly decompensate, thus rapid evaluation and resuscitation is needed. Initial evaluation should
begin with the ABCs airway , breathing, and circulation. This patient's hematemesis with a
depressed level of consciousness is a set up fo r aspiration, which could quickly exacerbate her
already tenuous clinical status. Endotracheal intubation would be indicated at this time to secure
15
16 her airway. Two large bore IVs have already been placed to begin aggressive fl uid resuscitation,
17 and a type and screen should be performed since the patient will likely need a blood transfusion.
18 Platelet transfusion is not generally indicated until levels fall below 50,000/mms Bleeding may be
19 exacerbated by this patient's elevated INR for which FFP could be considered, but it is often of
20 minimal effectiveness in cirrhotic patients receiving aggressive fluid resuscitation. Administration
21 of prophylactic antibiotics, and possibly a somatostatin analog such as octreotide, should also be
22 considered.
23
24 (Choice A) Diagnostic paracentesis could be considered at some point to evaluate for
25 spontaneous bacterial peritonitis (SSP) which can be a complicating factor in cirrhotic patients
26 with esophageal variceal hemorrhage. However , this is not as important currently as stabilizing
27
the patient and attempting to stop the bleeding
28
29
{Choice C) An abdominal x-ray rarely provides helpful info rmation in patients with gastrointestinal
30
31 bleeding
32
(Choice D) It is controversial as to whether nasogastric tube placement decreases the risk of
33
34 aspiration, but it certainly does not provide the same level of airway protection as endotracheal
35 intubation.
36
37 (Choice E) Upper endoscopy should be done in an attempt to stop this patient's variceal
38 hemorrhage, but this should be done after the patient is stabilized and intubated.
39
40
2
Item : 10of40 ~'?Mark <] C> !I ~ ~ , ~
0. ld : 49 27 Previous Next Lab Values Notes Calculator Reverse Color Text Zoom
3
4 rapidly decompensate, thus rapid evaluation and resuscitation is needed. Initial evaluation should
5 begin with the ABCs airway, breathing, and circulation. Thi s patient's hematemesis with a
6 depressed level of consciousness is a set up fo r aspiration, which could quickly exacerbate her
7 already tenuous clinical status. Endotracheal intubation would be indicated at this time to secure
8 her airway. Two large bore IVs have already been placed to begin aggressive fluid resuscitation,

--
9

11
12
13
14
and a type and screen should be performed since the patient will likely need a blood transfusion.
Platelet transfusion is not generally indicated until levels fall below 50,000/mms Bleeding may be
exacerbated by this patient's elevated INR for which FFP could be considered, but it is often of
minimal effectiveness in cirrhotic patients receiving aggressive fluid resuscitation. Administration
of prophylactic antibiotics, and possibly a somatostatin analog such as octreotide, should also be
15 considered.
16
17 (Choice A) Diagnostic paracentesis could be considered at some point to evaluate for
18 spontaneous bacterial peritonitis (SSP) which can be a complicating factor in cirrhotic patients
19 with esophageal variceal hemorrhage. However, this is not as important currently as stabilizing
20 the patient and attempting to stop the bleeding
21
22 {Choice C) An abdominal x-ray rarely provides helpful information in patients with gastrointestinal
23 bleeding
24
25 (Choice D) It is controversial as to whether nasogastric tube placement decreases the risk of
26 aspiration, but it certainly does not provide the same level of airway protection as endotracheal
27 intubation.
28
29 (Choice E) Upper endoscopy should be done in an attempt to stop this patient's variceal
30 hemorrhage, but this should be done after the patient is stabilized and intubated.
31
32 Educational objective:
33 Patients with upper gastrointestinal bleeding who have a depressed level of consciousness and
34
ongoing hematemesis should be intubated to protect the airway as a part of initial stabilization and
35
resuscitation. Prompt endoscopic treatment with band ligation or sclerotherapy should then be
36
37 performed to stop the bleeding
38
39 Time Spent 4 seconds Copyright© UWorld Last updated: [09/22/2016)
40

~
-------------------------------------------------------------------------------------------------------------------------------
Feedback SuWend EnQ ock
2
Item : 11 of40 ~'?Mark <] C> !I ~ ~ , ~
0. ld : 4697 Previous Next Lab Values Notes Calculator Reverse Color Text Zoom
3
4
5
6 A 70-year-old male presents to the emergency room complaining of weakness, dizziness and
7 back pain. He denies nausea, vomiting, diarrhea, chest pain, palpitations, shortness of breath,
8 urinary symptoms, or black stools. His past medical history is significant for diabetes mellitus,
9 diabetic nephropathy and retinopathy , hypertension, atrial fib rillation and chronic leg cellulitis. He

--
10

12
13
14
15
takes warfarin for chronic anticoagulation On physical examination, his blood pressure is 120/70
mmHg and his heart rate is 110 and irregular. His WBC count is 10,500/mm 3, hemoglobin level
is 7.0 g/dl and platelet count is 170,000/mm3. An abdominal CT image is shown on the slide
below.

16
17
18
19
20
21
22
23
24
25
26
27
28
29
30
31
32
33
34
35
36
37
38
39
40
2
3
4
5
6
7
8
9

--
10

12
13
14
15
16
17
18
19
20
21
22
23
24
25
26
27 W hich of the following is the most likely diagnosis?
28
29
30 <0 A Renal cell ca rc inoma
31 <0 B. Ve rtebral fracture
32
33 <0 C. Retroperitoneal hematoma
34 <0 D. Hydronephrosis
35
36
<0 E. Mesenteric ischemia
37
38 Submit
39
40
2
Item : 11 of40 ~'?Mark <] C> !I ~ ~ , ~
0. ld : 4697 Previous Next Lab Values Notes Calculator Reverse Color Text Zoom
3
4
5 Explanation:
6
7
8
9

--
10

12
13
14
15
16
17
18
19
20
21
22
23
24
25
26
27
28
29
30 This patient's history of anticoagulation, symptoms of weakness and dizziness, and evidence of
31 anemia and tachycardia should immediately raise concern for internal hemorrhage. The risk of
32
bleeding while on warfarin therapy is greatest in patients with risk factors such as diabetes, age >
33
34 60, hypertension and alcoholism. A supratherapeutic INR also increases one's risk of
35 hemorrhage, although in the case of retrope ritoneal hematomas, even therapeutic-range INRs
36 confer significant risk. This patient's back pain should put retroperitoneal hematoma high on the
37 diffe rential, and warrants an abdominal CT The above CT image reveals a large isodense
38 collection in the right retroperitoneum, which lies anterior to the psoas muscle and displaces the
39 right kidney anteriorly The CT findings are consistent with a spontaneous retroperitoneal
40
2
Item : 11 of40 ~'?Mark <] C> !I ~ ~ , ~
0. ld : 4697 Previous Next Lab Values Notes Calculator Reverse Color Text Zoom
3 eI • yp I " p p ., •
4
hemorrhage, although in the case of retrope ritoneal hematomas, even therapeutic-range INRs
5
6
confer significant risk. This patient's back pain should put retroperitoneal hematoma high on the
7 diffe rential, and warrants an abdominal CT The above CT image reveals a large isodense
8 collection in the right retroperitoneum, which lies anterior to the psoas muscle and displaces the
9 right kidney anteriorly The CT findings are consistent with a spontaneous retroperitoneal

--
10

12
13
14
15
hematoma (Choice C)

(Choice A) The classic triad of renal cell carcinoma (RCC) is hematuria, abdominal mass and
flank pain On abdominal CT, RCC presents as a lesion within the kidney parenchyma that
enhances with contrast administration. The above lesion is extrinsic to the kidney.
16
17 (Choice B) A vertebral fracture could certainly cause back pain, and may occur in osteoporotic
18 patients in the absence of trauma. However , the above CT image does not depict a vertebral
19 abnormality
20
21 (Choice D) Hydronephrosis is a condition in which the renal pelvis dilates due to distal
22 obstruction of the genitourinary system Patients present with flank pain CT scan in
23 hydronephrosis demonstrates dilation of the renal pelvis, and may help in identifying the source
24
of obstruction ( e g kidney stones)
25
26
27 {Choice E) Mesenteric ischemia presents with abdominal pain out of proportion to physical exam
28 findings. It is most common in elderly patients, and may result from a cardiac embolus (as in
29 atrial fibrillation), arterial thrombosis (as in atherosclerosis), or other etiology (as in shock or
30 vasoconstriction) CT findings include bowel wall thickening, pneumatosis intestinalis, and
31 mesenteric thrombi.
32
33 Educational objective:
34
Anticoagulation with warfarin places patients at risk fo r hemorrhage. Retrope ritoneal hematoma
35
may occur even without a supratherapeutic INR Back pain and signs and symptoms of
36
37 hemodynamic compromise should raise suspicion fo r retroperitoneal hematoma.
38
39 Time Spent 7 seconds Copyright © UWorld Last updated: [05/04/2016)
40
2
Item : 12 of40 ~'?Mark <] C> !I ~ ~ , ~
0. ld : 3 8 33 Previous Next Lab Values Notes Calculator Reverse Color Text Zoom
3
4
5
6 A 54-year-old man comes to the emergency department compla ining of several days of persistent
7 abdominal pain. He also complains of nausea, vomiting, low-grade fever, and appetite loss. He
8 does not drink alcohol. The patient is being treated fo r seizure disorder ; his last seizure was 3
9 months ago Chest and abdominal x-rays reveal no abnormalities. The patient's abdominal CT
10

-
scan is shown below.
11

13
14
15
16
17
18
19
20
21
22
23
24
25
26
27
28
29
30
31
32
33
34
35
36
37
38
39 W hich of the following is the most likely cause of this patient's current symptoms?
40
2
3
4
5
6
7
8
9
10

-
11

13
14
15
16
17
18
19
20
21
22
23
24
25
26
27 W hich of the following is the most likely cause of this patient's current symptoms?
28
29
30 <0 A Gallbladder disease
31 <0 B. Liver disease
32
33 <0 C. Mesenteric ischemia
34 <0 D. Pancreatic disease
35
36
<0 E. Peptic ulcer disease
37
38 Submit
39
40
2
Item : 12 of40 ~'?Mark <] C> !I ~ ~ , ~
0. ld : 3 8 33 Previous Next Lab Values Notes Calculator Reverse Color Text Zoom
3
4
5 Explanation:
6
7
8
9
10

-
11

13
14
15
16
17
18
19
20
21
22
23
24
25
26
27
28
29
30
31
32
33
34
35
36
37 This patient's presentation suggests acute pancreatitis Most cases of acute pancreatitis are
38 caused by alcohol ingestion or gallstones, but drug-induced pancreatitis accounts for 5% of
39 Th. I r t' r d. r · th t l'k 1 f h. d I · d d
40

~ Feedback SuWend EnQ ock


2
Item : 12 of40 ~'?Mark <] C> !I ~ ~ , ~
0. ld : 3 8 33 Previous Next Lab Values Notes Calculator Reverse Color Text Zoom
3
4 This patient's presentation suggests acute panc reatitis. Most cases of acute pancreatitis are
5 caused by alcohol ingestion or gallstones, but drug-induced pancreatitis accounts for 5% of
6 cases. This patient's anti-seizure medication is the most likely cause of his drug-induced
7 pancreatitis In particular , valproic acid is a commonly-used anti-seizure medication that is
8 associated with acute pancreatitis. Other common drugs associated with panc reatitis include
9
10 1. Diuretics (furosemide, thiazides)

-
11

13
14
15
16
2.
3.
4.
5.
D rugs for inflammatory bowel disease (sulfasalazine, 5-ASA)
Immunosuppressive agents (azathioprine)
HIV-related medications (didanosine, pentamidine)
Antibiotics (metronidazole, tetracycline)

17 Drug-induced pancreatitis is usually mild. Patients typically develop nausea, vom iting and
18 abdominal pain radiating to the back. Laboratory results usually show elevated serum amylase
19 and lipase. The CT scan reveals swelling of the pancreas with prominent peripancreatic fluid and
20 fat-stranding (red arrow) Supportive treatment with fluids and electrolyte replacement is
21
recommended.
22
23 (Choice A) Gallstones migrating into the bile duct or the ampulla of Vater can obstruct the
24
pancreatic duct and cause gallstone pancreatitis. However , this patient's abdominal CT does not
25
reveal an enlarged gallbladder shadow or gallbladder inflammation/stones.
26
27
(Choice B) Liver disease (eg, metastatic malignancy such as colon cancer , nodules, abscess,
28
29 hemangiomas) can cause abdominal discomfort, nausea, vomiting, and jaundice. However , this
30 patient's CT scan does not reveal any significant liver lesions.
31
32 (Choice C) Acute mesenteric ischemia typically presents with periumbilical abdominal pain
33 (often out of proportion to examination findings) that can be associated with nausea and
34 vom iting However , this patient's absence of risk factors (eg, advanced age, atrial fibrillation)
35 makes this less likely In addition, mesenteric ischemia does not usually cause the abnormal
36 pancreatic findings seen on this patient's abdominal CT scan.
37
38 (Choice E) Peptic ulcer disease can present with midepigastric pain and can sometimes be
39 relieved with food or antacids. However, it would not cause this patient's abnormal CT findings
40
2
Item : 12 of40 ~'?Mark <] C> !I ~ ~ , ~
0. ld : 3 8 33 Previous Next Lab Values Notes Calculator Reverse Color Text Zoom
3 ... . . . o1 . , .. . o1 I I - . .. .. • • o1 . • - I I , ,.. • I " --,,..- . -I o1 'o1-

4 and lipase. The CT scan reveals swelling of the panc reas with prominent peripancreatic fluid and
5 fat-stranding (red arrow) Supportive treatment with fluids and electrolyte replacement is
6
recommended.
7
8 (Choice A) Gallstones migrating into the bile duct or the ampulla of Vater can obstruct the
9
pancreatic duct and cause gallstone pancreatitis However, this patient's abdominal CT does not
10

-
11 reveal an enlarged gallbladder shadow or gallbladder inflammation/stones.

(Choice B) Liver disease (eg, metastatic malignancy such as colon cancer , nodules, abscess,
13
14 hemangiomas) can cause abdominal discomfort, nausea, vomiting, and jaundice. However , this
15 patient's CT scan does not reveal any significant liver lesions.
16
17 (Choice C) Acute mesenteric ischemia typically presents with periumbilical abdominal pain
18 (often out of proportion to examination findings) that can be associated with nausea and
19 vomiting However , this patient's absence of risk factors (eg, advanced age, atrial fibrillation)
20 makes this less likely In addition, mesenteric ischemia does not usually cause the abnormal
21 pancreatic findings seen on this patient's abdominal CT scan.
22
23 (Choice E) Peptic ulcer disease can present with midepigastric pain and can sometimes be
24 relieved with food or antacids. However, it would not cause this patient's abnormal CT findings
25
26 Educational objective:
27 Acute pancreatitis is most often due to alcohol use, gallstones, or medications. Commonly
28 implicated drugs include diuretics, anti-seizure drugs (eg, valproic acid) , and antibiotics (eg,
29
metronidazole) An abdominal CT scan can reveal diffuse or focal parenchymal changes,
30
31 edema, necrosis, or liquefaction Drug-induced pancreatitis is usually mild and resolves with
32 supportive care.
33
34 References:
35
36 1. Drug-induced pancreatitis: A potentially serious and underreported problem.
37
38
39 Time Spent 5 seconds Copyright © UWorld Last updated: [06/ 10/2016)
40
2
Item : 13 of40 ~'?Mark <J C> 61 ~ ~ , ~
0. ld: 4106 Previous Next Lab Values Notes Calculator Reverse Color Text Zoom
3
4
5
6 A 45-year-old man comes to the office with a 6-month history of recurrent, burning epigastric
7 pain and diarrhea. His stools are frothy and unusually foul smelling and they float The patient's
8 clothes fit loosely, and he believes that he may have lost some weight He has tried several
9 over-the-counter medications, including antacids, H-2 blockers, and proton pump inhibitors, with
10 moderate success. Vital signs are normal. Examination shows no abnormalities except for
11

-
midepigastric tenderness to deep palpation. Test of the stool for occult blood is positive, and stool
12
fat is also positive. Gastrointestinal endoscopy reveals two duodenal ulcers and a jejunal ulcer.
14
W hich of the following is the best explanation for this patient's impaired fat absorption?
15
16
<0 A. Auto immune mucosal injury
17
18 <0 B. Bacterial overgrowth
19
<0 C. Pancreatic enzyme deficiency
20
21 <0 D. Panc reatic enzyme inactivation
22
23 <0 E. Reduced bile acid absorption
24
25
26 Submit
27
28
29
30
31
32
33
34
35
36
37
38
39
40
2
Item : 13 of40
0. ld: 4106
~'?Mark <]
Previous
C>
Next
a
Lab Values
~
Notes
~
Calculator
,
Reverse Color
~
Text Zoom
3
4
5 Explanation:
6
7
8
9 Zollinger-EIIison syndrome
10
11

-
12

14
15
16
17
Epidemio logy

Clinical
• Age 20-50
• 80% Sporadic/20% MEN1

• Multiple & refractory peptic ulcers


• Ulcers distal to duodenum
features
18 • Chronic diarrhea
19
20
21 • Markedly elevated serum gastrin
22 Diagnosis (>1000 pg/ml) in the presence of
23 normal gastric acid (pH <4)
24
25
26 • Endoscopy
27 Workup • CT/MRI & somatostatin receptor
28 scintigraphy for tumor localization
29
30 MEN1 = multiple endocrine neoplasia type 1.
31 ~UWorld
32
33 This patient with multiple peptic ulcers and diarrhea has clinical features of Zollinger-EIIison
34 syndr ome (ZES). ZES is due to a gastrin-producing tumor, which usually occurs in the
35 pancreas or duodenum. Most cases are sporadic, but ZES also occurs frequently in the context
36
of multiple endocrine neoplasia type 1 (MEN1) Uncontrolled gastrin secretion leads to parietal
37
38 cell hyperplasia, with excessive production of gastric acid. Multiple duodenal (and sometimes
39 jejunal) ulcer s are typical, and the ulcers can be refractory to standard acid-reducing
40
2
Item : 13 of40 ~'?Mark <] C> !I ~ ~ , ~
0. ld: 4106 Previous Next Lab Values Notes Calculator Reverse Color Text Zoom
3 l' , , • 11 • 1 ,. • . • ttl t I t H I I .. J I I ...
4 pancreas or duodenum. Most cases are sporadic , but ZES also occurs frequently in the context
5
of multiple endocrine neoplasia type 1 (MEN1) Uncontrolled gastrin secretion leads to parietal
6
7 cell hyperplasia, with excessive production of gastric acid. Multiple duodenal (and sometimes
8 jejunal) ulcer s are typical, and the ulcers can be refractory to standard acid-reduc ing
9 medications. The excess gastric acid in the small intestine can cause diarrhea and steatorrhea
10 due to inactivation of pancreatic enzymes and injury to the mucosal brush border.
11

-
12

14
15
16
17
The diagnosis of ZES is suggested by a markedly elevated serum gastrin level (>1000 pg/ml) in
the presence of acidic gastric pH (<4). Endoscopy usually reveals ulcers (>90% of patients with
ZES) and occasionally will identify a primary duodenal gastrinoma. CT, MRI, and somatostatin
receptor scintigraphy can be used to identify pancreatic tumors and metastatic disease. If
gastrinoma is confirmed, patients should be screened fo r MEN1 with assays for parathyroid
hormone, ionized calcium, and prolactin.
18
19
(Choices A , B, and C) Celiac disease is an autoimmune disorder characterized by
20
malabsorption due to mucosal villous atrophy triggered by exposure to gluten-containing wheat
21
22 products Small intestinal bacterial overgrowth can also cause mucosal injury, possibly due to
23 bacterial toxins. Pancreatic exocrine deficiency can be seen in chronic (eg, alcoholic)
24 pancreatitis, cystic fibrosis, and pancreatic resection. However , duodenal and jejunal ulcerations
25 are not typical in these conditions.
26
27 (Choice E) Bile acids are primarily reabsorbed in the ileum; impaired absorption can cause
28 malabsorption of fats. This is typically seen following extensive ileal resection in Crohn disease
29 (CD), which presents with chronic or recurrent abdominal pain, fever , diarrhea, and weight loss.
30 It would be unusual for CD to present with multiple duodenal ulcers.
31
32 Educational objective:
33 Zollinger-EIIison syndrome should be suspected in patients with multiple duodenal ulce rs
34
refractory to treatment or ulcers distal to the duodenum or associated with chronic diarrhea. In
35
these patients, inactivation of pancreatic enzymes by inc reased production of stomach acid may
36
37 lead to malabsorption
38
39 Time Spent 3 seconds Copyright © UWorld Last updated [09/ 13/2016)
40
2
Item : 14 of 40 ~'?Mark <J C> 61 ~ ~ , ~
0. ld : 2899 Previous Next Lab Values Notes Calculator Reverse Color Text Zoom
3
4
5
6 A 55-year-old man comes to the office reporting "a swollen belly and puffy feet" that began
7 several months ago He has not seen a physic ian fo r several years but says that he was
8 previously diagnosed with depression. The patient takes no medications. He currently lives alone
9 and is unemployed He admits to drinking 6-12 beers a night, depending on what he can afford.
10 He has a remote history of cigarette smoking Phys ical examination reveals bilateral
11 gynecomastia, spider angiomas on the chest, Dupuytren contracture of both hands, and palmar
12

-
erythema D ilated periumbilical veins are also visible. Percussion of the abdomen demonstrates
13
shifting dullness. Splenomegaly is present There is no evidence of asterixis. There is pitting
15 edema of the lower extremities. Laboratory studies reveal increased prothrombin time and
16 thrombocytopenia. W hich of the following findings has the same pathogenesis as spider
17 angiomas?
18
19
20 <0 A Dilated abdominal veins
21 <0 B. Dupuytren contracture
22
23
<0 C. Palmar erythema
24 e; D. Pedal edema
25
<0 E. Splenomegaly
26
27
28
29 Submit
30
31
32
33
34
35
36
37
38
39
40
2
Item : 14 of 40
0. ld : 2899
~'?Mark <]
Previous
C>
Next
a
Lab Values
~
Notes
~
Calculator
,
Reverse Color
~
Text Zoom
3
4
5 Signs of liver cirrhosis
6
7
8
9
10
11
12

-
13

15
16
17
18
Portal hypertension

Esophageal varices
Hyperestrinism

Spider angiomata

19 Loss of sexual hair


20
21 Testicular atrophy
22
23 Anorectal - - - -- -
24 varices
25
26
27
28
29
30
Hepatic synthetic dysfunction
31
32
33
34
35
36
37
38
39
40
2
Item: 14 of 40 ~'?Mark <] C> !I ~ ~ , ~
0. ld : 2899 Previous Next Lab Values Notes Calculator Reverse Color Text Zoom
3 ---- * --- -- ---- -------· •••• - - - - --

4 type leads to progressive loss of liver function, which can be divided into 3 categories synthetic
5 (production of clotting factors, cholesterol, and proteins); metabolic (metabolism of drugs and
6 corticosteroids, including detoxification); and excretory (bile excretion)
7
8 In patients with cirrhosis, spider angioma and palmar erythema both arise from
9 hyperestrinism due to impaired hepatic metabolism of circulating estrogens, a process that
10
begins in the cytochrome P450 system Circulating estrogens affect vascular wall dilation.
11
12 Spider angioma consists of a central, dilated arteriole surrounded by smaller radiating vessels.

-
13 Palmar erythema is a result of increased normal speckling of the palm due to increased
vasodilation, especially at the thenar and hypothenar eminences. Other manifestations of
15 hyperestrinism in patients with cirrhosis include gynecomastia, testicular atrophy , and decreased
16 body hair in males.
17
18 {Choices A and E) Caput medusae arise from the dilation of superfic ial veins on the abdominal
19 wall due to portal hypertension (PH) PH results from inc reased resistance to portal flow at the
20 sinusoids and leads to increased pressure at the portosystemic collateral veins in the anterior
21 abdomen, rectum, and distal esophagus. This then predisposes to esophageal varices, rectal
22 va rices, and caput medusae. Similarly, PH causes enlargement of the spleen (which drains into
23
the portal ve in via the splenic ve in) due to vascular congestion of the red pulp.
24
25
(Choice B) Oupuytren contractur e occurs when the palmar fascia thickens and shortens,
26
deforming the hand. It is usually most notable initially in the fourth and fifth digits and occurs due
27
28 to fibroblast proliferation and collagen deposition, which are likely worsened by oxidative stress
29 from increased free radical production.
30
31 (Choice D) The live r is the primary site of protein synthesis, and the cirrhotic live r typically
32 produces insufficient amounts of proteins, including albumin. Hypoalbuminemia leads to both a
33 decrease in the intravascular oncotic pressure and fluid shifts to the extravascular space,
34 predisposing to edema of the lower extremities.
35
36 Educational objective:
37 Hyperestrinism in cirrhosis leads to gynecomastia, testicular atrophy , decreased body hair,
38 spider angiomas, and palmar erythema
39
40
2
Item : 15of40 ~'?Mark <J C> 61 ~ ~ , ~
0. ld : 21 66 Previous Next Lab Values Notes Calculator Reverse Color Text Zoom
3
4
5
6 A 52-year-old woman comes to the physician with recurrent episodes of vom iting and abdominal
7 discomfort over the last several weeks. Her appetite is poor and she has lost 6.8 kg (15 lb) over
8 the last year She has no back pain or blood in the stooL Her blood pressure is 110/70 mm Hg
9 and pulse is 104/min and regular The physician places the stethoscope over the upper abdomen
10 and rocks the patient back and forth at the hips This maneuver is most helpful in diagnosing
11 which of the following?
12
13

--
14

16
17
18
19
e> A. Chronic pancreatitis
(:) B. Esophageal cance r
e> C. Gastric outlet obstruction
0 D. Mesenteric ischemia
20 0 E. Ulcerative colitis
21
22
23
Submit
24
25
26
27
28
29
30
31
32
33
34
35
36
37
38
39
40
2
Item : 15of40 ~'?Mark <] C> !I ~ ~ , ~
0. ld : 21 66 Previous Next Lab Values Notes Calculator Reverse Color Text Zoom
3
4 D. Mesenteric ischemia [5%)
5
E. Ulcerative colitis [0%)
6
7
8 Explanation:
9
10 This physical examination maneuver is known as the abdominal succussion splash, which is
11
elicited by placing the stethoscope over the upper abdomen and rocking the patient back and
12
forth at the hips Retained gastric material >3 hours after a meal will generate a splash sound,
13

--
14 indicating the presence of a hollow viscus filled with fluid and gas Although the test has modest
sensitivity and specificity for diagnosing gastric outlet obstruction (GOO) , it may suggest the need
16 for more definitive evaluations. Initial management of GOO includes nasogastric suctioning to
17 decompress the stomach, intravenous hydration, and endoscopy for definitive diagnosis
18
19 (Choice A) Chronic pancreatitis may lead to inflammation and fibrosis in adjacent structures
20 (duodenum) that could r ar ely lead to obstruction. Succussion splash can be present in these
21 patients but would be more specific for GOO than underlying pancreatitis.
22
23 (Choice B) Esophageal cancer may present with dysphagia, weight loss, and vom iting
24 However, an esophageal obstruction would not usually produce a gastric succussion splash
25
26 (Choice D) Chronic mesenteric ischemia can cause intermittent abdominal pain and is usually
27 triggered by eating ("intestinal angina") Patients may also have significant weight loss.
28 However, GOO is uncommon as the stomach and pylorus are not usually involved.
29
30 {Choice E) Although Crohn's disease may occasionally involve the stomach or duodenum to
31 cause GOO, ulcerative colitis spares the upper gastrointestinal tract. Examination findings in
32
ulce rative col itis are often normal or nonspecific , but patients with moderate to severe flares may
33
34 have gross blood on rectal examination.
35
Educational objective:
36
37 The presence of "succussion splash" can indicate gastric outlet obstruction.
38
39 Time Spent 3 seconds Copyright © UWorld Last updated: [08/ 18/2016)
40
2
Item: 16 of40 ~'?Mark <J C> 61 ~ ~ , ~
0. ld : 4360 Previous Next Lab Values Notes Calculator Reverse Color Text Zoom
3
4
5
6 A 32-year-old man comes to the emergency department with intense midline chest pain and
7 diaphoresis of 4 hours duration. Prior to the onset of pain, he developed nausea and recurrent
8 vom iting after returning from a party His medical problems include HIV infection, medication
9 noncompliance, alcohol abuse, and alcoholic hepatitis. The patient admits to using cocaine
10 regularly His tempe rature is 37.8 C (100 F), blood pressure is 100/60 mm Hg, pulse is 120/min,
11 and respirations are 28/min. Examination shows injected conjunctivae and bilateral dilated
12
pupils. Chest x-ray demonstrates a widened mediastinum and moderate left-sided pleural
13
14 effusion. ECG shows sinus tachycardia Pleural fl uid is found to be a yellow exudate with high

-
15 amylase content W hich of the following is the most likely diagnosis in this patient?

17 e; A Acute pancreatitis
18
19 e> B. Ao rtic dissection
20 e; C. Asp iration pneumonia
21
22 e> D. Esophageal perforation
23
24 e; E. Myocardial infarction
25
26
Submit
27
28
29
30
31
32
33
34
35
36
37
38
39
40
2
Item: 16 of40
0. ld : 4360
~'?Mark <]
Previous
C>
Next
a
Lab Values
~
Notes
~
Calculator
,
Reverse Color
~
Text Zoom
3
4
5 Characteristics of gastroesophageal mural injury
6
7
8 Mallory-Weiss tear Boerhaave syndrome
9
10 • Upper gastrointestinal
11 • Esophageal transmural tear
mucosal tear
12 Etiology • Caused by forceful retch ing
13 • Caused by forceful retching (I pressure)
14 ( I pressure)

-
15

17
18
19
20
• Submucosal arterial or
venule plexus bleeding
• Esophageal air/fluid leakage
into nearby areas (eg, pleura)

• Vomiting, retching, chest & upper


abdominal pain
21 • Vomiting, retching
22 Clinical • Hematemesis • Odynophagia, fever, dyspnea
23 presentation • Epigastric pain & septic shock can ensue
24
• Subcutaneous emphysema may
25
26 be seen
27
28 • CT or contrast
29 esophagography with
30
Gastrografin confirms diagnosis
31
32 Laboratory/ • Chest x-ray:
33
imaging • EGO confirms diagnosis Pneumomediastinum & pleural
34 effusions
35
36 • Pleural fluid analysis: Exudative,
37 low pH, very high amylase
38 (>2,500 IU/L)
39
40
2
Item: 16 of40
0. ld : 4360
~'?Mark <]
Previous
C>
Next
a
Lab Values
~
Notes
~
Calculator
,
Reverse Color
~
Text Zoom
3 • " •"" "- ' ! #- . 4-
4 (>2,500 IU/L)
5
6
7 • Most tears heal • Surgery: For thoracic perforations
8 spontaneously
9 Treatment • Conservative measures
10 • Endoscopic therapy for (eg, antibiotics): For cervical
11 continual bleed perforations
12
13 EGO = esophagogastroduodenoscopy.
14

-
Cii>UWorfd
15
This patient's presentation is consistent with spontaneous esophageal rupture (Boerhaave
17 syndrome). Patients usually present with vomiting/retching and chest/upper abdominal pain with
18 rapid progression to odynophagia, dyspnea, and septic shock. Examination can be nonspecific
19 but often shows fever, tachyca rdia , tachypnea, cyanosis, and subcutaneous emphysema.
20
Chest x-ray may reveal pneumomediastinum or unilateral pleural effusion (usually left) with or
21
22 without pneumothorax. Mediastinal widening can be seen as air and fluid accumulate in the
23 mediastinum, causing inflammation (mediastinitis)
24
25 The diagnosis can be confirmed by CT scan or contrast esophagography with Gastrografin
26 (showing contrast extravasation from the esophagus into surrounding areas). Pleural fluid
27 analysis is typically exudative with low pH and very high amylase (>2500 lUlL due to saliva in the
28 esophageal contents) and may contain food particles.
29
30 {Choice A) Acute pancreatitis can cause a unilateral, left-sided pleural effusion with a high
31 amylase concentration. However , pancreatitis would typically not cause mediastinal widening
32 Pancreatitis classically presents as epigastric pain radiating to the back rather than chest pain
33
34 (Choice B) This patient's cocaine use can predispose him to an aortic dissection, which
35 classically presents as severe tearing chest pain radiating to the back. Chest x-ray can also
36 reveal a widened mediastinum and unilateral pleural effusion (hemothorax) However, the high
37
amylase content in the pleural fluid is more suggestive of esophageal perforation.
38
39 I I
40

~ Feedback SuWend EnQ ock


2
Item: 16 of40 ~'?Mark <] C> !I ~ ~ , ~
0. ld : 4360 Previous Next Lab Values Notes Calculator Reverse Color Text Zoom
3
4
(Choice A) Acute pancreatitis can cause a unilateral, left-sided pleural effusion with a high
5 amylase concentration. However, pancreatitis would typically not cause mediastinal widening
6 Pancreatitis classically presents as epigastric pain radiating to the back rather than chest pain
7
8 {Choice B) This patient's cocaine use can predispose him to an aortic dissection, which
9 classically presents as severe tearing chest pain radiating to the back. Chest x-ray can also
10 reveal a widened mediastinum and unilateral pleural effusion (hemothorax). However, the high
11 amylase content in the pleural fluid is more suggestive of esophageal perforation
12
13 (Choice C) Aspiration usually occurs in patients with altered mental status who are unable to
14 protect their airways Aspiration pneumonia is seen more commonly in the right lower lobe and

-
15

17
18
19
20
can present with a unilateral pleural effusion due to parapneumonic effusion or empyema.
Pleural fluid analysis would be expected to show many white blood cells, increased protein, and
elevated lactic acid dehydrogenase, but not an elevated amylase level. Also, this patient's x-ray
shows no lung infiltrates.

{Choice E) Myocardial infa rction (MI) could cause this patient's chest pain Cocaine can cause
21
22 co ronary vasospasm and Ml in otherwise healthy people However, Ml would not account for his
23 unilateral pleural effusion or widened mediastinum on chest x-ray
24
25 Educational objective:
26 Spontaneous rupture of the esophagus (Boerhaave syndrome) typically occurs after severe
27 retching and vomiting Chest x-ray can reveal unilateral pleural effusion with or without
28 pneumothorax, subcutaneous or mediastinal emphysema, and widened mediastinum. Pleural
29 fl uid analysis is typically exudative with low pH and very high amylase (>2500 IU/ L) and may
30 contain food particles CT or contrast esophagography with Gastrografin confirms the diagnosis
31
32
References:
33
34 1. Boerhaave syndrome: a diagnostic conundrum.
35
36 2. Evolving options in the management of esophageal perforation.
37
38
39 Time Spent 3 seconds Copyright© UWorld Last updated: [09/21 /2016)
40
2
Item : 17 of 40 ~'?Mark <] C> !I ~ ~ , ~
0. ld : 2937 Previous Next Lab Values Notes Calculator Reverse Color Text Zoom
3
4
5
6 A 45-year-old male presents for a routine checkup because he wishes to purchase life
7 insurance. He says he feels "perfectly healthy" and has no complaints His medical history is
8 significant for a remote bout of cellulitis and a fractured clavicle from playing football in high
9 school. He works as a supervisor at a corporate call center. He is monogamous and lives with
10 his wife and two children. Laboratory testing reveals the following:
11
12 Liver studies
13
14 Total bilirubin 0.9 mg/dL
15

-
16 Direct bilirubin 0.2 mg/dL

Aspartate aminotransferase (SGOT) 244 U/L


18
19 Alanine aminotransferase (SGPT) 155 U/L
20
21
22 Complete blood count
23
Hemoglobin 13 8 g/ L
24
25 Erythrocyte count 4.6 mln/mm3
26
27 MCV 95 fl
28
29 Platelets 290,000/mm3
30
31 Leukocyte count 5,500/mm3
32
33 Chemistry panel
34
35 Serum sodium 142 mEq/ L
36
37 Serum potassium 3.9 mEq/ L
38
Chloride 105 mEq/ L
39
40
2
Item : 17 of 40 ~'?Mark <J C> 61 ~ ~ , ~
0. ld : 2937 Previous Next Lab Values Notes Calculator Reverse Color Text Zoom
3
4 MCV 95 fl
5
6 Platelets 290,000/mm3
7
8 Leukocyte count 5,500/mm3
9
10 Chemistry panel
11
12 Serum sodium 142 mEq/L
13
14 Serum potassium 3.9 mEq/L
15

-
16 Chloride 105 mEq/L

Bicarbonate 26 mEq/L
18
19 Blood urea nitrogen ( BUN) 21 mg/dl
20
21 Serum creatinine 0.9 mg/dl
22
23 Calcium 8.3 mg/dl
24
Blood glucose 92 mg/dl
25
26
27 W hat is the next best step in the evaluation of this patient?
28
29
30 <0 A Order viral serology testing
31 <0 B. Obtain more history about alcohol or drug intake
32
33 <0 C . Obtain abdominal ultrasound
34 <0 D. Order urine copper, serum coppe r, and ceruloplasmin levels
35
<0 E. Order serum iron and fe rritin levels
36
37
38 Submit
39
40
2
Item : 17 of 40 ~'?Mark <] C> !I ~ ~ , ~
0. ld : 2937 Previous Next Lab Values Notes Calculator Reverse Color Text Zoom
3
4
5 Explanation:
6
7 Aspartate aminotransferase (AST) and alanine aminotransferase (A LT) are distributed widely
8 throughout the body, with AST present and very active in the liver , heart, kidney, and skeletal
9 muscle. W hile ALT is also present in reduced quantities in the kidney, heart, and skeletal
10 muscle, it is predominantly found in the liver and is therefore more specific for hepatocyte injury
11
12 The patient in this scenario has a mild (< 250 U/ L), asymptomatic elevation of his serum
13 transaminases. The fi rst step in the evaluation of his condition should be a careful screening for
14
all hepatitis risk factors, including drug and alcohol intake, travel outside the United States, blood
15

-
transfusions, or high-risk sexual practices. This aspect of the patient's history will provide insight
16
as to whether the transaminase elevation could be caused by alcohol, medications (eg, NSAIDs,
18 antibiotics, HMG-CoA reductase inhibitors, antiepileptic drugs, antituberculous drugs, herbal
19 preparations), or viral agents
20
21 Afte r thoroughly questioning the patient about his history and having him discontinue all alcohol
22 and drug use, the next step in the evaluation process would be to repeat the liver function tests. If
23 the transaminases persist in being elevated over a six-month period, they are categorized as
24 chronic. Testing for viral hepatitis Band C, hemochromatosis, and fatty liver should then be
25 undertaken to further evaluate chronically elevated transaminases. If these tests prove
26 unremarkable, a search for muscle disorders (eg, polymyositis) and thy roid disease should be
27
made.
28
29 {Choice A) This patient has no obvious risk factors for viral hepatitis. Therefore, his drug and
30
alcohol usage must be examined fi rst
31
32
{Choice C) An ultrasound of the abdomen would not be particularly enlightening in a patient with
33
34 asymptomatic aminotransferase elevation unless gallbladder pathology was suspected
35
{Choice D) These tests are employed to rule out W ilson's disease, which is not a likely diagnosis
36
37 in an asymptomatic middle-aged man with an unremarkable history and physical exam. However,
38 these tests could be ordered after all common causes of hepatitis were ruled out
39
40
... . ... .. .. .. ...... ... . .. . ....... ... . . . ........ .
~ Feedback SuWend EnQ ock
2
Item : 17 of 40 ~'?Mark <] C> !I ~ ~ , ~
0. ld : 2937 Previous Next Lab Values Notes Calculator Reverse Color Text Zoom
3
4 all hepatitis risk factors, incl uding drug and alcohol intake, travel outside the United States, blood
5 transfusions, or high-risk sexual practices This aspect of the patient's history will provide insight
6 as to whether the transaminase elevation could be caused by alcohol, medications (eg, NSAIDs,
7 antibiotics, HMG-CoA reductase inhibitors, antiepileptic drugs, antituberculous drugs, herbal
8 preparations), or viral agents.
9
10 After thoroughly questioning the patient about his history and having him discontinue all alcohol
11 and drug use, the next step in the evaluation process would be to repeat the liver function tests. If
12
the transaminases persist in being elevated over a six-month period, they are categorized as
13
14 chronic. Testing for viral hepatitis B and C, hemochromatosis, and fatty liver should then be
15 undertaken to further evaluate chronically elevated transaminases. If these tests prove

-
16

18
19
20
21
unremarkable, a search for muscle disorders (eg, polymyositis) and thyroid disease should be
made.

(Choice A) This patient has no obvious risk factors fo r viral hepatitis. Therefore, his drug and
alcohol usage must be examined first

22 {Choice C) An ultrasound of the abdomen would not be particularly enlightening in a patient with
23 asymptomatic aminotransferase elevation unless gallbladder pathology was suspected
24
25 (Choice D) These tests are employed to rule out Wilson's disease, which is not a likely diagnosis
26 in an asymptomatic middle-aged man with an unremarkable history and physical exam. However,
27 these tests could be ordered after all common causes of hepatitis were ruled out
28
29 (Choice E) These tests are employed to rule out hemochromatosis, which is a possible diagnosis
30 in an asymptomatic middle-aged man with a normal physical exam. However, the more common
31 causes of hepatitis should be ruled out before testing for the more unusual causes.
32
33 Educational objective:
34
In evaluating the asymptomatic elevation of aminotransferases, the first step is to take a thorough
35
history to rule out the more common hepatitis risk factors (eg, alcohol or drug use, travel outside
36
37 of the country, blood transfusions, high-risk sexual practices)
38
39 Time Spent 4 seconds Copyright© UWorld Last updated: [08/ 10/2016)
40
2
Item : 18 of40 ~'?Mark <J C> 61 ~ ~ , ~
0. ld : 2601 Previous Next Lab Values Notes Calculator Reverse Color Text Zoom
3
4
5
6 A 57-year-old man comes to the office due to chest pain. The pain is burning in nature and is
7 not related to meals. The patient has lost 18 kg ( 40 lb) over the last 6 months, although he
8 attributes this to eating a selective diet and ca refully chewing his food before swallowing. In
9 addition, the patient has had fatigue and malaise as well as decreased interest in his usual
10 activities. His medical history is notable for long-standing gastroesophageal reflux for which he
11 takes ranitidine. He has a 20-pack-year smoking history but quit 4 years ago. The patient
12
occasionally drinks alcohol. He works as a missionary and makes frequent trips abroad. His
13
14 father died at age 67 from lung cance r. The patient's vital signs are normal. Physical
15 examination (including cervical, cardiopulmonary, and abdominal examinations) is unremarkable.
16 Chest x-ray reveals no abnormalities. W hich of the following is the best next step in management

--
17

19
20
21
22
of this patient?

ID A Bronchoscopy
6 B. CT scan of the chest with and without contrast
23 ID C. Evaluation by a nutritionist
24 6 D. Lansoprazole and follow-up in 2 months
25
26 ® E. Testing for Helicobacter pylori infection
27 e; F. Upper gastro intestinal endoscopy
28
29
30 Submit
31
32
33
34
35
36
37
38
39
40
2
Item : 18 of40
0. ld : 2601
~'?Mark <]
Previous
C>
Next
a
· · Values
~
Notes
~
Calculator
,
Reverse Color
~
Text Zoom
3
4
5
6
7 Esophageal cancer
8
9
10 • Adenocarcinoma
11 o Distal esophagus, arises from Barrett esophagus
12 Subtypes
13 • Squamous cell carcinoma
14 o Anywhere in the esophagus
15
16

--
Risk • Acid reflux, obesity (adenocarcinoma)
17
factors • Smoking, alcohol, caustic injury (squamous cell)
19
20
21 • Chest pain
22 Symptoms • Weight loss
23 • Dysphagia (solids)
24
25
26 Diagnosis • Endoscopy with biopsy
27 • CT (PET/CT) for staging
28
29 PET = positron emission tomography.
30 @UWorld
31
32 Esophageal cancer should be strongly suspected in this patient given his age, persistent burning
33 chest pain, probable dysphagia (carefully chewing his food before swallowing), and significant
34 weight loss. Most esophageal malignancies are either adenocarcinoma or squamous cell
35 carcinoma. Adenocarcinoma usually arises within an area of Barrett esophagus near the
36
gastroesophageal junction; risk is increased with smoking and gastroesophageal refl ux disease
37
(GERD). Squamous cell carcinoma can occur anywhere in the esophagus; risk is increased
38
39 with smoking and heavy alcohol consumption
40
2
Item : 18 of40 ~'?Mark <J C> 61 ~ ~ , ~
0. ld : 260 1 Previous Next Lab Values Notes Calculator Reverse Color Text Zoom
3
© UWorld
4
5 Esophageal cancer should be strongly suspected in this patient given his age, persistent burning
6
chest pain, probable dysphagia (carefully chewing his food before swallowing), and significant
7
weight loss. Most esophageal malignancies are either adenocarcinoma or squamous cell
8
9 carcinoma. Adenocarcinoma usually arises within an area of Barrett esophagus near the
10 gastroesophageal junction; risk is increased with smoking and gastroesophageal reflux disease
11 (GERD). Squamous cell carcinoma can occur anywhere in the esophagus; risk is increased
12 with smoking and heavy alcohol consumption
13
14 Definitive diagnosis of esophageal cance r requires esophageal endoscopy with biopsy
15 Young, low-risk patients with undetermined esophageal symptoms may start with barium
16 esophagram, but those who are age >55 as well as those with alarm symptoms (eg, weight loss,

--
17

19
20
21
22
gross or occult bleeding, early satiety) will usually proceed directly to endoscopy Subsequent
staging procedures may include CT scan and positron emission tomography (Choice 8), and
surgery fo r definitive cure is advised fo r patients with limited-stage disease.

(Choice A) Bronchoscopy is indicated for evaluation of accessible pulmonary masses or for


patients with additional symptoms (eg, hemoptysis) of an endobronchial lesion. This patient has a
23
24 normal chest x-ray and possible dysphagia, making esophageal cancer more likely than lung
25 malignancy
26
27 {Choice C) Nutritional review is advised fo r patients with weight loss due to behavioral disorders,
28 swallowing dysfunction, or food intolerance.
29
30 {Choice D) Proton pump inhibitors (eg, omeprazole, lansoprazole) are useful for patients with
31 typical symptoms of GERD who fa il other medications. However , patients with alarm symptoms or
32 other features of malignancy should have a more definitive evaluation first
33
34 {Choice E) Helicobacter pylori infection is usually not associated with esophageal disorders.
35 Testing would be considered in some patients with unexplained dyspepsia (rather than dysphagia)
36 and no alarm symptoms for stomach malignancy
37
38 Educational objective:
39 Diagnosis of esophageal cancer requires esophageal endoscopy with biopsy Young, low-risk
40
Item : 18 of40 ~'?Mark <] C> !I ~ ~ , ~
2
3
4
0. ld :
.- .- . . . . . . .. . .-..
2601
- ... - - .. - - .. . .-... .. ..
Previous Next
'

Young, low-risk patients with undetermined esophageal symptoms may start with barium
., .... Lab Values Notes Calculator Reverse Color Text Zoom

5 esophagram, but those who are age >55 as well as those with alarm symptoms (eg, weight loss,
6
gross or occult bleeding, early satiety) will usually proceed directly to endoscopy. Subsequent
7
staging procedures may include CT scan and positron emission tomography (Choice B), and
8
9 surgery for definitive cure is advised fo r patients with limited-stage disease.
10
11 (Choice A) Bronchoscopy is indicated for evaluation of accessible pulmonary masses or for
12 patients with additional symptoms (eg, hemoptysis) of an endobronchial lesion. This patient has a
13 normal chest x-ray and possible dysphagia, making esophageal cancer more likely than lung
14 malignancy
15
16 (Choice C) Nutritional review is advised fo r patients with weight loss due to behavioral disorders,

--
17

19
20
21
22
swallowing dysfunction, or food intolerance.

(Choice D) Proton pump inhibitors (eg, omeprazole, lansoprazole) are useful for patients with
typical symptoms of GERD who fail other medications. However, patients with alarm symptoms or
other features of malignancy should have a more definitive evaluation first.

23 (Choice E) Helicobacter pylori infection is usually not associated with esophageal disorders.
24 Testing would be considered in some patients with unexplained dyspepsia (rather than dysphagia)
25 and no alarm symptoms fo r stomach malignancy
26
27 Educational objective:
28 Diagnosis of esophageal cancer requires esophageal endoscopy with biopsy Young, low-risk
29
patients with undetermined esophageal symptoms may start with barium esophagram, but those
30
31 who are age >55 or with alarm symptoms (eg, weight loss, gross or occult bleeding, early satiety)
32 should proceed directly to endoscopy.
33
34 References:
35
36 1. Endoscopic assessment and management of early esophageal adenocarcinoma.
37
38
39 Time Spent 2 seconds Copyright© UWorld Last updated: [09/ 15/2016)
40
2
Item : 19 of40 ~'?Mark <J C> 61 ~ ~ , ~
0. ld : 3790 Previous Next Lab Values Notes Calculator Reverse Color Text Zoom
3
4
5
6 A 42-year-old man with a history of Crohn disease comes to the office for follow-up He had a
7 partial ileal resection due to a stricture and also had multiple surgeries to treat an
8 enterocutaneous fistula. The patient has received parenteral nutrition for the past several weeks
9 and recently restarted oral feed ing He reports nonbloody diarrhea but no fever or abdominal
10 pain. The patient says that food does not taste the same as before. On examination, he has
11 patchy alopecia and a vesicular , crusting skin rash with scaling and erythema around the mouth
12
and on the extremities. Abdom inal examination shows that the fistula has healed; no abdominal
13
14 tenderness is present and bowel sounds are normal. The jugular venous pulse is normal and
15 there is no lower extremity edema. W hich of the following is most likely to improve this patient's
16 current condition?

...
17
18

20
21
(:) A Elemental copper
e; B. Gluten avoidance
e> C. Niacin therapy
22
23 e; D. Selenium supplement
24
25 (:) E. Zinc supplement
26
27
28 Submit
29
30
31
32
33
34
35
36
37
38
39
40
2
Item : 19 of40
0. ld : 3790
~'?Mark <]
Previous
C>
Next
a
Lab Values
~
Notes
~
Calculator
,
Reverse Color
~
Text Zoom
3
4
5 Clinical manifestations of trace mineral deficiencies
6
7 Chrom ium • Impaired glucose control in diabetics
8
9
10 • Brittle hair
11 • Skin depigmentation
12 Copper • Neurologic dysfunction (eg, ataxia, peripheral neuropathy)
13 • Sideroblastic anemia
14 • Osteoporosis
15
16

...
Iron • Microcytic anemia
17
18
• Thyroid dysfunction
20 Selenium • Cardiomyopathy
21 • Immune dysfunction
22
23 • Alopecia
24 • Pustular skin rash (perioral region & extremities)
25
26 • Hypogonadism
Zinc
27 • Impaired wound healing
28 • Impaired taste
29 • Immune dysfunction
30
(\)UWortd
31
32 This patient most likely has zinc deficiency. Zinc is a trace mineral that is obtained from meat,
33
34
nuts, and fo rtified cereal in the diet. It is an essential component of numerous enzymes in the
35 body and plays an important role in gene transcription and cell division. Clinical manifestations of
36 zinc deficiency in adults include hypogonadism, impaired wound healing, impaired taste , and
37 immune dysfunction Characteristic physical examination findings include alopecia as well as a
38 skin rash consisting of erythematous pustules around body orifices (eg, mouth) and on the
39 extremities.
40

~ Feedback
-------------------------------------------------------------------------------------------------------------------------------
SuWend EnQ ock
2
Item : 19 of40 ~'?Mark <] C> !I ~ ~ , ~
0. ld : 3790 Previous Next Lab Values Notes Calculator Reverse Color Text Zoom
3 I I -.~ ,. • • ... 11 I 1 .. '"' t•t"' I '"' I • · · · • I • - • . ..... • I
4 trace mineral deficiency (eg, zinc , selenium, copper , chrom ium) due to inadequate
5
supplementation of parenteral intake. Comorbid diar rhea and malabsorption further increase the
6
risk of trace mineral deficiency in patients on PN due to excessive gastrointestinal losses.
7
8
(Choice A) Clinical manifestations of copper deficiency include fragile hair, skin depigmentation,
9
10 neurologic dysfunction that can mimic vitamin 8 12 deficiency, and sideroblastic anemia. Impaired
11 taste and a pustular skin rash would be unusuaL
12
13 (Choice B) Gluten avoidance is used to treat celiac disease, which can lead to malabsorption
14 and deficiency of zinc and other vitamins and minerals. Celiac disease is associated with
15 dermatitis herpetiformis, an autoimmune-mediated vesicular skin rash that typically appears on
16 the extremities. Perioral involvement would be unusuaL In addition, celiac disease is less likely in

...
17
18

20
21
this patient with alternate explanations for zinc deficiency (eg, Crohn disease, recent PN)

(Choice C) Niacin (vitamin 8,) deficiency (rare in developed countries) causes pellagra, which
is characterized by dermatitis, diarrhea, and dementia. The rash associated with pellagra is
hyperpigmented, occurs symmetrically in sun-exposed areas, and is not pustular.
22
23
(Choice D) Selenium deficiency can lead to thyroid dysfunction as well as ca rdiomyopathy The
24
absence of heart fa ilure signs (eg, jugular venous distension, lower extremity edema) in this
25
26 patient makes selenium deficiency less likely
27
28
Educational objective:
29 Risk factors for trace mineral deficiency include malabsorption, bowel resection, poor nutritional
30 intake, and dependence on parenteral nutrition. Clinical manifestations of zinc deficiency include
31 hypogonadism, impaired taste, impaired wound healing, alopecia, and skin rash with perioral
32 involvement
33
34
Refer ences:
35
36 1. Zinc: physiology, deficiency, and parenteral nutrition.
37
38
39 I I I I
40

~ Feedback SuWend EnQ ock


2
Item: 20of40 ~'?Mark <] C> !I ~ ~ , ~
0. ld : 3732 Previous Next Lab Values Notes Calculator Reverse Color Text Zoom
3
4
5
6 A 59-year-old woman comes to the office for a routine visit She reports no symptoms She has
7 no medical problems and takes no medications. She grew up in the United States and has not
8 traveled outside the country The patient does not use tobacco, alcohol, or illicit drugs. Her blood
9 pressure is 120/70 mm Hg, pulse is 88/min, and respirations are 14/min. Physical examination
10 shows a firm , nontender mass in the right upper quadrant The remainder of the examination is
11 within normal limits. Laboratory testing is unremarkable. Abdominal CT scan is shown in the
12
image below.
13
14
15
16
17
18

-
19

21
22
23
24
25
26
27
28
29
30
31
32
33
34
35
36
37
38
39
40
2
3
4
5
6
7
8
9
10
11
12
13
14
15
16
17
18

-
19

21
22
23
24
25
26 This patient is at greatest risk for which of the following conditions?
27
28 ® A Anaphylactic shock
29
30 ® B. Gallbladder adenocarcinoma
31 ® C. Hepatic abscess
32
33 ® D. Intestinal perforation
34 ® E. Liver cirrhosis
35
36
® F. Renal cell carcinoma
37
38 Submit
39
40
2
Item : 20of40 ~'?Mark <] C> !I ~ ~ , ~
0. ld : 3732 Previous Next Lab Values Notes Calculator Reverse Color Text Zoom
3
4
5 Explanation:
6
7
8
9
10
11
12
13
14
15
16
17
18

-
19

21
22
23
24
25
26
27
28
29
30
31
32
33
34
35
36 This patient's abdominal CT scan shows a por celain gallbladder, a term used to describe the
37
calcium-laden gallbladder wall with bluish color and brittle consistency often associated with
38
39
chronic cholecystitis The pathogenesis of the condition remains unclea r, but it is thought that
~ II ~ * '"' ........ I ~ II ~ • '"' • I'"' I ~ ~. e1'"' • •• I e 1 1i~1111~ •••
40

~ Feedback SuWend EnQ ock


2
Item : 20of40 ~'?Mark <] C> !I ~ ~ , ~
3
4
0. ld : 3732
.. ....... . .. - . .. . - ~
Previous
. ..
..
Next
... . . . .. . Lab Values Notes Calculator Reverse Color Text Zoom

calcium-laden gallbladder wall with bluish color and brittle consistency often associated with
5 chronic cholecystitis The pathogenesis of the condition remains unclear, but it is thought that
6 calcium salts are deposited intramurally due to the natural progression of chronic inflammation or
7
chronic irritation from gallstones. Patients can be asymptomatic , have right upper quadrant pain,
8
or have a fi rm and nontender right upper quadrant mass on examination.
9
10
Plain x-rays can show a rimlike calcification in the area of the gallbladder, and CT scan typically
11
12 reveals a calcified rim in the gallbladder wall with a central bile-filled dark area. Porcelain
13 gallbladder has been associated with increased risk for gallbladder adenocarcinoma (2%-5% in
14 some studies). Cholecystectomy is typically considered fo r patients with porcelain gallbladder,
15 particularly if they are symptomatic or have incomplete mural calcification.
16
17 {Choice A) Ruptured echinococcal cyst can cause anaphylaxis with eventual shock. However,
18 an echinococcal cyst usually appears on abdominal CT as a cystic live r lesion (sometimes with

-
19

21
22
23
24
calcifications) without gallbladder involvement It would be unlikely in a patient who has never
traveled outside the United States.

(Choice C) Liver abscess is typically amebic or bacterial in origin (not a complication of


porcelain gallbladder) and is caused by portal vein bacteremia, systemic bacteremia, ascending
cholangitis, or trauma. Ultrasound or CT scan is usually diagnostic
25
26 (Choices 0 and E) Porcelain gallbladder is typically not directly associated with intestinal
27
perforation or cirrhosis of the liver. Gallstone ileus can cause a fistula between the gallbladder
28
and intestine, which allows gallstones to enter and obstruct the intestine.
29
30
(Choice F) Risk factors associated with renal cell ca rcinoma (RCC) include smoking,
31
32 hype rtension, and obesity However, porcelain gallbladder is not usually associated with
33 increased risk for RCC.
34
35 Educational objective:
36 Porcelain gallbladder is usually diagnosed on abdominal imaging showing a calcified rim in the
37 gallbladder wall with a central bile- filled dark area. It is associated with an increased risk for
38 gallbladder adenocarcinoma and usually requires cholecystectomy
39
40
2
Item : 2 1 of40 ~'?Mark <] C> !I ~ ~ , ~
0. ld : 3603 Previous Next Lab Values Notes Calculator Reverse Color Text Zoom
3
4
5
6 A 41-year-old woman comes to the physician with 3 months of daily abdominal bloating and
7 increased flatulence. She also has watery, nonbloody diarrhea several times a week, most often
8 at night She has no relief of her symptoms after a bowel movement The patient has no fever,
9 abdominal pain, weight loss, or recent travel. Her medical history includes systemic sclerosis,
10 gastroesophageal reflux disease, and type 2 diabetes mellitus. Ten years ago she underwent
11 open cholecystectomy fo r symptomatic gallstones. Her temperature is 37 C (98 7 F), blood
12
pressure is 144/67 mm Hg, and pulse is 78/min. The patient's body mass index is 32 kg/m2
13
14 Extremities examination shows skin hardening affecting the fingers on both hands and scattered
15 telangiectasias The abdomen is soft and nontender without organomegaly . She has a
16 well-healed abdominal scar. Stool test for occult blood is negative Laboratory results are as
17 follows:
18
19 Hemoglobin 11 .0 g/dL

-
20

22
23
24
25
Mean corpuscular volume

Platelets
Leukocytes
106 flL

275,000/IJL

9,000/IJL

26 Potassium 3.3 mEq/L


27
Bicarbonate 24 mEq/L
28
29 Creatinine 0.6 mg/dL
30
31 Calcium 7.8 mg/dL
32
33 Glucose 140 mg/dL
34
35 Albumin 4.0 g/dL
36 Erythrocyte sedimentation
37 11 mm/h
rate
38
39
40
2
Item : 2 1 of40 ~'?Mark <J C> 61 ~ ~ , ~
0. ld : 3603 Previous Next Lab Values Notes Calculator Reverse Color Text Zoom
3
~

4 follows:
5
6 Hemoglobin 11 0 g/dL
7
Mean corpuscular volume 106 fL
8
9 Platelets 275,000/IJL
10
11 Leukocytes 9,000/IJL
12
13 Potassium 3.3 mEq/L
14
Bica rbonate 24 mEq/ L
15
16 Creatinine 0.6 mg/dL
17
18 Calcium 7.8 mg/dL
19

-
20

22
23
24
25
Glucose

Albumin
Erythrocyte sedimentation
rate
140 mg/dL

4.0 g/dL

11 mm/h

26 -
27 W hich of the following is the most likely diagnosis in this patient?
28
29
30 <0 A Inflammatory bowel disease
31 <0 B. Irritable bowel syndrome
32
33 <0 C. Lactose intolerance
34 <0 D. Partial small-bowel obstruction
35
<0 E. Small-bowel bacterial overgrowth
36
37
38 Submit
39
40
2
Item : 2 1 of40
0. ld : 3603
~'?Mark <]
Previous
C>
Next
a
Lab Values
~
Notes
~
Calculator
,
Reverse Color
~
Text Zoom
3
4
5
6
7 Small intestinal bacterial overgrowth
8
9
• Anatomical abnormalities (eg, strictures, surgery)
10
11
Etiology • Motility disorders (eg, diabetes mellitus, scleroderma)
12
• Other causes (eg, end-stage renal disease, AIDS,
13
14
cirrhosis, advanced age)
15
16 • Abdominal pain, diarrhea, bloating, excess
17 Signs/
symptoms flatulence , malabsorption, weight loss, anemia, &
18
nutritional deficiencies
19

-
20

22
23
24
25
Diagnosis
• Endoscopy (gold standard) with jejunal aspirate
showing> 105 organisms/ml
• Glucose breath hydrogen testing

26
Common
27
organisms • Streptococci, Bacteroides, Escherichia, Lactobacillus
28
29
30 • 7-10-day course of antibiotics (eg, rifaximin ,
31 amoxicillin-clavulanate)
32
33 Treatment • Avoid antimotility agents (eg, narcotics)
34 • Dietary changes (eg, high-fat, low-carbohydrate)
35
36 • Trial of promotility agents (eg, metoclopramide)
37
© UWorld
38
39
40
II I ., ... I I '"' ... I ., I I I I ... I t I • '"' II ., I '"' I ... I ., '"' ., I '"' I I t' I

~ Feedback SuWend EnQ ock


2
Item : 2 1 of40 ~'?Mark <J C> 61 ~ ~ , ~
0. ld : 3603 Previous Next Lab Values Notes Calculator Reverse Color Text Zoom
3
4 ©UWorld
5
6 This patient's presentation is consistent with likely small-intestine bacterial overgrowth
7 (SIBO) SIBO is due to an increased number of native and non-native intestinal bacteria that alter
8
the normal flora and cause excessive fermentation, inflammation, or malabsorption The proximal
9
small intestine normally contains relatively minimal bacterial colonization due to gastric acidity and
10
11 peristalsis. Other protective mechanisms against SIBO include bacterial degradation by
12 proteolytic digestive enzymes, trapping of bacteria by intestinal mucus, and an intact ileocecal
13 valve preventing retrograde bacterial movement from the colon.
14
15 Causes of SIBO include anatomical changes (eg, surgically created anastomosis, strictures,
16 fistulas), motility disorders (eg, diabetes, systemic sclerosis, radiation enteritis) , and others (eg,
17 acid suppression, immunodeficiency states, chronic pancreatitis, cirrhosis) This patient likely
18 has systemic sclerosis as suggested by her skin hardening and telangiectasias Patients
19

-
typically develop bloating, increased flatulence, abdominal discomfort, and diarrhea. Laboratory
20 studies are usually normal in most patients, but some may have nutritional deficienc ies (eg,
calc ium, vitamin 812, fat-soluble vitamins). Severe cases may develop significant diarrhea,
22
23 malabsorption, and weight loss.
24
The gold standard for diagnosis is endoscopy with jejunal aspirate showing >1CP organisms/ml
25
26 (normal ~1 0' organisms/ml) Other tests, such as the hydrogen breath test using lactulose, show
27 an early peak in hydrogen levels due to rapid lactulose metabolism in the small bowel (normally
28 broken down in the colon)
29
30 (Choice A) Inflammatory bowel disease is associated with diarrhea, weight loss, and
31 anemia. Many patients report melena or hematochezia, and stool occult test is usually
32 positive Patients may also have extra-intestinal symptoms and elevated inflammatory markers
33 (eg, erythrocyte sedimentation rate, C-reactive protein)
34
35 {Choice B ) Irritable bowel syndrome is diagnosed based on the Rome criteria . Patients usually
36 have chronic abdominal pain P-3 days/month for at least 3 months), stool urgency, altered bowel
37 habits, and a feeling of incomplete evacuation after a bowel movement This patient's symptoms
38
are more suggestive of SIBO.
39
40
2
Item : 2 1 of40 ~'?Mark <] C> !I ~ ~ , ~
0. ld : 3603 Previous Next Lab Values Notes Calculator Reverse Color Text Zoom
3 . . .

4 an early peak in hydrogen levels due to rapid lactulose metabolism in the small bowel (normally
5 broken down in the colon)
6
7 (Choice A) Inflammatory bowel disease is associated with diarrhea, weight loss, and
8 anemia. Many patients report melena or hematochezia, and stool occult test is usually
9 positive. Patients may also have extra-intestinal symptoms and elevated inflammatory markers
10 (eg, erythrocyte sedimentation rate, C-reactive protein)
11
12 {Choice B) Irritable bowel syndrome is diagnosed based on the Rome criteria. Patients usually
13 have chronic abdominal pain (~3 days/month for at least 3 months), stool urgency, altered bowel
14
habits, and a feeling of incomplete evacuation after a bowel movement This patient's symptoms
15
are more suggestive of SIBO.
16
17
(Choices C) Patients with lactose intolerance usually have symptoms (eg, diarrhea with
18
19
abdominal bloating, cramps) after consuming lactose-containing products However, this patient

-
20 has daily symptoms that are more common at night without specific food triggers.

22 (Choice D) Partial small-bowel obstruction typically presents with postprandial abdominal


23 discomfort, nausea, and obstipation (ie, inability to pass flatus or stool) This patient has no such
24 symptoms
25
26 Educational objective:
27 Small-intestine bacterial overgrowth is a malabsorption syndrome due to anatomic or dysmotility
28 disorders. It typically presents with abdominal bloating, flatulence, and diarrhea. Weight loss
29 and nutritional deficiencies may occur in severe cases. Endoscopy with jejunal aspirate showing
30 >10' organisms/ml is the gold standard for diagnosis.
31
32
References:
33
34 1. Small intestinal bacterial overgrowth syndrome
35
36 2. Small intestinal bacterial overgrowth.
37
38
39 Time Spent 4 seconds Copyright© UWorld Last updated: [08/24/2016)
40
2
Item: 22 of 40 ~'?Mark <J C> 61 ~ ~ , ~
0. ld : 3591 Previous Next Lab Values Notes Calculator Reverse Color Text Zoom
3
4
5
6 A 45-year-old man comes to the physician due to epigastric pain and diarrhea. Past medical
7 history is significant fo r peptic ulcer disease. He has a 20-pack-year smoking history but does
8 not use alcohol or illicit drugs Physical examination shows abdominal tenderness without
9 rebound or rigidity . Endoscopy shows prominent gastric folds, 3 duodenal ulce rs, and upper
10 jejunal ulceration. W hich of the following is the most appropriate next step in the management of
11 this patient?
12
13
14 e') A Calcium infusion study
15
16 ® B. Jejunal ulcer biopsy
17
® C. Serum chromogranin A
18
19 ® D. Serum gastrin concentration
20

-
21 ® E. Triple therapy for Helicobacter pylori

23
24 Submit
25
26
27
28
29
30
31
32
33
34
35
36
37
38
39
40
2
Item: 22 of 40
0. ld : 3591
~'?Mark <]
Previous
C>
Next
a
Lab Values
~
Notes
~
Calculator
,
Reverse Color
~
Text Zoom
3
C. Serum chromogranin A [1%)
4
5 ~ D. Serum gastrin concentration [79%)
6 E. Triple therapy for Helicobacter pylori [13%)
7
8
9 Explanation:
10
11
12
13
Suspected gastrinoma
14
15
16
17
+
18 Endoscopy shows multiple
19 stomach ulcers & thickened
20 gastric folds

-
21

23
24
25
26
No
gastrinoma
<110
pg/ml Check serum gastrin level off
PPI therapy for 1 week
>1000
pg/ml
Check gastric pH off
PPI therapy for 1 week

27
28
110-1000 pg/ml pHsY~H>4
29
30 Negative Positive Further testing
Secretin No
31 to localize
stimulation test gastrinoma
32 gastrinoma
33
©UWorld
34
35
This patient's clinical findings - recurrent peptic ulcer disease with multiple ulcers and jejunal
36
ulceration - suggest gastrinoma (Zollinger-EIIison syndrome [ZESJ) ZES is usually sporadic but
37
38 is found in conjunction with multiple endoc rine neoplasia type 1 (MEN-1) in 20% of cases.
39 Gastrinomas usually occur in patients age 20-50 with dyspepsia, reflux symptoms, abdominal
40 - -- --- --- -- -- - - -- -- ---- - - --- -- - -- -- - - ----- --- ---

~ Feedback SuWend EnQ ock


2
Item : 22 of 40 ~'?Mark <] C> !I ~ ~ , ~
0. ld : 3 591 Previous Next Lab Values Notes Calculator Reverse Color Text Zoom
3 I '" -.~ I Itt"' 1, I ,. I t '"' I ' I I '" I 1 ... ' II ... t II
4 is found in conjunction with multiple endocrine neoplasia type 1 (MEN-1 ) in 20% of cases.
5
Gastrinomas usually occur in patients age 20-50 with dyspepsia, reflux symptoms, abdominal
6
pain, weight loss, diarrhea, or frank gastrointestinal bleeding. Endoscopy often shows thickened
7
8 gastric folds, multiple peptic ulcers, refractory ulcers despite proton pump inhibitor ( PPI) use, or
9 ulcers distal to the duodenum in the jejunum (suggesting excess gastric acid that cannot be fully
10 neutralized in the duodenum)
11
12 Fasting serum gastrin level (off PPI therapy for 1 week) should be checked in suspected
13 gastrinoma; a level <110 pg/ml rules it out, and a level >1000 pg/ml is diagnostic If gastrin is
14 elevated, gastric pH should also be measured as gastrin may also be elevated due to failure of
15 gastric acid secretion (achlorhydria) A gastrin level of 110-1000 pg/ml is non-diagnostic and
16 requires a follow-up secretin stimulation test Secretin stimulates the release of gastrin by
17 gastrinoma cells. Normal gastric G cells are inhibited by secretin; therefore, secretin
18
administration should not cause a rise in serum gastrin concentrations in patients with other
19
20 causes of hypergastrinemia.

-
21

23
24
25
26
(Choice A) The calcium infusion study is usually reserved fo r patients who have gastric acid
hypersecretion and are strongly suspected of having gastrinoma despite a negative secretin test
Calcium infusion can lead to an inc rease in serum gastrin levels in patients with gastrinoma

(Choice B) Jejunal ulcer biopsy would not likely be useful as jejunal ulcers are usually due to
27 gastric acid hypersecretion rather than local tumor invasion.
28
29 (Choice C) Serum chromogranin A is a marke r for well-differentiated neuroendocrine tumors. It
30 is elevated in several conditions such as carcinoid tumors, hyperthyroidism, chronic atrophic
31 gastritis, and even chronic PPI therapy For this reason, it is less sensitive and specific than
32 measurement of serum gastrin leveL However, it may be used as a confi rmatory test in
33 non-diagnostic cases.
34
35 (Choice E) Triple therapy (PPI, amoxicillin, and clarithromycin) treats HeJicobacter pylori
36 infection, which can cause gastric and duodenal ulcers. However , this patient's diarrhea and
37 multiple ulcers in different sites are more suggestive of gastrinoma.
38
39 Educafonal ob'ecfve·
40
Item : 22 of 40 ~'?Mark <] C> !I ~ ~ , ~
2
3
4
0. ld : 3 591
. .. ... . - - . . ..
... ... - . .. -.- . . ... ..
Previous Next
. .. . . . .
requires a follow-up secretin stimulation test Secretin stimulates the release of gastrin by
...
Lab Values Notes Calculator Reverse Color Text Zoom

5 gastrinoma cells. Normal gastric G cells are inhibited by secretin; therefore, secretin
6
administration should not cause a rise in serum gastrin concentrations in patients with other
7
causes of hypergastrinemia
8
9
{Choice A) The calcium infusion study is usually reserved for patients who have gastric acid
10
11 hypersecretion and are strongly suspected of having gastrinoma despite a negative secretin test
12 Calcium infusion can lead to an inc rease in serum gastrin levels in patients with gastrinoma
13
14 (Choice B) Jejunal ulcer biopsy would not likely be useful as jejunal ulce rs are usually due to
15 gastric acid hype rsecretion rather than local tumor invasion.
16
17 (Choice C) Serum chromogranin A is a marker for well-differentiated neuroendocrine tumors. It
18 is elevated in several conditions such as carcinoid tumors, hyperthyro idism, chronic atrophic
19 gastritis, and even chronic PPI therapy For this reason, it is less sensitive and specific than
20 measurement of serum gastrin level. However, it may be used as a confirmatory test in

-
21

23
24
25
26
non-diagnostic cases.

{Choice E) Triple therapy (PPI, amoxicillin, and clarithromycin) treats Helicobacter pylori
infection, which can cause gastric and duodenal ulcers. However, this patient's diarrhea and
multiple ulcers in diffe rent sites are more suggestive of gastrinoma.
27 Educational objective:
28 Gastrinoma (Zollinger-EIIison syndrome) should be suspected in patients with multiple stomach
29
ulce rs and thickened gastric folds on endoscopy The diagnosis is strongly suggested by a
30
31 fasting serum gastrin level >1 000 pg/ml Patients with non-diagnostic serum gastrin levels
32 should be evaluated with a secretin stimulation test
33
34 Refer ences:
35
36 1. Diagnosis and treatment of gastrinoma in the era of proton pump inhibitors.
37
38
39 Time Spent 2 seconds Copyright © UWorld Last updated: [10/03/2016)
40
2
Item: 23 of 40 ~'?Mark <J C> 61 ~ ~ , ~
0. ld : 3857 Previous Next Lab Values Notes Calculator Reverse Color Text Zoom
3
4
5
6 A 36-year-old man comes to the clinic with a 2-day history of traces of blood on the tissue paper
7 after bowel movements. He does not have gross blood mixed with stool, but on one occasion
8 there were drops of blood in the toilet after defecation. The patient otherwise feels well. Past
9 medical history is unremarkable, and the patient has no family history of malignancy. Vital signs
10 are normal. The abdomen is soft and nontender with normal bowel sounds and no
11 hepatosplenomegaly or masses. Digital rectal examination is unremarkable except for traces of
12
red blood visible on the glove W hich of the following is the most appropriate next step in
13
14 management?
15
16 6 A Anoscopy
17
18 e') B. Barium enema
19 6 C. Colonoscopy
20
21 e') D. Fecal occult blood test

--
22

24
25
26
27
6 E. Sigmoidoscopy

Submit

28
29
30
31
32
33
34
35
36
37
38
39
40
2
Item: 23 of 40
0. ld : 3857
~'?Mark <]
Previous
C>
Next
a
Lab Values
~
Notes
~
Calculator
,
Reverse Color
~
Text Zoom
3
4
5 Explanation:
6
7
8 Evaluation of minimal bright red blood per rectum
9
10 Minimal bright red blood
11 per rectum
12
13
14
15
16
17 Age <40 without red flags Age 4049 without red f lags Age <:50 or red flags•
18
19
20
21

--
22

24
25
26
27 I HemToKjs I..._Nidentified
_o_sou
_ r_
c e__.l---•
Sigmoidoscopy Colonoscopy
or colonoscopy
28
29
30 No further evaluation
necessary
31 =
*Red flags change in bowel habits, abdominal pain, weight
32 @UWorld loss, iron deficiency anemia, family history of colon cancer.
33
34 This patient has minimal bright red blood per rectum (BRBPR) This symptom, characterized
35 by small amounts of bright red blood on to ilet paper after wiping, a few drops of blood in the toilet
36
bowl after defecation, or small amounts of blood on the surface of the stool, is most often due to
37
38 benign disorders such as hemorrhoids or rectal fissures. However , more serious disorders (eg,
39 proctitis, rectal ulce rs, colorectal polyps, cancer) are possible.
40
2
Item: 23 of 40 ~'?Mark <] C> !I ~ ~ , ~
0. ld : 3857 Previous Next Lab Values Notes Calculator Reverse Color Text Zoom
3
loss, iron deficiency anemia, family history of colon cancer.
4
5
6
This patient has minimal bright red blood per r ectum (BRBPR) This symptom, characterized
7 by small amounts of bright red blood on toilet paper after wiping, a few drops of blood in the toilet
8 bowl after defecation, or small amounts of blood on the surface of the stool, is most often due to
9 benign disorders such as hemorrhoids or rectal fissures. However, more serious disorders (eg,
10 proctitis, rectal ulcers, colorectal polyps, cance r) are possible
11
12 The evaluation of BRBPR depends on the patient's presentation and risk factors. Clinical factors
13 associated with increased risk of serious disease include blood mixed with stool, systemic
14 symptoms (eg, fever, weight loss), diarrhea, anemia, change in bowel habits, and abdominal
15 pain. Age also co rrelates strongly with risk of malignancy in patients with BRBPR. Patients age
16 :::so are at elevated risk for colorectal cancer and should undergo colonoscopy unless they have
17
had a normal colonoscopy within the last 2-3 years Patients age 40-49 are at intermediate risk
18
19 and could be considered fo r sigmoidoscopy as an alternative to colonoscopy. If the patient is
20 age <40 and has no other risk factors fo r colon cance r, office-based anoscopy or proctoscopy
21 should be performed first If no etiology is found, colonoscopy or sigmoidoscopy is then

--
22

24
25
26
27
considered (Choices C and E) Anoscopy is also useful in older patients to visualize a palpable
abnormality found on physical examination.

(Choice B) Barium enema is occasionally used for colorectal cancer screening (often in
addition to other procedures) in asymptomatic patients It has low sensitivity for evaluating rectal
bleeding
28
29 (Choice D) This patient has gross blood confirmed on examination. Occult blood testing is not
30 necessary.
31
32 Educational objective:
33 Minimal rectal bleeding is usually due to hemorrhoids or other benign conditions. Evaluation
34
depends on the patient's presentation and risk factors. Patients age ~50 or with clinical features
35
suggesting malignancy should undergo colonoscopy For younger patients (age <40) and no
36
37 other risk factors, office-based anoscopy may be performed first
38
39 Time Spent 2 seconds Copyright © UWorld Last updated: [08/30/2016)
40
2
Item : 24 of 40 ~'?Mark <J C> 61 ~ ~ , ~
0. ld: 2 2 12 Previous Next Lab Values Notes Calculator Reverse Color Text Zoom
3
4
5
6 An 82-year-old man is brought to the hospital with 3 days of fever, productive cough, and
7 shortness of breath. He was admitted to the hospital 2 months ago with right lower lobe
8 pneumonia. His wife reports that he coughs, chokes, and has nasal regurgitation when
9 swallowing liquids or solids. The patient admits the food "gets stuck" in his th roat when
10 swallowing He has a history of hypertension, type 2 diabetes, ischemic stroke , chronic kidney
11
disease, co ronary artery disease, and advanced dementia. His temperature is 38.7 C (101 .7 F) ,
12
blood pressure is 142/90 mm Hg, pulse is 104/min, and respirations are 20/min. Neck
13
14 examination shows midline trachea without palpable masses. Lung examination shows crackles in
15 the right lower lung field. Abdomen is soft and nontender. Chest imaging reveals right lower lobe
16 pneumonia without additional mass lesions. W hich of the following is the most appropriate next
17 step in management of this patient's dysphagia?
18
19
20 <0 A 4-week proton pump inhibitor trial
21 <0 B. 24-hour pH monitoring
22

--
<0 C. Esophageal motility studies
23
e; D. Upper gastrointestinal endoscopy
25
<0 E. Videofluoroscopic modified barium swallow
26
27
28
29 Submit
30
31
32
33
34
35
36
37
38
39
40
2
Item: 24 of 40 ~'?Mark <] C> !I ~ ~ , ~
0. ld: 2212 Previous Next Lab Values Notes Calculator Reverse Color Text Zoom
3
4 Dysphagia can be classified as oropharyngeal or esophageal. Oropharyngeal dysphagia
5 presents with difficulty initiating swallowing due to inability to properly transfer food from the
6 mouth to the pharynx. Underlying etiologies fo r oropharyngeal dysphagia can include stroke,
7
advanced dementia, oropharyngeal malignancy, or neuromuscular disorders (eg, myasthenia
8
9
gravis).
10
Patients with oropharyngeal dysphagia can also have associated coughing, choking, or nasal
11
12 regurgitation on swallowing Other complications can include aspiration pneumonia and weight
13 loss. This patient's history of stroke with coughing and choking on swallowing suggests
14 oropharyngeal dysphagia. Recurrent right lower lobe pneumonia suggests likely aspiration
15 pneumonia. Videofluoroscopic modified barium swallow study is preferred initially in these
16 patients to evaluate swallowing mechanics, degree of dysfunction, and severity of aspiration
17
18 (Choices A and B) Proton pump inhibitors can be used for treatment of typical
19 gastroesophageal refl ux disease (GERD) Obtaining 24-hour pH monitoring might be helpful in
20 confirming a diagnosis of GERD. However, this patient's dysphagia associated with coughing
21
and choking along with his history of stroke and recurrent aspiration pneumonia require barium
22

--
swallow to evaluate for oropharyngeal dysphagia
23
{Choices C and D) Esophageal motility studies and upper gastrointestinal endoscopy are
25
typically used fo r evaluating esophageal dysphagia, which presents with a sensation of food
26
27 getting stuck in the esophagus (not throat) a few seconds after a swallow but does not cause
28 difficulty initiating swallowing. This patient's findings are more concerning for oropharyngeal
29 dysphagia
30
31 Educational objective:
32 Oropharyngeal dysphagia presents as difficulty initiating swallowing associated with coughing,
33 choking, aspiration, or nasal regurgitation Typically, patients are evaluated initially with
34 videofluoroscopic modified barium swallow to evaluate swallowing mechanics, degree of
35 dysfunction, and severity of aspiration
36
37
38 References:
39 1. Role of barium esophagography in evaluating dysphagia.
40

~
-------------------------------------------------------------------------------------------------------------------------------
Feedback SuWend EnQ ock
2
Item: 25of40 ~'?Mark <J C> 61 ~ ~ , ~
0. ld : 2943 Previous Next Lab Values Notes Calculator Reverse Color Text Zoom
3
4
5
6 A 52-year-old obese woman comes to the clinic complaining of intermittent right upper-quadrant
7 pain and nausea. She had an elective cholecystectomy for gallstones a year ago. The pain is
8 located in the right subcostal area and generally lasts 30-60 minutes. The patient recalls that she
9 had similar pain episodes before the surgery Laboratory results after one of the episodes are as
10 follows:
11
12 Total bilirubin 2.1 mg/dl
13
14 Direct bilirubin 1.2 mg/dl
15
Alkaline phosphatase 185 U/ L
16
17 Aspartate aminotransferase (AST,
18 84 U/ L
SGOT)
19
20 Alanine aminotransferase (ALT, SGPT) 72 U/L
21
22
23 An ultrasound of the abdomen reveals mild dilation of the common bile duct The pancreas is

-
24

26
27
28
29
visualized and appears normal. W hich of the following is the best next step in management of this
patient?

e') A Antimitochondrial antibodies


6 B. Endoscopic retrograde cholangiopancreatog raphy
30
6 C . Helicobacter pylori stool antigen testing
31
32 6 D. Liver biopsy
33 e') E. Ursodeoxycholic acid therapy
34
35
36 Submit
37
38
39
40
2
Item: 25of40 ~'?Mark <J C> 61 ~ ~ , ~
0. ld : 2943 Previous Next Lab Values Notes Calculator Reverse Color Text Zoom
3
4 C. Helicobacter pylori stool antigen testing [0%)
5 D. Liver biopsy [1%)
6
7 E. Ursodeoxycholic acid therapy [7%)
8
9
Explanation:
10
11
This patient's presentation is conce rning for postcholecystectomy syndrome ( PCS). PCS refers
12
to persistent abdominal pain or dyspepsia (eg, nausea) that occurs either postoperatively (early)
13
14 or months to years (late) after a cholecystectomy. PCS can be due to biliary (eg, retained
15 common bile duct or cystic duct stone, biliary dyskinesia) or extra-biliary (eg, pancreatitis, peptic
16 ulcer disease, coronary artery disease) causes. Patients usually notice the same pain they had
17 prior to surgery, new pain just after surgery, or the same pain that never went away.
18
19 Laboratory findings can include elevated alkaline phosphatase, mildly abnormal serum
20 aminotransferases, and dilated common bile duct on abdominal ultrasound. These findings
21 usually suggest common bile duct stones or biliary sphincter of Oddi dysfunction The next step
22 involves endoscopic ultrasound, endoscopic retrograde cholangiopancreatography ( ERCP) , or
23

-
magnetic resonance cholangiopancreatography for final diagnosis and guiding therapy
24
Treatment for PCS is directed at the causative factor.
26 (Choice A) Antimitochondrial antibodies are usually elevated in primary biliary cirrhosis. These
27
patients typically have other symptoms (eg, pruritus, fatigue, jaundice) along with abdominal
28
29 pain. Although primary biliary cirrhosis can present with laboratory findings similar to those seen
30 in this patient, imaging typically does not show a dilated common bile duct
31
32 {Choice C) Helicobacter pylori stool antigen testing is used for diagnosing H pylori infection in
33 patients with peptic ulce r disease. It is not typically associated with abnormal liver function tests
34 and/or dilated common bile duct
35
36 (Choice D) Liver biopsy should be done when initial evaluation (laboratory analysis and imaging)
37 cannot provide a definitive diagnosis or determine the severity of liver disease. This patient may
38 require liver biopsy if ERCP and other laboratory tests for hepatitis are unremarkable.
39
40
2
Item: 25of40 ~'?Mark <] C> !I ~ ~ , ~
0. ld : 2943 Previous Next Lab Values Notes Calculator Reverse Color Text Zoom
3
4 Treatment for PCS is directed at the causative factor.
5
6 (Choice A) Antimitochondrial antibodies are usually elevated in primary biliary cirrhosis. These
7 patients typically have other symptoms (eg, pruritus, fatigue, jaundice) along with abdominal
8 pain Although primary biliary cirrhosis can present with laboratory findings similar to those seen
9 in this patient, imaging typically does not show a dilated common bile duct
10
11 (Choice C) Helicobacter pylori stool antigen testing is used for diagnosing H pylori infection in
12 patients with peptic ulcer disease. It is not typically associated with abnormal liver function tests
13 and/or dilated common bile duct.
14
15 (Choice D) Liver biopsy should be done when initial evaluation {laboratory analysis and imaging)
16 cannot provide a definitive diagnosis or determine the severity of liver disease. This patient may
17 require liver biopsy if ERCP and other laboratory tests for hepatitis are unremarkable.
18
19 (Choice E) Ursodeoxycholic acid can treat cholesterol gallstones in patients with mild symptoms
20 who are not candidates for cholecystectomy It is also used to treat primary biliary cirrhosis and
21
primary sclerosing cholangitis
22
23

-
Educational objective:
24
Postcholecystectomy syndrome is persistent abdominal pain or dyspeps ia either postoperatively
26 (early) or months to years {late) after cholecystectomy Etiologies include biliary (eg, retained
27 common bile duct, cystic duct stone) or extra-biliary (eg, pancreatitis, peptic ulcer disease)
28 causes. Abdominal imaging (eg, ultrasound) followed by direct visualization (eg, endoscopic
29 retrograde cholangiopancreatography, magnetic resonance cholangiopanc reatography) can
30 establish the diagnosis and guide therapy toward the causative factor.
31
32
Refer ences:
33
34 1. Biliary causes of postcholecystectomy syndrome.
35
36 2. Postcholecystectomy syndrome (PCS).
37
38
39 Time Spent 2 seconds Copyright © UWorld Last updated [08/28/2016)
40
2
Item: 26 of 40 ~'?Mark <J C> 61 ~ ~ , ~
0. ld: 2912 Previous Next Lab Values Notes Calculator Reverse Color Text Zoom
3
4
5
6 A 42-year-old woman comes to the office with worsening fatigue and dark yellow urine. She also
7 has anorexia, nausea, mild abdominal pain, and malaise. The patient is unce rtain as to when her
8 symptoms began but felt normal during her vacation 4 weeks ago. She has no known medical
9 problems and does not take any medications. The patient does not use alcohol, tobacco, or illicit
10 drugs. She has had unprotected sexual intercourse with 6 different partners within the past yea r.
11 The patient does not remember getting a hepatitis 8 vaccination. Laboratory results are as
12
follows:
13
14 5.3
15 Total bilirubin
mg/ dl
16
17 Alkaline phosphatase 170 U/L
18
19 Aspa rtate
992 U/L
20 aminotransferase
21
22 Alanine
1290 U/L
23 aminotransferase
24

-
25

27
28
29
30
W hich of the following is the most appropriate test to diagnose hepatitis 8 infection in this patient?

e> A H8cAg
6 8 . H8eAg
31 6 C . H8sAg and anti-H8s
32 6 D. H8sAg and lgM anti-H8c
33
34 e> E. Hepatitis 8 virus DNA
35
36
37 Submit
38
39
40
Item: 26 of 40 ~'?Mark <] C> a ~ ~ , ~
2
3
4
. . .
a. ld: 2912
. .
Previous Next I ..coh Values Notes Calculator Reverse Color Text Zoom

5
6
7
8 Acute hepatitis B infection
9
10
11 Symptoms
12
13 HBsAg b~==:::::;j)
14
15 HBeAg 1:::::::::::::::::::J
16
1 ~----
HBV DNA .. ::::;;-
17
18
19
20 c Total anti-HBc
0
21
~
22
23
-~
c
24 c

-
0
25 (.)

27 Anti-HBs
28
29
30
31
32
33
34
35
36
0 1 2 3 24 36
37
38 Months after hepatitis B virus infection
39
40
2
Item : 26 of 40 ~'?Mark <] C> !J ~ ~ , ~
3
4
0. ld: 291 2
. -
Previous
. Next
- . Lab Values Notes Calculator Reverse Color Text Zoom

5
Months after hepatitis B virus infection
6
7 © UWorld
8
9 This patient's new-onset symptoms (eg, fatigue, nausea/abdominal pain, dark urine) and
10 abnormal liver function tests (eg, elevated serum transaminases, hyperbilirubinemia) in the setting
11 of unprotected sex with multiple partners is concerning for acute hepatitis B infection. The first
12 serologic marker to appear in the serum with acute hepatitis B is HBsAg, which appears usually
13 4-8 weeks after infection. lgM anti-HBc appears shortly thereafter, which is around the time
14 clinical symptoms occur and patients develop elevations in hepatic aminotransferase levels (often
15 >25 times the normal limit)
16
17 There can be a time lag (weeks to months) between the disappearance of HBsAg and the
18
appearance of anti-HBs, which is termed the window period. lgM anti-HBc may be the only
19
detectable marker for acute hepatitis B infection during this period As such, HBsAg and
20
21
anti-HBc are the most appropriate diagnostic tests for acute hepatitis B infection as these are
22 both elevated during initial infection and anti-HBc will remain elevated during the window period
23
24 (Choice A) HBcAg is not detectable in serum and is therefore not useful for establishing the

-
25 diagnosis.

27 {Choice B) HBeAg is a good indicator of infectivity but is a poor test for acute hepatitis B
28 infection as levels typically fall early in the course of the disease.
29
30 {Choice C) Testing for HBsAg and anti-HBs can miss the diagnosis of acute hepatitis B infection
31 in patients who are in the window period
32
33 (Choice E) Although hepatitis B virus DNA may be detectable prior to the appearance of HBsAg
34 or HBeAg, this test is generally not performed to diagnose acute infection. Instead, hepatitis B
35 virus DNA is obtained in patients with chronic hepatitis B to determine candidacy for antiviral
36 therapy or monitor response to treatment
37
38 Educational objective:
39 HBsAg and anti-HBc are the most appropriate diagnostic tests for acute hepatitis B infection as
40
2
Item: 26 of 40 ~'?Mark <] C> !I ~ ~ , ~
0. ld: 2912 Previous Next Lab Values Notes Calculator Reverse Color Text Zoom
3 . .. ... . . . .. . .. .. .. . . . . .
4 >25 times the normal limit)
5
6 There can be a time lag (weeks to months) between the disappearance of HBsAg and the
7 appearance of anti-HBs, which is te rmed the window period lgM anti-HBc may be the only
8 detectable marker for acute hepatitis B infection during this period As such, HBsAg and
9
anti-HBc are the most appropriate diagnostic tests fo r acute hepatitis B infection as these are
10
both elevated during initial infection and anti-HBc will remain elevated during the window period
11
12
{Choice A) HBcAg is not detectable in serum and is therefore not useful for establishing the
13
14 diagnosis
15
(Choice B) HBeAg is a good indicato r of infectivity but is a poor test for acute hepatitis B
16
17 infection as levels typically fall early in the course of the disease.
18
19
(Choice C) Testing for HBsAg and anti-HBs can miss the diagnosis of acute hepatitis B infection
20 in patients who are in the window period
21
22 (Choice E) Although hepatitis B virus DNA may be detectable prior to the appearance of HBsAg
23 or HBeAg, this test is generally not performed to diagnose acute infection. Instead, hepatitis B
24 virus DNA is obtained in patients with chronic hepatitis B to determine candidacy for antiviral

-
25

27
28
29
30
therapy or monitor response to treatment.

Educational objective:
HBsAg and anti-HBc are the most appropriate diagnostic tests for acute hepatitis B infection as
they are both elevated during initial infection and anti-HBc will remain elevated during the window
period
31
32
References:
33
34 1. Hepatitis 8 : diagnosis and treatment.
35
36 2. Hepatitis B virus infection.
37
38
39 Time Spent 3 seconds Copyright © UWorld Last updated: [04/26/2016)
40
2
Item : 27 of 40 ~'?Mark <J C> 61 ~ ~ , ~
0. ld : 3 582 Previous Next Lab Values Notes Calculator Reverse Color Text Zoom
3
4
5
6 A 50-year-old white male comes to the physician's office because of diarrhea, abdominal pain
7 and weight loss. He has bulky, foul-smelling stools, abdominal distension and flatulence. He also
8 has arthralgias and a chronic cough. His temperature is 37.8° C (100° F), blood pressure is
9 120/80 mm Hg, pulse is 80/min, and respirations are 18/min. Physical examination shows
10 generalized lymphadenopathy , skin hyperpigmentation and a diastolic murmur in the aortic area.
11 Small bowel biopsy shows villous atrophy with numerous PAS-positive materials in the lamina
12
propria W hich of the following is the most likely diagnosis?
13
14
15 <D A. Celiac disease
16
17 <D B. Tropical sprue
18 <D C. Crohn's disease
19
20 <D D. W hipple's disease
21 <D E. Cystic fibrosis
22
23
24

...
Submit
25
26

28
29
30
31
32
33
34
35
36
37
38
39
40
2
Item : 27 of 40 ~'?Mark <] C> !I ~ ~ , ~
0. ld : 3 582 Previous Next Lab Values Notes Calculator Reverse Color Text Zoom
3
4
5 Explanation:
6
7 W hipple's disease is a rare multi-systemic illness. It is an infectious disease caused by the
8 bacillus Tropheryma whippelii. It is most commonly seen in white men in the fourth-to-sixth
9 decades of life , and often presents with weight loss. Gastro intestinal symptoms include abdominal
10 pain, diarrhea, and malabsorption with distension, flatulence, and steatorrhea. Extraintestinal
11 manifestations include migratory polyarthropathy , chronic cough, and myocardial or valvular
12 involvement leading to congestive failure or valvular regurgitation. Later stages of the disease
13 may be characterized by dementia and other central nervous system findings, such as
14
supranuclear ophthalmoplegia and myoclonus Intermittent low-grade feve r, pigmentation and
15
lymphadenopathy may also be occasionally seen. PAS-positive material in the lamina propria of
16
17 the small intestine is a classical biopsy finding
18
19 (Choice A) Celiac disease, although associated with malabsorption, is not associated with
20 pigmentation and lymphadenopathy .
21
22 (Choice B) Tropical sprue is a chronic diarrheal disease, possibly of infectious origin, that
23 should be considered in patients who have lived for more than a month in a tropical area.
24

...
25
26

28
29
(Choice C) Crohn's disease can be associated with malabsorption, abdominal pain, fever and
arthralgias; however , this diagnosis cannot explain the patient's skin hyperpigmentation, chronic
cough and biopsy findings. Furthermore, patients with Crohn's disease usually present with
bloody diarrhea.
30 (Choice E) Cystic fibrosis can be associated with chronic cough and malabsorption. It is not
31 associated with arthralgias or skin hyperpigmentation
32
33 Educational Objective:
34
W hipple's disease is a multi-systemic illness characterized by arthralgias, weight loss, feve r,
35
diarrhea and abdominal pain PAS-positive material in the lamina propria of the small intestine is
36
37 a classical biopsy finding of W hipple's disease.
38
39 Time Spent 2 seconds Copyright © UWorld Last updated: [08/ 15/2016)
40
2
Item: 28 of 40 ~'?Mark <J C> 61 ~ ~ , ~
0. ld : 4903 Previous Next Lab Values Notes Calculator Reverse Color Text Zoom
3
4
5
6 A 34-year-old man comes to the office due to foul-smelling anal discharge and perianal
7 discomfort for the past several weeks. The patient has a history of recurrent anal fissures and
8 takes a fiber supplement, stool softeners, and a topical analgesic He also has occasional canke r
9 sores and recurrent abdominal pain and diarrhea, which he attributes to the use of laxatives. The
10 patient smokes a pack of cigarettes daily but does not use alcohol or illicit drugs He has not
11 traveled since a trip to South America a yea r ago Temperature is 38 C (100.4 F) , blood
12
pressure is 120/76 mm Hg, and pulse is 86/min. Tenderness is present in the right lower
13
14 quadrant on deep palpation. Perianal examination shows a large posterior skin tag. A fistula
15 anterolateral to the anus is draining whitish material. Further evaluation of this patient is most
16 likely to reveal which of the following?
17
18
(:) A Colonic ulcers containing protozoal cysts
19
20 e; B. Diffuse colitis with no skip areas
21
(:) C. Lymph nodes with caseating granulomas
22
23 e; 0 . Segmental inflammation of mesenteric arteries
24
25 (:) E. Transmural bowel inflammation
26

-
27

29
30
31
32
Submit

33
34
35
36
37
38
39
40
2
Item: 28 of 40
0. ld : 4903
~'?Mark <]
Previous
C>
Next
a
Lab Values
~
Notes
~
Calculator
,
Reverse Color
~
Text Zoom
3
4
5 Crohn disease
6
7 • Extending from the mouth to the anus
8
(mostly ileum & colon)
9
10 Involvement • Rectum spared
11
• Perianal disease
12
13 • Skip lesions
14
15
16
Microscopy • Noncaseating granulomas
17
18
• Transmural inflammation
19
20 • Linear mucosal ulcerations
21
Gross
• Cobblestoning
22
23 • Creeping fat
24
25
26 Clin ical
• Diarrhea (bloody if colitis)

-
27
manifestations

29 • Fistulas
30 Intestinal
31 • Strictures (bowel obstruction)
complications
32 • Abscesses
33
34 @UWotld
35
36 This patient most likely has Crohn disease (CD) CD can involve any component of the
37 gastrointestinal tract from the mouth to the anus and characteristically has skip areas of
38 involvement Anal fissu res are seen in approximately 20% of patients. Aphthous ulcers (ie,
39 canker sores) are an occasional component of the disease but are fairly nonspecific by
40
2
Item: 28 of 40 ~'?Mark <J C> 61 ~ ~ , ~
0. ld : 4903 Previous Next Lab Values Notes Calculator Reverse Color Text Zoom
3
4 Intestinal
• Sttictures (bowel obstruction)
5 complications
6 • Abscesses
7
@UWotld
8
9 This patient most likely has Crohn disease (CD) CD can involve any component of the
10 gastrointestinal tract from the mouth to the anus and characteristically has skip areas of
11 involvement Anal fissures are seen in approximately 20% of patients. Aphthous ulcers (ie,
12
canke r sores) are an occasional component of the disease but are fairly nonspecific by
13
14 themselves. Skin tags and fistulas are common perianal manifestations. Other gastrointestinal
15 symptoms include chronic diarrhea and abdominal pain Extraintestinal manifestations include
16 arthritis, skin lesions, and lung disease. CD commonly presents in patients age 15-40, and there
17 appears to be a slight female predominance Smoking is a risk factor.
18
19 Diagnosis can be confirmed with endoscopy or other imaging studies that demonstrate consistent
20 findings (eg, strictures) Biopsy typically reveals focal ulcerations with transmural
21 inflammation. Granulomas are also common. Treatment involves 5-aminosalicylic acid drugs for
22 patients without systemic symptoms and corticosteroids or biologic agents (eg, infliximab) for
23 patients with more severe disease.
24
25 (Choice A) Intestinal amebiasis due to Entamoeba histolytica can also cause abdominal pain
26

-
with chronic diarrhea and, rarely, perianal disease; however, stools are most commonly bloody
27
and E histolytica would not explain this patient's oral ulcers.
29 (Choice B) Ulcerative colitis is associated with diffuse colitis with no skip lesions and presents
30
with colicky abdominal pain and inflammatory diarrhea (eg, blood, mucus) However,
31
32 gastrointestinal involvement is limited to the colon, and both oral and perianal disease would be
33 unusual.
34
35 (Choice C) Tuberculous enteritis should be included in the differential of CD as it may present
36 similarly with abdominal pain, diarrhea, strictures, and fistula. However , patients classically have
37 feve r, night sweats, and weight loss, and oral ulcers would be unusual.
38
39 (Choice D) Vasculitides such as polyarteritis nodosa can present with recurrent abdominal pain
40
2
Item: 28 of 40 ~'?Mark <] C> !I ~ ~ , ~
0. ld : 4903 Previous Next Lab Values Notes Calculator Reverse Color Text Zoom
3
4 patients without systemic symptoms and corticoste roids or biologic agents (eg, infl iximab) for
5 patients with more severe disease.
6
7 (Choice A) Intestinal amebiasis due to Entamoeba histolytica can also cause abdominal pain
8 with chronic diarrhea and, rarely , perianal disease; however, stools are most commonly bloody
9 and E histolytica would not explain this patient's oral ulcers.
10
11 (Choice B) Ulcerative col itis is associated with diffuse colitis with no skip lesions and presents
12 with colicky abdominal pain and inflammatory diarrhea (eg, blood, mucus). However ,
13 gastrointestinal involvement is limited to the colon, and both oral and perianal disease would be
14
unusual.
15
16 (Choice C) Tuberculous enteritis should be included in the differential of CD as it may present
17
similarly with abdominal pain, diarrhea, strictures, and fistula. However, patients classically have
18
19 fever, night sweats, and weight loss, and oral ulcers would be unusual.
20
{Choice D) Vasculitides such as polyarteritis nodosa can present with recurrent abdominal pain
21
22 and diarrhea, but the abdominal pain is frequently postprandial as a result of chronic mesenteric
23 ischemia. Patients also commonly have multiple other manifestations, including skin (eg,
24 erythema nodosum), renal, and neurologic disease (eg, mononeuropathy multiplex). Fistulas are
25 not a common complication
26

-
27

29
30
31
32
Educational objective:
Grahn disease can involve any component of the gastrointestinal tract from the mouth to the anus,
such as aphthous ulcers and perianal skin tags and fistulas. Biopsy typically reveals focal
ulcerations with transmural inflammation.

References:
33
34 1. Diagnosis and management of Crohn's disease.
35
36 2. Treatment of peri-anal fistula in Crohn's disease.
37
38
39 Time Spent 2 seconds Copyright © UWorld Last updated: [10/ 12/2016)
40
2
Item : 29 of 40 ~'?Mark <J C> 61 ~ ~ , ~
0. ld : 3704 Previous Next Lab Values Notes Calculator Reverse Color Text Zoom
3
4
5
6 A 35-year-old Caucasian male presents to the emergency department with two episodes of
7 bloody vomiting which occurred one-half hour ago He has a history of migraines and takes
8 aspirin For the past two days, he has been having severe headaches and has taken 20 tablets
9 of aspirin without relief. He then resorted to heavy drinking and forgot about the pain. He drinks
10 alcohol "occasionally" and has been smoking 1 pack of cigarettes daily for the past 18 yea rs.
11 W hich of the following is the most likely explanation fo r this patient's hematemesis?
12
13
14 (:) A. Esophageal variceal bleeding
15
16 10 B. Acute erosive gastritis
17 e> C. Mallory Weiss syndrome
18
19 o D. Fulminant hepatic fa ilure
20 10 E. Acute platelet dysfunction
21
22
23
Submit
24
25
26

...
27
28

30
31
32
33
34
35
36
37
38
39
40
2
Item : 29 of 40 ~'?Mark <] C> !I ~ ~ , ~
0. ld : 3704 Previous Next Lab Values Notes Calculator Reverse Color Text Zoom
3 - ~ - ... . . . . .
4 E. Acute platelet dysfunction (6%)
5
6
7 Explanation:
8
9 Acute erosive gastritis is characterized by the development of severe hemorrhagic erosive lesions
10 after the exposure of gastric mucosa to various injurious agents or after a substantial reduction in
11 blood flow. Asp irin decreases the protective prostaglandin production In addition, aspirin and
12
alcohol cause direct mucosal injury (as in this case). Acute mucosal injury decreases the normal
13
14 protective barriers (decreased levels of secreted mucins and bica rbonate, and decreased
15 integrity of the epithelium) , thus permitting acid and other luminal substances (e.g , proteases and
16 bile acids) to penetrate into the lamina propria, thereby causing additional injury to the
17 vasculature and subsequent hemorrhage As in this case, patients may manifest with
18 hematemesis and abdominal pain.
19
20 {Choice A) Esophageal variceal bleeding is the other common cause of upper Gl bleeding, but it
21 is seen in patients with live r disease. This patient is a young, occasional drinke r and has no
22 other findings suggestive of liver disease.
23
24 {Choice C) Mallory-Weiss tears occur in the distal esophagus at the gastro-esophageal junction
25 after repeat bouts of retching and vomiting No such history is given in this case.
26

...
27
28

30
31
(Choice D) This patient has no other signs suggestive of fulminant hepatic fa ilure, which would be
characterized by hepatic encephalopathy .

{Choice E) Platelet dysfunction might be an aggravating facto r in this patient's bleeding since he
took large amounts of aspirin; however , platelet dysfunction by itself cannot cause erosive
32
gastritis
33
34
Educational Objective:
35
Massive doses of aspirin and NSAIDs can cause acute erosive gastritis and upper Gl bleeding
36
37 Alcohol can aggravate their effect
38
39 Time Spent 2 seconds Copyright © UWorld Last updated: [08/ 10/2016)
40
2
Item: 30 of 40 ~'?Mark <J C> 61 ~ ~ , ~
0. ld : 4492 Previous Next Lab Values Notes Calculator Reverse Color Text Zoom
3
4
5
6 A 20-year-old university student comes to the clinic with a 2-month history of diarrhea. The
7 patient has had 4-6 watery bowel movements a day associated with occasional tenesmus, fecal
8 urgency, and abdominal cramps He has also had intermittent bright red blood per rectum over
9 the last 2 weeks. His appetite and energy levels are normal and his weight is stable. The patient
10 is otherwise healthy , and his fam ily history is unremarkable. He has not recently used antibiotics
11 or traveled outside the country The patient does not use tobacco, alcohol, or illicit drugs
12
Sigmoidoscopy demonstrates mild erythema invoMng the rectum and distal sigmoid colon, and
13
14 rectal biopsy confirms mucosal inflammation and crypt abscesses. Besides normal
15 age-appropriate screening, this patient will require future screening for which of the following
16 disease complications?
17
18
0 A. Colorectal carcinoma
19
20 0 B. Nonmelanoma skin cancer
21
0 C. Perianal fistula
22
23 6 D. Sclerosing cholangitis
24 0 E. Toxic megacolon
25
26 6 F. Uveitis
27
28

--
29

31
32
33
34
Submit

35
36
37
38
39
40
2
Item: 30 of 40
0. ld : 4492
~'?Mark <]
Previous
C>
Next
a
Lab Values
~
Notes
~
Calculator
,
Reverse Color
~
Text Zoom
3
4
5
6 Ulcerative colitis
7
8
9 • Bloody diarrhea
Symptoms
10 • Weight loss, fever
11
12
13 • Erythema, friable mucosa
14 Endoscopic • Pseudopolyps
15 findings • Involvement of rectosigmoid
16 • Continuous colonic involvement (no skip lesions)
17
18
19 • Mucosal & submucosal inflammation
Biopsy
20 • Crypt abscesses
21
22
23 • Toxic megacolon
24 • Primary sclerosing cholangitis
25 Complications • Colorectal cancer
26 • Erythema nodosum, pyoderma gangrenosum
27 • Spondyloarthritis
28

--
29

31
32
33
34
@UWortd

Ulcerative colitis (UC) typically presents with persistent diarrhea and gross rectal bleeding, often
with systemic symptoms such as weight loss and fever. It is slightly more frequent in males and
individuals of Ashkenazi Jewish descent, with a peak incidence at age 15-25. UC invariably
involves the rectosigmoid, and severe cases can have continuous involvement of the entire
35
36 colon. Colonoscopy in UC will show erythematous, friable mucosa, with biopsy characterized by
37 mucosal inflammation and crypt abscesses. Crohn disease limited to the colon (Crohn colitis)
38 can present with similar clinical findings but can be differentiated from UC by the presence of
39 skip lesions (discontinuous areas of inflammation), noncaseating granulomas, and transmural
40
2
Item: 30 of 40 ~'?Mark <] C> !I ~ ~ , ~
0. ld : 4492 Previous Next Lab Values Notes Calculator Reverse Color Text Zoom
3 I • I I
4
can present with similar clinical findings but can be differentiated from UC by the presence of
5
6 skip lesions (discontinuous areas of inflammation), noncaseating granulomas, and transmural
7 inflammation.
8
9 Common extracolonic manifestations of UC include erythema nodosum and pyoderma
10 gangrenosum in the skin, episcleritis, spondyloarthritis, and sclerosing cholangitis. In addition,
11 the incidence of colorectal cancer is increased in patients with UC, with the risk proportionate to
12 the duration and extent of disease. As a result, surveillance colonoscopy is advised in patients
13 with UC even in the absence of symptoms (typically beginning 8-10 years after the initial
14 diagnosis)
15
16 {Choice B) The risk of melanoma is increased in inflammatory bowel disease, and the risk of
17 nonmelanoma skin cancer may also be elevated in patients treated with certain
18
immunosuppressive agents However, a dedicated surveillance program is not currently advised.
19
20 (Choice C) Perianal fistula is a common complication of Crohn disease but is not typical in UC.
21
22 {Choice D) Patients with UC are at inc reased risk for primary sclerosing cholangitis (PSG) An
23 elevated serum alkaline phosphatase level in a patient with UC should raise suspicion for PSG,
24
but routine surveillance is not currently recommended.
25
26 (Choice E) Toxic megacolon is a severe complication of UC that can lead to peritonitis and
27
death, but no form of regular surveillance has been shown to prevent this compl ication.
28

--
29

31
32
33
34
{Choice F) Uveitis is an extraintestinal manifestation of UC but does not necessitate regular
surve illance.

Educational objective:
Ulce rative col itis (UC) presents with bloody diarrhea, often with weight loss and fever. UC
35 inva riably involves the rectosigmoid but can have continuous involvement of the entire colon.
36 Biopsy shows mucosal inflammation and crypt abscesses. The risk of colo rectal cancer is
37 elevated in UC, and surveillance colonoscopy is advised.
38
39 I I I I
40

~ Feedback SuWend EnQock


2
Item: 3 1 of40 ~'?Mark <J C> 61 ~ ~ , ~
0. ld : 4074 Previous Next Lab Values Notes Calculator Reverse Color Text Zoom
3
4
5
6 A 25-year-old man comes to the emergency department due to a 3-month history of abdominal
7 pain and diarrhea. Bowel movements occur 3 or 4 times a day , and he has occasionally noticed
8 blood in his stool. The patient has no fever but displays "low energy " He was diagnosed with
9 anterior uveitis 3 years ago but has no other medical problems and takes no medications. His
10 parents and 2 siblings are alive and healthy . Vital signs are normal. Abdomen is tender and
11 bowel sounds are hype ractive The perianal area appears normal. Laboratory results show
12
anemia and an elevated erythrocyte sedimentation rate. W hich of the following features would
13
14 most favo r the diagnosis of Grahn disease compared with ulcerative colitis?
15
16 6 A Absence of perianal abnormalities
17
18 e') B. Continuous inflammation on colonoscopy
19 6 C. History of bloody diarrhea
20
21 e') D. Inflammation in the rectum on colonoscopy
22 6 E. Inflammatory pseudopolyps on colonoscopy
23
24 e') F. Noncaseating granulomas on biopsy
25
26
27 Submit
28
29

--
30

32
33
34
35
36
37
38
39
40
2
Item: 3 1 of40 ~'?Mark <] C> !I ~ ~ , ~
0. ld : 4074 Previous Next Lab Values Notes Calculator Reverse Color Text Zoom
3
4
5
6 Inflammatory bowel disease is characterized by idiopathic and chronic gastro intestinal (GI)
7 inflammation, and the 2 major types are ulcerative colitis (UC) and Crohn disease (CD)
8
9 Several features are more suggestive of CD than UC and are helpful in diffe rentiating between the
10 2 conditions:
11
12
• Involvement of multiple portions of the Gl tract (extending from the mouth to the anus)
13 • Presence of noncaseating granulomas (up to 30% of patients with CD)
14 • Rectal sparing (Choice D)
15 • Fistula formation
16
17 Other characteristic features of CD are transmural involvement of the colon, skip lesions
18
(disease-free regions of Gl tract), cobblestone appearance of the colon, fissures, abscesses,
19
and perianal disease (Choice A) Conversely, UC is more commonly characterized by bloody
20
21 diarrhea and continuous involvement of the rectum and colon (Choices Band C)
22
23
Endoscopy is central to diagnosis of both CD and UC and frequently helpful in distinguishing
24 them from each other. Initial management of both CD and UC involves administration of
25 5-aminosalicylic acids and, frequently, topical corticosteroids. Maintenance therapy for both
26 diseases may involve azathioprine or antitumor necrosis factors.
27
28 (Choice E) Pseudopolyps are growths due to chronic inflammatory disease. These are more
29 commonly seen in UC but frequently seen in CD.

--
30

32
33
34
35
Educational objective:
A number of features are helpful in diffe rentiating ulcerative colitis (UC) and Crohn disease (CD),
including involvement of multiple portions of the gastrointestinal tract, rectal sparing, presence of
noncaseating granulomas, and fistula formation, all of which are more suggestive of CD than UC.
36
References:
37
38 1. Diagnosis and management of Crohn's disease.
39
40
2
Item: 32 of40 ~'?Mark <J C> 61 ~ ~ , ~
0. ld : 3087 Previous Next Lab Values Notes Calculator Reverse Color Text Zoom
3
4
5
6 A 42-year-old man who recently emigrated from Northern Africa comes to the clinic due to a
7 1-month history of abdominal pain and watery diarrhea. He has also had a skin rash for the last
8 2 months that gets worse with sun exposure In addition, the patient has felt depressed recently
9 and has loss of appetite with mild weight loss. Medical history is notable for latent tuberculosis,
10 for which he is currently taking isoniazid and pyridoxine The patient takes no other medications
11 and does not use tobacco, alcohol, or illicit drugs He is vegetarian, and his diet consists mostly
12
of corn and other ce real grains. On examination, there is a pigmented scaly skin rash in the
13
14 malar distribution of his face , on his neck, and on the back of his hands. He also has mild,
15 diffuse abdominal tenderness. Neurologic examination including distal sensation, and gait is
16 normal. Liver function tests are normal. W hich of the following is the most likely diagnosis?
17
18
0 A. Acute intermittent porphyria
19
20 0 B. Isoniazid hypersensitivity
21
0 C. Pellagra
22
23 6 D. Seborrheic dermatitis
24
0 E. Systemic lupus erythematosus
25
26 6 F. Ulcerative col itis
27
28
29 Submit
30

-
31

33
34
35
36
37
38
39
40
Item: 32 of40 ~'?Mark <] C> !I ~ ~ , ~
2
3
4
.. . .
0. ld : 3087 Previous Next alues Notes Calculator Reverse Color Text Zoom

5
6
7
8
9
10
11
12
13
14
15
16
17
18
19
20
21
22
23
24
25
26
27
28
29
30

-
31

33
34
35
36 Pellagra ("rough skin" in Italian vernacular) is due to niacin deficiency and is characterized by
37 the "3 Ds" dermatitis, diarrhea, and dementia
38
39 • Dermatitis is primarily on sun-exposed areas of the body and is characterized by rough ,
40
2
3
4
5
6
7
8
9
Pellagra ("rough skin" in Italian vernacular) is due to niacin deficiency and is characterized by
10
11
the "3 Ds": dermatitis, diarrhea, and dementia:
12 • Dermatitis is primarily on sun-exposed areas of the body and is characterized by rough,
13
hyperpigmented, scaly skin.
14
• Diarrhea is often associated with abdominal pain, nausea, and loss of appetite.
15
16 • Dementia is due to neuronal degeneration in the brain and spinal cord and can lead to
17 memory loss, affective symptoms (eg, depressed mood in this patient), and psychosis.
18
19 Niacin is present in a broad variety of foods and can be synthesized endogenously from
20 tryptophan. In developing countries, niacin deficiency is seen in populations that subsist
21 primarily on corn products (niacin in corn occurs in a bound, unabsorbable form) . In developed
22
countries, it is primarily seen in patients with impaired nutritional intake (eg, alcoholism, chronic
23
24 illness). Pellagra can also be seen occasionally in those with carcinoid syndrome (due to
25 depletion of tryptophan) or Hartnup disease (congenital disorder of tryptophan absorption).
26 Prolonged isoniazid therapy can interfere with metabolism of tryptophan and occasionally lead to
27 pellagra .
28
29 (Choice A) Acute intermittent porphyria (AlP) causes abdominal pain, vom iting, and diarrhea,
30 often with neurologic symptoms (eg, agitation, paresthesias, confusion). Although A lP may be

-
31

33
34
35
36
triggered by isoniazid, the symptoms are episodic rather than chronic, chronic transaminase
elevation is common, and it is more frequent in women than in men.

(Choice B) Isoniazid hypersensitivity can present as hives (maculopapular rash) with pruritus,
fever, and hepatitis. This patient's chronic scaly rash is not consistent with a hypersensitivity
rash .
37
38 (Choice D) Seborrheic dermatitis is characterized by erythematous, scaly plaques affecting the
39
,. I ,. .. I : ,. I I I '" I I I t "' '" "' • I t... .. I .. :,.1 • I I '" II '" I .. '" I • .. • I t I
40
~ -------- - ----------
Feedback
-
SuW.nd EnQ ock
.
2
Item: 32 of40 ~'?Mark <] C> !I ~ ~ , ~
0. ld : 3087 Previous Next Lab Values Notes Calculator Reverse Color Text Zoom
3
4 depletion of tryptophan) or Hartnup disease (congenital disorder of tryptophan absorption)
5 Prolonged isoniazid therapy can interfere with metabolism of tryptophan and occasionally lead to
6 pellagra.
7
8 {Choice A) Acute intermittent porphyria (AlP) causes abdominal pain, vom iting, and diarrhea,
9 often with neurologic symptoms (eg, agitation, paresthesias, confusion). Although A lP may be
10 triggered by isoniazid, the symptoms are episodic rather than chronic , chronic transaminase
11
elevation is common, and it is more frequent in women than in men.
12
13 (Choice B) Isoniazid hypersensitivity can present as hives (maculopapular rash) with pruritus,
14
fever , and hepatitis This patient's chronic scaly rash is not consistent with a hypersensitivity
15
16 rash.
17
{Choice D) Seborrheic dermatitis is characterized by erythematous, scaly plaques affecting the
18
19 scalp, face , chest, and intertriginous areas. It can be associated with dementia (eg, Parkinson
20 disease). However, the hands are not typically affected, and it does not cause gastrointestinal
21 symptoms.
22
23 (Choice E) Systemic lupus erythematosus causes a photosensitive rash in a malar distribution.
24 Central nervous system manifestations (eg, psychosis) can occur. Isoniazid can cause
25 drug-induced lupus (eg, positive antinuclear antibodies) although rash is uncommon. W ith lupus,
26 gastrointestinal involvement typically causes impaired motility, and diarrhea is not typical. In
27 addition, this patient's diet makes pellagra more likely
28
29 (Choice F) Ulcerative colitis (UC) causes bloody diarrhea as opposed to the watery diarrhea
30

-
seen with pellagra. Although UC can have extraintestinal manifestations, associated skin findings
31 include erythema nodosum and pyoderma gangrenosum, not scaly dermatitis.
33 Educational objective:
34
Pellagra is due to niacin deficiency and is characterized by the "3 Ds": dermatitis, diarrhea, and
35
dementia. Prolonged isoniazid therapy can interfere with niacin metabolism and occasionally
36
37 cause pellagra
38
39 Time Spent 3 seconds Copyright © UWorld Last updated: [09/ 13/2016)
40
2
Item: 33 of40 ~'?Mark <] C> !I ~ ~ , ~
0. ld : 2627 Previous Next Lab Values Notes Calculator Reverse Color Text Zoom
3
4
5
6 A 62-year-old woman comes to the physician complaining of 6-8 weeks of fatigue and yellowish
7 skin discoloration. She feels tired, has lost 4.5 kg (10 lb), and has occasional mild nausea. She
8 has no change in bowel habits or abdominal pain The patient had a colposcopy for an abnormal
9 Pap smear 15 years ago Her last mammogram 6 months ago was normal. She is a nonsmoker
10 and drinks 2 or 3 beers each night The patient's father has hype rtension and hyperlipidemia and
11 had a stroke. Her temperature is 36.7 C (98 F), blood pressure is 110/70 mm Hg, pulse is
12
88/min, and respirations are 14/min. There is marked scleral icterus. Abdominal examination
13
14 shows normal bowel sounds and no organomegaly No occult blood is detected in her stool.
15 Laboratory results are as follows
16 6.1 g/dl
Total protein
17
18 Albumin 3.9 g/dl
19
20 Aspartate
67 U/L
21 aminotransferase
22
23 Alanine aminotransferase 52 U/L
24
25 Alkaline phosphatase 890 U/L
26 Total bilirubin 14.6 mg/dl
27
28 Direct bilirubin 10.9 mg/dl
29
30 46 U/L (normal <95
Serum lipase
31 U/L)

-
32

34
35
36
37
Anti-mitochondrial
antibodies
Negative

W hich of the following is the most likely cause of this patient's condition?
38
39
40
2
Item: 33 of40 ~'?Mark <J C> 61 ~ ~ , ~
0. ld : 2627 Previous Next Lab Values Notes Calculator Reverse Color Text Zoom
3 . .

4 Laboratory results are as follows


5
Total protein 6.1 g/dl
6
7 Albumin 3.9 g/dl
8
9 Aspartate
10 67 U/L
aminotransferase
11
12 Alanine aminotransferase 52 U/L
13
14 Alkaline phosphatase 890 U/ L
15
Total bilirubin 14.6 mg/dl
16
17 Direct bilirubin 10.9 mg/dl
18
19 46 U/L (normal <95
20 Serum lipase
U/L)
21
22 Anti-mitochondrial
Negative
23 antibodies
24
25
26 W hich of the following is the most likely cause of this patient's condition?
27
28 ® A Acute choledocholithiasis
29
30 ® B. Chronic autoimmune hepatitis
31 ® C. Chronic pancreatitis

-
32

34
35
36
37
® D. Hemochromatosis
® E. Malignant biliary obstruction
® F. Primary biliary cirrhosis

38 Submit
39
40
2
Item: 33 of40
0. ld : 2627
~'?M
ar
k <]
Previous
C>
Next
a
Lab Values
~
Notes
~
Calculator
,
Reverse Color
~
Text Zoom
3
4
5 Explanation:
6
7
8 Approach to hyperbilirubinemia In adults
9
10 Possible causes
11 Overproduction (eg, hemolysis)
12 Mainly Reduced uptake (eg, drugs,
13 Hyperbilirubinemia 1----'-----
unconjugated portosystemic shunt)
14
15 Mainly Conjugation defect (eg,
conjugated Gilbert's syndrome)
16
17
18 Evaluate liver
19 enzyme pattern
20
21
22
23
24
25 Predominantly elevated Normal AST, ALT, alkaline Predominantly elevated
26 AST&ALT phosphatase alkaline phosphatase
27 .Viral hepatitis . Dubin-Johnson syndrome • Cholestasis of pregnancy
28 . Autoimmune hepatitis • Rotor's syndrome • Malignancy
29
30
. Toxin/drug-related (eg, pancreas, ampullary)
31 hepatitis . Cholangiocarcinoma
. Hemochromatosis
-
32

34
35
36
37
. Ischemic hepatitis
. Alcoholic hepatitis
. Primary biliary cirrhosis
. Primary sclerosing cholangitis
. Choledocholithiasis

38
39
40
2
Item: 33 of40 ~'?Mark <] C> !J ~ ~ , ~
0. ld : 2627 Previous Next Lab Values Notes Calculator Reverse Color Text Zoom
3
4
5 • Abdominal imaging
6 (ultrasound or CT)
7 AlT =alanine transaminase: AST =aspartate transamnase. • Antimitochondrial antibody
8 © UWorld
9
10 This patient presents with predominantly conjugated (direct) hyperbilirubinemia Predominantly
11 unconjugated hyperbilirubinemia is due to inc reased bilirubin production (eg, hemolysis),
12 decreased bilirubin uptake by the live r (eg, portosystemic shunt), or abnormal bilirubin
13 conjugation (eg, Gilbert's syndrome) Predominant conjugated hype rbilirubinemia is due to
14 hepatocellular injury, decreased bilirubin excretion in bile canaliculi (eg, Dubio-Johnson
15
syndrome) , intrahepatic cholestasis (eg, primary biliary cirrhosis), or extrahepatic cholestasis
16
from biliary obstruction ( eg, malignancy)
17
18
The live r enzyme pattern can help differentiate the etiology of conjugated hyperbilirubinemia
19
20 Normal transaminases (aspartate aminotransferase, alanine aminotransferase) and alkaline
21 phosphatase suggest inherited bilirubin metabolism disorders (eg, Dubio-Johnson syndrome).
22 Predominantly elevated transaminases with normal alkaline phosphatase favor intrinsic live r
23 disease (eg, viral hepatitis, hemochromatosis). Elevated alkaline phosphatase out of proportion to
24 the transaminases suggests intrahepatic cholestasis or biliary obstruction.
25
26
27
28
29
30 This patient's conjugated hype rbilirubinemia, elevated alkal ine phosphatase, painless jaundice,
31 and systemic symptoms (fatigue, weight loss) suggest malignant obstruction of the biliary

-
32

34
35
36
37
system Likely etiologies include pancreatic adenocarcinoma or cholangiocarcinoma The next
step in this patient's evaluation should be abdominal imaging (ultrasound or computed
tomography), with consideration for endoscopic retrograde cholangiopancreatography if initial
imaging is nondiagnostic

(Choice A) Acute choledocholithiasis can present similarly , with conjugated hype rbilirubinemia
38
and markedly elevated alkaline phosphatase However , patients typically have acute-onset right
39
40 .. .. .. . .. . ... .. .. . ... . .. .. .. .. .. ... . ... .. . .. .. .. .. . . .. . .
~ Feedback SuWend EnQ ock
2
Item: 33 of40 ~'?Mark <] C> !I ~ ~ , ~
0. ld : 2627 Previous Next Lab Values Notes Calculator Reverse Color Text Zoom
3 "' - ., "' - oil - - .e o~ • .., - .,. • • • • .., - ., e . , - . . .. I-

4 choledocholithiasis, the pain is more chronic and typically worsens in the supine position and at
5 night
6
7 {Choice B) Chronic autoimmune hepatitis usually causes elevated aminotransferases and
8 conjugated hype rbilirubinemia However , it is usually not associated with significantly elevated
9
alkaline phosphatase.
10
11
(Choice C) Chronic pancreatitis typically presents with recurrent bouts of abdominal pain and/or
12
symptoms of fat malabsorption with steatorrhea. Liver function tests are usually normal in chronic
13
14 pancreatitis Recurrent episodes of panc reatitis can lead to fibrosis, stricture of the
15 intrapancreatic portion of the bile duct, and elevated bilirubin and alkaline phosphatase However,
16 this usually develops later in patients with known chronic panc reatitis
17
18 {Choice D) Intrinsic liver disorders (eg, sarcoidosis, hemochromatosis, hepatic malignancy)
19 usually cause hepatocellular injury with predominantly elevated transaminases. Hemochromatosis
20 is also associated with diabetes mellitus, erectile dysfunction, skin pigmentation, arth ropathy , and
21 inc reased risk of cirrhosis and hepatocellular carcinoma. However , bilirubin and alkaline
22 phosphatase levels are usually normal or mildly elevated without clinically obvious jaundice
23
24 (Choice F) Primary biliary c irrhosis may present with fatigue, itching, jaundice, weight loss, and
25 elevated alkaline phosphatase levels. However, antimitochondrial antibodies would be positive
26
27 Educational objective:
28 Painless jaundice in a patient with conjugated hyperbilirubinemia and markedly elevated alkaline
29
phosphatase should raise conce rn fo r biliary obstruction due to pancreatic or biliary cancer.
30
31 Other common causes of biliary obstruction include choledocholithiasis and benign biliary

-
32 strictures.

34 References:
35
36 1. Malignant biliary tract obstruction: evaluation and therapy.
37
38
39 Time Spent 4 seconds Copyright © UWorld Last updated: [07/27/2016)
40
2
Item: 34 of40 ~'?Mark <J C> 61 ~ ~ , ~
0. ld : 2625 Previous Next Lab Values Notes Calculator Reverse Color Text Zoom
3
4
5
6 A 52-year-old woman comes to the physician with symptoms of a "pounding" sensation in her
7 neck and lower extremity edema. She has lost weight recently Her other medical problems
8 include migraine disorder, chronic diarrhea, and severe post-menopausal flushes. The patient
9 has a 30-pack-year smoking history. Physical examination shows poor oral hyg iene A 2/6
10 murmur is heard at the lower sternal border that increases with inspiration Echocardiography
11 shows retracted and immobile tricuspid valve leaflets with poor coaptation and severe tricuspid
12
insuffic iency W hich of the following is the most likely diagnosis?
13
14
15 e A Carcinoid syndrome
16
17 e B. Chronic thromboembolic disease
18 e C. Infective endocarditis
19
20 e D. Myxomatous valve disease
21
22
e E. Primary pulmonary hype rtension
23 e F. Systemic lupus erythematosus
24
25
26 Submit
27
28
29
30
31
32

--
33

35
36
37
38
39
40
2
Item: 34 of40
0. ld : 2625
~'?Mark <]
Previous
C>
Next
a
Lab Values
~
Notes
~
Calculator
,
Reverse Color
~
Text Zoom
3
4
5 Explanation:
6
7
8
9 Features of carcinoid syndrome
10
11
12 • Skin: flushing, telangiectasias, cyanosis
13 • Gastrointestinal: diarrhea, cramping
14
Clinical • Cardiac: valvular lesions (right> left side)
15
manifestations • Pulmonary: bronchospasm
16
17 • Miscellaneous: Niacin deficiency (dermatitis,
18 diarrhea & dementia)
19
20
21 • Elevated 24-hour urinary excretion of 5-HIAA
22 • CT/MRI of abdomen & pelvis to localize tumor
23 Diagnosis
24 • OctreoScan to detect metastases
25 • Echocardiogram (if symptoms of carcinoid
26 heart disease are present)
27
28
29 • Octreotide for symptomatic patients & prior to
30 Treatment surgery/anesthesia
31
• Surgery for liver metastases
32

--
33

35
36
37
38
@UWorld

This patient's presentation- episodic pounding sensation (due to flushing and associated rise in
pulse rate), chronic diarrhea, weight loss, and valvular heart disease with tricuspid regurgitation -
is consistent with carcinoid syndrome Carcinoids are well-differentiated neuroendocrine tumors
found most commonly in the distal small intestine, proximal colon, and lung, with a strong
39
I ll'"'l .: I II '"' .,. "' I • I'"' '"' Ill .,J '"' '"' . . . . '"' ... I II I I II
40

~ Feedback SuWend EnQock


2
Item: 34 of40 ~'?Mark <] C> !I ~ ~ , ~
0. ld : 2625 Previous Next Lab Values Notes Calculator Reverse Color Text Zoom
3 -·-~-·-··· -· -·- ····- ------ ··--------
4 is consistent with carcinoid syndrome Carcinoids are well-diffe rentiated neuroendocrine tumors
5 found most commonly in the distal small intestine, proximal colon, and lung, with a strong
6 propensity fo r metastasis to live r. These tumors can secrete several products including
7 hi stamine, serotonin, and vasoactive intestinal peptide that are metabolized in the liver. In patients
8
with liver metastasis, these hormones are released directly into the systemic circulation, leading
9
10 to ca rcinoid syndrome
11
12 Episodic flushing is the hallmark of ca rcinoid syndrome and occurs in almost 85% of
13 patients Severe flushing may be associated with hypotension and tachycardia. Secr etory
14 diarr hea may be accompanied by abdominal cramping. Other common features include
15 cutaneous telangiectasias, bronchospasm, and tricuspid r egurgitation Pathognomonic
16 plaque-like deposits of fibrous tissue occur most commonly on the endocardium on the right side
17 of the heart, leading to tricuspid regurgitation and right-sided heart failure. Elevated 24-hour
18 urinary 5-hydroxyindoleacetic acid can confirm the diagnosis in most patients
19
20 {Choice B) Patients with chronic thromboembolic disease and recurrent pulmonary emboli
21 typically have symptoms of progressive dyspnea and decreased exercise tolerance. There is
22 usually a history of deep venous thrombosis or a prior thromboembolic event
23
24 (Choice C) Right-sided infective endocarditis (IE) is often associated with intravenous drug
25
use. Patients with IE typically have constitutional symptoms (fever, malaise, and arthralgias),
26
vascular or immunologic phenomena, and a new valvular regurgitant murmur. However, flushing
27
28 and chronic diarrhea are not symptoms of IE.
29
30 (Choice D) Myxomatous valve disease occurs due to weakening of the connective tissue of
31 valvular structures. It most commonly affects the mitral valve, resulting in mitral valve
32 prolapse Although the tricuspid valve may be affected by myxomatous degeneration, symptoms

--
33

35
36
37
38
of fl ushing and diarrhea are more suggestive of carc inoid syndrome

(Choice E) Patients with primary pulmonary hypertension typ ically have exertional symptoms
including chest pain, dyspnea, decreased exerc ise tolerance, and/or syncope These patients do
not have any associated fl ushing, weight loss, or diarrhea.

39 (Choice F) Systemic lupus erythematosus (SLE) typically has skin (butterfly rash and
40
2
Item: 34 of40 ~'?Mark <] C> !I ~ ~ , ~
0. ld : 2625 Previous Next Lab Values Notes Calculator Reverse Color Text Zoom
3
4 (Choice B) Patients with chronic thromboembolic disease and recurrent pulmonary emboli
5 typ ically have symptoms of progressive dyspnea and decreased exercise tolerance. There is
6
usually a history of deep venous th rombosis or a prior thromboembolic event
7
8 (Choice C) Right-sided infective endocarditis (IE) is often associated with intravenous drug
9
use. Patients with IE typically have constitutional symptoms (feve r, malaise, and arthralgias),
10
11 vascular or immunologic phenomena, and a new valvular regurgitant murmur. However, flushing
12 and chronic diarrhea are not symptoms of IE.
13
14 (Choice D) Myxomatous valve disease occurs due to weakening of the connective tissue of
15 valvular structures. It most commonly affects the mitral valve, resulting in mitral valve
16 prolapse Although the tricuspid valve may be affected by myxomatous degeneration, symptoms
17 of flushing and diarrhea are more suggestive of carcinoid syndrome.
18
19 (Choice E) Patients with primary pulmonary hypertension typically have exertional symptoms
20 including chest pain, dyspnea, decreased exercise tolerance, and/or syncope These patients do
21 not have any associated fl ushing, we ight loss, or diarrhea.
22
23 (Choice F) Systemic lupus erythematosus (SLE) typically has skin (butterfly rash and
24 photosensitivity), musculoskeletal (arthralgias), hematologic (cytopenias), cardiac (perica rditis),
25 renal, and neurologic manifestations (cognitive defects, seizures) However, isolated tricuspid
26 regurgitation is not a frequent presentation of SLE.
27
28 Educational objective:
29
Carcinoid syndrome typically presents with episodic flushing, secretory diarrhea, wheezing, and
30
31 murmur of tricuspid regurgitation Elevated 24-hour urinary 5-hydroxyindoleacetic acid can
32 confi rm the diagnosis in most patients

--
33

35
36
37
38
References:
1. Carcinoid syndrome: diagnosis and medical management.

39 Time Spent 2 seconds Copyright © UWorld Last updated: [09/26/2016)


40
2
Item: 35 of 40 ~'?Mark <J C> 61 ~ ~ , ~
0. ld : 2950 Previous Next Lab Values Notes Calculator Reverse Color Text Zoom
3
4
5
6 A 45-year-old woman is noted to have abnormal liver chemistries. She has no symptoms The
7 patient has hype rcholesterolemia and her only medication is a statio. She lives with her husband
8 and 3 children and works in a grocery store. Temperature is 37 C (98.6 F) , blood pressure is
9 118/72 mm Hg, pulse is 68/min, and respirations are 12/min. BMI is 25 kg/m2 Physical
10 examination reveals no abnormalities. Laboratory results are as follows
11
12 Albumin 4.2 g/dl
13
14 Total bilirubin 1.0 mg/dl
15
Alkaline phosphatase 248 U/L
16
17 Aspartate aminotransferase (AST,
18 32 U/L
SGOT)
19
20 Alanine aminotransferase (ALT, SGPT) 28 U/L
21 -
22
23
24 Antinuclear antibody negative
25
Anti-mitochondrial antibody positive in high titers
26
27
28 Right upper quadrant ultrasonogram shows no abnormalities. Afte r confi rmation of the diagnosis,
29 which of the following is the best next step in management of this patient?
30
31
32 6 A Discontinue statio and repeat testing
33 6 B. Measure serum alpha-fetoprotein level

--
34

36
37
38
39
e; C. Refer for liver transplantation
6 D. Start glucocorticoids
6 E. Start ursodeoxycholic acid

40
2
Item: 35 of 40
0. ld : 2950
~'?Mark <]
Previous
C>
Next
a
Lab Values
~
Notes
~
Calculator
,
Reverse Color
~
Text Zoom
3
4
5 Explanation:
6
7
8
9
Evaluation of elevated alkaline phosphatase
10
11 Elevated serum
12 alkaline phosphatase
13
14
15 Normal Alkaline phosphatase
CheckGGT
16 GGT likely of bone origin
17
Elevated GGT
18
19
20 Alkaline phosphate
likely of biliary origin
21
22
23
24 AMA positive
OR CheckRUQ Dilated bile
25
Abnormal hepatic parenchyma on ultrasound & AMA ducts
26 ultrasound
27
28
Both normal
29
30
31
32
33
I Liver biopsy I Consider liver biopsy,
ERCP, observation
I ERCP I

--
34

36
37
38
39
AMA = antimitochondrial anbbody; ERCP = endoscopic retrograde cholangiopancreatograrn; GGT= garnrna-glutarnyltransferase;
RUQ =right upper quadrant

<i>UWorld

II " II I I II • I " ... I I " '"' ... " ... I " ~~, " I '"' I I I I I " .. ... • I I I II " I ... I " " I " II I " ...
40

~ Feedback SuWend EnQ ock


2
Item: 35 of 40 ~'?Mark <J C> 61 ~ ~ , ~
0. ld : 2950 Previous Next Lab Values Notes Calculator Reverse Color Text Zoom
3
4 <i>UWorld
5
6 This asymptomatic patient has elevated alkaline phosphatase with normal hepatic transaminase
7 (alanine aminotransferase [ALT], aspartate aminotransferase [AST]) levels, normal right upper
8 quadrant ultrasound, and positive antimitochondrial antibody assay This is consistent with
9
early primary biliary cholangitis (PBC, previously primary biliary cirrhosis), a chronic ,
10
11 progressive live r disease characterized by cholestasis with autoimmune destruction of
12 intrahepatic bile ducts. It is most common in middle-aged women and has an insidious onset.
13 Pruritus and fatigue are usually the first symptoms As the disease progresses, jaundice,
14 steatorrhea, hepatomegaly , eyelid xanthelasma, portal hype rtension, and osteopenia may develop
15
16 Ursodeoxycholic acid (UDCA) is used in a number of cholestatic disorders and is the drug of
17 choice in PBC. UDCA is a hydrophilic bile acid that decreases biliary injury by the more
18 hydrophobic endogenous bile acids. It also increases biliary secretion and may have additional
19 anti-inflammatory and immunomodulatory effects. UDCA delays histologic progression in PBC
20 and may improve symptoms and possibly survivaL It should be initiated as soon as the diagnosis
21
is made, even in asymptomatic patients. Treatment is less effective in advanced disease, and
22
many patients will go on to require liver transplantation.
23
24
(Choice A) Statins cause a hepatocellular rather than cholestatic pattern of injury, characterized
25
26 by elevated live r transaminases (ALT, AST) This patient has normal transaminases and an
27 elevated alkaline phosphatase leveL
28
29 (Choice B) Alpha-fetoprotein (AFP) is used to screen for hepatocellular carc inoma in patients
30 with chronic viral hepatitis or cirrhosis.
31
32 {Choice C) Live r transplantation is the definitive cure for progressive PBC but is indicated only
33 in those with severe liver damage or cirrhosis. As this patient has no signs or symptoms of

--
34

36
37
38
39
cirrhosis, a trial of UDCA is more appropriate

(Choice D) Glucocorticoids are used for autoimmune hepatitis, which is characterized by


elevated liver transaminases and a positive antinuclear antibody titer. Glucocorticoids are
ineffective in PBC.
40
2
Item: 35 of 40 ~'?Mark <] C> !I ~ ~ , ~
0. ld : 2950 Previous Next Lab Values Notes Calculator Reverse Color Text Zoom
3 • "' • ' " • ' •• - I - . o1 I . - • e - o1 I I . - o1 e - I II I

4 choice in PBC. UDCA is a hydrophilic bile acid that decreases biliary injury by the more
5 hydrophobic endogenous bile acids. It also inc reases biliary secretion and may have additional
6
anti-inflammatory and immunomodulatory effects. UDCA delays histologic progression in PBC
7
and may improve symptoms and possibly survivaL It should be initiated as soon as the diagnosis
8
9 is made, even in asymptomatic patients Treatment is less effective in advanced disease, and
10 many patients will go on to require liver transplantation
11
12 {Choice A) Statins cause a hepatocellular rather than cholestatic pattern of injury, characterized
13 by elevated live r transaminases (A LT, AST). This patient has normal transaminases and an
14 elevated alkaline phosphatase leveL
15
16 (Choice B) Alpha-fetoprotein (AFP) is used to screen for hepatocellular carcinoma in patients
17 with chronic viral hepatitis or cirrhosis.
18
19 (Choice C) Liver transplantation is the definitive cure for progressive PBC but is indicated only
20 in those with severe liver damage or cirrhosis. As this patient has no signs or symptoms of
21 cirrhosis, a trial of UDCA is more appropriate
22
23 {Choice D) Glucocorticoids are used for auto immune hepatitis, which is characterized by
24 elevated liver transaminases and a positive antinuclear antibody titer. Glucocorticoids are
25 ineffective in PBC.
26
27 Educational objective:
28 Ursodeoxycholic acid (UDCA) is the drug of choice in primary biliary cholangitis (PBC) It
29
delays histologic progression and may improve symptoms and possibly survivaL Therefore,
30
31 UDCA should be initiated as soon as the diagnosis of PBC is made. Patients with advanced
32 disease require liver transplantation
33

--
34

36
37
38
39
References:
1. Primary biliary cirrhosis.

Time Spent 3 seconds Copyright © UWorld Last updated: [09/ 11/2016)


40
2
Item: 36 of40 ~'?Mark <J C> 61 ~ ~ , ~
0. ld : 3887 Previous Next Lab Values Notes Calculator Reverse Color Text Zoom
3
4
5
6 A 34-year-old man comes to the office with a 4-week history of diarrhea. He has no blood in the
7 stools. The patient recently returned from a trip to South America , where he developed
8 foul-smelling stools, abdominal cramps, and bloating. A 3-day course of ciprofloxacin did not
9 relieve his symptoms. He has no previous medical problems and has never had similar
10 symptoms. The patient takes no other medication and does not use tobacco, alcohol, or illicit
11 drugs Temperature is 36.8 C (98 2 F), blood pressure is 126/80 mm Hg, and pulse is 86/min.
12
Examination shows a soft, nontender abdomen with increased bowel sounds. Which of the
13
14 following is the most appropriate next step in management?
15
16 ® A Ciprofloxacin fo r 5 more days
17
18 ® B. Colonoscopy with mucosal biopsy
19 ® C. CT scan of the abdomen
20
21 ® D. Stool antigen testing for Clostridium difficile
22
® E. Stool antigen testing for Giardia
23
24 ® F. Upper endoscopy with biopsy of the small bowel
25
26
27 Submit
28
29
30
31
32
33
34

-
35

37
38
39
40
2
Item: 36 of40 ~'?Mark <J C> 61 ~ ~ , ~
0. ld : 3887 Previous Next Lab Values Notes Calculator Reverse Color Text Zoom
3 .., I I ,. I.. .. I I .. I • i e.
4
5 F. Upper endoscopy with biopsy of the small bowel [3% J
6
7
Explanation:
8
9
This patient has typical features of giardiasis, including diar rhea acquired during international
10
travel, abdominal cramps, foul-smelling stools, bloating, and benign findings on abdominal
11
12 examination. Giardia duodena/is (sometimes noted as G Iamblia or G intestinalis) is common in
13 rural areas and developing countries, and has an incubation period of 1-2 weeks. It is most
14 commonly transmitted by contaminated water but can be foodborne or transmitted
15 person-to-person via a fecal-oral route. Most patients are asymptomatic ; however , a significant
16 minority of those who do develop clinical illness may go on to develop chronic giardiasis
17 characterized by malabsorption, weight loss, or persistent gastrointestinal distress.
18
19 The preferred confi rmatory test for giardiasis is a stool antigen assay (direct
20 immunofluorescence or ELISA). Stool microscopy for oocysts and t rophozoites can also
21 identify the organism and is useful in resource-poor settings or if other parasitic organisms are
22
suspected Some facilities also offer a nucleic acid amplification assay Metronidazole is the
23
24 preferred treatment Asymptomatic ca rriers do not usually need treatment
25
(Choice A) A short course of ciprofloxacin is advised for empiric treatment of traveler's diarrhea
26
27 (most commonly due to Escherichia colt) However, the preferred treatment for giardiasis is
28 metronidazole. Diagnostic confi rmation should be obtained prior to additional antibiotic therapy in
29 this patient with a prolonged course of diarrhea who has failed ciprofloxacin
30
31 {Choices 8 and F) Upper endoscopy with small bowel biopsy (to rule out tropical sprue) and
32 colonoscopy would be considered in patients with persistent diarrhea who remain undiagnosed
33 after initial noninvasive testing (eg, microscopy, stool antigen testing)
34

-
35

37
38
39
40
(Choice C) CT imaging is not useful in routine evaluation of travel-associated diarrhea.

(Choice D) Clostridium difficile causes antibiotic-associated colitis. This patient was


symptomatic prior to receiving antibiotics, so C difficile would be less likely

~
-------------------------------------------------------------------------------------------------------------------------------
Feedback SuWend EnQ ock
2
Item: 36 of40 ~'?Mark <] C> !I ~ ~ , ~
0. ld : 3887 Previous Next Lab Values Notes Calculator Reverse Color Text Zoom
3
4 t ravel, abdominal cramps, foul-smelling stools, bloating, and benign findings on abdominal
5 examination. Giardia duodena/is (sometimes noted as G Iamblia or G intestinalis) is common in
6 rural areas and developing countries, and has an incubation period of 1-2 weeks. It is most
7 commonly transmitted by contaminated water but can be foodborne or transmitted
8 person-to-person via a fecal-oral route. Most patients are asymptomatic ; however , a significant
9
minority of those who do develop clinical illness may go on to develop chronic giardiasis
10
characterized by malabsorption, weight loss, or persistent gastrointestinal distress.
11
12
The preferred confi rmatory test for giardiasis is a stool antigen assay (direct
13
14 immunofluorescence or ELISA). Stool microscopy for oocysts and trophozoites can also
15 identify the organism and is useful in resource-poor settings or if other parasitic organisms are
16 suspected Some facilities also offer a nucle ic acid amplification assay Metronidazole is the
17 preferred treatment Asymptomatic ca rriers do not usually need treatment
18
19 (Choice A) A short course of ciprofloxacin is advised for empiric treatment of traveler's diarrhea
20 (most commonly due to Escherichia colt) However , the preferred treatment for giardiasis is
21 metronidazole. Diagnostic confi rmation should be obtained prior to additional antibiotic therapy in
22 this patient with a prolonged course of diarrhea who has failed ciprofloxacin
23
24 {Choices 8 and F) Upper endoscopy with small bowel biopsy (to rule out tropical sprue) and
25 colonoscopy would be considered in patients with persistent diarrhea who remain undiagnosed
26 after initial noninvasive testing (eg, microscopy, stool antigen testing)
27
28 (Choice C) CT imaging is not useful in routine evaluation of travel-associated diarrhea.
29
30 {Choice D) Clostridium difficile causes antibiotic-associated colitis. This patient was
31 symptomatic prior to receiving antibiotics, so C difficile would be less likely
32
33 Educational objective:
34

-
Giardiasis is characterized by persi stent diarrhea and abdominal c ramps, and is most commonly
35
transmitted via water in rural areas and developing countries. The preferred diagnostic test is a
37 stool antigen assay. Metronidazole is the first-line treatment
38
39 Time Spent 2 seconds Copyright © UWorld Last updated: [09/08/2016)
40
2
Item: 37 of40 ~'?Mark <J C> 61 ~ ~ , ~
0. ld : 4188 Previous Next Lab Values Notes Calculator Reverse Color Text Zoom
3
4
5
6 A 65-year-old man comes to the physician with a several month history of difficulty swallowing
7 and frequent coughing during meals. He has noticed a right-sided neck mass which increases in
8 size while drinking fluids. His wife reports that there has been a change in his breath odor , and
9 he occasionally regurgitates medications taken earlier in the day His past medical history is
10 significant for hype rtension, gastroesophageal reflux disease, and right-knee osteoarthritis. His
11
current medications include hydrochlorothiazide, ranitidine, and occasional naproxen as needed.
12
W hich of the following is the most important pathogenic factor in the development of this patient's
13
14
problem?
15
16 6 A Abnormal cellular prolife ration
17
18 e; B. Acid reflux
19 6 C. Inflammation
20
21 e; D. Metabolic abnormalities
22 6 E. Motor dysfunction
23
24
25
Submit
26
27
28
29
30
31
32
33
34
35

-

36

38
39
40
2
Item: 37 of40
0. ld : 4188
~'?Mark <]
Previous
C>
Next
a
Lab Values
~
Notes
~
Calculator
,
Reverse Color
~
Text Zoom
3
4
5 Explanation:
6
7
8
9 Zenker diverticulum
10
11
12 • Usually <::age 60
13
• More common in males
14
15 • Dysphagia
Clinical features
16 • Halitosis
17 • Regurgitation & aspiration
18
• Variable neck mass
19
20
21 • Barium esophagram
22 Diagnosis
23 • Esophageal manometry
24
25
26 • Open/endoscopic surgery
Management
27 • Cricopharyngeal myotomy
28
29 ©USMLEWorld, LLC
30
31 This patient has dysphagia, coughing, regurgitation, halitosis, and a neck mass; these are
32 typical features of Zenke r (pharyngoesophageal) diverticulum (ZD) Patients with ZD are usually
33 over 60 years of age, and characteristically report oropharyngeal dysphagia associated with a
34 neck mass. Depending on food and fl uid intake, the neck mass may vary in size and cause
35 gurgling in the throat. Potential complications include tracheal compression, ulceration with

-

36

38
39
40
bleeding, regurgitation, and pulmonary aspiration

ZD develops immediately above the upper esophageal sphincter with posterior herniation between
th fib f th I h I I I UI I I h I I t h' t d f f d

~ Feedback SuWend EnQock


2
Item: 37 of40 ~'?Mark <] C> !I ~ ~ , ~
0. ld: 4188 Previous Next Lab Values Notes Calculator Reverse Color Text Zoom
3 -·~-- ·-- .. - .. -- -- -- ~-

4
5 ZD develops immediately above the upper esophageal sphincte r with posterior herniation between
6 the fibers of the cricopharyngeal muscle. Upper esophageal sphincter dysfunction and
7 esophageal dysmotility are believed to cause ZD. A barium esophagram is the preferred
8 imaging modality to confirm the diagnosis Surgical treatment includes excision and,
9 frequently , cricopharyngeal myotomy.
10
11 {Choice A) Abnormal cell ular proliferation is the underlying pathophysiologic defect in
12
neoplasia Esophageal cancer may cause dysphagia, but regurgitation and a va riable neck mass
13
are more characteristic of ZD.
14
15
(Choice B) Gastroesophageal reflux disease (GERD) is frequently associated with ZD but is not
16
causative. It is a common cause of dysphagia, often due to esophageal spasm or
17
18 stricture. However, GERD is not typically associated with neck masses.
19
20 {Choice C) External inflammation may cause traction diverticula (usually in the mid-esophageal
21 zone), but this is uncommon.
22
23 {Choice D) Metabolic abnormalities (eg, iron deficiency) may be associated with upper-
24 esophageal webs (Plummer-Vinson syndrome) These patients may have dysphagia and
25 stomatitis. However, this is more common in women than in men, and regurgitation and a neck
26 mass are not typical features.
27
28 Educational objective:
29 Zenker (pharyngoesophageal) diverticulum develops immediately above the upper esophageal
30 sphincter. It occurs due to posterior herniation between the fibers of the cricopharyngeal
31 muscle. Symptoms include dysphagia, halitosis, aspiration, and regurgitation Diagnosis is
32 confi rmed with a barium esophagram and treatment is surgical
33
34
35 References:

-

36

38
39
40
1. Zenker' s diverticula: pathophysiology, clinical presentation, and flexible
endoscopic management.
2
Item: 38 of 40 ~'?Mark <] C> !I ~ ~ , ~
0. ld : 2986 Previous Next Lab Values Notes Calculator Reverse Color Text Zoom
3
4
5
6 A 20-year-old college student is brought to the emergency department after his friend found that
7 he ingested large amounts of acetaminophen in a suicide attempt The patient has been
8 depressed since his girlfriend broke up with him a week ago, and he took approximately 30 pills
9 of 500-mg acetaminophen 2 days ago to end his life . He has nausea and mild abdominal pain
10 His temperature is 36.7 C (98 F), blood pressure is 114/68 mm Hg, and pulse is 94/min.
11 Abdominal examination shows mild tender hepatomegaly. Serum acetaminophen level is high,
12
and the patient is hospitalized for N-acetylcysteine therapy During the hospital stay, he
13
14 becomes markedly confused and incoherent Repeat examination shows scleral icterus and
15 asterixis. Current laboratory results and those obtained at the time of admission are as follows
16
At
17 Current
18 admission -
19 Liver function studies
20
21 4.1
22 Total bilirubin 1.2 mg/dl
mg/dl
23
24 Aspartate
8456 U/L 96 U/L
25 aminotransferase
26
27 Alanine
9634 U/L 70 U/L
28 aminotransferase
29
30
31
Prothrombin time 120 sec 18 sec
32
33 3.5
34 Serum creatinine 1.1 mg/dl
mg/dl
35
36

--

37

39
40
Which of the following is the best next step in management of this patient?
2
Item: 38 of 40 ~'?Mark <J C> 61 ~ ~ , ~
0. ld : 2986 Previous Next Lab Values Notes Calculator Reverse Color Text Zoom
3 I ry I I I I
4
5 At
6 Current
admission
7
8 Liver function studies
9
10 4.1
Total bilirubin 1.2 mg/dl
11 mg/dl
12
13 Aspartate
8456 U/L 96 U/ L
14 aminotransferase
15
16 Alanine
9634 U/L 70 U/L
17 aminotransferase
18
19
20 Prothrombin time 120 sec 18 sec
21
22 3.5
Serum c reatinine 1.1 mg/dl
23 mg/dl
24
25
26 W hich of the following is the best next step in management of this patient?
27
28 ® A Close monitoring only
29
30 ® B. Discontinue N-acetylcysteine
31 ® C. Initiate hemodialys is
32
33 ® D. Perform liver biopsy
34 ® E. Refer to liver transplant center
35
36
® F. Start glucocortico id

--

37

39
40
Submi t
2
Item: 38 of 40 ~'?Mark <] C> !I ~ ~ , ~
0. ld : 2986 Previous Next Lab Values Notes Calculator Reverse Color Text Zoom
3
4
5 Explanation:
6
7 This patient has acute liver failure (ALF) due to acetaminophen toxicity ALF is defined as
8 severe acute liver injury without underlying liver disease and is characterized by elevated
9 aminotransferases (often >1000 U/L), hepatic encephalopathy (HE), and synthetic liver
10 dysfunction (defined as pr olonged prothrombin time [Pl] with INR <::1 5) Approximately only
11 half of patients with ALF will survive without liver transplantation (L T) Reliable indicators of
12
worsening ALF include rising ser um bilirubin and PT, as seen in this patient Acute renal
13
14 insufficiency, likely due to decreased renal perfusion, is common and portends a lower chance
15 of recovery without LT The degree of HE is also of prognostic importance as grade Ill HE
16 (characterized by marked confusion and incoherence, as seen in this patient) is associated with
17 only a 40%-50% chance of spontaneous recovery. Cerebral edema is a potential compl ication
18 of ALF that may lead to coma and brain stem herniation, and is the most common cause of death.
19
20 In A LF due to acetaminophen toxicity , LT is fi rmly indicated in patients with grade Ill or IV HE, PT
21 >100 seconds, and serum c reatinine >3.4 mg/dl (such as this patient). One-year survival
22 following LT fo r A LF is approximately 80%.
23
24 {Choices A and D) Live r biopsy is sometimes helpful in ALF of unclear etiology This patient's
25 ALF is due to acetaminophen toxicity and his condition is worsening Close monitoring alone
26 would not be appropriate, and liver biopsy is unlikely to prevent the need fo r LT
27
28 (Choice B) W hen administered within 8 hours of acetaminophen ingestion, N-acetylcysteine
29 markedly improves the rate of recovery in acetaminophen overdose and may be beneficial in
30
other etiologies of ALF. This patient, who presented 2 days after acetaminophen ingestion, may
31
32 obtain some benefit from N-acetylcysteine and should continue therapy while undergoing
33 evaluation for LT
34
35 {Choice C) Acetaminophen can be cleared by hemodialysis, which is occasionally used to treat
36 acetaminophen toxicity However, initiation of hemodialysis is unlikely to reverse this patient's

--

37

39
40
worsening ALF and should not precede referral for LT

(Choice F) Glucocorticoids are generally not indicated in ALF as they inc rease the risk of
2
Item: 38 of 40 ~'?Mark <] C> !I ~ ~ , ~
0. ld : 2986 Previous Next Lab Values Notes Calculator Reverse Color Text Zoom
3 ..
g pp y :I I
4
5 {Choices A and D) Liver biopsy is sometimes helpful in ALF of unclear etiology This patient's
6
ALF is due to acetaminophen toxicity and his condition is worsening. Close monitoring alone
7
8 would not be appropriate, and live r biopsy is unlikely to prevent the need for LT.
9
(Choice B) W hen administered within 8 hours of acetaminophen ingestion, N-acetylcysteine
10
11 markedly improves the rate of recovery in acetaminophen overdose and may be beneficial in
12 other etiologies of ALF. This patient, who presented 2 days after acetaminophen ingestion, may
13 obtain some benefit from N-acetylcysteine and should continue therapy while undergoing
14 evaluation for LT
15
16 (Choice C) Acetaminophen can be cleared by hemodialysis, which is occasionally used to treat
17 acetaminophen toxicity However , initiation of hemodialysis is unlikely to reverse this patient's
18 worsening ALF and should not precede referral for LT
19
20 (Choice F) Glucocorticoids are generally not indicated in ALF as they increase the risk of
21 infection and have not demonstrated benefit in most etiologies of ALF. They may provide benefit
22 in alcoholic hepatitis and in ALF due to autoimmune hepatitis.
23
24 Educational objective:
25 Liver transplantation should be considered in all patients with acute live r failure and indications
26
that the disease is worsening or failing to improve
27
28
29 References:
30
1. Determinants of outcome among patients with acute liver failure listed for liver
31
32 transplantation in the United States.
33 2. Fulminant hepatic failure : etiology and indications for liver transplantation.
34
3. Acetaminophen-induced acute liver failure : results of a United States
35
36 multicenter, prospective study.

--

37

39
40
Time Spent 5 seconds Copyright © UWorld Last updated: [08/08/2016)
2
Item: 39 of 40 ~'?Mark <] C> !I ~ ~ , ~
0. ld : 2917 Previous Next Lab Values Notes Calculator Reverse Color Text Zoom
3
4
5
6 A 45-year-old African American alcoholic female presents to clinic complaining of cough with
7 hemoptysis, night sweats, and subjective fever . A chest radiograph demonstrates right upper lobe
8 cavity that is consistent with a diagnosis of pulmonary tuberculosis. Sputum staining for acid-fast
9 bacteria is positive. She is begun on a regimen that includes isoniazid, rifamp in, ethambutol, and
10 pyrazinamide She reports having taken oral contraceptive pills for the past five years . Two
11 months later when she returns fo r follow-up examination, her sputum is clear of acid-fast bacilli.
12
However, she now complains of fatigue , malaise, and nausea that began approximately ten days
13
14 ago. Physical examination reveals icteric sclerae and tender hepatomegaly . Her laboratory
15 evaluation returns with the following :
16
17 Liver studies
18 Albumin 3.9 g/dl
19 Total bilirubin 4.8 mg/dl
20 Alkaline phosphatase 98 U/ L
21 Aspartate aminotransferase (SGOT) 372 U/L
22 Alanine aminotransferase (SGPT) 410 U/L
23
24
25 Coagulation studies
26 Prothrombin time 13 sec
27 Partial thromboplastin time 27 sec
28
29
Hepatitis Panel
30
31 HBsAg Negative
32 Anti-HBsAg Negative
33 lgM Anti-HBcAg Negative
34 HBeAg Negative
35 Anti-HBeAg Negative
36 Anti-HAV lgM Negative
37

--
Anti-HAV lgG Negative
38
Anti-HCV Negative
• 40
2
Item: 39 of 40 ~'?Mark <J C> 61 ~ ~ , ~
0. ld : 2917 Previous Next Lab Values Notes Calculator Reverse Color Text Zoom
3 .. I I • I I
4 Total bilirubin 4.8 mg/dl
5
Alkaline phosphatase 98 U/L
6
Aspartate aminotransferase (SGOT) 372 U/L
7
8 Alanine aminotransferase (SGPT) 410 U/L
9
10 Coagulation studies
11 Prothrombin time 13 sec
12
Partial thromboplastin time 27 sec
13
14
15 Hepatitis Panel
16 HBsAg Negative
17 Anti-HBsAg Negative
18 lgM Anti-HBcAg Negative
19 HBeAg Negative
20 Anti-HBeAg Negative
21
Anti-HAV lgM Negative
22
23 Anti-HAV lgG Negative
24 Anti-HCV Negative
25
26 Liver biopsy demonstrates panlobular mononuclear infiltration and hepatic cell necrosis. W hich of
27 the following is the most likely diagnosis?
28
29
30 <0 A Acute alcoholic hepatitis
31 <0 B. Hepatitis secondary to oral contraceptive usage
32
33 <0 C. Hepatitis secondary to isoniazid usage
34 <0 D. Autoimmune hepatitis
35
36
<0 E. Tuberculous hepatitis
37

--

38

40
Submit
2
Item: 39 of 40 ~'?Mark <] C> !I ~ ~ , ~
0. ld : 2917 Previous Next Lab Values Notes Calculator Reverse Color Text Zoom
3
4
5 Explanation:
6
7 This patient's liver function tests and liver biopsy result are consistent with a diagnosis of acute
8 hepatitis However, the negative viral serology results indicate an infectious cause of her hepatitis
9 is unlikely Since this patient is on isoniazid, idiosyncratic liver injury with a histological picture
10 similar to viral hepatitis must be considered. While extrahepatic hypersensitivity manifestations
11
like rash, arthralgias, fever, leukocytosis, and eosinophilia are common in patients with
12
drug-induced liver injury, they are characteristically absent in cases of isoniazid-induced hepatic
13
14 cell injury
15
Drugs and toxins typically cause hepatic injury, either through direct toxic effects or through
16
17 idiosync ratic reactions. The direct toxic effects are dose-dependent and have short latent
18 periods Some examples of direct toxins include carbon tetrachloride, acetaminophen,
19 tetracycline, and substances found in the Amanita phalloides mushroom. Idiosyncratic reactions
20 are not dose-dependent and have variable latent periods. Some examples of pharmacological
21 agents that cause idiosync ratic reactions include isoniazid, chlorpromazine, halothane, and
22 antiretroviral therapy
23
24 Drug-induced liver disease can also be broadly categorized according to morphology 1)
25 cholestasis, which is caused by medications such as chlorpromazine, nitrofurantoin,
26 erythromycin, and anabolic steroids; 2) fatty liver , which is caused by medications such as
27
tetracycline, valproate , and anti-retrovirals; 3) hepatitis, which is caused by medications such as
28
29 halothane, phenytoin, isoniazid, and alpha-methyldopa; 4) toxic or fulminant liver failure, which is
30 caused by medications such as ca rbon tetrachloride and acetaminophen; and 5) granulomatous,
31 which is caused by medications such as allopurinol and phenylbutazone
32
33 (Choice A) The patient's laboratory profile is not suggestive of alcoholic hepatitis Classically, the
34 ASTALT ratio is > 2 in alcoholic hepatitis Moreover , the timeline correlation between the initiation
35 of treatment for tuberculosis and the onset of her hepatic dysfunction mandates close scrutiny of
36 her pharmacotherapy regimen.
37

--

38

40
(Choice B) Oral contraceptives are unusual in that they can cause abnormalities in liver function
tests without evidence of necrosis or fatty change Since this patient's liver biopsy demonstrated
2
Item: 39 of 40 ~'?Mark <] C> !I ~ ~ , ~
0. ld : 2917 Previous Next Lab Values Notes Calculator Reverse Color Text Zoom
3
4 are not dose-dependent and have variable latent periods. Some examples of pharmacological
5 agents that cause idiosyncratic reactions include isoniazid, chlorpromazine, halothane, and
6 antiretroviral therapy.
7
8 Drug-induced liver disease can also be broadly catego rized according to morphology 1)
9 cholestasis, which is caused by medications such as chlorpromazine, nitrofurantoin,
10 erythromycin, and anabolic steroids; 2) fatty live r, which is caused by medications such as
11 tetracycline, valproate, and anti-retrovirals ; 3) hepatitis, which is caused by medications such as
12
halothane, phenytoin, isoniazid, and alpha-methyldopa; 4) toxic or fulminant liver failure, which is
13
14 caused by medications such as ca rbon tetrachloride and acetaminophen; and 5) granulomatous,
15 which is caused by medications such as allopurinol and phenylbutazone
16
17 (Choice A) The patient's laboratory profile is not suggestive of alcoholic hepatitis. Classically, the
18 ASTALT ratio is > 2 in alcoholic hepatitis Moreover, the timeline correlation between the initiation
19 of treatment for tuberculosis and the onset of her hepatic dysfunction mandates close scrutiny of
20 her pharmacotherapy regimen
21
22 {Choice B) Oral contraceptives are unusual in that they can cause abnormalities in live r function
23 tests without evidence of necrosis or fatty change Since this patient's live r biopsy demonstrated
24 necrosis, oral contraceptives are not likely responsible fo r her hepatic dysfunction
25
26 (Choice D) Autoimmune hepatitis should be considered once all possible causes of drug-induced
27 or viral hepatitis have been ruled out Positive antinuclear antibody (ANA) and/or positive smooth
28 muscle antibody suggest the diagnosis of classic (type 1) autoimmune hepatitis
29
30 {Choice E) Tuberculous hepatitis may develop in cases of miliary tuberculosis but would be
31 characterized by granulomas on liver biopsy. Moreover , since this patient had a sputum
32
conversion, it is highly unlikely that she is now suffering from miliary tuberculosis.
33
34
Educational objective:
35
Isoniazid causes idiosyncratic live r injury with histological features similar to those seen in
36
37 patients with viral hepatitis

--

38

40
Time Spent 4 seconds Copyright © UWorld Last updated: [09/25/2016)
2
Item: 40 of 40
0. ld: 2911
~'?Mark <]
Previous
C>
Next
a
Lab Values
~
Notes
~
Calculator
,
Reverse Color
~
Text Zoom
3
4
5
6 A 45-year-old man comes to the emergency department with poor appetite and nausea for the
7 last 3-4 weeks. He admits to drinking vodka daily He attempted to quit on several occasions
8 over the last year but relapsed each time. Two years ago, he was admitted for seizures but has
9 had no medical follow-up since. The patient takes no medications and does not use any illicit
10 drugs. He has had no recent sexual encounters. He has no recent unintentional weight loss. His
11 temperature is 37.2 C (99 F) , blood pressure is 120/70 mm Hg, and pulse is 102/min. His
12
sclerae appear icteric. The live r is enlarged and mildly tender. The gallbladder is not palpable,
13
14 and Murphy's sign is negative. The patient has no abdominal distension, peripheral edema,
15 spider angiomas, gynecomastia, or asterixis. W hich of the following laboratory findings are most
16 likely to be present?
17 Aspa rtate Alanine
18 aminotransfe rase aminotransferase Gamma-glutamyltransferase Ferritin
19
20
21
22 ® A 230 111 i i
23
24 ® B. 2,500 1,050 i i
25
26 ® C. 2,640 2,355 l l
27
28 6 D. 266 105 Normal Normal
29
30 ® E. 32 55 i Normal
31
32
33 Submit
34
35
36
37
38

--
39
2
Item: 40 of 40
0. ld: 2911
~'?Mark <]
Previous
C>
Next
a
Lab Values
~
Notes
~
Calculator
,
Reverse Color
~
Text Zoom
3 - -- - -
4
5
6 Explanation:
7
8
9
10 Clinical features of alcoholic hepatitis
11
12
• Jaundice, anorexia, fever
13
14 • Right upper quadrant &l or epigastric pain
15 Clinical • Abdominal distension due to ascites
16 presentation • Proximal muscle weakness from muscle
17
wasting (if malnourished)
18
19 • Possible hepatic encephalopathy
20
21 • Elevated AST & ALT, usually <300 U/L
22
• AST:ALT ratio '2::2
23
24 Laboratory/ • Elevated gamma-glutamyllransferase, bilirubin,
25 imaging studies &lor international normalized ratio
26 • Leukocytosis, predominantly neutrophils
27
• Decreased albumin if malnourished
28
29 • Abdominal imaging may show fatty liver
30 0 UW0<1d
31
32 This patient, who has an extensive history of alcohol use, most likely has alcoholic hepatitis with
33 jaundice, anorexia, and tender hepatomegaly There is no evidence of ascites or cirrhosis (eg,
34
spider angiomas, gynecomastia, asterixis) He has no weight loss or systemic symptoms to
35
36 suggest malignancy, and there are no signs of infection.
37
Alcoholic liver disease is generally characterized by modest elevations in aspartate
38
.. .
--
aminotransferase (AST) and alanine aminotransferase (ALT), usually <300 IU/L and almost
39
. ... .. .. ' . ..
... ... ... . .. .
... ... .... .. . . ..
~ Feedback SuWend EnQ ock
2
Item : 40 of 40 ~'?Mark <J C> 61 ~ ~ , ~
0. ld: 2911 Previous Next Lab Values Notes Calculator Reverse Color Text Zoom
3
4
5 0 UW0<1d
6
7 This patient, who has an extensive history of alcohol use, most likely has alcoholic hepatitis with
8 jaundice, anorexia, and tender hepatomegaly There is no evidence of ascites or cirrhosis (eg,
9
spider angiomas, gynecomastia, asterixis) He has no weight loss or systemic symptoms to
10
11 suggest malignancy, and there are no signs of infection.
12
13
Alcoholic live r disease is generally characterized by modest elevations in aspartate
14 aminotransferase (AST) and alanine aminotransferase (A LT), usually <300 IU/L and almost
15 always <500 IU/L A ratio of AST to AL T >2 (thought to be due to hepatic deficiency of pyridoxal
16 5'-phosphate, an ALT enzyme cofactor) is very common in alcoholic live r d isease (>90% in one
17 study) However, it is rarely seen with other forms of live r disease, in whic h A LT is typically
18 higher than AST. This can be used as an important diagnostic marke r. There is no correlation
19 between the degree of elevation and liver disease severity.
20
21 Elevations in gamma-glutamyltransferase (GGT), an enzyme present in .liver and other cells,
22 and in ferritin , an acute phase reactant, would likely be seen in alcoholic live r disease. Normal
23 values of AST and ALT (Choice E) or of GGT and ferritin (Choice D) wourd be less likely
24
25 {Choices 8 and C) If marked elevations (>25 times the upper limit) of AST and ALTare present,
26 toxin-induced (eg, acetaminophen), ischemic , or viral hepatitis should be suspected
27
28 Educational objective:
29
Alcoholic hepatitis is generally characterized by a ratio of aspartate aminotransferase (AST) to
30
31 alanine aminotransferase (ALT) >2, elevated gamma-glutamyltransferase, and elevated fe rritin .
32 The absolute values of AST and ALTare almost always <500 IU/ L in alcoholic liver disease.
33
34 Refer ences:
35
36 1. Diagnosis and management of alcoholic hepatitis .
37
38

--
39

~
Time Spent 3 seconds Copyright © UWorld Last updated: [04/20/2016)

-------------------------------------------------------------------------------------------------------------------------------
Feedback SuWend EnQ ock

You might also like